You are on page 1of 263

The Ultimate

Krok 2 Bases
Companion:
Comprehensive
Explanations and Insights

By Mohammed
Abdul Danish
The Ultimate Krok 2
Bases Companion:
Comprehensive
Explanations and
Insights

By Danish.
Disclaimer:

The information provided in this book, "The Ultimate Krok 2 Bases Companion: Comprehensive
Explanations and Insights," is intended for educational purposes only. It is not meant to substitute
professional medical advice, diagnosis, or treatment. The content presented herein is based on the
author's knowledge, research, and personal experiences as a medical student, and it may not reflect
the most current medical practices or guidelines. Medical knowledge is constantly evolving, and
readers are advised to consult with qualified healthcare professionals or authoritative sources for
specific medical advice or information.

The author and publisher of this book make no representations or warranties, express or implied,
regarding the accuracy, completeness, or reliability of the information provided. They disclaim any
liability, loss, or risk incurred as a direct or indirect consequence of the use or application of any
content presented in this book.

Furthermore, it is important to note that certain responses or explanations within this book may
have been generated with the assistance of an AI language model called ChatGPT. While efforts
have been made to ensure the accuracy and reliability of the responses, ChatGPT's outputs are based
on patterns and examples from a vast dataset, and it may not always provide correct or up-to-date
information. Therefore, readers are advised to independently verify and cross-reference any
information provided by ChatGPT or seek professional advice when necessary.

The inclusion of any medications, treatments, or procedures in this book does not imply
endorsement or recommendation. Readers are urged to consult healthcare professionals or refer to
authoritative sources for specific guidance related to medical treatments, medications, dosages, and
potential contraindications.

Readers are responsible for their own actions and decisions based on the information presented in
this book. The author and publisher shall not be held liable for any damages, losses, or injuries
arising from the use or misuse of the information contained within.

For any correspondence, please email at doctordoctor.doc.doc@gmail.com


Thank you
I want to take a moment to express my heartfelt gratitude and extend a sincere thank you to all of you who have
supported me throughout the creation of "The Ultimate Krok 2 Bases Companion: Comprehensive Explanations
and Insights." Your belief in me and your unwavering support have been instrumental in bringing this project to life.

I would like to extend a special thank you to my dear friends Rehan, Tanveer, and Val, who have been with me
every step of the way. Your friendship, encouragement, and unwavering support have been a constant source of
strength and inspiration.

To my mentor, Dr. Natalia Ponzel, I am incredibly grateful for your guidance, kindness, and unwavering support.
Your expertise and inspiration have shaped me as a medical student and fueled my passion for learning.

I would also like to express my gratitude to Dr. Pavlo Kolesnyk, whose innovative approach to education and
teaching has inspired me to think differently and strive for excellence. Your dedication to teaching and mentorship
is truly commendable.

To Dr. Vlad, thank you for your invaluable support during my internship and clinical rotations. Your guidance and
expertise have been invaluable in shaping my practical skills as a future clinician.

A special thanks goes to Dr. Vlasta Vysochanska for helping me gain a deeper understanding of infectious diseases.
Your knowledge and guidance have been invaluable in my studies.

To Dr. Mariana Rostoka-Reznikova, thank you for your teaching and guidance in the field of hematology.

I am immensely grateful to Dr. Ivanna Shushman for her kindness and assistance in helping me understand the
intricacies of endocrinology.

To Dr. Natalia Lishchynska, thank you for your expertise in the field of endocrinology. Your support and teachings
have expanded my knowledge and enriched my understanding.

To Dr. Valeri Isak, thank you for your invaluable assistance in the field of neurology. Your expertise and dedication
to teaching have inspired me to become a better clinician.

To Dr. Vitalya, thank you for your exceptional explanations in the field of immunology.

To Coral, for your support and assistance in reviewing and researching. Your unwavering support and
understanding have been a source of inspiration throughout this journey.

To all my other teachers, I am immensely grateful for your kindness, understanding, and dedication to my growth as
a medical student. Your unwavering support and guidance have contributed significantly to my journey.

Lastly, I want to express my gratitude to my parents, who have supported me unconditionally throughout this
endeavor. Your love, encouragement, and sacrifices have been the foundation of my success. I am forever grateful
for your unwavering support.

Each of you has played a vital role in my journey, and I am truly blessed to have such incredible individuals in my
life. Thank you for believing in me and for being there every step of the way.

With heartfelt appreciation,

Mohammed Abdul Danish


About the Author

My name is Mohammed Abdul Danish, and I am a student of Medical Faculty No.2 at Uzhhorod
National University (UzhNU) in Ukraine. It is a great honor and privilege for me to have the opportunity
to pursue my medical education in this esteemed institution.
My journey as a student has been filled with ups and downs, and I must admit that I haven't always been
the best student. I have faced my fair share of challenges and obstacles along the way. However, I firmly
believe that these experiences have shaped me into a stronger individual and have taught me valuable
lessons about perseverance, determination, and the importance of never giving up.

Living in Ukraine during these unprecedented times of pandemic and war has further added to the
difficulties. Despite the adversities, I remain hopeful and optimistic about the future. I have immense
faith in the resilience and strength of Ukraine, and I am certain that it will overcome these challenges and
emerge victorious. In the spirit of solidarity, I proudly proclaim, "Slava Ukraini!"

As a student at UzhNU, I have had the opportunity to learn from inspiring professors, engage in clinical
experiences, and immerse myself in the fascinating world of medicine. My time here has been a
transformative experience, allowing me to grow both academically and personally.

Through my book, "The Ultimate Krok 2 Bases Companion," I aim to share my knowledge, insights, and
personal experiences with fellow medical students who are preparing for the Krok 2 examination. It is my
sincere hope that this comprehensive guide will serve as a valuable resource, providing clarity and support
in their journey towards success.

I am grateful for the support and encouragement I have received from my family, friends, mentors, and
the medical community. Their unwavering belief in me has been a driving force in my pursuit of
excellence in medicine.

Thank you for joining me on this journey. Together, let us strive for knowledge, embrace challenges, and
make a positive impact in the field of medicine.

Warm regards,
Mohammed Abdul Danish
Navigation
Welcome to "The Ultimate Krok 2 Bases Companion," an extensive guide specifically curated for
medical students preparing for the Krok 2 examination. This book aims to provide you with
comprehensive explanations and insights to enhance your understanding of the subject matter. To
make the most of this resource, please take note of the following features:
Book Structure:

• Subjects: The book covers five main subjects: Therapy, Surgery, Pediatrics, Obstetrics and
Gynecology, and Hygiene.
• Subspecialties: Each subject is further divided into relevant subspecialties, allowing for
focused study and targeted learning.
• Topics: Within each subspecialty, topics are organized alphabetically, providing convenient
access to specific areas of interest.
Question Structure:
Each question is presented with a specific structure for easy comprehension and effective learning.

• The following information is provided for each question:


• Specialty, Subspecialty, and Topic: Clearly identified to help you locate questions related to
specific areas of study.
• Correct Answer: Explanation of why a particular option (e.g., option A) is the correct
answer.
• Key Points: Essential components of the question that are necessary for accurate answering.
• Explanation: Detailed information about the correct answer, including etiology,
pathophysiology, clinical features, diagnosis, treatment, and prevention (if applicable). Lab
test findings are also explained, including normal values and diagnostic significance.
• Other Options: An explanation of why the other options are incorrect, with suggestions for
modifications to make them correct.
• Medication: Information on the treatment options mentioned in the question and options,
including mechanism of action, side effects, and usage. Treatment recommendations are also
provided for conditions without specific medication mentions.
• Words: Etymology or explanation of any uncommon words used in the question.
• Findings: Normal values for findings mentioned in the question when no disease is present,
along with the diagnostic implications.
Additional Features:

• Resources Section: Towards the end of the book, you will find a dedicated section containing
links to download similar question sets, highlighting areas of similarity and reinforcing your
understanding.
• Normal Values: Two pages following the resources section provide a reference table of
normal values for Krok 2 lab tests and other relevant information.
Please note that while the book strives to provide comprehensive coverage, there may be a few
questions that lack certain steps or details. However, the information presented overall aims to assist
you in your preparation effectively.

Happy learning and best of luck on your Krok 2 examination!


Contents

Therapy

Surgery

Pediatrics

Obstetrics & Gynecology

Hygiene
Therapy
cardiology

1 Acute Coronary Syndrome

A 57-year-old male patient had an attack of retrosternal pain that lasted more than 1,5 hours.
Objectively: the patient is inert, adynamic, has pale skin, cold extremities, poor volume pulse,
heart rate - 120/min, AP - 70/40 mm Hg. ECG shows ST elevation in II, III, aVF leads. What
condition are these changes typical for?
A Cardiogenic shock
B Arrhythmogenic shock
C Perforated gastric ulcer
D Acute pericarditis
E Acute pancreatitis

Correct Answer: A - Cardiogenic shock - is the condition that these changes are typical for. The
patient's symptoms, ECG findings, and objective findings are consistent with a myocardial
infarction (MI) complicated by cardiogenic shock, a serious and potentially life-threatening
complication of MI.
Key Points:
• A 57-year-old male patient with retrosternal pain lasting over 1.5 hours.
• Patient is inert, adynamic, has pale skin, cold extremities, poor volume pulse, heart rate -
120/min, AP - 70/40 mm Hg.
• ECG shows ST elevation in II, III, aVF leads.
• Changes are typical for a specific condition.

Therapy
Explanation:
The patient's symptoms, ECG findings, and objective findings are consistent with a myocardial
infarction (MI) complicated by cardiogenic shock. The retrosternal pain lasting over 1.5 hours,
along with the ST elevation in the inferior leads (II, III, aVF), suggest an inferior wall MI. The
patient's objective findings, including low blood pressure, tachycardia, and poor perfusion,
suggest the development of cardiogenic shock, a serious and potentially life-threatening
complication of MI. In cardiogenic shock, the heart is unable to pump enough blood to meet the
body's needs, leading to hypotension, poor organ perfusion, and other serious complications.
Other Options:
B) Arrhythmogenic shock - This is not a recognized medical term. It is possible that the author
of the question meant to refer to arrhythmic shock, which is a type of cardiogenic shock that
occurs due to severe arrhythmias.
C) Perforated gastric ulcer - This is unlikely to be the cause of the patient's symptoms and ECG
findings. Perforated gastric ulcers can cause abdominal pain, but they do not typically cause ST
elevation on ECG.
D) Acute pericarditis - Acute pericarditis can cause chest pain and ECG changes, but ST
elevation is typically seen diffusely across multiple leads rather than localized to the inferior
leads.
E) Acute pancreatitis - Acute pancreatitis can cause abdominal pain, but it is not typically
associated with ST elevation on ECG.
Medication:
• Medications used to treat cardiogenic shock may include vasopressors, inotropes, and
diuretics. The specific medications and dosages will depend on the patient's individual
characteristics and comorbidities.
Words:
• Retrosternal pain: pain behind the sternum, or breastbone
• Inert: lacking energy or movement
• Adynamic: lacking or deficient in energy or vitality
• Pale skin: skin that appears abnormally pale or white due to reduced blood flow or decreased
hemoglobin levels
• Cold extremities: extremities that are cool to the touch due to reduced blood flow
cardiology

• Volume pulse: a measure of the strength and amplitude of the arterial pulse
• Cardiogenic shock: a type of shock that occurs due to impaired cardiac function, resulting in
inadequate blood flow to the body's organs and tissues
Findings:
• Heart rate: 120/min (elevated)
• Blood pressure: 70/40 mm Hg (low)
• ST elevation in II, III, aVF leads on ECG (suggestive of inferior wall MI)

Therapy
cardiology

2 Acute Coronary Syndrome

3 hours before, a 68-year-old male patient got a searing chest pain radiating to the neck and
left
forearm, escalating dyspnea. Nitroglycerin failed to relieve pain but somewhat reduced
dyspnea. Objectively: there is crimson cyanosis of face. Respiratory rate is 28/min. The patient
has vesicular breathing with isolated sibilant rales. Heart sounds are muffled, with a gallop
rhythm. Ps - 100/min, AP - 100/65 mmHg. ECG shows negative T-wave in V2-V6 leads. What
drug can reduce the heart's need for oxygen without aggravating the disease?
A Isosorbide dinitrate
B Corinfar
C Atenolol
D Streptokinase
E Aminophylline

Correct Answer: A Isosorbide dinitrate - The drug that can reduce the heart's need for oxygen
without aggravating the disease is Isosorbide dinitrate.
Key Points:
• 68-year-old male patient
• Symptoms of searing chest pain radiating to the neck and left forearm, escalating dyspnea
• Nitroglycerin failed to relieve pain but somewhat reduced dyspnea
• Objective findings of crimson cyanosis of the face, respiratory rate of 28/min, vesicular

Therapy
breathing with isolated sibilant rales, muffled heart sounds with a gallop rhythm, Ps - 100/min,
AP - 100/65 mmHg, and ECG showing negative T-wave in V2-V6 leads
• Drug that can reduce the heart's need for oxygen without aggravating the disease: Isosorbide
dinitrate
Explanation: The patient's symptoms and objective findings are suggestive of acute coronary
syndrome, which is a medical emergency that requires prompt treatment. Isosorbide dinitrate is
a drug that belongs to the class of nitrates and works by dilating the blood vessels, reducing the
workload of the heart, and improving blood flow to the heart muscle. It can reduce the heart's
need for oxygen without aggravating the disease. Isosorbide dinitrate is commonly used in the
treatment of acute coronary syndrome.
Other Options:
• B Corinfar: Corinfar is a calcium channel blocker that is used in the treatment of hypertension
and angina. It can reduce the heart's workload and oxygen demand, but it is not the first-line
treatment for acute coronary syndrome.
• C Atenolol: Atenolol is a beta-blocker that is used in the treatment of hypertension and
angina. It can reduce the heart's workload and oxygen demand, but it is not the first-line
treatment for acute coronary syndrome.
• D Streptokinase: Streptokinase is a thrombolytic agent that is used in the treatment of acute
myocardial infarction. It works by dissolving the blood clot that is causing the heart attack, but
it is not the first-line treatment for unstable angina.
• E Aminophylline: Aminophylline is a bronchodilator that is used in the treatment of asthma
and chronic obstructive pulmonary disease. It does not have a role in the treatment of acute
coronary syndrome.
Making Other Options Right:
• B Corinfar: Corinfar could be useful in the treatment of hypertension and angina, but it is not
the first-line treatment for acute coronary syndrome.
• C Atenolol: Atenolol could be useful in the treatment of hypertension and angina, but it is not
the first-line treatment for acute coronary syndrome.
• D Streptokinase: Streptokinase is the first-line treatment for acute myocardial infarction, not
unstable angina.
cardiology

• E Aminophylline: Aminophylline does not have a role in the treatment of acute coronary
syndrome.
Medication: Treatment for acute coronary syndrome includes antithrombotic agents, antiplatelet
agents, beta-blockers, nitrates, and reperfusion therapy. The choice of treatment depends on
the type and severity of the acute coronary syndrome. In this case, the first-line treatment is
nitrates, such as isosorbide dinitrate, to reduce the heart's oxygen demand. Antiplatelet agents,
such as aspirin and P2Y12 inhibitors, and antithrombotic agents, such as heparin or low-
molecular-weight heparin, are also commonly used in the treatment of acute coronary
syndrome.
Words: Acute coronary syndrome - a group of conditions that includes unstable angina and
myocardial infarction, characterized by the sudden onset of chest pain or discomfort due to
reduced blood flow to the heart. Cyanosis - a bluish discoloration of the skin and mucous
membranes due to decreased oxygen saturation in the blood. Gallop rhythm - an abnormal
heart rhythm that sounds like a horse galloping, usually indicative of heart failure.
Findings:
• Ps - 100/min (normal range: 60-100 beats/min)
• AP - 100/65 mmHg (normal range: less than 120/80 mmHg)
• ECG shows negative T-wave in V2-V6 leads (normal: positive T-wave)
• Respiratory rate is 28/min (normal: 12-20 breaths/min)
Antiplatelet Agents:
• MOA: Inhibit platelet aggregation and the formation of blood clots.
• Names: Aspirin, Clopidogrel, Ticagrelor, Prasugrel, Dipyridamole
• Dosage:
◦ Aspirin: 81-325 mg once daily
◦ Clopidogrel: 75 mg once daily
◦ Ticagrelor: 90 mg twice daily

Therapy
◦ Prasugrel: 10 mg once daily
◦ Dipyridamole: 200-400 mg twice daily
• Side Effects: Bleeding, gastrointestinal upset, allergic reactions, increased risk of bleeding in
combination with other anticoagulant medications.
Antithrombotic Agents:
• MOA: Inhibit the formation of blood clots by interfering with the coagulation cascade.
• Names: Heparin, Low-molecular-weight heparin (LMWH), Warfarin, Direct oral anticoagulants
(DOACs) - Apixaban, Rivaroxaban, Dabigatran, Edoxaban
• Dosage:
◦ Heparin: IV bolus of 80 units/kg followed by a continuous infusion of 18 units/kg/hour
(adjusted based on patient weight and laboratory monitoring)
◦ LMWH: Enoxaparin 1 mg/kg subcutaneously every 12 hours or Dalteparin 200 units/kg
subcutaneously once daily
◦ Warfarin: Initial dose of 5 mg/day, adjusted based on international normalized ratio (INR)
monitoring
◦ DOACs: Dosage varies depending on the specific medication and indication
• Side Effects:
◦ Heparin/LMWH: Bleeding, thrombocytopenia, osteoporosis, hypersensitivity reactions
◦ Warfarin: Bleeding, skin necrosis, hypersensitivity reactions
◦ DOACs: Bleeding, gastrointestinal upset, renal impairment, hypersensitivity reactions.
Direct Anticoagulants:
Direct anticoagulants are medications that directly inhibit specific coagulation proteins, such as
thrombin or factor Xa. They have a more predictable anticoagulant effect than indirect
anticoagulants and do not require routine laboratory monitoring. Examples of direct
anticoagulants include:
• Dabigatran (direct thrombin inhibitor)
• Rivaroxaban, Apixaban, and Edoxaban (direct factor Xa inhibitors)
Indirect Anticoagulants:
Indirect anticoagulants are medications that inhibit the synthesis or function of vitamin K-
dependent coagulation factors. They have a more variable anticoagulant effect than direct
cardiology

anticoagulants and require routine laboratory monitoring to ensure that the anticoagulant effect
is within the therapeutic range. Examples of indirect anticoagulants include:
• Warfarin
• Heparin and low-molecular-weight heparin (LMWH)
In summary, direct anticoagulants directly inhibit specific coagulation proteins, while indirect
anticoagulants inhibit the synthesis or function of vitamin K-dependent coagulation factors.
Direct anticoagulants have a more predictable anticoagulant effect and do not require routine
laboratory monitoring, while indirect anticoagulants have a more variable anticoagulant effect
and require routine laboratory monitoring. The choice of anticoagulant depends on the specific
clinical situation and individual patient factors.

Therapy
cardiology

3 Acute Coronary Syndrome

On the 3rd day after the acute anterior myocardial infarction a 55 y.o. patient complains of
dull ache behind his breast bone, that can be reduced by bending forward, and of dyspnea.
Objectively: AP- 140/180 mm Hg, heart sounds are dull. ECG results: atrial fibrillation with
frequence
of ventricular contractions at the rate of 110/min, pathological Q wave and S-T segment raising
in the
right chest leads. The patient refused from thrombolisis. What is the most probable diagnosis?
A Acute pericarditis
B Pulmonary embolism
C Tietze's syndrome
D Dissecting aortic aneurysm
E Dressler's syndrome

Correct Answer: The most probable diagnosis is A) Acute pericarditis.


Key Points:
• Dull ache behind the breastbone.
• Dyspnea.
• Elevated blood pressure (AP- 140/180 mm Hg).
• Dull heart sounds.
• ECG findings of atrial fibrillation with a frequency of ventricular contractions at the rate of

Therapy
110/min, pathological Q wave, and S-T segment raising in the right chest leads.
• Refusal of thrombolysis.
• Occurring on the 3rd day after acute anterior myocardial infarction.
Explanation:
Acute pericarditis is an inflammation of the pericardium, the sac-like structure that surrounds
the heart. It commonly presents with chest pain that is typically sharp and worsened by deep
breathing and lying flat, but it can also be dull and aching, as in this case. The pain can be
relieved by leaning forward, and dyspnea may be present due to the compression of the lungs
by the inflamed pericardium. Dull heart sounds may be heard due to the presence of fluid
around the heart. ECG changes in pericarditis can include diffuse ST-segment elevation or PR-
segment depression, which can be mistaken for an acute myocardial infarction. Atrial fibrillation
can also occur as a result of pericarditis.
Other Options:
B) Pulmonary embolism - unlikely, as the symptoms and ECG findings are not typical of
pulmonary embolism.
C) Tietze's syndrome - unlikely, as Tietze's syndrome is characterized by swelling of the
costochondral joints and tenderness on palpation, and it does not cause dyspnea or atrial
fibrillation.
D) Dissecting aortic aneurysm - unlikely, as this would typically present with sudden, severe
chest or back pain, and the ECG findings would not be typical.
E) Dressler's syndrome - less likely, as it typically presents later, several weeks to months after
an acute myocardial infarction, and the symptoms and ECG findings are not typical of Dressler's
syndrome.
Making Other Options Right:
B) To make pulmonary embolism the correct answer, the patient would need to have classic
symptoms such as sudden onset dyspnea, pleuritic chest pain, and tachycardia. The ECG would
typically show sinus tachycardia, right ventricular strain pattern, or S1Q3T3 pattern. Imaging,
such as a CT angiogram, would be needed to confirm the diagnosis.
C) To make Tietze's syndrome the correct answer, the patient would need to have characteristic
swelling and tenderness on palpation of the costochondral joints. There would be no ECG
changes, and dyspnea and atrial fibrillation would not be present.
cardiology

D) To make dissecting aortic aneurysm the correct answer, the patient would need to have
sudden, severe chest or back pain, with radiation to the neck or back, and a difference in blood
pressure between the arms. The ECG would not typically show ST-segment elevation, and atrial
fibrillation would not be present.
E) To make Dressler's syndrome the correct answer, the patient would need to present several
weeks to months after an acute myocardial infarction, with symptoms of pericarditis and a
fever. The ECG changes would be similar to those seen in acute pericarditis.
Words:
• Acute anterior myocardial infarction: A sudden blockage of the blood supply to the front of the
heart muscle.
• Dyspnea: Shortness of breath or difficulty breathing.
• AP: Arterial pressure, the force of blood against the walls of the arteries.
• Atrial fibrillation: A heart rhythm disorder where the upper chambers of the heart beat
irregularly and rapidly.
• Ventricular contractions: Contractions of the lower chambers of the heart that pump blood out
to the body.
• Q wave: A wave on an ECG that represents depolarization of the interventricular septum.
• S-T segment: A part of an ECG that represents the time between ventricular depolarization
and repolarization.
Findings:
• Blood pressure: 140/180 mm Hg (high)
• Heart sounds: dull (abnormal)
• ECG results: atrial fibrillation with a frequency of ventricular contractions at the rate of
110/min, pathological Q wave, and S-T segment raising in the right chest leads. (abnormal)

Therapy
cardiology

4 Acute Coronary Syndrome

A 58 y.o. patient developed acute myocardium infarction 4 hours ago, now he is in the
acute care department. ECG registers short paroxysms of ventricular tachycardia. The
most appropriate measure will be to introduct:
A Lidocain
B Flecainid
C Amyodaron
D Propafenone
E Veropamil

Correct Answer: A) Lidocaine


Key Points: A 58 y.o. patient who developed acute myocardial infarction 4 hours ago and is now
in the acute care department has short paroxysms of ventricular tachycardia.
Explanation: In the case of acute myocardial infarction, ventricular tachycardia is a potentially
life-threatening complication. Lidocaine is a class 1B antiarrhythmic agent that is used to treat
ventricular arrhythmias, such as ventricular tachycardia. It works by blocking the sodium
channels in the myocardium, which reduces automaticity and increases the electrical threshold
of the ventricular myocardium. This action reduces the incidence and severity of ventricular
arrhythmias.
Other Options:
B) Flecainide is a class 1C antiarrhythmic agent used to treat atrial and ventricular arrhythmias,

Therapy
but it has a limited effect on ventricular arrhythmias in acute myocardial infarction.
C) Amiodarone is a class 3 antiarrhythmic agent used to treat ventricular and supraventricular
arrhythmias, including ventricular tachycardia. However, it has a slow onset of action, and its
use in the acute setting is limited due to the risk of hypotension.
D) Propafenone is a class 1C antiarrhythmic agent used to treat supraventricular and ventricular
arrhythmias, but it is not a first-line agent in the treatment of ventricular tachycardia in acute
myocardial infarction.
E) Verapamil is a calcium channel blocker used to treat supraventricular arrhythmias, but it can
worsen ventricular arrhythmias and should not be used to treat ventricular tachycardia in acute
myocardial infarction.
Medication:
Lidocaine is a local anesthetic and class 1B antiarrhythmic agent. Its mechanism of action is to
block the sodium channels in the myocardium, which reduces automaticity and increases the
electrical threshold of the ventricular myocardium. Its side effects include dizziness, confusion,
convulsions, and arrhythmias.
Words:
Myocardial infarction is a condition where the blood flow to the heart muscle is blocked, leading
to damage or death of the heart muscle cells.
Ventricular tachycardia is a type of arrhythmia where the heart beats abnormally fast and
originates from the ventricles of the heart.
Paroxysms are sudden episodes or attacks of a symptom or disease.
A class 1 antiarrhythmic agent blocks the sodium channels in the myocardium and reduces
automaticity and increases the electrical threshold of the ventricular myocardium.
A class 3 antiarrhythmic agent prolongs the action potential duration in the atria and ventricles
and prolongs the refractory period of the myocardium.
cardiology

5 Acute Coronary Syndrome

A patient has got a sudden attack of severe substernal pain at night. On examination:
confusion, pallor of the skin, acrocyanosis, cold sweat, BP- 80/50 mm Hg, Ps- 120/min,
irregular and weak pulse. What condition are these symptoms typical for?
A Cardiogenic shock
B Acute left-side heart failure
C Acute right-side heart failure
D Radicular syndrome
E Acute vascular insufficienc

Correct Answer: A Cardiogenic shock


Key Points: Sudden attack of severe substernal pain at night, confusion, pallor of the skin,
acrocyanosis, cold sweat, BP- 80/50 mm Hg, Ps- 120/min, irregular and weak pulse.
Explanation: The symptoms described in the question are typical for cardiogenic shock, which is
a life-threatening condition that occurs when the heart cannot pump enough blood to meet the
body's needs. In this case, the patient has had a sudden attack of severe substernal pain, which
is likely caused by a myocardial infarction (heart attack). The decreased blood flow to the heart
can cause confusion, pallor of the skin, acrocyanosis, cold sweat, and irregular and weak pulse.
The low blood pressure and tachycardia are compensatory mechanisms to maintain adequate
perfusion to the organs. Cardiogenic shock requires immediate treatment with medications, such
as vasopressors and inotropic agents, and urgent revascularization procedures, such as

Therapy
percutaneous coronary intervention or coronary artery bypass grafting, to restore blood flow to
the heart muscle and improve cardiac function.
Other Options:
B. Acute left-side heart failure and C. Acute right-side heart failure can present with symptoms
of dyspnea, orthopnea, paroxysmal nocturnal dyspnea, and peripheral edema.
D. Radicular syndrome is a neurological condition that occurs when a nerve root is compressed
or irritated. It can cause pain, weakness, numbness, and tingling in the area supplied by the
affected nerve.
E. Acute vascular insufficiency can present with symptoms of sudden onset pain, pallor,
pulselessness, paresthesia, and paralysis in the affected limb.
Making Other Options Right:
B. Acute left-side heart failure and C. Acute right-side heart failure can cause cardiogenic shock
if the heart cannot maintain adequate cardiac output.
D. Radicular syndrome and E. Acute vascular insufficiency are not related to the symptoms
described in the question.
Words: Acrocyanosis is a bluish discoloration of the skin and mucous membranes due to
reduced oxygen supply to the tissues.
Findings: The blood pressure is decreased to 80/50 mm Hg, and the pulse rate is increased to
120/min.
cardiology

6 Acute Coronary Syndrome

A 60-year-old female patient was admitted to a hospital for acute transmural infarction. An
hour ago the patient's contition got worse. She developed progressing dyspnea, dry
cough. Respiratory rate - 30/min, heart rate - 130/min, AP- 90/60 mm Hg. Heart sounds are
muffled, diastolic shock on the pulmonary artery. There are medium moist rales in the
lower parts of lungs on the right and on the left. Body temperature - 36,4
oC. What drug should be given in the first place?
A Promedol
B Aminophylline
C Dopamine
D Heparin
E Digoxin

Correct Answer: A Promedol is the correct answer because the patient is experiencing
progressive dyspnea, dry cough, and has medium moist rales in both lungs, which are
symptoms of acute pulmonary edema. Promedol is a synthetic opioid that can be used to relieve
pain and anxiety and can also help alleviate respiratory distress by reducing the sensation of
breathlessness and the work of breathing.
Key Points:
• 60-year-old female patient with acute transmural infarction
• Developed progressing dyspnea, dry cough, respiratory rate - 30/min, heart rate - 130/min,

Therapy
AP- 90/60 mm Hg, muffled heart sounds, diastolic shock on the pulmonary artery, medium
moist rales in both lungs
• Body temperature - 36,4°C
• Drug to be given in the first place
Explanation:
• Acute pulmonary edema is a complication that can occur after acute myocardial infarction and
is characterized by the accumulation of fluid in the lungs, leading to respiratory distress.
• Promedol is a synthetic opioid that can relieve pain and anxiety and can also help alleviate
respiratory distress by reducing the sensation of breathlessness and the work of breathing.
• Aminophylline is a bronchodilator that is used to treat asthma and chronic obstructive
pulmonary disease (COPD) and is not indicated for the treatment of acute pulmonary edema.
• Dopamine is a medication that can increase blood pressure and heart rate and is not indicated
for the treatment of acute pulmonary edema.
• Heparin is an anticoagulant that can help reduce pulmonary congestion and prevent blood
clots from forming or getting larger. Although it is a standard treatment for acute pulmonary
edema in patients with acute myocardial infarction, it is not the drug of choice in this case.
Medication:
• Promedol is a synthetic opioid that works by binding to opioid receptors in the brain and spinal
cord, reducing pain and producing sedation. It can also help alleviate respiratory distress by
reducing the sensation of breathlessness and the work of breathing. Promedol can cause
respiratory depression, nausea, vomiting, and other side effects.
• Aminophylline is a bronchodilator that works by relaxing the smooth muscles in the airways,
improving breathing. It is used to treat asthma, COPD, and other respiratory conditions.
Aminophylline can cause nausea, vomiting, tremors, and other side effects.
• Dopamine is a medication that can increase blood pressure and heart rate by activating
dopamine receptors in the brain and peripheral tissues. It is used to treat low blood pressure,
shock, and other conditions. Dopamine can cause arrhythmias, hypertension, and other side
effects.
• Heparin is an anticoagulant that works by inhibiting the activity of clotting factors and
preventing the formation of blood clots. It is commonly used to prevent and treat blood clots in
patients with a variety of conditions, including acute myocardial infarction, deep vein
cardiology

thrombosis, and pulmonary embolism. Heparin can cause bleeding, thrombocytopenia, and
other side effects.
Words:
• Transmural: Refers to damage that extends through the entire thickness of a tissue or organ
wall.
• Diastolic shock: Refers to a sudden drop in diastolic blood pressure, which can be a sign of
severe heart failure or other cardiac conditions.
Findings:
• Respiratory rate: Normal range is 12-20 breaths per minute.
• Heart rate: Normal range is 60-100 beats per minute.
• AP: Normal blood pressure is generally considered to be below 120/80 mm Hg.
• Body temperature: Normal range is 36.5–37.5°C.

Therapy
cardiology

7 Arrhythmia

A 63-year-old male patient with persistent atrial fibrillation complains of moderate dyspnea.
Objectively: peripheral edemata are absent, vesicular breathing is present, heart rate - 72/min,
AP - 140/90 mm Hg. What combination of drugs will be most effective for the secondary
prevention of heart failure?
A Beta-blockers, ACE inhibitors
B Beta-blockers, cardiac glycosides
C Cardiac glycosides, diuretics
D Cardiac glycosides, ACE inhibitors
E Diuretics, beta-blockers

Correct Answer: A - Beta-blockers and ACE inhibitors - is the most effective combination of
drugs for the secondary prevention of heart failure in a patient with persistent atrial fibrillation.
Both beta-blockers and ACE inhibitors have been shown to reduce the risk of heart failure in
patients with atrial fibrillation by improving ventricular function and reducing ventricular
remodeling.
Key Points:
• A 63-year-old male patient with persistent atrial fibrillation and moderate dyspnea.
• Peripheral edema absent, vesicular breathing present, heart rate - 72/min, AP - 140/90 mm
Hg.
• Goal is secondary prevention of heart failure.

Therapy
Explanation:
Persistent atrial fibrillation is a common arrhythmia that can lead to a variety of complications,
including heart failure. The goal of secondary prevention in this patient is to reduce the risk of
heart failure by improving ventricular function and reducing ventricular remodeling.
Beta-blockers and ACE inhibitors are both effective in achieving this goal. Beta-blockers reduce
heart rate and myocardial oxygen demand, while also improving ventricular function and
reducing ventricular remodeling. ACE inhibitors reduce afterload and improve ventricular
function, as well as reducing ventricular remodeling and fibrosis.
Other Options:
B) Beta-blockers and cardiac glycosides - Cardiac glycosides, such as digoxin, are often used to
control the heart rate in patients with atrial fibrillation. However, while they may be effective in
controlling the heart rate, they do not have the same beneficial effects on ventricular function
and remodeling as beta-blockers and ACE inhibitors.
C) Cardiac glycosides and diuretics - Diuretics are often used to reduce fluid overload and
edema in patients with heart failure, but they do not have a direct effect on ventricular function
or remodeling. Cardiac glycosides may be used to control the heart rate, but again, they do not
have the same beneficial effects on ventricular function and remodeling as beta-blockers and
ACE inhibitors.
D) Cardiac glycosides and ACE inhibitors - ACE inhibitors are effective in reducing ventricular
remodeling and improving ventricular function in patients with heart failure, but cardiac
glycosides do not have the same beneficial effects on ventricular function and remodeling as
beta-blockers and ACE inhibitors.
E) Diuretics and beta-blockers - Diuretics may be used to reduce fluid overload and edema in
patients with heart failure, but they do not have a direct effect on ventricular function or
remodeling. Beta-blockers are effective in reducing ventricular remodeling and improving
ventricular function, but they do not have a direct effect on fluid overload or edema.
Medication:
• Beta-blockers and ACE inhibitors are the most effective combination of drugs for the
secondary prevention of heart failure in patients with atrial fibrillation. The specific dosages and
titration schedules may vary depending on the patient's individual characteristics and
comorbidities.
cardiology

Words:
• Atrial fibrillation: a common arrhythmia characterized by irregular and often rapid heart rate
• Dyspnea: shortness of breath
• Peripheral edema: swelling in the extremities due to fluid accumulation
• Vesicular breathing: normal breath sounds heard on auscultation of the chest
• Heart rate: the number of times the heart beats per minute
• AP: arterial blood pressure
• Ventricular function: the ability of the ventricles to contract effectively and pump blood
• Ventricular remodeling: changes in the size, shape, and function of the ventricles that occur in
response to injury or stress
Findings:
• Heart rate: 72/min (normal range: 60-100 beats per minute)
• AP: 140/90 mm Hg (elevated; normal range: < 120/80 mm Hg)
• No laboratory or additional physical findings were mentioned in the question.
Anti-arrhythmic drugs:
• Anti-arrhythmic drugs are medications used to treat arrhythmias by suppressing or controlling
abnormal heart rhythms.
• These drugs can be classified into several different classes based on their MOA and the specific
types of arrhythmias they are used to treat.
• Class I drugs are sodium channel blockers and are used to treat ventricular arrhythmias and
supraventricular tachycardia.
• Class II drugs are beta blockers and are used to treat tachyarrhythmias such as atrial
fibrillation and ventricular tachycardia.
• Class III drugs are potassium channel blockers and are used to treat ventricular arrhythmias
and atrial fibrillation.
• Class IV drugs are calcium channel blockers and are used to treat supraventricular

Therapy
tachycardia.
• Other anti-arrhythmic drugs include adenosine, digoxin, and amiodarone, among others.
Beta-blockers:
• Beta-blockers are a class of medications that block the effects of epinephrine and
norepinephrine on the heart, resulting in a decrease in heart rate and contractility.
• Beta-blockers are commonly used to treat tachyarrhythmias, such as atrial fibrillation and
ventricular tachycardia.
• Beta-blockers can also reduce the risk of sudden cardiac death in patients with heart failure or
a history of myocardial infarction.
• Examples of beta-blockers used to treat arrhythmias include metoprolol, propranolol, and
atenolol.
ACE inhibitors:
• ACE inhibitors are a class of medications that block the conversion of angiotensin I to
angiotensin II, resulting in a decrease in systemic vascular resistance and blood pressure.
• ACE inhibitors are commonly used to treat hypertension, heart failure, and other
cardiovascular conditions.
• In patients with atrial fibrillation, ACE inhibitors can improve ventricular function and reduce
the risk of heart failure.
• Examples of ACE inhibitors used to treat arrhythmias include lisinopril, enalapril, and ramipril.
Cardiac glycosides:
• Cardiac glycosides, such as digoxin, are a class of medications that increase the force of
myocardial contraction and decrease heart rate.
• Cardiac glycosides are commonly used to control the heart rate in patients with atrial
fibrillation or atrial flutter.
• In some cases, cardiac glycosides may also be used to treat heart failure.
• Cardiac glycosides have a narrow therapeutic index and can have significant side effects,
including toxicity.
• Examples of cardiac glycosides used to treat arrhythmias include digoxin and digitoxin.
Diuretics:
• Diuretics are a class of medications that increase urine output and decrease fluid overload in
the body.
cardiology

• Diuretics are commonly used to treat hypertension, heart failure, and other cardiovascular
conditions.
• In patients with heart failure or edema, diuretics can reduce fluid overload and improve
symptoms.
• Examples of diuretics used to treat arrhythmias include furosemide, hydrochlorothiazide, and
spironolactone.

Therapy
cardiology

8 Arrhythmia

A 76-year-old male consulted a therapist about slow discharge of urine with a small jet. The
patient reported no cardiac problems. Examination revealed atrial fibrillation with a heart rate of
72/min and without pulse deficit. There are no signs of heart failure. ECG confirms the presence
of atrial fibrillation. From history we know that the arrhythmia was detected three years ago.
What tactics for the treatment of atrial fibrillation in the patient should be chosen?
A Does not require treatment
B Digoxin
C Verapamil
D Obzidan
E Ajmaline

Correct Answer: The correct answer is A - Does not require treatment.


Key Points:
• 76-year-old male with slow discharge of urine with a small jet.
• Atrial fibrillation with a heart rate of 72/min and without pulse deficit.
• No signs of heart failure.
• ECG confirms the presence of atrial fibrillation.
• History of atrial fibrillation detected three years ago.
Explanation:
Atrial fibrillation is a common arrhythmia in the elderly, and its incidence increases with age. In

Therapy
this case, the patient has a history of atrial fibrillation that was detected three years ago. The
patient's current presentation with slow discharge of urine with a small jet is likely related to
benign prostatic hyperplasia, a common condition in older men that can cause urinary
symptoms.
The patient's atrial fibrillation is asymptomatic and without any signs of heart failure. Therefore,
the patient does not require treatment for atrial fibrillation. Treatment for atrial fibrillation is
usually indicated when the patient is symptomatic or when there is a risk of complications such
as stroke or heart failure.
Option B (Digoxin) is not a first-line treatment for atrial fibrillation and is mainly used for rate
control in patients with heart failure. Option C (Verapamil) and option D (Obzidan) are not
recommended for rate control in patients with atrial fibrillation due to the risk of adverse effects.
Option E (Ajmaline) is not used for the treatment of atrial fibrillation.
Other Options:
• Option B (Digoxin): Digoxin is not a first-line treatment for atrial fibrillation. It is mainly used
for rate control in patients with heart failure.
• Option C (Verapamil): Verapamil is not recommended for rate control in patients with atrial
fibrillation due to the risk of adverse effects such as hypotension and heart failure.
• Option D (Obzidan): Obzidan is not recommended for rate control in patients with atrial
fibrillation due to the risk of adverse effects such as hypotension and heart failure.
• Option E (Ajmaline): Ajmaline is not used for the treatment of atrial fibrillation.
Medication:
• No medication is required for the treatment of atrial fibrillation in this patient.
Words:
• Atrial fibrillation: A condition in which the heart's rhythm is irregular and often rapid.
• Benign prostatic hyperplasia: A condition in which the prostate gland enlarges and causes
urinary symptoms.
Findings:
• Atrial fibrillation confirmed on ECG.
• Heart rate of 72/min without pulse deficit.
• No signs of heart failure.
cardiology

9 Arrhythmia

A 26-year-old female patient has an 11-year history of rheumatism. Four years ago she
suffered 2 rheumatic attacks. Over the last 6 months there have been paroxysms of atrial
fibrillation every 2-3 months. What option of antiarrhythmic therapy or tactics should be
proposed?
A Prophylactic administration of cordarone
B Immediate hospitalization
C Defibrillation
D Lidocaine administration
E Heparin administration

Correct Answer: A Prophylactic administration of cordarone - correct.


Key Points:
• 26-year-old female patient with an 11-year history of rheumatism and 2 prior rheumatic
attacks.
• Paroxysms of atrial fibrillation every 2-3 months over the last 6 months.
Explanation:
The patient's history of rheumatism and rheumatic attacks suggests that she may have
rheumatic heart disease, a condition in which the heart valves become damaged due to
rheumatic fever. This can lead to arrhythmias such as atrial fibrillation. Prophylactic
administration of cordarone (amiodarone), an antiarrhythmic medication, is an appropriate

Therapy
treatment option for preventing future episodes of atrial fibrillation in this patient. Cordarone
can help stabilize the heart rhythm and prevent the formation of blood clots that can lead to
stroke or other complications associated with atrial fibrillation.
Other Options:
B. Immediate hospitalization may be necessary if the patient is experiencing severe symptoms
or complications related to atrial fibrillation, but is not typically necessary for every episode of
paroxysmal atrial fibrillation.
C. Defibrillation is a treatment option for ventricular fibrillation or pulseless ventricular
tachycardia, but is not typically used for atrial fibrillation.
D. Lidocaine administration is not typically used as a first-line treatment for atrial fibrillation.
E. Heparin administration may be necessary if the patient has atrial fibrillation with evidence of
blood clots, but is not typically used as a first-line treatment for paroxysmal atrial fibrillation.
Making Other Options Right:
B. To make immediate hospitalization the correct answer, the question would need to specify
that the patient is experiencing severe symptoms or complications related to atrial fibrillation,
such as hemodynamic instability or heart failure.
C. To make defibrillation the correct answer, the question would need to specify that the patient
is in ventricular fibrillation or pulseless ventricular tachycardia, rather than atrial fibrillation.
D. To make lidocaine administration the correct answer, the question would need to specify that
the patient has a ventricular arrhythmia, rather than atrial fibrillation.
E. To make heparin administration the correct answer, the question would need to specify that
the patient has evidence of blood clots, such as a history of stroke or thromboembolism.
Medication:
• Prophylactic administration of cordarone (amiodarone), an antiarrhythmic medication, can
help prevent future episodes of atrial fibrillation in patients with rheumatic heart disease.
Words:
• Rheumatism: a general term used to describe any pain or discomfort in the joints, muscles, or
connective tissue.
• Rheumatic heart disease: a condition in which the heart valves become damaged due to
rheumatic fever, leading to irregular heart rhythms and other complications.
Findings:
cardiology

• Atrial fibrillation: an irregular heart rhythm characterized by rapid and disorganized electrical
activity in the atria.
• Paroxysmal: a term used to describe a sudden and brief episode of atrial fibrillation that
typically resolves on its own.
• Cordarone (amiodarone): an antiarrhythmic medication used to treat and prevent a variety of
cardiac arrhythmias.

Therapy
cardiology

10 Arrhythmia

A 53-year-old woman complained of cardiac pain and rhythm intermissions. She had
experienced these presentations since childhood. The patient's father had a history of cardiac
arrhythmias. Objectively: the patient was in grave condition, Ps- 220 bpm, AP- 80/60 mm Hg.
ECG: heart rate - 215/min, widening and deformation of $QRS$ complex accompanied by
atrioventricular dissociation; positive $P$ wave. Some time later heart rate reduced down to
45/min, there was a complete dissociation of $P$ wave and $QRST$ complex. Which of the
following will be the most effective treatment?
A Implantation of the artificial pacemaker
B $¥beta$-adrenoreceptor blocking agents
C Cholinolytics
D Calcium antagonists
E Cardiac glycosides

Correct Answer: A - Implantation of the artificial pacemaker is the most effective treatment
because the patient is presenting with a severe brady-tachy syndrome, which can be life-
threatening. The patient's ECG findings suggest supraventricular tachycardia with
atrioventricular dissociation followed by complete heart block, which is a classic presentation of
brady-tachy syndrome. Implantation of an artificial pacemaker can restore normal heart rhythm
and prevent further complications.
Key Points:

Therapy
• 53-year-old woman with a history of cardiac pain and rhythm intermissions since childhood
• Family history of cardiac arrhythmias
• Grave condition with Ps- 220 bpm, AP- 80/60 mm Hg
• ECG findings of supraventricular tachycardia with atrioventricular dissociation followed by
complete heart block
Explanation:
The patient's history and ECG findings suggest brady-tachy syndrome, which is a condition
characterized by alternating episodes of supraventricular tachycardia and complete heart block.
This condition can be life-threatening and requires urgent treatment. Implantation of an artificial
pacemaker is the most effective treatment for brady-tachy syndrome as it can restore normal
heart rhythm and prevent further complications.
Other Options:
• B Beta-adrenoreceptor blocking agents: Beta-blockers are used in the treatment of
supraventricular tachycardia to slow down the heart rate. However, they are contraindicated in
complete heart block as they can worsen the condition.
• C Cholinolytics: Cholinolytics like atropine are used in the treatment of bradycardia or
complete heart block to increase heart rate. However, they are not effective in supraventricular
tachycardia with atrioventricular dissociation.
• D Calcium antagonists: Calcium antagonists like verapamil or diltiazem are used in the
treatment of supraventricular tachycardia to slow down the heart rate. However, they are
contraindicated in complete heart block as they can worsen the condition.
• E Cardiac glycosides: Cardiac glycosides like digoxin are used in the treatment of heart failure
and atrial fibrillation. They are not effective in brady-tachy syndrome.
Making Other Options Right:
• Option B Beta-adrenoreceptor blocking agents: If the question mentioned that the patient had
only supraventricular tachycardia without complete heart block, beta-blockers would be the
correct answer.
• Option C Cholinolytics: If the question mentioned that the patient had only complete heart
block without supraventricular tachycardia, cholinolytics would be the correct answer.
cardiology

• Option D Calcium antagonists: If the question mentioned that the patient had only
supraventricular tachycardia without complete heart block, calcium antagonists would be the
correct answer.
Medication:
• Artificial pacemaker: An electronic device that is implanted under the skin and connected to
the heart to regulate its rhythm. It delivers electrical impulses to the heart muscle to stimulate
contractions when the heart's natural pacemaker is not functioning properly.
• Beta-blockers: Medications that block the effects of adrenaline on the heart and slow down the
heart rate. Side effects include fatigue, dizziness, and hypotension.
• Cholinolytics: Medications that block the effects of acetylcholine on the heart and increase
heart rate. Side effects include dry mouth, blurred vision, and constipation.
• Calcium antagonists: Medications that block the entry of calcium into the heart cells, leading
to a decrease in heart rate and blood pressure. Side effects include headache, flushing, and
hypotension.
• Cardiac glycosides: Medications that increase the force of heart muscle contractions and slow
down the heart rate. Side effects include nausea, vomiting, and arrhythmias.
Words:
• Brady-tachy syndrome: A condition characterized by alternating episodes of supraventricular
tachycardia and complete heart block.
• Atrioventricular dissociation: A condition in which the atria and ventricles are beating
independently of each other.
• QRS complex: A wave on the ECG that represents the depolarization of the ventricles.
• P wave: A wave on the ECG that represents the depolarization of the atria.
Findings:
• Ps- 220 bpm, AP- 80/60 mm Hg: Severe brady-tachy syndrome with hypotension.
• ECG findings of supraventricular tachycardia with atrioventricular dissociation followed by

Therapy
complete heart block: Classic findings in brady-tachy syndrome.
cardiology

11 Arrhythmia

A 46-year-old patient complains of sudden palpitation, which is accompanied by pulsation


in the neck and head, fear, nausea. The palpitation lasts for 15-20 minutes and is over
after straining when holding her breath. What kind of cardiac disorder may be suspected?
A An attack of supraventricular paroxysmal tachycardia
B An attack of ventricular paroxysmal tachycardia
C An attack of atrial flutter
D An attack of ciliary arrhythmia
E An attack of extrasystolic arrhythmia

Correct Answer: A An attack of supraventricular paroxysmal tachycardia - The patient's


symptoms of sudden palpitation, pulsation in the neck and head, fear, and nausea, which are
relieved by straining and holding her breath, are consistent with an attack of supraventricular
paroxysmal tachycardia (SVT).
Key Points:
• 46-year-old patient with sudden palpitation
• Palpitation is accompanied by pulsation in the neck and head, fear, and nausea
• Palpitation lasts for 15-20 minutes
• Palpitation is relieved by straining and holding breath
Explanation:
Supraventricular paroxysmal tachycardia (SVT) is a type of arrhythmia that originates above the

Therapy
ventricles, typically in the atria or AV node. SVT can present with a variety of symptoms,
including sudden palpitation, chest discomfort, shortness of breath, and lightheadedness.
Pulsation in the neck and head, fear, and nausea are also commonly seen in patients with SVT.
In this case, the patient's symptoms of sudden palpitation, pulsation in the neck and head, fear,
and nausea, along with the relief of symptoms by straining and holding her breath, are
consistent with an attack of SVT. Diagnosis of SVT is typically made through electrocardiography
(ECG) during an episode of palpitation, and treatment may include vagal maneuvers,
medications, or cardioversion.
Other Options:
B An attack of ventricular paroxysmal tachycardia - Ventricular paroxysmal tachycardia is a type
of arrhythmia that originates in the ventricles, and typically presents with rapid and regular
heartbeats. The patient's symptoms of pulsation in the neck and head, fear, and nausea are not
typical of ventricular tachycardia.
C An attack of atrial flutter - Atrial flutter is a type of arrhythmia that originates in the atria, and
typically presents with a rapid and regular heart rate. The patient's symptoms of pulsation in the
neck and head, fear, and nausea are not typical of atrial flutter.
E An attack of extrasystolic arrhythmia - Extrasystolic arrhythmia refers to occasional irregular
heartbeats, rather than sustained episodes of palpitation. The patient's symptoms of sudden
palpitation lasting for 15-20 minutes are not typical of extrasystolic arrhythmia.
Making Other Options Right:
B An attack of ventricular paroxysmal tachycardia - This option could be made right if the
patient had symptoms more suggestive of ventricular tachycardia, such as chest discomfort or
shortness of breath.
C An attack of atrial flutter - This option could be made right if the patient had symptoms more
suggestive of atrial flutter, such as a regular and rapid heart rate.
E An attack of extrasystolic arrhythmia - This option could be made right if the patient had
occasional irregular heartbeats rather than sustained episodes of palpitation.
Medication:
• Treatment for supraventricular paroxysmal tachycardia (SVT) may include vagal maneuvers,
medications such as adenosine, beta-blockers, or calcium channel blockers, and cardioversion if
necessary.
cardiology

Words:
• Supraventricular paroxysmal tachycardia (SVT): a type of arrhythmia that originates above
the ventricles, typically in the atria or AV node. SVT can present with a variety of symptoms,
including sudden palpitation, chest discomfort, shortness of breath, and lightheadedness.
Findings:
• Sudden palpitation lasting for 15-20 minutes
• Palpitation is accompanied by pulsation in the neck and head, fear, and nausea
• Palpitation is relieved by straining and holding breath.

Therapy
cardiology

12 Arrhythmia

Adenosine triphosphate may be expected to convert which of the following arrhythmias to


sinus rhythm?
A Paroxysmal supraventricular tachycardia
B Paroxysmal ventricular tachycardia
C Atrial fibrillation
D Atrial flutter
E Ventricular fibrillation

Correct Answer: A Paroxysmal supraventricular tachycardia


Key Points: Adenosine triphosphate (ATP), convert, arrhythmias, sinus rhythm
Explanation: Adenosine triphosphate (ATP) is a naturally occurring nucleotide that plays a
crucial role in energy transfer within cells. ATP is also used as a medication to convert certain
types of arrhythmias to sinus rhythm, including paroxysmal supraventricular tachycardia
(PSVT). PSVT is a type of arrhythmia that originates above the ventricles and is often caused by
re-entry circuits within the atrioventricular node. The mechanism of action of ATP in this setting
is not fully understood, but it is thought to work by blocking the atrioventricular node,
interrupting the re-entry circuit and allowing the sinus node to regain control of the heart
rhythm.
Other Options:
B Paroxysmal ventricular tachycardia - ATP is not effective in converting ventricular tachycardia.

Therapy
C Atrial fibrillation - While ATP has been used in the past to treat atrial fibrillation, it is not a
first-line treatment for this condition and is less effective than other medications.
D Atrial flutter - ATP is less effective in converting atrial flutter than other medications such as
beta-blockers and calcium channel blockers.
E Ventricular fibrillation - ATP is not effective in converting ventricular fibrillation.
Making Other Options Right:
B Paroxysmal ventricular tachycardia - Intravenous verapamil, amiodarone, or lidocaine may be
used to convert ventricular tachycardia to sinus rhythm.
C Atrial fibrillation - Cardioversion, antiarrhythmic drugs such as amiodarone or flecainide, or
rate control with beta-blockers or calcium channel blockers are the primary treatment options
for atrial fibrillation.
D Atrial flutter - Cardioversion, antiarrhythmic drugs such as beta-blockers or calcium channel
blockers, or catheter ablation are the primary treatment options for atrial flutter.
E Ventricular fibrillation - Defibrillation and cardiopulmonary resuscitation (CPR) are the primary
treatment options for ventricular fibrillation.
Medication: Adenosine triphosphate (ATP) is a naturally occurring nucleotide that plays a crucial
role in energy transfer within cells. As a medication, ATP is used to convert certain types of
arrhythmias to sinus rhythm.
Words: Adenosine - from adenine, one of the nucleotide bases; triphosphate - three phosphates.
Findings: Sinus rhythm - Normal sinus rhythm is typically defined as a heart rate between 60
and 100 beats per minute with a regular rhythm.
cardiology

13 Arrhythmia

A 67 y.o. patient complains of palpitation, dizziness, noise in ears, feeling of shortage of air.
Objectively: pale, damp skin. Vesicular respiration, respiratory rate- 22 per min, pulse- 200
bpm, AP- 100/70 mm Hg. On ECG: heart rate- 200 bmp, ventricular complexes are
widened, deformed, location of segments ST and of wave T is discordant. The wave Р is
not changed, superimposes QRST, natural conformity between Р and QRS is not present.
What kind of arrhythmia is present?
A Paroxismal ventricular tachycardia
B Sinus tachycardia
C Atrial flutter
D Ventricular extrasystole
E Atrial tachycardia

Correct Answer: A Paroxismal ventricular tachycardia.


Key Points: 67 y.o. patient with palpitation, dizziness, noise in ears, feeling of shortage of air.
Objective findings include pale, damp skin, respiratory rate 22 per minute, pulse rate 200 bpm,
and blood pressure 100/70 mm Hg. ECG reveals ventricular complexes that are widened and
deformed, and ST segments and T waves are discordant. The P wave is not changed,
superimposes QRST, and natural conformity between P and QRS is not present.
Explanation: Paroxysmal ventricular tachycardia (PVT) is a rapid heart rhythm originating from
the ventricles, usually occurring in individuals with underlying structural heart disease. In this

Therapy
patient, the symptoms and objective findings on ECG are consistent with PVT. The widened and
deformed ventricular complexes with a rapid heart rate, in the absence of any changes in the P
wave, suggest a ventricular origin of the arrhythmia. Widening of the QRS complex to >120 ms
is characteristic of ventricular arrhythmias, and the discordant ST segments and T waves
indicate a risk of ventricular arrhythmia.
Other Options:
B) Sinus tachycardia: is an increased heart rate originating from the sinoatrial (SA) node in
response to a physiological stimulus, such as exercise or stress, but with a normal rhythm and
ECG.
C) Atrial flutter: is an abnormal heart rhythm characterized by a fast atrial rate, usually around
300 bpm, with a characteristic sawtooth-shaped flutter wave on the ECG.
D) Ventricular extrasystole: is a premature ventricular contraction (PVC) originating from the
ventricles, with a wide QRS complex followed by a compensatory pause.
E) Atrial tachycardia: is a rapid heart rate originating from the atria, typically between 150 and
250 bpm, with normal P waves on the ECG.
Making Other Options Right:
B) Sinus tachycardia would be wrong as the ECG shows a ventricular rhythm with widened QRS
complexes.
C) Atrial flutter would be wrong as the ECG does not show the characteristic sawtooth-shaped
flutter waves, and the rate is faster than what is typical of atrial flutter.
D) Ventricular extrasystole would be wrong as the patient's ECG shows a continuous ventricular
rhythm with no evidence of extra systoles.
E) Atrial tachycardia would be wrong as the ECG shows a ventricular rhythm with no evidence of
an atrial origin of the arrhythmia.
Medication: None mentioned in the question.
Words: Widened, deformed, discordant, paroxysmal.
Findings: Respiratory rate 22/min, pulse rate 200 bpm, blood pressure 100/70 mm Hg. ECG
shows ventricular complexes that are widened and deformed, and ST segments and T waves are
discordant.
cardiology

14 Arrhythmia

A 45-year-old male patient was admitted to the intensive care unit because of myocardial
infarction. An hour later the ventricular facilitation occurred. Which of the following should
be administered?
A Defibrillation
B External chest compression
C Lidocaine injection
D Adrenalin injection
E Cardiac pacing

Correct Answer: A Defibrillation should be administered.


Key Points: A 45-year-old male patient with myocardial infarction has developed ventricular
tachycardia. The correct treatment option for this condition should be identified.
Explanation: Ventricular tachycardia is a cardiac arrhythmia that originates in the ventricles of
the heart. It is defined as three or more consecutive ventricular beats at a rate of more than
100 beats per minute. Ventricular tachycardia is a potentially life-threatening condition that can
lead to ventricular fibrillation, which is a fatal arrhythmia. The treatment for ventricular
tachycardia includes immediate defibrillation, which is the delivery of an electric shock to the
heart. Defibrillation can be accomplished with an external defibrillator or an implantable
defibrillator.
Other Options:

Therapy
B. External chest compression: External chest compression is not an appropriate treatment for
ventricular tachycardia. It is used in cardiac arrest when the patient is pulseless and
unresponsive.
C. Lidocaine injection: Lidocaine injection can be used for the treatment of ventricular
tachycardia. However, in this situation, immediate defibrillation is the preferred option.
D. Adrenalin injection: Adrenalin injection is not an appropriate treatment for ventricular
tachycardia.
E. Cardiac pacing: Cardiac pacing is not an appropriate treatment for ventricular tachycardia. It
is used for bradyarrhythmias or slow heart rhythms.
Making Other Options Right:
B. External chest compression: External chest compression is the correct option for a patient in
cardiac arrest who is pulseless and unresponsive.
C. Lidocaine injection: Lidocaine injection is an appropriate treatment for ventricular
tachycardia, but immediate defibrillation is the preferred option.
D. Adrenalin injection: Adrenalin injection is not an appropriate treatment for ventricular
tachycardia.
E. Cardiac pacing: Cardiac pacing is not an appropriate treatment for ventricular tachycardia.
Medication:
Lidocaine is a class IB antiarrhythmic medication that is used to treat ventricular tachycardia. It
blocks the sodium channels in the heart and stabilizes the cell membranes, reducing the
automaticity of the myocardial cells. The common side effects of lidocaine include dizziness,
blurred vision, nausea, vomiting, and confusion.
Adrenaline is a sympathomimetic medication that stimulates the alpha and beta-adrenergic
receptors in the body. It increases the heart rate, contractility, and cardiac output, and is used
to treat cardiac arrest and anaphylaxis. The common side effects of adrenaline include
tachycardia, hypertension, and arrhythmias.
Words:
Myocardial infarction: A heart attack, caused by the blockage of blood flow to the heart muscle.
Ventricular fibrillation: A rapid, uncoordinated contraction of the ventricles of the heart, resulting
in the cessation of effective blood flow and cardiac arrest.
Findings:
cardiology

Ventricular tachycardia: A cardiac arrhythmia that originates in the ventricles of the heart. It is
defined as three or more consecutive ventricular beats at a rate of more than 100 beats per
minute.

Therapy
cardiology

15 Arrhythmia

A 70 y.o. patient complains of weakness, dizziness, short periods of unconsciousness, pain


in the cardiac area. Objectively: HR- 40 bpm, heart sounds are rhythmic, the S1 is dull,
periodically amplified. AP is 180/90 mm Hg. What is the most probable cause of
hemodynamic disturbances?
A Atrioventricular block type III
B Atrioventricular block type I
C Bradysystolic form of ciliary arrhythmia
D Sinus bradycardia
E Complete left bandle-branch block

Correct Answer: Option A, Atrioventricular (AV) block type III, is the most probable cause of the
hemodynamic disturbances.
Key Points:
• A 70-year-old patient complains of weakness, dizziness, short periods of unconsciousness, and
pain in the cardiac area.
• On examination, the patient has a heart rate of 40 bpm, rhythmic heart sounds, dull S1, and
periodically amplified sounds. The patient's blood pressure is 180/90 mm Hg.
Explanation:
• The patient's symptoms and examination findings suggest the possibility of a conduction
disorder, which can cause hemodynamic disturbances.

Therapy
• AV block type III, also known as complete heart block, is a conduction disorder that occurs
when the electrical impulses from the atria are completely blocked from reaching the ventricles.
As a result, the ventricles beat independently of the atria at a slower rate, leading to a slow
heart rate, which can cause hemodynamic disturbances and symptoms such as weakness,
dizziness, and syncope.
• Atrioventricular block type I, bradysystolic form of ciliary arrhythmia, sinus bradycardia, and
complete left bundle-branch block can also cause hemodynamic disturbances, but they are less
likely to cause the patient's symptoms and examination findings.
Other Options:
• Option B, Atrioventricular block type I, is not the correct answer because it is a less severe
form of AV block in which the electrical impulses from the atria are delayed before reaching the
ventricles. This delay can cause a slow heart rate and hemodynamic disturbances, but it is less
likely to cause the patient's symptoms and examination findings.
• Option C, Bradysystolic form of ciliary arrhythmia, is not the correct answer because it is a
rare arrhythmia that occurs in patients with ciliary dyskinesia, a respiratory disorder. This
arrhythmia is characterized by a slow heart rate and hemodynamic disturbances, but it is less
likely to cause the patient's symptoms and examination findings.
• Option D, Sinus bradycardia, is not the correct answer because it is a slow heart rate that
originates from the sinus node, the heart's natural pacemaker. Sinus bradycardia can cause
hemodynamic disturbances, but it is less likely to cause the patient's symptoms and
examination findings.
• Option E, Complete left bundle-branch block, is not the correct answer because it is a
conduction disorder that occurs when the electrical impulses that travel through the left bundle
branch are blocked or delayed. This block can cause a slow heart rate, but it is less likely to
cause the patient's symptoms and examination findings.
Medication:
• Temporary pacing: In some cases, temporary pacing with a transvenous or transcutaneous
pacemaker may be necessary to stabilize the patient's heart rate and blood pressure.
• Permanent pacing: If the AV block is persistent or recurrent, permanent pacing with a
pacemaker may be necessary. A pacemaker is a small device that is implanted under the skin of
cardiology

the chest and connected to the heart through wires. It sends electrical impulses to the heart to
regulate the heart rate and prevent hemodynamic disturbances.
• Medications: In some cases, medications such as atropine, isoproterenol, or dopamine may be
used to increase the heart rate and improve hemodynamic stability temporarily. However,
medications are not a substitute for permanent pacing.
Making Other Options Right:
• Option B could be made correct by changing the question to "What is the less severe form of
AV block in which the electrical impulses from the atria are delayed before reaching the
ventricles?"
• Option C could be made correct by changing the question to "What arrhythmia occurs in
patients with ciliary dyskinesia, a respiratory disorder, and is characterized by a slow heart rate
and hemodynamic disturbances?"
• Option D could be made correct by changing the question to "What is a slow heart rate that
originates from the sinus node, the heart's natural pacemaker?"
• Option E could be made correct by changing the question to "What is the conduction disorder
that occurs when the electrical impulses that travel through the left bundle branch are blocked
or delayed?"
Findings:
• The patient has a heart rate of 40 bpm, which is slower than the normal range of 60-100 bpm.
• The patient's blood pressure is 180/90 mm Hg, which is elevated and can cause hemodynamic
disturbances.
• The dull S1 and periodically amplified sounds are not specific findings and may indicate
underlying heart disease that can worsen AV block type III.

Therapy
cardiology

16 Arterial Embolism

A patient with a history of coronary artery disease and atrial fibrillation has the onset of
sudden pain and weakness of the left leg. Examination reveals a cool, pale extremity with
absent pulses below the groin and normal contralateral leg. The most likely diagnosis is:
A Arterial embolism
B Arterial thrombosis
C Acute thrombophlebitis
D Cerebrovascular accident
E Dissecting aortic aneurysm

Correct Answer: A Arterial embolism.


Key Points:
• Patient with a history of coronary artery disease and atrial fibrillation.
• Sudden onset of pain and weakness of the left leg.
• Examination reveals a cool, pale extremity with absent pulses below the groin and normal
contralateral leg.
Explanation:Arterial embolism is the most likely diagnosis in this patient. Arterial embolism is a
condition in which a blood clot or other debris travels from its site of origin and lodges in a
smaller artery, blocking blood flow to the affected area. Risk factors for arterial embolism
include atrial fibrillation, coronary artery disease, and other conditions that promote the
formation of blood clots.

Therapy
In this patient, the sudden onset of pain and weakness of the left leg, along with a cool, pale
extremity and absent pulses below the groin, suggests an acute arterial occlusion. The normal
contralateral leg indicates that the occlusion is localized to the affected limb.
Other Options:
• Option B, Arterial thrombosis, is less likely given the sudden onset of symptoms and absence
of previous vascular disease in the affected limb.
• Option C, Acute thrombophlebitis, is less likely given the absence of redness, warmth, and
tenderness in the affected limb.
• Option D, Cerebrovascular accident, is less likely given the symptoms are isolated to the left
leg and not indicative of a stroke.
• Option E, Dissecting aortic aneurysm, is less likely given the symptoms are isolated to the left
leg and not indicative of an aortic emergency.
Treatment:
• Arterial embolism is a medical emergency that requires prompt intervention to restore blood
flow to the affected area.
• Treatment options may include thrombolysis (dissolving the clot with medication),
thrombectomy (surgical removal of the clot), or bypass surgery (rerouting blood flow around the
occlusion).
• Anticoagulation therapy may be initiated to prevent further clot formation and reduce the risk
of recurrent embolism.
• Management of the underlying risk factors, such as atrial fibrillation or coronary artery
disease, is also important to prevent future episodes.
Words:
• Arterial embolism: A condition in which a blood clot or other debris travels from its site of
origin and lodges in a smaller artery, blocking blood flow to the affected area.
• Contralateral: On the opposite side of the body.
Findings:
• No specific values or findings are mentioned in the question.
cardiology

17 Arterial Occlusion

A 65 y.o. patient has acute pain, paresthesia, paleness of his left extremity. Pulse in the a.
dorsalis pedis is absent. There is skin coldness and paleness that gradually spreads
upwards. These symptoms are most likely to be the evidence of:
A Arterial occlusion
B Thrombophlebitis of superficial veins
C Hernia of lumbar disc
D Thrombophelebitis of deep veins
E-

Correct Answer: A. Arterial occlusion.


Key Points: Acute pain, paresthesia, paleness, absent pulse in the dorsalis pedis artery, and skin
coldness and paleness that gradually spreads upwards.
Explanation: Arterial occlusion is the most likely cause of the symptoms described in the patient.
Acute pain, paresthesia, and paleness of the extremity are typical symptoms of acute arterial
occlusion. The absence of the pulse in the dorsalis pedis artery is a strong indicator of arterial
occlusion. Skin coldness and paleness that spread upwards is an indication of ischemia. Arterial
occlusion can be caused by emboli or thrombi, and it can lead to tissue ischemia and necrosis if
left untreated.
Other Options:
B. Thrombophlebitis of superficial veins: Thrombophlebitis of superficial veins can cause

Therapy
redness, swelling, and pain in the affected area, but it does not cause paleness, paresthesia, or
absence of pulse.
C. Hernia of lumbar disc: Hernia of lumbar disc can cause radicular pain and paresthesia, but it
does not cause paleness, absence of pulse, or skin coldness.
D. Thrombophelebitis of deep veins: Thrombophelebitis of deep veins can cause swelling,
warmth, and tenderness in the affected area, but it does not cause paleness, paresthesia, or
absence of pulse.
E. Empty.
Making Other Options Right:
B. Thrombophlebitis of superficial veins: Thrombophlebitis of superficial veins can cause
redness, swelling, and pain in the affected area, but it does not cause paleness, paresthesia, or
absence of pulse.
C. Hernia of lumbar disc: Hernia of lumbar disc can cause radicular pain and paresthesia, but it
does not cause paleness, absence of pulse, or skin coldness.
D. Thrombophelebitis of deep veins: Thrombophelebitis of deep veins can cause swelling,
warmth, and tenderness in the affected area, but it does not cause paleness, paresthesia, or
absence of pulse.
E. Empty.
Medication: There is no medication mentioned in the question.
Words:
Arterial: Related to an artery, a blood vessel that carries oxygenated blood away from the heart
to the body.
Occlusion: The complete or partial blockage of a blood vessel.
Findings: The absence of the pulse in the dorsalis pedis artery is an important finding in the
diagnosis of arterial occlusion. The normal value of the dorsalis pedis pulse is present and easily
palpable. The absence of this pulse indicates the possibility of arterial occlusion.
cardiology

18 Cardiomyopathy

A 47-year-old male patient complains of compressive chest pain that occurs both at rest and
during light physical activity; irregular heartbeat. These problems arose 3 months ago. The
patient's brother died suddenly at the age of 30. Objectively: Ps - 84/min, arrhythmic, AP -
130/80 mm Hg. ECG confirms signs of left ventricular hypertrophy, abnormal Q-waves in V4-V6
leads. EchoCG reveals that interventricular septum is 1,7 cm, left ventricular wall thickness is
1,2 cm. What is the most likely diagnosis?
A Hypertrophic cardiomyopathy
B Neurocirculatory asthenia
C Exertional angina
D Myocarditis
E Pericarditis

Correct Answer: A - Hypertrophic cardiomyopathy - is the most likely diagnosis.


Key Points:
• 47-year-old male patient with compressive chest pain at rest and during light physical activity
and irregular heartbeat.
• Family history of sudden death in a younger brother.
• Ps - 84/min, arrhythmic, AP - 130/80 mm Hg.
• ECG shows signs of left ventricular hypertrophy and abnormal Q-waves in V4-V6 leads.
• EchoCG reveals an interventricular septum of 1,7 cm and left ventricular wall thickness of 1,2

Therapy
cm.
• Most likely diagnosis.
Explanation:
The patient's symptoms, physical examination findings, and diagnostic test results suggest the
presence of hypertrophic cardiomyopathy (HCM), a genetic disorder characterized by the
thickening of the heart muscle, particularly the left ventricular wall. Symptoms of HCM can
include chest pain, shortness of breath, and arrhythmias, and can be triggered by physical
activity or emotional stress. Family history is an important risk factor for HCM, as it is an
autosomal dominant genetic disorder.
The ECG findings of left ventricular hypertrophy and abnormal Q-waves in V4-V6 leads are
common in patients with HCM. The EchoCG findings of an interventricular septum of 1,7 cm and
left ventricular wall thickness of 1,2 cm are also consistent with the diagnosis of HCM.
Other Options:
B) Neurocirculatory asthenia - Neurocirculatory asthenia, also known as neurasthenia, is a term
used to describe a group of symptoms related to fatigue, weakness, and autonomic dysfunction.
It is not a cardiac disorder.
C) Exertional angina - Exertional angina is chest pain or discomfort that occurs during physical
activity due to an inadequate supply of oxygen to the heart muscle. While this symptom can be
present in patients with HCM, the ECG and EchoCG findings are not consistent with exertional
angina.
D) Myocarditis - Myocarditis is inflammation of the heart muscle, typically due to a viral
infection. While this condition can cause chest pain and arrhythmias, the ECG and EchoCG
findings are not consistent with myocarditis.
E) Pericarditis - Pericarditis is inflammation of the pericardium, the sac that surrounds the heart.
While this condition can cause chest pain and arrhythmias, the ECG and EchoCG findings are not
consistent with pericarditis.
Medication:
• Treatment for HCM typically involves medications to control symptoms, such as beta-blockers
or calcium channel blockers, and anticoagulation therapy to prevent blood clots. In some cases,
surgical intervention may be necessary to remove excess heart muscle tissue.
Words:
cardiology

• Left ventricular hypertrophy: thickening of the left ventricular wall of the heart
• Arrhythmia: abnormal heart rhythm
• EchoCG: Echocardiogram, a diagnostic test that uses ultrasound to visualize the heart
Findings:
• Ps - 84/min, arrhythmic
• AP - 130/80 mm Hg
• ECG shows signs of left ventricular hypertrophy and abnormal Q-waves in V4-V6 leads
• EchoCG reveals an interventricular septum of 1,7 cm and left ventricular wall thickness of 1,2
cm

Therapy
cardiology

19 Cardiomyopathy

A 28-year-old patient complains of periodic compressing heart pain. His brother died at the
age of 34 from a cardiac disease with similar symptoms. Objectively: the patients skin is
pale. Heart borders display no significant deviations. Heart sounds are loud, there is a
systolic murmur above all the points with a peak above the aorta. Echocardioscopy reveals
thickening of the interventricular septum in the basal parts, reduction of left ventricular
cavity. What drug should be administered in order to prevent the disease progression?
A Metoprolol
B Digoxin
C Nitroglycerin
D Captopril
E Furosemide

Correct Answer: A Metoprolol - Option A is the correct answer because it is a beta-blocker that
can help reduce the workload on the heart and slow the progression of hypertrophic
cardiomyopathy, a genetic condition that is characterized by thickening of the heart muscle.
Key Points:
• 28-year-old patient
• Periodic compressing heart pain
• Brother died from a cardiac disease with similar symptoms
• Skin is pale

Therapy
• Systolic murmur above all the points with a peak above the aorta
• Echocardiography reveals thickening of the interventricular septum in the basal parts,
reduction of left ventricular cavity
• Requires drug to prevent disease progression
Explanation:
• Hypertrophic cardiomyopathy is a genetic condition that is characterized by thickening of the
heart muscle, particularly the interventricular septum, which can lead to obstruction of blood
flow and symptoms such as chest pain, shortness of breath, and palpitations.
• The patient's symptoms of periodic compressing heart pain, family history of similar
symptoms, and echocardiographic findings of interventricular septal thickening and left
ventricular cavity reduction are consistent with hypertrophic cardiomyopathy.
• The systolic murmur above all the points with a peak above the aorta is also consistent with
hypertrophic cardiomyopathy, as it may indicate obstruction of blood flow through the aorta.
• Metoprolol is a beta-blocker that can help reduce the workload on the heart and slow the
progression of hypertrophic cardiomyopathy by reducing the heart rate and blood pressure.
Other Options:
• Digoxin (Option B) is not indicated for the treatment of hypertrophic cardiomyopathy.
• Nitroglycerin (Option C) is a medication that can help relieve angina symptoms by dilating
blood vessels, but it is not indicated for the treatment of hypertrophic cardiomyopathy.
• Captopril (Option D) is an ACE inhibitor that can help reduce blood pressure and improve heart
function in some patients with heart failure, but it is not indicated for the treatment of
hypertrophic cardiomyopathy.
• Furosemide (Option E) is a loop diuretic that can help reduce fluid buildup in the body, but it is
not indicated for the treatment of hypertrophic cardiomyopathy.
Making Other Options Right:
• To make Option B (Digoxin) right, the question could describe a patient with systolic heart
failure and symptoms such as shortness of breath and edema, with echocardiography showing
reduced left ventricular function.
• To make Option C (Nitroglycerin) right, the question could describe a patient with stable
angina and symptoms such as chest pain or discomfort, with electrocardiography showing
evidence of myocardial ischemia.
cardiology

• To make Option D (Captopril) right, the question could describe a patient with heart failure
and symptoms such as fatigue, shortness of breath, and edema, with echocardiography showing
reduced left ventricular function.
• To make Option E (Furosemide) right, the question could describe a patient with fluid overload
and symptoms such as edema or shortness of breath, with echocardiography showing normal
heart function.
Medication:
• Metoprolol is a beta-blocker that can help reduce the workload on the heart and slow the
progression of hypertrophic cardiomyopathy.
Words:
• Hypertrophic cardiomyopathy: A genetic condition that is characterized by thickening of the
heart muscle, particularly the interventricular septum, which can lead to obstruction of blood
flow and symptoms such as chest pain, shortness of breath, and palpitations.
Findings:
• There are no specific lab findings mentioned in the question.

Therapy
cardiology

20 Cardiomyopathy

A 32-year-old patient complains of cardiac irregularities, dizziness, dyspnea at physical


stress. He has never suffered from this before. Objectively: Ps- 74 bpm, rhythmic. AP130/80
mm Hg. Auscultation revealed systolic murmur above aorta, the first heart sound
was normal. ECG showed hypertrophy of the left ventricle, signs of repolarization
disturbance in the I, V5 and V6 leads. Echocardiogram revealed that
interventricular septum was 2 cm. What is the most likely diagnosis?
A Hypertrophic cardiomyopathy
B Aortic stenosis
C Essential hypertension
D Myocardium infarction
E Coarctation of aorta

Correct Answer: A) Hypertrophic cardiomyopathy is the most likely diagnosis.


Key Points:
• 32-year-old patient
• Cardiac irregularities, dizziness, dyspnea at physical stress
• Ps- 74 bpm, rhythmic
• AP- 130/80 mm Hg
• Systolic murmur above aorta
• Normal first heart sound

Therapy
• ECG showed hypertrophy of the left ventricle and repolarization disturbance in I, V5, and V6
leads
• Echocardiogram revealed an interventricular septum of 2 cm.
Explanation:Hypertrophic cardiomyopathy is a genetic disorder characterized by the thickening
of the heart muscle, which leads to an impaired ability of the heart to pump blood efficiently.
The symptoms of hypertrophic cardiomyopathy can vary from mild to severe and include
dyspnea, cardiac irregularities, and dizziness. In this patient, the systolic murmur above the
aorta, a normal first heart sound, and the hypertrophy of the left ventricle seen on the ECG and
echocardiogram are all consistent with hypertrophic cardiomyopathy.
ECG findings in hypertrophic cardiomyopathy include repolarization disturbances, such as ST-T
changes and T-wave inversion, which can be seen in leads I, V5, and V6. Echocardiography is
the most useful diagnostic tool for hypertrophic cardiomyopathy, and it can identify the
characteristic thickening of the interventricular septum seen in this patient.
Other Options:B) Aortic stenosis can cause a systolic murmur and may present with some
similar symptoms, but the hypertrophy of the left ventricle and repolarization disturbances on
the ECG make hypertrophic cardiomyopathy a more likely diagnosis.
C) Essential hypertension can lead to left ventricular hypertrophy, but it is unlikely to cause the
other symptoms and findings in this patient.
D) Myocardial infarction is unlikely, as there is no evidence of acute coronary syndrome.
E) Coarctation of the aorta can also cause a systolic murmur, but it is usually detected earlier in
life and would not explain the other findings in this patient.
Making Other Options Right:B) Aortic stenosis could be the correct answer if the patient had
evidence of calcification of the aortic valve on echocardiography and an elevated systolic
gradient across the valve.
C) Essential hypertension could be the correct answer if the patient had no evidence of
hypertrophy on echocardiography and responded to treatment with antihypertensive
medications.
D) Myocardial infarction could be the correct answer if the patient had evidence of acute
coronary syndrome on ECG or cardiac biomarkers.
E) Coarctation of the aorta could be the correct answer if the patient had evidence of a
narrowing in the aorta on imaging studies.
cardiology

Medication:Treatment for hypertrophic cardiomyopathy depends on the severity of the disease


and the patient's symptoms. Medications such as beta-blockers, calcium channel blockers, and
disopyramide can be used to reduce symptoms and prevent complications. In severe cases,
surgical intervention may be necessary, such as septal myectomy or alcohol septal ablation.
Words:Hypertrophy - from the Greek words "hyper," meaning excessive, and "trophe," meaning
nourishment or growth.
Findings:Ps- 74 bpm, rhythmic (normal heart rate is 60-100 bpm)AP- 130/80 mm Hg (normal
blood pressure is less than 120/80 mm Hg)Interventricular septum of 2 cm (normal range is
0.6-1.1 cm)

Therapy
cardiology

21 Cardiomyopathy

A 39 y.o. patient complains of having dyspnea during physical activity, crus edemata,
palpitation, heart intermissions. Objectively: HR is 150 bpm, atrial fibrillation. Heart is both
ways enlarged. Heart sounds are muted. Liver is 6 cm below the costal margin.
Echocardiogram reveals dilatation of heart chambers (end diastolic volume of left ventricle
is 6,8 cm) is 29% EF, valve apparatus is unchanged. What is the most probable
diagnosis?
A Dilated cardiomyopathy
B Exudative pericarditis
C Restrictive cardiomyopathy
D Hypertrophic cardiomyopathy
E Thyreotoxic cardiomyopathy

Correct Answer: The correct answer is A, Dilated Cardiomyopathy.


Key Points:
• A 39-year-old patient has dyspnea during physical activity, crus edema, palpitations, and
heart intermissions.
• The patient has a heart rate of 150 bpm and atrial fibrillation.
• The patient has an enlarged heart in both directions with muted heart sounds.
• The liver is 6 cm below the costal margin.
• Echocardiogram reveals dilatation of heart chambers, 29% EF, and unchanged valve

Therapy
apparatus.
• The question is asking about the most probable diagnosis.
Explanation:
• Dilated cardiomyopathy is a condition in which the heart becomes enlarged and weakened,
leading to decreased cardiac function and symptoms such as dyspnea, edema, palpitations, and
arrhythmias.
• Atrial fibrillation is a common arrhythmia associated with dilated cardiomyopathy.
• Enlargement of the heart in both directions and muted heart sounds are consistent with
dilated cardiomyopathy.
• Liver enlargement is also a common finding in patients with dilated cardiomyopathy due to
congestive hepatopathy.
• The echocardiogram findings of dilatation of heart chambers and reduced EF are also
consistent with dilated cardiomyopathy.
• Exudative pericarditis, restrictive cardiomyopathy, hypertrophic cardiomyopathy, and
thyrotoxic cardiomyopathy do not fit with the clinical presentation and findings in this case.
Other Options:
• Option B, exudative pericarditis, is a condition in which fluid accumulates in the pericardial
sac, causing compression of the heart and symptoms such as dyspnea and palpitations.
However, the echocardiogram findings of dilatation of heart chambers and reduced EF are not
consistent with pericarditis.
• Option C, restrictive cardiomyopathy, is a condition in which the heart becomes stiff, leading
to decreased cardiac function and symptoms such as dyspnea and edema. However, the
echocardiogram findings of dilatation of heart chambers and reduced EF are not consistent with
restrictive cardiomyopathy.
• Option D, hypertrophic cardiomyopathy, is a condition in which the heart muscle becomes
thickened, leading to decreased cardiac function and symptoms such as dyspnea and
palpitations. However, the echocardiogram findings of dilatation of heart chambers and reduced
EF are not consistent with hypertrophic cardiomyopathy.
• Option E, thyrotoxic cardiomyopathy, is a condition in which the heart becomes enlarged and
weakened due to excess thyroid hormone, leading to symptoms such as dyspnea and
cardiology

palpitations. However, the echocardiogram findings of dilatation of heart chambers and reduced
EF are not consistent with thyrotoxic cardiomyopathy.
Making Other Options Right:
• Option B, exudative pericarditis, could be the correct answer if the echocardiogram findings
showed signs of pericardial effusion or pericardial thickening.
• Option C, restrictive cardiomyopathy, could be the correct answer if the echocardiogram
findings showed signs of diastolic dysfunction or restrictive filling patterns.
• Option D, hypertrophic cardiomyopathy, could be the correct answer if the echocardiogram
findings showed signs of left ventricular hypertrophy or asymmetric septal hypertrophy.
• Option E, thyrotoxic cardiomyopathy, could be the correct answer if the patient had a history
of hyperthyroidism or signs of thyroid dysfunction on laboratory testing.
Medication:
• The treatment for dilated cardiomyopathy includes medications such as ACE inhibitors, beta-
blockers, diuretics, and aldosterone antagonists to improve cardiac function and reduce
symptoms.
• Anticoagulation therapy is also necessary for patients with atrial fibrillation to prevent stroke.
• In severe cases, cardiac transplantation may be necessary.
Words:
• Edema: swelling caused by excess fluid in the body tissues.
• EF: ejection fraction, a measure of the percentage of blood pumped out of the heart with each
beat.
• Pericarditis: inflammation of the pericardium, the sac that surrounds the heart.
• Thyrotoxic: relating to excess thyroid hormone in the body.
Findings:
• Liver enlargement: The liver is normally palpable 1-2 cm below the costal margin. A liver that
is 6 cm below the costal margin is significantly enlarged.

Therapy
• EF: The normal range for EF is 50-70%. The patient's EF of 29% is significantly reduced,
indicating decreased cardiac function.
cardiology

22 Complications after surgery

Five days after a total hip joint replacement a 72 year old woman becomes acutely short of
breath, diaphoretic and hypotensive. Both lung fields are clear to auscultation and
percussion, but examination of the neck reveals mild jugular venous distension with
prominent A waves. Heart sounds are normal. ECG shows sinus tachycardia with a new
right bundle branch block and minor nonspecific ST-T wave changes. The most likely
diagnosis is:
A Pulmonary thromboembolism
B Acute myocardial infarction
C Aortic dissection
D Pericarditis
E Aspiration

Correct Answer: A Pulmonary thromboembolism is the most likely diagnosis.


Key Points: Total hip joint replacement, acute onset of shortness of breath, diaphoresis,
hypotension, jugular venous distension with prominent A waves, normal heart sounds, sinus
tachycardia with a new right bundle branch block and minor nonspecific ST-T wave changes.
Explanation: The clinical scenario is highly suggestive of a pulmonary embolism, which is a
serious and potentially fatal complication of hip replacement surgery. The patient has acute
onset of shortness of breath, diaphoresis, and hypotension, which are classic symptoms of
pulmonary embolism. The jugular venous distension with prominent A waves is a sign of right

Therapy
ventricular dysfunction, which is common in pulmonary embolism. The ECG findings of sinus
tachycardia with a new right bundle branch block and minor nonspecific ST-T wave changes are
also suggestive of pulmonary embolism.
Other Options: Acute myocardial infarction (Option B) is less likely because the patient has no
chest pain, and the heart sounds are normal. Aortic dissection (Option C) is less likely because
the patient has no chest pain or back pain, and the blood pressure is not elevated. Pericarditis
(Option D) is less likely because the heart sounds are normal, and the ECG does not show any
specific changes suggestive of pericarditis. Aspiration (Option E) is less likely because there is
no history of aspiration, and the clinical presentation is not typical of aspiration.
Making Other Options Right: To make option B, Acute myocardial infarction, right, the patient
should have chest pain, and ECG should show ST-T wave changes and elevated cardiac
enzymes. To make option C, Aortic dissection, right, the patient should have severe chest or
back pain, and blood pressure should be elevated or asymmetric. To make option D, Pericarditis,
right, the patient should have pericardial friction rub on physical examination, and ECG should
show specific changes suggestive of pericarditis. To make option E, Aspiration, right, the patient
should have a history of aspiration and coughing, and chest X-ray should show findings of
aspiration pneumonitis.
Medication: The question does not mention any medication.
Words: Diaphoresis means excessive sweating.
Findings: The normal value for jugular venous distension is less than 3 cm above the sternal
angle. The prominent A waves suggest right ventricular dysfunction, which is common in
pulmonary embolism.
cardiology

23 Congenital Heart Disease

A 29-year-old female patient complains of dyspnea and palpitations on exertion. According to


her mother, as a child she had heart murmur, did not undergo any examinations. Objectively:
the
patient has pale skin, Ps- 94/min, rhythmic. AP- 120/60 mm Hg. In the II intercostal space on
the left auscultation reveals a continuous rasping systolodiastolic murmur, diastolic shock above
the pulmonary artery. Blood and urine are unremarkable. What is the most likely diagnosis?
A Patent ductus arteriosus
B Atrial septal defect
C Ventricular septal defect
D Aortarctia
E Tetralogy of Fallot

Correct Answer: A Patent ductus arteriosus - The continuous rasping systolodiastolic murmur
and diastolic shock above the pulmonary artery are indicative of patent ductus arteriosus, a
common congenital heart defect.
Key Points:
• Complaints of dyspnea and palpitations on exertion
• History of heart murmur in childhood
• Objective findings of pale skin, continuous rasping systolodiastolic murmur, and diastolic shock
above the pulmonary artery

Therapy
Explanation:
Patent ductus arteriosus (PDA) is a congenital heart defect that occurs when the ductus
arteriosus, a blood vessel that connects the pulmonary artery to the aorta during fetal
development, fails to close after birth. This results in a continuous shunt of blood flow from the
aorta to the pulmonary artery. PDA is more common in females and in premature infants.
The clinical presentation of PDA depends on the size of the defect and the degree of left-to-right
shunting. Patients may be asymptomatic or may present with symptoms of dyspnea, fatigue,
and poor exercise tolerance. On physical examination, a continuous murmur is heard in the left
upper sternal border, which may be louder during systole or diastole. A palpable thrill and a
diastolic shock may be present over the pulmonary artery. The diagnosis can be confirmed by
echocardiography.
Treatment of PDA depends on the size of the defect and the degree of shunting. Small PDAs
may close spontaneously over time, while larger PDAs may require medical or surgical
intervention. Medical treatment may include administration of indomethacin, a nonsteroidal anti-
inflammatory medication that can stimulate closure of the ductus. Surgical intervention may
involve ligation or division of the ductus.
Other Options:
B) Atrial septal defect: Atrial septal defect (ASD) is another congenital heart defect that results
in a shunt of blood flow between the atria. However, the location of the murmur and the
presence of a diastolic shock above the pulmonary artery make PDA a more likely diagnosis.
C) Ventricular septal defect: Ventricular septal defect (VSD) is a congenital heart defect that
results in a shunt of blood flow between the ventricles. The location of the murmur and the
presence of a diastolic shock above the pulmonary artery make PDA a more likely diagnosis.
D) Aortarctia: Aortarctia, also known as coarctation of the aorta, is a congenital heart defect
that results in narrowing of the aorta. It can cause hypertension in the upper extremities and
decreased blood flow to the lower extremities. The location of the murmur and the presence of a
diastolic shock above the pulmonary artery make PDA a less likely diagnosis.
E) Tetralogy of Fallot: Tetralogy of Fallot is a congenital heart defect that involves four
abnormalities, including a ventricular septal defect, pulmonary stenosis, right ventricular
hypertrophy, and overriding aorta. The location of the murmur and the presence of a diastolic
shock above the pulmonary artery make PDA a less likely diagnosis.
cardiology

Making Other Options Right:


B) Atrial septal defect could be a possible diagnosis if the location of the murmur and the
presence of a diastolic shock were over the right upper sternal border instead of the left.
C) Ventricular septal defect could be a possible diagnosis if the location of the murmur and the
presence of a diastolic shock were over the left lower sternal border instead of the left upper
sternal border.
D) Aortarctia could be a possible diagnosis if the location of the murmur and the presence of a
diastolic shock were over the right upper sternal border and if the patient had hypertension in
the upper extremities and decreased blood flow to the lower extremities.
E) Tetralogy of Fallot could be a possible diagnosis if the patient had additional findings of
pulmonary stenosis, right ventricular hypertrophy, and overriding aorta.
Medication:
• Indomethacin is a nonsteroidal anti-inflammatory medication that can be used to stimulate
closure of a PDA in some cases.
• Other medications may be used to manage symptoms of PDA, such as diuretics or medications
to lower blood pressure.
Words:
• Patent ductus arteriosus: A congenital heart defect that occurs when the ductus arteriosus
fails to close after birth, resulting in a continuous shunt of blood flow from the aorta to the
pulmonary artery.
• Diastolic shock: A palpable, pulsatile sensation that occurs during diastole when there is a
large pressure gradient across a vessel or valve.
• Left-to-right shunting: A type of shunt in which blood flows from the left side of the heart to
the right side of the heart.
Findings:
• Normal values for Ps, AP, and urine and blood tests are not mentioned in the question.

Therapy
• The presence of a continuous rasping systolodiastolic murmur and diastolic shock above the
pulmonary artery are abnormal findings indicative of a congenital heart defect such as PDA.
cardiology

24 Congenital Heart Disease

A 56 year old man complains of fatigue, dyspnea on exertion and palpitations. He has had
a murmur since childhood. Examination reveals a lift at the left sternal border, split S1,
and fixed splitting of S2. There is a grade 3/6 midsystolic pulmonic murmur and a 1/6
middiastolic tricuspid murmur at the lower left sternal border. Chest x-ray shows right
ventricular enlargement and prominent pulmonary arteries. ECG demonstrates atrial
fibrillation with a right bundle branch block. The most likely diagnosis is:
A Arterial septal defect
B Coarctation of the aorta
C Patent ductus arteriosus
D Tetralogy of Fallot
E Ventricular septal defect

Correct Answer: Arterial septal defect (Option A) is the most likely diagnosis.
Key Points: The patient has a history of a murmur since childhood, and examination reveals a
lift at the left sternal border, split S1, and fixed splitting of S2. There is a grade 3/6 midsystolic
pulmonic murmur and a 1/6 middiastolic tricuspid murmur at the lower left sternal border.
Chest x-ray shows right ventricular enlargement and prominent pulmonary arteries. ECG
demonstrates atrial fibrillation with a right bundle branch block.
Explanation: Arterial septal defect (ASD) is a congenital heart defect in which there is a
communication between the left and right atria, allowing shunting of blood. The condition allows

Therapy
oxygenated blood from the left atrium to mix with deoxygenated blood from the right atrium,
leading to increased blood flow to the lungs and right-sided heart enlargement. Over time, this
can lead to pulmonary hypertension, right ventricular enlargement, and heart failure. ASD is
typically asymptomatic in childhood, and symptoms may not appear until adulthood. Symptoms
may include fatigue, dyspnea on exertion, palpitations, and right-sided heart failure. Physical
examination findings in ASD may include a fixed splitting of S2, a systolic ejection murmur at
the left upper sternal border, and a diastolic murmur at the lower left sternal border. The ECG
may show right atrial enlargement, right ventricular hypertrophy, and right bundle branch block.
The chest x-ray may show right atrial and ventricular enlargement and prominent pulmonary
arteries.
Other Options:
• Coarctation of the aorta (B) is a narrowing of the aorta, typically occurring at the site of the
ductus arteriosus insertion, which can cause hypertension in the upper extremities, diminished
pulses in the lower extremities, and radiofemoral delay.
• Patent ductus arteriosus (C) is an abnormal connection between the pulmonary artery and the
aorta, which can cause pulmonary hypertension, bounding pulses, and a continuous murmur.
• Tetralogy of Fallot (D) is a congenital heart disease characterized by a combination of four
defects, including pulmonary stenosis, right ventricular hypertrophy, ventricular septal defect,
and an overriding aorta, leading to cyanosis and clubbing of the fingers.
• Ventricular septal defect (E) is a congenital heart disease characterized by a hole in the wall
(septum) that separates the two lower chambers of the heart (ventricles), leading to increased
blood flow to the lungs and left-sided heart enlargement.
Making Other Options Right:
• Coarctation of the aorta (B) can present with hypertension in the upper extremities,
diminished pulses in the lower extremities, and radiofemoral delay.
• Patent ductus arteriosus (C) can cause pulmonary hypertension, bounding pulses, and a
continuous murmur.
• Tetralogy of Fallot (D) can present with cyanosis and clubbing of the fingers.
• Ventricular septal defect (E) can cause increased blood flow to the lungs and left-sided heart
enlargement.
Findings:
cardiology

• Right ventricular enlargement: An increase in the size of the right ventricle of the heart.
• Prominent pulmonary arteries: Enlargement of the pulmonary arteries, which can be seen on a
chest X-ray.
• Split S1: A finding on physical examination where the first heart sound (S1) is heard in two
distinct components instead of a single sound.
• Fixed splitting of S2: A finding on physical examination where the second heart sound (S2) is
heard in two distinct components and does not vary with respiration.
• Midsystolic pulmonic murmur: A heart murmur that occurs between the first and second heart
sounds and is heard best over the pulmonic valve area.
• Middiastolic tricuspid murmur: A heart murmur that occurs in mid-diastole and is heard best
over the tricuspid valve area.

Therapy
cardiology

25 Congestive Heart Failure

A 30-year-old patient complains of breathlessness, pain in the right rib arc region, dry
cough and the edema of legs. He is ill for 2 months. He had been treated for rheumatic
fever without any effect. On exam: cyanosis, edema of legs, BT of 36,6oC, RR of
28/min, HR of 90/min, BP of 110/80 mm Hg, crackles above low parts of both lungs, heart
borders are displaced to the left and to the right, weak sounds, systolic murmur above the
apex. What is the preliminary diagnosis?
A Dilated cardiomyopathy
B Infectious endocarditis
C Acute myocarditis
D Rheumatic fever, mitral stenosis
E Acute pericarditis

correct answer is Option A, Dilated cardiomyopathy.


The patient's symptoms of breathlessness, pain in the right rib arc region, dry cough, and
edema of legs, along with the presence of crackles above low parts of both lungs, heart borders
displaced to the left and to the right, and weak sounds, are consistent with congestive heart
failure. The systolic murmur above the apex may be present due to the dilated left ventricle,
which is the hallmark of dilated cardiomyopathy. Additionally, the history of rheumatic fever is
not suggestive of mitral stenosis in this case.
Therefore, the preliminary diagnosis is likely to be Option A, Dilated cardiomyopathy.

Therapy
Other Options:
• Option A, Dilated cardiomyopathy, is the correct answer.
• Option B, Infectious endocarditis, is less likely given the absence of fever, heart murmurs, and
other typical signs of infection.
• Option C, Acute myocarditis, may present with similar symptoms, but usually has a more
acute onset and is associated with a recent viral infection.
• Option D, Rheumatic fever, mitral stenosis, is less likely given the absence of typical signs of
mitral stenosis such as a diastolic murmur, loud S1, and an opening snap.
• Option E, Acute pericarditis, may present with similar symptoms, but is usually associated with
a sharp chest pain that worsens with inspiration and is relieved by sitting up or leaning forward.
Treatment:
• The treatment for dilated cardiomyopathy typically involves medications to improve heart
function, such as beta-blockers, ACE inhibitors, or angiotensin receptor blockers. Diuretics may
also be used to reduce fluid accumulation in the lungs and peripheral tissues. In severe cases,
heart transplantation may be necessary.
Words:
• Dilated cardiomyopathy: A condition characterized by dilation of the heart chambers and
impaired cardiac function.
• Congestive heart failure: A condition characterized by impaired cardiac function leading to the
accumulation of fluid in the lungs and peripheral tissues.
• Systolic murmur: A heart murmur that is heard during systole, which is the phase of the
cardiac cycle when the heart is contracting.
• Rheumatic fever: A systemic inflammatory disease that can lead to various cardiac
complications, including mitral stenosis.
• Infectious endocarditis: An infection of the heart valves or the endocardium, which can cause
various cardiac complications.
• Acute myocarditis: An inflammation of the myocardium, which can cause various symptoms
including chest pain, dyspnea, and palpitations.
• Acute pericarditis: An inflammation of the pericardium, which can cause chest pain and other
symptoms.
Findings:
cardiology

• The patient's clinical findings include breathlessness, pain in the right rib arc region, dry
cough, and edema of legs, as well as cyanosis, edema of legs, crackles above low parts of both
lungs, heart borders displaced to the left and to the right, and weak sounds.
• These findings are consistent with congestive heart failure, which is a condition characterized
by impaired cardiac function leading to the accumulation of fluid in the lungs and peripheral
tissues.
• The presence of a systolic murmur above the apex suggests dilated cardiomyopathy, which is
a common cause of congestive heart failure.
• The absence of typical signs of mitral stenosis such as a diastolic murmur, loud S1, and an
opening snap makes rheumatic fever, mitral stenosis (Option D) less likely.
• The absence of fever, heart murmurs, and other signs of infection makes infectious
endocarditis (Option B) less likely.
• Acute myocarditis (Option C) and acute pericarditis (Option E) may present with similar
symptoms, but have different underlying causes.

Therapy
cardiology

26 Cor Pulmonale

A 57-year-old male patient complains of dyspnea on exertion, heaviness in the right


hypochondrium and shin edemata towards evening. Objectively: temperature - $38,1^oC$,
HR20/min, HR=Ps=92/min, AP- 140/90 mm Hg. There is apparent kyphoscoliosis. In the lungs
single dry rales can be auscultated. Heart sounds are muffled, rhythmic. ECG: Rv1+Sv5=15
mm. X-ray picture shows the bulging of pulmonary artery cone, right ventricle enlargement.
What
is the most likely cause of this condition?
A Pulmonary heart
B Atherosclerotic cardiosclerosis
C Dilatation cardiomyopathy
D Mitral stenosis
E Primary pulmonary hypertension

Correct Answer: A - Pulmonary heart - is the correct answer.


Key Points:
• 57-year-old male patient with dyspnea on exertion, heaviness in the right hypochondrium, and
shin edema towards evening
• Objective findings: temperature - $38,1^oC$, HR=20/min, HR=Ps=92/min, AP- 140/90 mm
Hg, kyphoscoliosis, single dry rales in lungs, muffled and rhythmic heart sounds, Rv1+Sv5=15
mm on ECG, bulging of pulmonary artery cone, right ventricle enlargement on X-ray

Therapy
• Most likely cause of this condition
Explanation:
• Pulmonary heart disease, also known as cor pulmonale, is a condition in which the right side of
the heart becomes enlarged and weakened due to pulmonary hypertension or lung disease.
• The symptoms of dyspnea on exertion, heaviness in the right hypochondrium, and shin edema
towards evening are consistent with cor pulmonale.
• Kyphoscoliosis is a condition in which there is abnormal curvature of the spine, which can lead
to respiratory and cardiovascular problems.
• Dry rales in the lungs may be a sign of pulmonary congestion or fluid buildup.
• Muffled and rhythmic heart sounds may be a sign of right ventricular enlargement and
dysfunction.
• The bulging of the pulmonary artery cone and right ventricular enlargement on X-ray are
consistent with cor pulmonale.
• The ECG finding of Rv1+Sv5=15 mm may also be seen in cor pulmonale due to right
ventricular hypertrophy.
• The treatment for cor pulmonale depends on the underlying cause and may include
medications to reduce pulmonary hypertension, oxygen therapy, and treatment of the
underlying lung disease.
Other Options:
• B: Atherosclerotic cardiosclerosis is a condition in which there is narrowing of the coronary
arteries due to plaque buildup. This condition is not consistent with the patient's presentation.
• C: Dilatation cardiomyopathy is a condition in which there is enlargement and weakening of
the heart muscle. This condition can cause symptoms such as dyspnea and edema, but it is not
consistent with the patient's presentation.
• D: Mitral stenosis is a condition in which there is narrowing of the mitral valve, which can
cause symptoms such as dyspnea and pulmonary congestion. However, this condition is not
consistent with the patient's presentation.
• E: Primary pulmonary hypertension is a condition in which there is high blood pressure in the
pulmonary arteries, leading to right ventricular hypertrophy and dysfunction. This condition is a
potential cause of cor pulmonale, but it is not the most likely cause in this patient given the
presence of lung disease and kyphoscoliosis.
cardiology

Making Other Options Right:


• To make option B, Atherosclerotic cardiosclerosis, right, the question would need to be
changed to describe a patient with symptoms of angina or myocardial infarction, which are
typical of coronary artery disease.
• To make option C, Dilatation cardiomyopathy, right, the question would need to be changed to
describe a patient with symptoms of heart failure such as dyspnea, edema, and fatigue.
• To make option D, Mitral stenosis, right, the question would need to be changed to describe a
patient with symptoms such as dyspnea, orthopnea, and hemoptysis, which are typical of mitral
stenosis.
• To make option E, Primary pulmonary hypertension, right, the question would need to be
changed to describe a patient with no underlying lung disease or kyphoscoliosis and with a
history of exposure to drugs or toxins that can cause pulmonary hypertension.
Medication:
• The treatment for cor pulmonale depends on the underlying cause and may include
medications to reduce pulmonary hypertension, oxygen therapy, and treatment of the
underlying lung disease.
Words:
• Cor pulmonale: a condition in which the right side of the heart becomes enlarged and
weakened due to pulmonary hypertension or lung disease.
• Pulmonary hypertension: high blood pressure in the pulmonary arteries, which can lead to
right ventricular hypertrophy and dysfunction.
• Kyphoscoliosis: abnormal curvature of the spine in both the sagittal and coronal planes.
• Rales: abnormal crackling or rattling sounds heard on auscultation of the lungs.
Findings:
• Normal body temperature range is 36.5°C to 37.5°C.
• Normal heart rate range is 60-100 beats per minute.

Therapy
• Normal blood pressure range is 90/60 mm Hg to 120/80 mm Hg.
• Normal ECG findings do not show Rv1+Sv5 > 10.5 mm in males.
cardiology

27 Coronary Artery Disease (CAD)

A 24-year-old female teacher complains of dizziness and heart pain irradiating to the left nipple.
Pain is not associated with physical activity and cannot be relieved by nitroglycerin, it abates
after taking Valocordin and lasts an hour or more. The patient has a nearly 2-year history of this
disease. Objectively: Ps- 76 bpm. AP- 110/70 mm Hg. Heart borders are normal, heart sounds
are clear. The ECG shows respiratory arrhythmia. Radiograph of the cervicothoracic spine
shows no pathology. Lungs, abdomen are unremarkable. What changes in blood formula can be
expected?
A No changes
B Leukocytosis
C Thrombocytopenia
D Leukemic hiatus
E Increased ESR

Correct Answer: A No changes - No changes in blood formula can be expected.


Key Points:
• A 24-year-old female teacher complains of dizziness and heart pain irradiating to the left
nipple.
• Pain is not associated with physical activity and cannot be relieved by nitroglycerin, it abates
after taking Valocordin and lasts an hour or more.
• The patient has a nearly 2-year history of this disease.

Therapy
• Objectively, the patient has a heart rate of 76 bpm and a blood pressure of 110/70 mm Hg.
• The ECG shows respiratory arrhythmia, and a radiograph of the cervicothoracic spine shows no
pathology.
• The question asks what changes in blood formula can be expected.
Explanation:
The patient's symptoms of dizziness and heart pain irradiating to the left nipple, along with the
history of the disease, suggest the possibility of coronary artery disease. However, the fact that
the pain is not associated with physical activity and cannot be relieved by nitroglycerin, but
abates after taking Valocordin, makes this diagnosis less likely.
The absence of any significant findings on physical examination, including normal heart borders
and clear heart sounds, further reduces the likelihood of significant cardiovascular disease.
Therefore, it is unlikely that any changes in blood formula can be expected in this patient.
Other Options:
• B Leukocytosis: Leukocytosis is an increase in the number of white blood cells in the blood.
This can occur in response to infection or inflammation, but is unlikely in the absence of other
clinical findings.
• C Thrombocytopenia: Thrombocytopenia is a decrease in the number of platelets in the blood.
This can occur in a variety of conditions, but is unlikely in the absence of other clinical findings.
• D Leukemic hiatus: Leukemic hiatus refers to a period of time during which leukemia may be
present but not producing any clinical symptoms or abnormalities on laboratory testing. This is
unlikely in this patient, as there are no other clinical findings to suggest leukemia.
• E Increased ESR: The erythrocyte sedimentation rate (ESR) is a non-specific marker of
inflammation. It can be elevated in a variety of conditions, but is unlikely in the absence of
other clinical findings.
Making Other Options Right:
• B Leukocytosis: If the patient had signs of infection or inflammation, leukocytosis may be a
more likely finding.
• C Thrombocytopenia: If the patient had a history of bleeding or bruising, or other clinical
findings suggestive of a bleeding disorder, thrombocytopenia may be a more likely finding.
• D Leukemic hiatus: If the patient had other clinical findings suggestive of leukemia, such as
unexplained weight loss or fatigue, leukemic hiatus may be a more likely finding.
cardiology

• E Increased ESR: If the patient had signs of inflammation, such as joint pain or swelling,
increased ESR may be a more likely finding.
Medication:
• Valocordin is a brand name for a medication containing phenobarbital,
ethylbromisovalerianate, and hop extract. Phenobarbital is a sedative that can help relieve
anxiety and tension, while ethylbromisovalerianate and hop extract are used to relieve spasms
and improve digestion.
• Nitroglycerin is a medication used to treat angina by dilating the blood vessels and improving
blood flow to the heart.
Words:
• Coronary artery disease: a condition in which the arteries that supply blood to the heart
become narrowed or blocked, leading to reduced blood flow and oxygen supply to the heart
muscle.
• Nitroglycerin: a medication used to treat angina by dilating the blood vessels and improving
blood flow to the heart.
• Valocordin: a brand name for a medication containing phenobarbital, ethylbromisovalerianate,
and hop extract.
• Leukocytosis: an increase in the number of white blood cells in the blood.
• Thrombocytopenia: a decrease in the number of platelets in the blood.
• Leukemic hiatus: a period of time during which leukemia may be present but not producing
any clinical symptoms or abnormalities on laboratory testing.
• Erythrocyte sedimentation rate (ESR): a non-specific marker of inflammation.
Findings:
• Ps (pulse rate): 76 bpm (normal range: 60-100 bpm)
• AP (blood pressure): 110/70 mmHg (normal range: <120/80 mmHg)
• Normal values for blood formula (white blood cell count, platelet count, etc.) are not applicable

Therapy
as there is no indication for these tests in this patient.
cardiology

28 Diagnostic Method

An 18-year-old patient presents no problems. Percussion reveals that heart borders are
displaced to the right and left by 1 cm, there is a coarse systolic murmur with its epicenter
within
the 4th intercostal space on the left. What is the most informative examination to confirm the
clinical diagnosis?
A Ventriculography
B ECG
C PCG
D Echocardiography
E Polycardiography

Correct Answer: A - Ventriculography - is the correct answer.


Key Points:
• 18-year-old patient with heart border displacement and a coarse systolic murmur with its
epicenter within the 4th intercostal space on the left.
• Most informative examination to confirm the clinical diagnosis.
Explanation:
• The clinical findings of heart border displacement and a systolic murmur suggest a possible
diagnosis of congenital heart disease.
• Ventriculography is an imaging technique that involves injecting a contrast agent into the

Therapy
heart's chambers to assess its structure and function. This procedure can help confirm the
diagnosis of congenital heart disease and identify the specific type and severity of the defect.
• ECG (electrocardiogram) is a non-invasive test that records the electrical activity of the heart
and can help identify abnormal rhythms and other cardiac abnormalities. However, it is less
informative than ventriculography for diagnosing structural abnormalities.
• PCG (phonocardiogram) is a recording of heart sounds and murmurs and is useful in
characterizing the timing and intensity of murmurs. However, it does not provide information on
the underlying structural abnormality.
• Echocardiography is an imaging technique that uses ultrasound to visualize the heart's
structure and function. It is useful in diagnosing structural abnormalities and assessing cardiac
function, but ventriculography may be more informative in certain cases.
• Polycardiography is a non-invasive technique that records multiple leads of electrocardiograms
simultaneously and can provide information on cardiac electrical activity, but it is not useful for
diagnosing structural abnormalities.
Other Options:
• B: ECG is a non-invasive test that records the electrical activity of the heart and can help
identify abnormal rhythms and other cardiac abnormalities. However, it is less informative than
ventriculography for diagnosing structural abnormalities.
• C: PCG is a recording of heart sounds and murmurs and is useful in characterizing the timing
and intensity of murmurs. However, it does not provide information on the underlying structural
abnormality.
• D: Echocardiography is an imaging technique that uses ultrasound to visualize the heart's
structure and function. It is useful in diagnosing structural abnormalities and assessing cardiac
function, but ventriculography may be more informative in certain cases.
• E: Polycardiography is a non-invasive technique that records multiple leads of
electrocardiograms simultaneously and can provide information on cardiac electrical activity, but
it is not useful for diagnosing structural abnormalities.
Making Other Options Right:
• To make option B, ECG, right, the question would need to be changed to describe a patient
with symptoms suggestive of an arrhythmia or electrical abnormality.
cardiology

• To make option C, PCG, right, the question would need to be changed to describe a patient
with symptoms suggestive of a heart murmur that requires further characterization.
• To make option D, Echocardiography, right, the question would need to be changed to
describe a patient with a suspected structural cardiac abnormality that can be visualized with
ultrasound imaging.
• To make option E, Polycardiography, right, the question would need to be changed to describe
a patient with symptoms suggestive of an arrhythmia or electrical abnormality that requires
further characterization with multiple leads of electrocardiograms simultaneously.
Medication:
• No medication is mentioned or required for the diagnostic procedure of ventriculography.
Words:
• Ventriculography: an imaging technique that involves injecting a contrast agent into the
heart's chambers to assess its structure and function.
• Congenital heart disease: a group of structural heart defects present at birth that can affect
the heart's structure and function.
Findings:
• No specific lab or physical exam findings are mentioned in the question.

Therapy
cardiology

29 Diagnostic Method

A 37 y.o. woman is suffering from squeezing substernal pain on physical exertion. On


examination: AP- 130/80 mm Hg, heart rate=pulse rate 72 bpm, heart boarders are dilated
to the left side, aortic systolic murmur. ECG- signs of the left venticle hypertrophy. What
method of examination is the most informative in this case?
A Echocardiography
B Phonocardiography
C Coronarography
D Sphygmography
E X-ray

Correct Answer: A Echocardiography is the most informative method of examination in this


case.Key Points:
• 37 y.o. woman
• Squeezing substernal pain on physical exertion
• AP- 130/80 mm Hg
• Heart rate=pulse rate 72 bpm
• Heart boarders are dilated to the left side
• Aortic systolic murmur
• ECG shows signs of left ventricular hypertrophyExplanation:
• The clinical presentation, physical examination, and ECG findings in this case suggest the

Therapy
possibility of a cardiac pathology, most likely valvular or ischemic heart disease.
• Echocardiography is a noninvasive diagnostic tool that can provide detailed information about
the structure and function of the heart, including the size and shape of the chambers, the
thickness of the heart muscle, the function of the valves, and the presence of any abnormalities
or defects.
• In this case, echocardiography can help diagnose the underlying cardiac pathology responsible
for the patient's symptoms and physical findings, such as aortic stenosis, mitral regurgitation, or
coronary artery disease.Other Options:
• B Phonocardiography is a method of recording and analyzing the sounds of the heart using a
microphone and electronic equipment. While it may provide additional information about the
nature and timing of the heart sounds and murmurs, it is not as informative as
echocardiography in diagnosing the underlying cardiac pathology.
• C Coronarography is an invasive diagnostic procedure that involves injecting contrast dye into
the coronary arteries and taking X-ray images to assess blood flow and detect any blockages or
narrowing. While it may be indicated in some cases of suspected coronary artery disease, it is
not the most informative method of examination in this case, given the patient's presentation
and findings.
• D Sphygmography is a method of recording and analyzing the arterial pulse waveforms using
a pressure transducer and electronic equipment. It may provide information about the arterial
stiffness and compliance and the presence of any peripheral vascular disease, but it is not as
informative as echocardiography in diagnosing the underlying cardiac pathology.
• E X-ray may show signs of cardiomegaly or pulmonary congestion, but it is not as informative
as echocardiography in diagnosing the underlying cardiac pathology.Making Other Options
Right:
• B Phonocardiography may be useful in recording and analyzing the nature and timing of the
heart sounds and murmurs, but it is not as informative as echocardiography in diagnosing the
underlying cardiac pathology.
• C Coronarography may be indicated in some cases of suspected coronary artery disease, but it
is not the most informative method of examination in this case, given the patient's presentation
and findings.
cardiology

• D Sphygmography may provide information about the arterial stiffness and compliance and
the presence of any peripheral vascular disease, but it is not as informative as echocardiography
in diagnosing the underlying cardiac pathology.
• E X-ray may show signs of cardiomegaly or pulmonary congestion, but it is not as informative
as echocardiography in diagnosing the underlying cardiac pathology.Medication:
• The treatment for the underlying cardiac pathology will depend on the specific diagnosis, but it
may include medication, such as beta blockers, calcium channel blockers, or nitrates, to improve
symptoms and reduce the risk of complications.
• For example, in the case of aortic stenosis, surgical intervention, such as valve replacement,
may be necessary.
• Findings:
• AP: 130/80 mm Hg (normal range: <120/80 mm Hg)
• Heart rate=pulse rate: 72 bpm (normal range: 60-100 bpm)
• Left ventricular hypertrophy on ECG: indicative of increased thickness or mass of the left
ventricular wall, which may be caused by conditions such as hypertension, aortic stenosis, or
cardiomyopathy.

Therapy
cardiology

30 Diagnostic Method

A 52-year-old male patient complains about attacks of asphyxia, pain in his right side
during respiration. These manifestations turned up all of a sudden. It is known from his
anamnesis that he had been treated for thrombophlebitis of the right leg for the last month.
In the admission ward the patient suddenly lost consciousness, there was a sudden attack
of asphyxia and pain in his side. Objectively: heart rate - 102/min, respiratory rate - 28/min,
AP- 90/70 mm Hg. Auscultation revealed diastolic shock above the pulmonary artery,
gallop rhythm, small bubbling rales above the lungs under the scapula on the right, pleural
friction rub. What examination method will be the most informative for a diagnosis?
A Angiography of pulmonary vessels
B Echocardioscopy
C Study of external respiration function
D ECG
E Coagulogram

Correct Answer: A Angiography of pulmonary vessels is the correct answer because the patient's
symptoms and clinical findings suggest that he is likely experiencing pulmonary embolism,
which is a potentially life-threatening condition caused by the obstruction of one or more
pulmonary arteries by a blood clot. Angiography of pulmonary vessels is the most informative
test to confirm the diagnosis, as it can directly visualize the pulmonary arteries and identify any
obstruction.

Therapy
Key Points:
• 52-year-old male patient with sudden onset of asphyxia and pain in the side.
• History of thrombophlebitis of the right leg.
• Loss of consciousness in the admission ward.
• Heart rate - 102/min, respiratory rate - 28/min, AP- 90/70 mm Hg.
• Diastolic shock above the pulmonary artery, gallop rhythm, small bubbling rales above the
lungs under the scapula on the right, pleural friction rub.
• Most informative test for diagnosis.
Explanation:
• Pulmonary embolism is a serious condition that occurs when a blood clot travels to the lungs
and blocks one or more pulmonary arteries, leading to reduced blood flow and oxygenation.
• The patient's symptoms and clinical findings suggest that he is likely experiencing pulmonary
embolism, including sudden onset of asphyxia and pain in the side, as well as diastolic shock
above the pulmonary artery and small bubbling rales above the lungs under the scapula on the
right.
• Angiography of pulmonary vessels is the most informative test to confirm the diagnosis of
pulmonary embolism, as it can directly visualize the pulmonary arteries and identify any
obstruction.
• Echocardiography can also be useful in the diagnosis of pulmonary embolism, as it can identify
signs of right heart strain and dysfunction, but it is not as specific as angiography.
• Study of external respiration function, ECG, and coagulogram can provide some information
about the patient's respiratory and cardiovascular status and coagulation profile, but they are
not specific for the diagnosis of pulmonary embolism.
Other Options:
• B Echocardioscopy is not the correct answer because while it can help in the diagnosis of
pulmonary embolism by identifying signs of right heart strain and dysfunction, such as dilated
right ventricle and tricuspid regurgitation, it is not as specific as angiography.
• C Study of external respiration function is not the correct answer because it can provide
information about the patient's respiratory status, but it is not specific for the diagnosis of
pulmonary embolism.
cardiology

• D ECG is not the correct answer because it can provide information about the patient's heart
rate, rhythm, and signs of ischemia or injury, but it is not specific for the diagnosis of pulmonary
embolism.
• E Coagulogram is not the correct answer because it can provide information about the
patient's coagulation profile, including the risk of thrombosis or bleeding, but it is not specific for
the diagnosis of pulmonary embolism.
Making Other Options Right:
• B Echocardioscopy could be the correct answer if the patient's symptoms and clinical findings
suggest that he is experiencing right heart strain and dysfunction, which can be confirmed by
echocardiography.
• C Study of external respiration function could be the correct answer if the patient's symptoms
and clinical findings suggest that he is experiencing respiratory distress, which can be assessed
by spirometry or arterial blood gas analysis.
• D ECG could be the correct answer if the patient's symptoms and clinical findings suggest that
he is experiencing cardiac ischemia, arrhythmia, or other cardiac conditions, which can be
identified by ECG.
• E Coagulogram could be the correct answer if the patient's symptoms and clinical findings
suggest that he is at high risk of thrombosis or bleeding, which can be assessed by coagulation
testing.
Words:
• Thrombophlebitis: Refers to inflammation of a vein caused by a blood clot.
• Pulmonary embolism: Refers to the obstruction of one or more pulmonary arteries by a blood
clot, leading to reduced blood flow and oxygenation.
• Diastolic shock: Refers to a sudden drop in diastolic blood pressure, which can be a sign of
severe heart failure or other cardiac conditions.
• Gallop rhythm: Refers to an abnormal heart rhythm that sounds like a horse galloping,

Therapy
indicating the presence of a third or fourth heart sound.
• Bubbling rales: Refers to abnormal breath sounds caused by the presence of fluid or mucus in
the lungs.
• Pleural friction rub: Refers to a creaking or rubbing sound heard during breathing, caused by
inflammation of the pleura.
Findings:
• Heart rate: Normal range is 60-100 beats per minute.
• Respiratory rate: Normal range is 12-20 breaths per minute.
• AP: Normal blood pressure is generally considered to be below 120/80 mm Hg.
cardiology

31 Diagnostic Method

During examination at a military commissariat a 15-year-old teenager was found to have


interval sysolic murmur on the cardiac apex, diastolic shock above the pulmonary artery,
tachycardia. Which of the suuplemental examination methods will be the most informative
for the diagnosis specification?
A Echocardiography
B Electrocardigraphy
C Roengenography
D Phonocardiography
E Rheography

Correct Answer: A Echocardiography.


Key Points:
• 15-year-old teenager with an interval systolic murmur on the cardiac apex, diastolic shock
above the pulmonary artery, and tachycardia.
• The question asks for the most informative supplemental examination method for diagnosis
specification.
Explanation:
• The patient's clinical presentation is suggestive of a congenital heart defect, such as an atrial
septal defect (ASD) or a ventricular septal defect (VSD).
• An interval systolic murmur on the cardiac apex is consistent with a VSD, while a diastolic

Therapy
shock above the pulmonary artery is consistent with an ASD.
• Tachycardia may be a compensatory response to the hemodynamic changes caused by the
heart defect.
• The most informative supplemental examination method for diagnosis specification is
echocardiography, which is a non-invasive imaging technique that uses ultrasound waves to
visualize the heart structures and blood flow patterns.
• Echocardiography can confirm the presence and location of the heart defect, determine its size
and severity, and assess its impact on the heart function and pulmonary circulation.
• Electrocardiography (ECG) can provide information on the heart rhythm and electrical
conduction, but it is less specific and sensitive for diagnosing congenital heart defects.
• Roentgenography can show the size and shape of the heart and lungs, but it is less
informative than echocardiography for diagnosing congenital heart defects.
• Phonocardiography and rheography are techniques that record and analyze heart sounds and
the blood flow patterns in the limbs, respectively. They may be useful in some cases, but they
are not as informative as echocardiography for diagnosing congenital heart defects.
Other Options:
• B Electrocardiography (ECG) can provide information on the heart rhythm, electrical
conduction, and signs of chamber enlargement or hypertrophy, but it is less specific and
sensitive for diagnosing congenital heart defects.
• C Roentgenography can show the size and shape of the heart and lungs, as well as signs of
pulmonary congestion or edema, but it is less informative than echocardiography for diagnosing
congenital heart defects.
• D Phonocardiography is a technique that records and analyzes heart sounds, including
murmurs, but it is not as informative as echocardiography for diagnosing congenital heart
defects.
• E Rheography is a technique that records and analyzes the blood flow patterns in the limbs,
but it is not as informative as echocardiography for diagnosing congenital heart defects.
Making Other Options Right:
• B would be correct if the question described a patient with suspected arrhythmia, conduction
abnormalities, or chamber hypertrophy or enlargement.
cardiology

• C would be correct if the question described a patient with suspected pulmonary or cardiac
pathology, such as pulmonary hypertension or cardiomegaly.
• D would be correct if the question described a patient with suspected heart sounds
abnormalities, such as murmur timing or intensity.
• E would be correct if the question described a patient with suspected peripheral vascular
disease, such as arterial or venous insufficiency.
Medication:
• Treatment of congenital heart defects depends on the type, size, and severity of the defect, as
well as the patient's age, symptoms, and hemodynamic status.
• Small and asymptomatic defects may not require treatment and may close spontaneously over
time.
• Large or symptomatic defects may require surgical or catheter-based interventions, such as
closure devices, patches, or open-heart surgery.
• Medications may be used to manage symptoms, such as heart failure, arrhythmias, or
pulmonary hypertension.
Words:
• Systolic: from Greek "systolē", meaning contraction or squeezing.
• Diastolic: from Greek "diastolē", meaning relaxation or dilation.
• Hemodynamic: from Greek "haima", meaning blood, and "dynamis", meaning power or force.
Findings:
• Interval systolic murmur on the cardiac apex.
• Diastolic shock above the pulmonary artery.
• Tachycardia.

Therapy
cardiology

32 Drug-induced Toxicity.

A patient has chronic heart failure of the II stage. He takes furosemide regularly three times a
week. He had developed bronchopneumonia and had been administered combined
pharmacotherapy. On the fifth day of therapy the patient complained of hearing impairment.
What drug coadministered with furosemide might have caused the hearing loss?
A Gentamicin
B Linex
C Nystatin
D Tavegil
E Mucaltin

Correct Answer: A - Gentamicin - is the correct answer.


Key Points:
• Patient with chronic heart failure stage II taking furosemide regularly
• Developed bronchopneumonia and received combined pharmacotherapy
• Complaint of hearing impairment on fifth day of therapy
• Drug coadministered with furosemide that may cause hearing loss
Explanation:
• Chronic heart failure is a progressive condition in which the heart is unable to pump blood
effectively, leading to symptoms such as shortness of breath and fatigue.
• Furosemide is a loop diuretic commonly used in the treatment of heart failure to reduce fluid

Therapy
buildup in the body.
• Bronchopneumonia is a type of lung infection that can occur as a complication of heart failure.
• Gentamicin is an aminoglycoside antibiotic that is commonly used to treat infections, including
pneumonia.
• One of the potential side effects of gentamicin is ototoxicity or hearing loss.
• The mechanism of gentamicin-induced ototoxicity is related to damage to the hair cells in the
inner ear, which are responsible for hearing.
• The treatment for gentamicin-induced hearing loss depends on the severity of the damage and
may include discontinuing the medication, hearing aids, or cochlear implants.
Other Options:
• B: Linex is a probiotic supplement that is not known to cause hearing loss as a side effect.
• C: Nystatin is an antifungal medication that is not known to cause hearing loss as a side
effect.
• D: Tavegil is an antihistamine medication that is not known to cause hearing loss as a side
effect.
• E: Mucaltin is a cough medication that is not known to cause hearing loss as a side effect.
Making Other Options Right:
• To make option B, Linex, right, the question would need to be changed to ask for a drug that
can cause gastrointestinal upset or diarrhea as a side effect.
• To make option C, Nystatin, right, the question would need to be changed to ask for a drug
that can cause oral thrush or other fungal infections as a side effect.
• To make option D, Tavegil, right, the question would need to be changed to ask for a drug
that can cause drowsiness or sedation as a side effect.
• To make option E, Mucaltin, right, the question would need to be changed to ask for a drug
that can cause nausea or vomiting as a side effect.
Medication:
• Furosemide is a loop diuretic used to reduce fluid buildup in heart failure patients.
• Gentamicin is an aminoglycoside antibiotic used to treat infections, including pneumonia.
• The treatment for gentamicin-induced hearing loss depends on the severity of the damage and
may include discontinuing the medication, hearing aids, or cochlear implants.
Words:
cardiology

• Heart failure: a condition in which the heart is unable to pump blood effectively, leading to
symptoms such as shortness of breath and fatigue.
• Furosemide: a loop diuretic commonly used in the treatment of heart failure to reduce fluid
buildup in the body.
• Bronchopneumonia: a type of lung infection that can occur as a complication of heart failure.
• Gentamicin: an aminoglycoside antibiotic commonly used to treat infections, including
pneumonia.
• Ototoxicity: damage to the hair cells in the inner ear, leading to hearing loss.
Findings:
• Hearing impairment is not a typical side effect of furosemide.
• Gentamicin-induced hearing loss may be temporary or permanent and can occur with
prolonged use or high doses of the medication.

Therapy
cardiology

33 Dystonia

A 20-year-old patient complains of breath shortness, continuous dull heart pain, irritability.
Objectively: general condition is satisfactory, the pulse is labile, AP- 130/60 mm Hg. ECG shows
repolarization disorder. The patient has been diagnosed with cardiac-type neurocirculatory
dystonia. The patient should receive treatment under the following conditions:
A Outpatient treatment
B Inpatient treatment at the therapeutic department
C Inpatient treatment at the cardiology department
D Inpatient treatment at the cardiac surgery department
E Inpatient treatment at the psychiatric department

Correct Answer: A. Outpatient treatment - cardiac-type neurocirculatory dystonia is a benign


condition that does not require inpatient treatment, and can be managed through outpatient
treatment.
Key Points:
• 20-year-old patient with breath shortness, dull heart pain, irritability
• General condition satisfactory, labile pulse, elevated blood pressure, ECG shows repolarization
disorder
• Diagnosis of cardiac-type neurocirculatory dystonia
Explanation:
Cardiac-type neurocirculatory dystonia is a benign condition that does not require inpatient

Therapy
treatment. The patient's symptoms of breath shortness, dull heart pain, and irritability are likely
due to anxiety or stress, and can be managed through outpatient treatment, such as relaxation
techniques or therapy. The patient's labile pulse and elevated blood pressure may also be
related to anxiety, and can be monitored through regular visits to a primary care physician or
cardiologist. The repolarization disorder seen on ECG is a nonspecific finding that can occur with
many different conditions, including anxiety.
Other Options:
B. Inpatient treatment at the therapeutic department - this is not necessary for cardiac-type
neurocirculatory dystonia, which is a benign condition that can be managed through outpatient
treatment.
C. Inpatient treatment at the cardiology department - this is not necessary for cardiac-type
neurocirculatory dystonia, which is a benign condition that can be managed through outpatient
treatment.
D. Inpatient treatment at the cardiac surgery department - this is not necessary for cardiac-type
neurocirculatory dystonia, which is a benign condition that can be managed through outpatient
treatment.
E. Inpatient treatment at the psychiatric department - this may be appropriate if the patient's
symptoms are related to a psychiatric condition, but there is no indication for inpatient
treatment for cardiac-type neurocirculatory dystonia.
Medication:
• No specific medication mentioned in the question
Words:
• Neurocirculatory dystonia: a condition characterized by symptoms of autonomic nervous
system dysfunction, such as palpitations, chest pain, and shortness of breath, that are often
related to stress or anxiety
Findings:
• Elevated blood pressure and repolarization disorder on ECG are not normal, but are
nonspecific findings that can occur with many different conditions, including anxiety.
cardiology

34 ECG

Generalized low voltage on an ECG (QRS deflection < 5 mm in limb leads and < 10 mm in
precordial leads) may be a marker for all of the following disorders EXCEPT:
A Hyperthyroidism
B Pericardial effusion
C Cardiac transplant rejection
D Amyloidosis
E Coronary artery disease

Correct Answer: Option A (Hyperthyroidism) is the correct answer.


Key Points: Generalized low voltage on an ECG (QRS deflection < 5 mm in limb leads and < 10
mm in precordial leads).
Explanation: Generalized low voltage on an ECG refers to a decrease in amplitude of QRS
complex in limb leads (less than 5 mm) and in precordial leads (less than 10 mm). It is a non-
specific finding that may indicate several underlying disorders such as cardiomyopathies,
infiltrative disorders, connective tissue diseases, and pericardial effusion. However, it is not a
marker for hyperthyroidism, as the thyroid gland does not directly affect the electrical
conduction system of the heart.
Other Options: Options B, C, D, and E are all disorders that can be associated with generalized
low voltage on an ECG.

Therapy
cardiology

35 Heart Failure

A 62 year old patient complains of rest dyspnea, heart pains. 3 years ago he had
myocardial infarction. Physical examination: orthopnea, acrocyanosis, swollen cervical
veins. Ps - 92, total heart enlargement, the liver is enlarged by 7 cm, shin edema. What is
the stage of chronic heart failure (CHF)?
A CHF- 2 B
B CHF- 1
C CHF- 2 А
D CHF- 0
E CHF- 3

Correct Answer: Option A (CHF-2) is the correct answer.


Key Points:
• 62-year-old patient with a history of myocardial infarction
• Symptoms of rest dyspnea, heart pains, orthopnea, acrocyanosis, swollen cervical veins, total
heart enlargement, liver enlargement by 7 cm, and shin edema
• Asking for the stage of chronic heart failure
Explanation:
Based on the patient's symptoms and physical examination findings, the stage of chronic heart
failure is CHF-2. CHF-2 is defined as "mild to moderate symptoms of heart failure at rest or with
normal activity."

Therapy
The patient's symptoms of dyspnea and chest pain at rest, along with orthopnea, suggest that
they are experiencing pulmonary congestion. Swollen cervical veins, liver enlargement, and shin
edema suggest that the patient is experiencing peripheral edema.
The combination of these symptoms indicates that the patient has mild to moderate symptoms
of heart failure, which puts them in CHF-2 stage.
The New York Heart Association (NYHA) functional classification system is commonly used to
classify CHF, which categorizes patients into four stages based on their symptoms and level of
physical activity.
NYHA class 1: Patients with no limitation of physical activity. Ordinary physical activity does not
cause undue fatigue, palpitation, or dyspnea (shortness of breath).
NYHA class 2: Patients with slight limitation of physical activity. They are comfortable at rest.
Ordinary physical activity results in fatigue, palpitation, or dyspnea.
The symptoms of heart failure in NYHA class 2 include fatigue, palpitations, and shortness of
breath during ordinary physical activity. Patients may also experience mild chest discomfort,
cough, or decreased exercise tolerance.
Diagnosis of heart failure stage 2 involves a physical examination, including a thorough medical
history, auscultation of the heart and lungs, and evaluation of peripheral edema. Tests such as
electrocardiogram (ECG), echocardiogram, chest X-ray, and blood tests may also be used to
diagnose heart failure and assess its severity.
The treatment of heart failure in NYHA class 2 aims to improve the patient's quality of life by
managing symptoms, slowing disease progression, and reducing complications. Treatment
options may include medications such as angiotensin-converting enzyme (ACE) inhibitors,
angiotensin receptor blockers (ARBs), beta-blockers, diuretics, and aldosterone antagonists.
Lifestyle changes such as weight management, exercise, smoking cessation, and a low-sodium
diet may also be recommended. In some cases, cardiac resynchronization therapy or
implantable cardioverter-defibrillator (ICD) may be necessary.
Stage 2 of heart failure is further divided into 2 sub-stages: 2a and 2b.
2a - Patients with stage 2a heart failure have slight limitations of physical activity. They may
experience symptoms such as fatigue, palpitations, dyspnea, and angina or chest pain,
especially during physical exertion. They are also at risk for fluid retention and swelling in the
legs, ankles, or feet.
cardiology

2b - Patients with stage 2b heart failure have marked limitations of physical activity. They may
experience symptoms even at rest, such as fatigue, palpitations, dyspnea, and chest pain. They
may also have swelling in the legs, ankles, or feet, and are at risk for developing shortness of
breath or wheezing with activity.
Stage 3 of CHF is characterized by marked limitations of physical activity, comfortable only at
rest. Less than ordinary physical activity causes fatigue, palpitation, dyspnea, or anginal pain.
Patients with Stage 3 CHF are usually symptomatic and can present with orthopnea, paroxysmal
nocturnal dyspnea, and peripheral edema. The diagnosis is confirmed by echocardiography,
which shows structural abnormalities of the heart and reduced ejection fraction. Treatment
includes optimizing medical therapy, such as ACE inhibitors, beta-blockers, aldosterone
antagonists, and diuretics. Cardiac resynchronization therapy or implantable cardioverter-
defibrillator may be considered in select patients.
Stage 4 of CHF is characterized by symptoms of heart failure at rest, and any physical activity
worsens the symptoms. Patients with Stage 4 CHF are severely symptomatic and have marked
limitations of physical activity. They can present with the symptoms of Stage 3 CHF, along with
other symptoms such as persistent fatigue, lethargy, confusion, and decreased urine output.
The diagnosis is confirmed by echocardiography, which shows severe structural abnormalities of
the heart and markedly reduced ejection fraction. Treatment includes optimizing medical
therapy and considering advanced heart failure therapies, such as left ventricular assist device
or heart transplantation.

Therapy
cardiology

36 Heart Failure

A 57-year-old man complains of shortness of breath, swelling on shanks, irregularity in


cardiac work, pain in the left chest half with irradiation to the left scapula.Treatment is
uineffective. On physical exam: heart's sounds are diminished, soft systolic murmur on the
apex. Ps - 100/min, arrhythmical, BP - 115/75 mm Hg. The liver is +2 cm, painful.
Roentgenoscopy:
enlargement of heart shadow to all sides, pulsation is weak. Electrocardiogram (ECG):
leftventricled
extrasystolia, decreased voltage. What method of investigation is necessary to do to determine
the
diagnosis?
A Echocardiography
B Veloergometria
C X-ray kymography
D ECG in the dynamics
E Coronarography

Correct Answer: Option A, Echocardiography, is the necessary investigation to determine the


diagnosis.
Key Points:
• A 57-year-old man complains of shortness of breath, swelling on shanks, irregularity in cardiac

Therapy
work, pain in the left chest half with irradiation to the left scapula, and ineffective treatment.
• On physical examination, the patient has diminished heart sounds, a soft systolic murmur on
the apex, a +2 cm painful liver, and an enlarged heart shadow with weak pulsation on X-ray.
• The electrocardiogram (ECG) shows left ventricular extrasystolia and decreased voltage.
• The question asks for the necessary investigation to determine the diagnosis.
Explanation:
• Heart failure is a condition in which the heart is unable to pump enough blood to meet the
body's needs.
• Echocardiography is a noninvasive imaging test that uses sound waves to create detailed
images of the heart's structure and function.
• The test can help to diagnose heart failure by assessing the size and function of the heart
chambers, the thickness and movement of the heart muscle, the function of the heart valves,
and the presence of any fluid around the heart.
• Symptoms of heart failure may include shortness of breath, swelling in the legs or abdomen,
fatigue, and chest pain or discomfort.
• Physical examination findings in heart failure may include diminished heart sounds, murmurs,
hepatomegaly, and an enlarged heart on X-ray.
• Electrocardiogram (ECG) findings in heart failure may include arrhythmias, such as
extrasystoles, and decreased voltage.
Other Options:
• Option B, Veloergometria, is not necessary to determine the diagnosis of heart failure.
Veloergometria, also known as exercise stress testing, is used to evaluate the heart's response
to exercise and may be used to diagnose coronary artery disease or assess the effectiveness of
treatment in patients with heart failure.
• Option C, X-ray kymography, is not a recognized medical investigation. X-ray imaging is
commonly used in the diagnosis of heart failure, but the specific type of X-ray is not specified in
the question.
• Option D, ECG in the dynamics, may be useful in monitoring changes in the heart's electrical
activity over time, but it is not necessary for the initial diagnosis of heart failure.
• Option E, Coronarography, is not necessary for the diagnosis of heart failure. Coronary
angiography, also known as coronary arteriography or angiogram, is a test that uses dye and X-
cardiology

rays to visualize the coronary arteries and diagnose blockages or narrowing that may cause
chest pain or heart attack.
Making Other Options Right:
• Option B would be correct if the question described symptoms or findings suggestive of
coronary artery disease, such as chest pain or ST segment changes on ECG, and the test was
used to diagnose the condition or assess treatment effectiveness.
• Option C would be correct if the question specified the type of X-ray used, such as chest X-
ray, and the imaging findings were consistent with heart failure.
• Option D would be correct if the question described changes in the patient's symptoms or ECG
findings over time and the test was used to monitor treatment effectiveness.
• Option E would be correct if the question described symptoms or findings suggestive of
coronary artery disease, such as chest pain or ST segment changes on ECG, and the test was
used to diagnose the condition or assess the need for revascularization.
Treatment:
• The treatment of heart failure may include lifestyle modifications, such as reducing salt and
fluid intake, regular exercise, and smoking cessation.
• Medications, such as diuretics, ACE inhibitors, beta-blockers, and aldosterone antagonists,
may also be used to reduce symptoms and improve heart function.
• In severe cases of heart failure, surgical interventions, such as coronary artery bypass
grafting, valve repair or replacement, or implantation of a cardiac resynchronization therapy
device or left ventricular assist device, may be necessary.
Words:
• Echocardiography: A noninvasive imaging test that uses sound waves to create detailed
images of the heart's structure and function.
• Extrasystolia: An abnormal heart rhythm in which there is an extra heartbeat.
• Hepatomegaly: Enlargement of the liver.

Therapy
Findings:
• The patient complains of shortness of breath, swelling on shanks, irregularity in cardiac work,
and pain in the left chest half with irradiation to the left scapula.
• On physical examination, the patient has diminished heart sounds, a soft systolic murmur on
the apex, a +2 cm painful liver, and an enlarged heart shadow with weak pulsation on X-ray.
• The electrocardiogram (ECG) shows left ventricular extrasystolia and decreased voltage.
• The patient's blood pressure is 115/75 mm Hg, and the pulse rate is 100 beats per minute
with arrhythmia.
• Heart failure is a likely diagnosis based on the patient's symptoms and physical examination
findings, and echocardiography is necessary to confirm the diagnosis and assess the severity of
the condition.
cardiology

37 Heart Failure

A 60-year-old patient complains about asphyxia, palpitation, rapid fatiguability. He has 8


year history of essential hypertension. Objectively: the left cardiac border is 2 cm deviated
to the left from the medioclavicular line, heart sounds are rhythmic and weak; there is
diastolic shock above aorta. AP- 170/100 mm Hg. Liver - +2 cm; shin pastosity is present.
ECG shows deviation of cardiac axis to the left, left ventricle hypertrophy. Ejection fraction -
63%. What type of cardiac insufficiency is observed?
A Diastolic
B Systolic
C Combined
D It's a norm
E Unspecified

Correct Answer: A Diastolic


Key Points:
• 60-year-old patient with a history of essential hypertension
• Complaints of asphyxia, palpitation, and rapid fatiguability
• Objective findings: left cardiac border deviated to the left, weak heart sounds, diastolic shock
above aorta, elevated blood pressure, liver enlargement, and peripheral edema
• ECG findings: deviation of cardiac axis to the left, left ventricular hypertrophy, and normal
ejection fraction

Therapy
• Type of cardiac insufficiency
Explanation:
The patient described has signs and symptoms of heart failure, which is a condition where the
heart is unable to pump enough blood to meet the body's needs. Heart failure can be classified
as either systolic or diastolic, depending on the underlying cause.
Systolic heart failure is characterized by a reduced ejection fraction, which is the percentage of
blood that is pumped out of the left ventricle with each heartbeat. Diastolic heart failure, on the
other hand, is characterized by normal or near-normal ejection fraction, but impaired filling of
the left ventricle during the relaxation phase of the cardiac cycle, leading to high filling
pressures.
The patient described has objective findings consistent with diastolic heart failure, including left
ventricular hypertrophy, diastolic shock above the aorta, and peripheral edema. The normal
ejection fraction also suggests diastolic heart failure, as systolic heart failure would typically
have a reduced ejection fraction. The patient's history of essential hypertension is a risk factor
for the development of diastolic heart failure, as chronic hypertension can lead to left ventricular
hypertrophy and impaired diastolic function.
Other Options:
B) Systolic: Systolic heart failure is characterized by a reduced ejection fraction, which is not
consistent with the patient's ejection fraction of 63%.
C) Combined: Combined heart failure is a type of heart failure that involves both systolic and
diastolic dysfunction. While some patients with heart failure may have both systolic and diastolic
dysfunction, the patient described has findings that are more consistent with diastolic heart
failure.
D) It's a norm: The patient has signs and symptoms of heart failure as well as objective findings
consistent with diastolic heart failure, so it is not a normal finding.
E) Unspecified: The patient's signs and symptoms, as well as objective findings, suggest
diastolic heart failure, so it is not an unspecified type of cardiac insufficiency.
Making Other Options Right:
B) Systolic: Systolic heart failure is characterized by a reduced ejection fraction.
C) Combined: Combined heart failure is a type of heart failure that involves both systolic and
diastolic dysfunction.
cardiology

D) It's a norm: A normal finding would not have the signs and symptoms, as well as objective
findings, consistent with heart failure.
E) Unspecified: Unspecified type of cardiac insufficiency refers to cases where the type of heart
failure is not specified or cannot be determined.
Medication:
The treatment of diastolic heart failure typically involves controlling blood pressure, reducing
salt intake, and using diuretics to manage fluid overload. Other medications that may be used
include beta-blockers, calcium channel blockers, and angiotensin-converting enzyme (ACE)
inhibitors. Lifestyle modifications such as regular exercise and weight loss may also be
beneficial.
Words:
• Heart failure: a condition where the heart is unable to pump enough blood to meet the body's
needs
• Systolic heart failure: heart failure characterized by a reduced ejection fraction
• Diastolic heart failure: heart failure characterized by impaired filling of the left ventricle during
the relaxation phase of the cardiac cycle, leading to high filling pressures
• Ejection fraction: the percentage of blood that is pumped out of the left ventricle with each
heartbeat
• Left ventricular hypertrophy: enlargement and thickening of the muscle of the left ventricle of
the heart
• Essential hypertension: high blood pressure with no identifiable cause
• Peripheral edema: swelling of the legs, ankles, or feet due to fluid buildup
• Diuretics: medications that promote diuresis (increased urine output) and are used to treat
fluid overload
Findings:
• Patient: 60-year-old

Therapy
• Symptoms: Asphyxia, palpitation, rapid fatiguability
• Objective findings: Left cardiac border deviated to the left, weak heart sounds, diastolic shock
above aorta, elevated blood pressure, liver enlargement, and peripheral edema
• ECG findings: Deviation of cardiac axis to the left, left ventricular hypertrophy, and normal
ejection fraction
• Diagnosis: Diastolic heart failure
cardiology

38 Heart Murmurs

During the preventive examination a 17-year-old young man reports no health problems.
Objectively: the patient is undernourished, asthenic; blood pressure is 110/70 mm Hg, Ps -
80/min. Heart borders are within normal range. Auscultation reveals three apical heart sounds,
murmurs are absent. ECG shows no pathological changes, PCG registers the S3 occurring
0,15 seconds after the S2. How can you interpret these changes?
A Physiologic S3
B Fout-ta-ta-rou (three-component rhythm)
C Protodiastolic gallop rhythm
D Presystolic gallop rhythm
E Physiologic S4

Correct Answer: A Physiologic S3 - correct.


Key Points:
• 17-year-old young man with no reported health problems.
• Undernourished, asthenic.
• Blood pressure is 110/70 mm Hg, Ps - 80/min.
• Heart borders are within normal range.
• Auscultation reveals three apical heart sounds, murmurs are absent.
• ECG shows no pathological changes.
• PCG registers the S3 occurring 0,15 seconds after the S2.

Therapy
• Interpretation is being asked for.
Explanation:
The patient's clinical findings are most consistent with a physiologic S3, which is a normal, low-
pitched sound that occurs after the S2 heart sound during early diastole. A physiologic S3 is
typically heard in young individuals with a healthy heart, and is caused by the rapid filling of the
ventricles during early diastole. The S3 is often referred to as a "ventricular gallop" because it
sounds like the galloping of a horse. In this case, the patient is a 17-year-old young man with
no reported health problems, and his ECG shows no pathological changes. The presence of a
physiologic S3 is not associated with any adverse clinical outcomes, and does not require any
specific treatment.
Other Options:
B. Fout-ta-ta-rou (three-component rhythm) is not a recognized heart sound or rhythm.
C. Protodiastolic gallop rhythm refers to a pathologic heart sound that occurs before S1 and is
associated with heart failure or other cardiac abnormalities.
D. Presystolic gallop rhythm refers to a pathologic heart sound that occurs just before S1 and is
also associated with heart failure or other cardiac abnormalities.
E. Physiologic S4 is a normal, low-pitched heart sound that occurs just before S1 and is
associated with the late diastolic filling of the ventricles. However, the patient in this question
has an S3, not an S4.
Making Other Options Right:
B. The option is not a recognized heart sound or rhythm.
C. To make protodiastolic gallop rhythm the correct answer, the question would need to specify
additional clinical findings that are consistent with this diagnosis, such as symptoms of heart
failure or evidence of structural heart disease on imaging.
D. To make presystolic gallop rhythm the correct answer, the question would need to specify
additional clinical findings that are consistent with this diagnosis, such as symptoms of heart
failure or evidence of structural heart disease on imaging.
E. To make physiologic S4 the correct answer, the question would need to describe a heart
sound occurring just before S1, rather than after S2.
Medication:
• No medication is mentioned in the question.
cardiology

Words:
• Auscultation: The act of listening to sounds within the body, typically using a stethoscope.
• S3: A low-pitched heart sound that occurs after S2 during early diastole. Also known as a
"ventricular gallop."
• Murmurs: Abnormal heart sounds that are caused by turbulent blood flow through the heart
valves or chambers.
• ECG: Electrocardiogram, a test that measures the electrical activity of the heart.
• PCG: Phonocardiogram, a recording of the sounds produced by the heart.

Therapy
cardiology

39 Heart Murmurs

Routine examination of a 16-year-old boy revealed the presence of three heart sounds on
auscultation. The third sound is low and occurs in early diastole, there is no additional murmur.
In history: pneumonia six months ago. The patient presents no problems. Examination revealed
hyposthenia, underdevelopment of muscles. Laboratory and instrumental studies reveald no
peculiarities. What is the origin of the additional heart sound?
A Physiological III sound
B The sound of the mitral valve opening
C Protodiastolic gallop rhythm
D Pericardial diastolic sound
E The sound of the tricuspid valve opening

Correct Answer: A. Physiological III sound - the patient's history of pneumonia and examination
findings of hyposthenia and underdevelopment of muscles suggest a diagnosis of physiological
third heart sound.
Key Points:
• 16-year-old boy with three heart sounds on auscultation
• Third sound is low and occurs in early diastole, no additional murmur
• History of pneumonia six months ago
• Examination reveals hyposthenia, underdevelopment of muscles
• No peculiarities on laboratory and instrumental studies

Therapy
Explanation:
The third heart sound (S3) is a low-frequency sound that occurs in early diastole and is caused
by the rapid filling of the ventricles with blood. In young individuals, a physiological third heart
sound may be heard, which is also known as the "ventricular gallop." The presence of a
physiological third heart sound is more likely in individuals who are thin or underdeveloped,
such as in this patient with examination findings of hyposthenia and underdevelopment of
muscles. The patient's history of pneumonia is not directly related to the third heart sound, but
may be an indication of a previous respiratory infection that could contribute to the presence of
a physiological third heart sound.
Other Options:
B. The sound of the mitral valve opening - this is not a likely cause of the third heart sound, as
the mitral valve opening sound occurs in early systole.
C. Protodiastolic gallop rhythm - this would be characterized by the presence of an S3 and an
S4, and is not consistent with the patient's auscultation findings.
D. Pericardial diastolic sound - this is not a likely cause of the third heart sound, as pericardial
sounds are often associated with inflammation or other pathology of the pericardium.
E. The sound of the tricuspid valve opening - this is not a likely cause of the third heart sound,
as the tricuspid valve opening sound occurs in early systole.
Medication:
• No specific medication mentioned in the question
Words:
• Physiological third heart sound: a low-frequency sound that occurs in early diastole, which can
be a normal finding in young individuals
• Ventricular gallop: another term for the physiological third heart sound
Findings:
• The presence of a physiological third heart sound is not necessarily abnormal, but can be a
normal finding in young individuals, particularly those who are thin or underdeveloped.
• No peculiarities on laboratory and instrumental studies suggests that there are no significant
abnormalities in the patient's cardiac function.
cardiology

40 Heart Murmurs

Heart auscultation of a 16 y.o. boy without clinical symptoms revealed accent of the S II and
systolic murmur above the pulmonary artery. Heart sounds are resonant, rhythmic. What is
the most probable diagnosis?
A Functional murmur
B Stenosis of pulmonary artery valve
C Insufficiency of pulmonary artery valve
D Nonclosure of Botallo's duct
E Defection of interatrial septum

Correct Answer: option A (Functional murmur) is the most probable diagnosis based on the
given clinical findings.
Explanation:
• A functional murmur is a benign heart murmur that is not caused by structural abnormalities
of the heart, but rather by changes in blood flow due to factors such as fever, anemia, or
exercise.
• It is a common finding in young individuals and usually does not require treatment.
• The accentuated S2 and systolic murmur above the pulmonary artery heard in this patient can
be caused by increased blood flow through the pulmonary valve, which is a common feature of a
functional murmur.
• Other causes of functional murmurs include increased cardiac output due to exercise or fever,

Therapy
anemia, or anxiety.
Other Options:
• Option B (Stenosis of pulmonary artery valve) is not the most probable diagnosis in this case
as the patient has no symptoms or risk factors for pulmonary valve stenosis, and the
accentuated S2 and systolic murmur are not typical findings in this condition.
• Option C (Insufficiency of pulmonary artery valve) and option D (Nonclosure of Botallo's duct)
are unlikely as these conditions would produce different types of murmurs.
• Option E (Defection of interatrial septum) can cause a heart murmur, but the accentuated S2
and systolic murmur above the pulmonary artery are not typical findings in this condition.
Making Other Options Wrong:
• Options B, C, D, and E do not match the clinical findings of a functional murmur with an
accentuated S2 and systolic murmur above the pulmonary artery.
Medication:
• As a functional murmur is a benign condition, no specific medication or treatment is required.
• However, if the murmur is causing anxiety or concern, the patient may benefit from
reassurance and education about the condition.
Words:
• Functional murmur: A benign heart murmur that is not caused by structural abnormalities of
the heart, but rather by changes in blood flow due to factors such as fever, anemia, or exercise.
• Accentuated S2: An abnormally loud sound heard during auscultation of the heart, typically
caused by increased pressure in the pulmonary artery.
• Systolic murmur: A heart murmur heard during systole (when the heart is contracting), often
caused by turbulence of blood flow through narrowed or damaged heart valves.
• Pulmonary valve stenosis: A condition in which the pulmonary valve becomes narrowed,
reducing blood flow from the right ventricle to the lungs.
Findings:
• A functional murmur is a benign heart murmur that is commonly found in young individuals
and is not associated with any structural abnormalities of the heart.
• The accentuated S2 and systolic murmur above the pulmonary artery heard in this patient are
typical findings in a functional murmur, and do not require any specific treatment.
cardiology

41 Heart Murmurs

During preventive examination a 16 year old patient presented no problems. Objectively:


the patient has signs of malnutrition, he is asthenic, AP is 110/70 mm Hg, Ps is 80 bpm,
cardiac border is normal, auscultation above the cardiac apex reveals three sounds,
cardiac murmur is absent. ECG shows no pathological changes, phonocardiogram shows
that the third sound comes 0,15 s after the second one above the apex. How are these
changes called?
A III physiological sound
B Fout-ta-ta-rou (reduplication of the 2nd sound)
C Protodiastolic gallop rhythm
D Atrial gallop rhythm
E IV physiological sound

Correct Answer: A III physiological sound.


Key Points:
• 16-year-old patient with signs of malnutrition and asthenia.
• Normal blood pressure (110/70 mm Hg) and pulse rate (80 bpm).
• Auscultation above the cardiac apex reveals three sounds.
• ECG shows no pathological changes.
• Phonocardiogram shows that the third sound comes 0,15 s after the second one above the
apex.

Therapy
Explanation:
• The third heart sound, also called S3, is a low-frequency sound that occurs in early diastole
and is associated with rapid filling of the ventricles.
• S3 is a normal physiological sound in children and young adults, but it can also be found in
conditions that increase ventricular filling, such as heart failure, mitral regurgitation, and
systemic hypertension.
• In this case, the S3 is most likely a physiological sound due to the patient's young age and
absence of pathological changes in the ECG and auscultation.
Other Options:
• B Fout-ta-ta-rou (reduplication of the 2nd sound) is a type of splitting of the second heart
sound that occurs in pulmonary hypertension and right bundle branch block.
• C Protodiastolic gallop rhythm, also called S4, is a low-frequency sound that occurs in late
diastole and is associated with atrial contraction and increased resistance to ventricular filling. It
is a sign of left ventricular hypertrophy and is commonly found in patients with hypertension
and aortic stenosis.
• D Atrial gallop rhythm is a type of gallop rhythm that occurs in atrial fibrillation, atrial flutter,
and other supraventricular tachycardias.
• E IV physiological sound is a rare and benign sound that occurs in late diastole and is
associated with atrial contraction.
Making Other Options Right:
• B would be correct if the question described a patient with pulmonary hypertension or right
bundle branch block.
• C would be correct if the question described a patient with hypertension or aortic stenosis.
• D would be correct if the question described a patient with atrial fibrillation or other
supraventricular tachycardias.
• E would be correct if the question described a patient with atrial contraction.
Medication:
• No medication is mentioned in the question.
• Treatment for malnutrition and asthenia may include nutritional support, dietary counseling,
and physical therapy.
Words:
cardiology

• Asthenia: from Greek "asthenēs", meaning weak or feeble.


• Auscultation: from Latin "auscultare", meaning to listen.
• Phonocardiogram: from Greek "phōnē", meaning sound, and "kardiogramma", meaning
recording of the heart.
Findings:
• Normal blood pressure: 110/70 mm Hg.
• Normal pulse rate: 80 bpm.
• The third sound comes 0.15 s after the second one above the apex, which is a normal finding
for S3.

Therapy
cardiology

42 Hemodynamics

During dynamic investigation of a patient the increase of central venous pressure is


combined with the decrease of arterial pressure. What process is proved by such
combination?
A Increase of bleeding speed
B Developing of cardiac insufficiency
C Shunting
D Depositing of blood in venous channel
E Presence of hypervolemia

Correct Answer: A increase in bleeding speed is the process that is proved by the combination of
an increase in central venous pressure (CVP) and a decrease in arterial pressure (AP) during
dynamic investigation of a patient.
Key Points:
• Central venous pressure (CVP) is the pressure in the superior vena cava, which reflects the
pressure in the right atrium.
• Arterial pressure (AP) is the pressure in the arteries, which reflects the pressure in the left
ventricle.
Explanation:
• The combination of an increase in CVP and a decrease in AP suggests a decrease in cardiac
output, which can be caused by a decrease in blood volume or an increase in bleeding speed.

Therapy
• An increase in bleeding speed is the process that is proved by the combination of an increase
in CVP and a decrease in AP during dynamic investigation of a patient.
• In this situation, the increase in CVP is due to an increase in venous return, while the decrease
in AP is due to a decrease in stroke volume and cardiac output.
• The decrease in stroke volume and cardiac output can be caused by a decrease in blood
volume or an increase in bleeding speed, which reduces the amount of blood that is available to
the heart to pump out into the arterial circulation.
• Therefore, an increase in bleeding speed is the most likely process that is proved by the
combination of an increase in CVP and a decrease in AP during dynamic investigation of a
patient.
Other Options:
A) Increase of bleeding speed: This option is correct, as an increase in bleeding speed is the
most likely process that is proved by the combination of an increase in CVP and a decrease in AP
during dynamic investigation of a patient.
B) Developing of cardiac insufficiency: This option is less likely to be the cause of the
combination of an increase in CVP and a decrease in AP, as the underlying mechanism is
different, and there may be other signs and symptoms of heart failure.
C) Shunting: This option is not related to the combination of an increase in CVP and a decrease
in AP.
D) Depositing of blood in venous channel: This option is not related to the combination of an
increase in CVP and a decrease in AP.
E) Presence of hypervolemia: This option may result in an increase in CVP, but it is less likely to
cause a decrease in AP.
Words:
• Central venous pressure (CVP): the pressure in the superior vena cava, which reflects the
pressure in the right atrium.
• Arterial pressure (AP): the pressure in the arteries, which reflects the pressure in the left
ventricle.
• Bleeding speed: the rate at which blood flows out of the arterial circulation due to bleeding or
other causes.
• Stroke volume: the amount of blood that is ejected by the heart with each beat.
cardiology

• Cardiac output: the amount of blood that is pumped by the heart per minute.

Therapy
cardiology

43 Hypertension

A 42-year-old male patient wth essential hypertension presents with headache, palpitations,
unexplained fear. Objectively: Ps- 100/min, AP- 200/100 mm Hg, the left border of cardiac
dullness is displaced by 1,5 cm to the left, vesicular breathing is present. ECG shows sinus
tachycardia, signs of left ventricular hypertrophy. What drug should be administered as an
emergency?
A Obzidan
B Dibazol
C Reserpine
D Magnesium sulfate
E Furosemide

Correct Answer: Option A, Obzidan (metoprolol), is the correct drug to administer in this
hypertensive emergency.
Key Points:
• A 42-year-old male patient with essential hypertension presents with headache, palpitations,
and unexplained fear.
• Objective findings include PS - 100/min, AP - 200/100 mm Hg, displacement of the left border
of cardiac dullness to the left by 1.5 cm, and vesicular breathing.
• ECG shows sinus tachycardia and signs of left ventricular hypertrophy.
• The question is asking for the drug that should be administered as an emergency.

Therapy
Explanation:
• Hypertensive emergency is a severe elevation in blood pressure that is associated with acute
end-organ damage.
• The patient's presentation with headache, palpitations, and unexplained fear along with
objective findings of elevated blood pressure, displacement of the left border of cardiac dullness,
and left ventricular hypertrophy suggest hypertensive emergency.
• The goal of treatment in hypertensive emergency is to reduce blood pressure quickly to
prevent further end-organ damage.
• Obzidan (metoprolol) is a beta-blocker that reduces blood pressure by decreasing heart rate
and cardiac output. It is a first-line drug in hypertensive emergencies, especially in patients with
signs of left ventricular hypertrophy.
• Other options such as Dibazol, Reserpine, Magnesium sulfate, and Furosemide are not
appropriate for this hypertensive emergency.
Other Options:
• Option B, Dibazol, is a vasodilator that is used for the treatment of hypertension. However, it
is not appropriate for the management of hypertensive emergency since it has a slow onset of
action.
• Option C, Reserpine, is an antihypertensive drug that works by depleting norepinephrine and
dopamine from presynaptic nerve terminals. It has a slow onset of action and is not appropriate
for hypertensive emergency.
• Option D, Magnesium sulfate, is a drug that is used for the treatment of preeclampsia and
eclampsia. It is not appropriate for the management of hypertensive emergency.
• Option E, Furosemide, is a loop diuretic that is used for the treatment of fluid overload and
hypertension. It is not appropriate for the management of hypertensive emergency.
Medication:
• Obzidan (metoprolol) is a beta-blocker that reduces blood pressure by decreasing heart rate
and cardiac output. The recommended dose for hypertensive emergency is 2-5 mg IV every 5
minutes, up to a maximum dose of 15 mg.
• Dibazol (B) is a vasodilator that is used for the treatment of hypertension. The recommended
dose is 20-40 mg orally three times daily.
cardiology

• Reserpine (C) is an antihypertensive drug that works by depleting norepinephrine and


dopamine from presynaptic nerve terminals. The recommended dose is 0.1-0.25 mg orally once
daily.
• Magnesium sulfate (D) is a drug that is used for the treatment of preeclampsia and eclampsia.
The recommended dose is 4-5 g IV over 20-30 minutes.
• Furosemide (E) is a loop diuretic that is used for the treatment of fluid overload and
hypertension. The recommended dose is 20-80 mg orally once daily.
Words:
• Hypertensive emergency: A severe elevation in blood pressure that is associated with acute
end-organ damage.
• Left ventricular hypertrophy: A condition in which the muscle mass of the left ventricle of the
heart increases.
• Vesicular breathing: A normal breath sound heard on auscultation of the lungs.
• ECG: Electrocardiogram, a test that measures the electrical activity of the heart.
Findings:
• Normal values for blood pressure are less than 120/80 mm Hg.
• Normal heart rate is 60-100 beats per minute.
• Signs of left ventricular hypertrophy on ECG include increased amplitude of the QRS
complexes and ST-segment changes.
• Normal values for the displacement of the left border of cardiac dullness are less than 1 cm.
• Normal breath sounds on auscultation of the lungs are vesicular.

Therapy
cardiology

44 Hypertension

A 67-year-old female patient with hypertensive crisis has asthma, cough with expectoration of
frothy pink sputum, moist rales in the lungs. The patient stays in sitting position, respiratory
rate
is 40/min, AP- 214/136 mm Hg, heart rate - 102/min. What is the most rational tactics of this
patient management?
A Intravenous administration of furosemide
B Urgent pneumography
C Bed rest, lying position
D Intravenous administration of a β-blocker
E Tactics can be determined after ECG and chest radiography

Correct Answer: Intravenous administration of furosemide - The patient's symptoms and signs
are consistent with pulmonary edema, likely due to hypertensive crisis. The most appropriate
initial management is intravenous administration of a diuretic such as furosemide.
Key Points:
• Patient with hypertensive crisis and symptoms of pulmonary edema, including asthma, cough
with frothy pink sputum, and moist rales in the lungs.
• Sitting position, respiratory rate of 40/min, elevated blood pressure and heart rate.
Explanation:
Hypertensive crisis is a severe elevation in blood pressure that can cause end-organ damage,

Therapy
including pulmonary edema. The patient's symptoms and signs, including asthma, cough with
frothy pink sputum, and moist rales in the lungs, are consistent with pulmonary edema. The
patient's high respiratory rate, elevated blood pressure, and heart rate indicate that urgent
intervention is needed.
The initial management of hypertensive crisis with pulmonary edema includes reducing blood
pressure and relieving fluid overload. Intravenous administration of a loop diuretic such as
furosemide is the most appropriate initial treatment. Furosemide inhibits sodium and chloride
reabsorption in the loop of Henle, leading to increased urine output and reduction of fluid
overload. However, care must be taken to monitor the patient's electrolyte levels and kidney
function.
Other Options:
B) Urgent pneumography: Pneumography is not a standard diagnostic or treatment option for
hypertensive crisis with pulmonary edema. The patient's symptoms and signs strongly suggest
pulmonary edema, and urgent intervention is needed to prevent further end-organ damage.
C) Bed rest, lying position: The patient's symptoms and signs indicate a need for urgent
intervention to reduce fluid overload and blood pressure. Bed rest and lying position are not
appropriate initial interventions for hypertensive crisis with pulmonary edema.
D) Intravenous administration of a β-blocker: β-blockers are contraindicated in the initial
management of hypertensive crisis with pulmonary edema, as they can worsen fluid overload
and exacerbate symptoms of heart failure.
E) Tactics can be determined after ECG and chest radiography: ECG and chest radiography may
be useful in evaluating the patient's condition, but urgent intervention is needed to reduce fluid
overload and blood pressure and prevent further end-organ damage.
Medication:
• Furosemide is a loop diuretic that inhibits sodium and chloride reabsorption in the loop of
Henle, leading to increased urine output and reduction of fluid overload.
Words:
• Hypertensive crisis: A severe elevation in blood pressure that can cause end-organ damage.
• Pulmonary edema: Accumulation of fluid in the lungs, leading to symptoms such as cough,
frothy pink sputum, and moist rales.
cardiology

• Loop diuretic: A type of diuretic that inhibits sodium and chloride reabsorption in the loop of
Henle, leading to increased urine output and reduction of fluid overload.
Findings:
• Normal values for blood pressure, heart rate, and respiratory rate are not mentioned in the
question.
• The patient's symptoms and signs, including asthma, cough with frothy pink sputum, and
moist rales in the lungs, are abnormal findings indicative of pulmonary edema

Therapy
cardiology

45 Hypertension

A 58-year-old patient complains of a headache in the occipital region, nausea, choking,


opplotentes. The presentations appeared after a physical exertion. Objectively: the patient is
excited. Face is hyperemic. Skin is pale. Heart sounds are regular, the 2nd aortic sound is
accentuated. AP- 240/120 mm Hg, HR- 92/min. Auscultation reveals some fine moisr rales in
the lower parts of the lungs. Liver is not enlarged. ECG shows signs of hypertrophy and left
ventricular overload. What is the most likely diagnosis?
A Complicated hypertensic crisis
B Acute myocardial infarction, pulmonary edema
C Bronchial asthma exacerbation
D Uncomplicated hypertensic crisis
E Community-acquired pneumonia

Correct Answer: A - Complicated hypertensive crisis is the most likely diagnosis because the
patient is presenting with a headache in the occipital region, nausea, and choking, which are
common symptoms of severe hypertension. The patient's physical exertion may have triggered
the hypertensive crisis. The patient's high blood pressure, tachycardia, and accentuated 2nd
aortic sound suggest severe hypertension, and the presence of fine moist rales in the lower
parts of the lungs suggests pulmonary edema, which can occur in severe hypertensive crises.
Key Points:
• 58-year-old patient with a headache in the occipital region, nausea, and choking

Therapy
• Symptoms appeared after physical exertion
• Excited, hyperemic face, pale skin
• Regular heart sounds, accentuated 2nd aortic sound
• AP- 240/120 mm Hg, HR- 92/min
• Fine moist rales in lower parts of lungs
• No hepatomegaly
• ECG shows signs of hypertrophy and left ventricular overload
Explanation:
A hypertensive crisis is a medical emergency characterized by severely elevated blood pressure,
which can cause organ damage and life-threatening complications. The patient's symptoms and
clinical findings are consistent with a complicated hypertensive crisis, including headache,
nausea, and choking. The patient's physical exertion may have triggered the hypertensive crisis.
The patient's high blood pressure, tachycardia, and accentuated 2nd aortic sound suggest
severe hypertension. The presence of fine moist rales in the lower parts of the lungs suggests
pulmonary edema, which can occur in severe hypertensive crises. The ECG findings of
hypertrophy and left ventricular overload suggest long-standing hypertension.
Other Options:
• B Acute myocardial infarction, pulmonary edema: Acute myocardial infarction can cause chest
pain and shortness of breath, but the patient's symptoms and clinical findings are more
consistent with a hypertensive crisis than acute myocardial infarction. The ECG findings are also
more consistent with long-standing hypertension than acute myocardial infarction.
• C Bronchial asthma exacerbation: Bronchial asthma exacerbation can cause shortness of
breath and wheezing, but the patient's symptoms and clinical findings are more consistent with
a hypertensive crisis than bronchial asthma exacerbation.
• D Uncomplicated hypertensive crisis: The presence of pulmonary edema and left ventricular
overload on ECG suggests that this is a complicated hypertensive crisis rather than an
uncomplicated hypertensive crisis.
• E Community-acquired pneumonia: Community-acquired pneumonia can cause fever, cough,
and shortness of breath, but the patient's symptoms and clinical findings are more consistent
with a hypertensive crisis than community-acquired pneumonia.
Making Other Options Right:
cardiology

• Option B Acute myocardial infarction, pulmonary edema: If the question mentioned that the
patient had chest pain and ST-segment elevation on the ECG, acute myocardial infarction with
pulmonary edema would be the correct answer.
• Option C Bronchial asthma exacerbation: If the question mentioned that the patient had a
history of wheezing and shortness of breath with a reversible airway obstruction, bronchial
asthma exacerbation would be the correct answer.
• Option D Uncomplicated hypertensive crisis: If the question mentioned that the patient had
severely elevated blood pressure without any evidence of end-organ damage, uncomplicated
hypertensive crisis would be the correct answer.
Medication:
• Antihypertensive medications: Medications such as nitroprusside, labetalol, or nicardipine may
be used to rapidly lower blood pressure in a hypertensive crisis.
• Diuretics: Diuretics such as furosemide may be used to reduce pulmonary edema and fluid
overload.
Words:
• Hypertensive crisis: A medical emergency characterized by severely elevated blood pressure
that can cause organ damage and life-threatening complications.
• Pulmonary edema: The accumulation of fluid in the lungs, which can cause shortness of
breath, cough, and rales on auscultation.
• Fine moist rales: Crackling sounds heard on auscultation of the lungs due to fluid accumulation
in the air spaces.
Findings:
• AP- 240/120 mm Hg, HR- 92/min: Severely elevated blood pressure with tachycardia.
• Accentuated 2nd aortic sound: Suggests aortic valve stenosis or hypertension.
• No hepatomegaly: Absence of hepatomegaly rules out liver congestion due to heart failure.
• ECG shows signs of hypertrophy and left ventricular overload: Suggests long-standing

Therapy
hypertension with left ventricular hypertrophy.
cardiology

46 Hypertension

A 52 year old patient has hypervolaemic type of essential hypertension. Which of the
following medications is to be prescribed either as monotherapy or in complex with other
antihypertensive drugs?
A Hypothiazid
B Dibazol
C Clonidine
D Kapoten
E Nifedipin

Correct Answer: A Hypothiazid is the correct medication to be prescribed either as monotherapy


or in complex with other antihypertensive drugs for hypervolaemic type of essential
hypertension.
Key Points:
• 52-year-old patient
• Hypervolaemic type of essential hypertension
• Medication to be prescribed either as monotherapy or in complex with other antihypertensive
drugs
Explanation: Hypertension is classified based on the underlying cause, and treatment options
vary accordingly. In hypervolaemic type of essential hypertension, the volume of blood in the
body is high, leading to increased blood pressure. Diuretics such as thiazides are effective in

Therapy
treating hypervolaemic type of essential hypertension as they increase the excretion of sodium
and water, resulting in decreased blood volume and blood pressure. Hypothiazid is a thiazide
diuretic that can be prescribed either alone or in combination with other antihypertensive drugs
for the treatment of hypertension. It reduces blood pressure by decreasing the amount of fluid
in the body and widening the blood vessels.
Other Options:
B. Dibazol is a smooth muscle relaxant that is not commonly used in the treatment of
hypertension.
C. Clonidine is a centrally acting antihypertensive drug that is not commonly used for
hypervolaemic type of essential hypertension.
D. Kapoten (captopril) is an ACE inhibitor that is not commonly used as a first-line treatment for
hypervolaemic type of essential hypertension.
E. Nifedipine is a calcium channel blocker that is not commonly used as a first-line treatment for
hypervolaemic type of essential hypertension.
Making Other Options Right: Dibazol, Clonidine, Kapoten, and Nifedipine are not typically used
as first-line treatments for hypervolaemic type of essential hypertension, but they may be used
in combination with other medications if necessary.
cardiology

47 Hypertension

A 60 y.o. woman has had increased BP up to 210/110 mm Hg for the last 7 years. On
examination: heart apex is displaced to the left. There are signs of left ventricular
hypertrophy on ECG. What is the most probable diagnosis?
A Essential hypertension, 2nd stage
B Essential hypertension, 1st stage
C Symptomatic hypertension
D Cardiomyopathy
E Ischemic heart disease

Correct Answer: Option A


The patient's clinical presentation and examination findings are consistent with Essential
Hypertension, specifically 2nd stage hypertension, which is characterized by persistently
elevated blood pressure levels and signs of target organ damage. The most probable diagnosis
is option A, Essential hypertension, 2nd stage.
Key Points:
• 60-year-old woman with increased blood pressure up to 210/110 mm Hg for the last 7 years
• Heart apex displaced to the left, signs of left ventricular hypertrophy on ECG
• Most probable diagnosis
Explanation:The patient's clinical presentation and examination findings are consistent with
Essential Hypertension, which is a chronic medical condition characterized by persistently

Therapy
elevated blood pressure levels. The patient's persistent elevation of blood pressure to 210/110
mm Hg for the last 7 years is indicative of poorly controlled hypertension. The presence of left
ventricular hypertrophy on ECG and displacement of the heart apex to the left is a sign of target
organ damage in the form of cardiac involvement. The patient's clinical presentation and
examination findings are consistent with 2nd stage hypertension, which is characterized by
persistently elevated blood pressure levels and signs of target organ damage.
Among the given options, the most probable diagnosis is option A, Essential hypertension, 2nd
stage, given the patient's clinical presentation and examination findings. Essential hypertension,
option B, is also a possibility, but 2nd stage hypertension is a more specific diagnosis given the
severity and duration of the patient's hypertension and the presence of target organ damage.
Symptomatic hypertension, option C, is a type of secondary hypertension that is caused by an
underlying medical condition and is characterized by symptoms, such as headache and
palpitations, which are not present in this patient. Cardiomyopathy, option D, is a possibility
given the presence of left ventricular hypertrophy, but it is not the most probable diagnosis
given the patient's history of hypertension. Ischemic heart disease, option E, is also a possibility
given the patient's cardiac involvement, but it is less likely given the absence of symptoms or
history of ischemic events.
Other Options:
• Option A, Essential hypertension, 2nd stage, is the most probable diagnosis.
• Option B, Essential hypertension, 1st stage, is also a possibility, but 2nd stage hypertension is
a more specific diagnosis given the severity and duration of the patient's hypertension and the
presence of target organ damage.
• Option C, Symptomatic hypertension, is a type of secondary hypertension that is caused by an
underlying medical condition and is characterized by symptoms, such as headache and
palpitations, which are not present in this patient.
• Option D, Cardiomyopathy, is a possibility given the presence of left ventricular hypertrophy,
but it is not the most probable diagnosis given the patient's history of hypertension.
• Option E, Ischemic heart disease, is also a possibility given the patient's cardiac involvement,
but it is less likely given the absence of symptoms or history of ischemic events.
Medication:
cardiology

• The treatment for hypertension typically involves lifestyle modifications, such as weight loss,
exercise, and dietary changes, as well as pharmacotherapy with antihypertensive medications,
such as diuretics, ACE inhibitors, angiotensin receptor blockers, beta blockers, and calcium
channel blockers.
Words:
• Essential Hypertension: A chronic medical condition characterized by persistently elevated
blood pressure levels.
• 2nd stage hypertension: A classification of hypertension based on the severity and duration of
elevated blood pressure levels and the presence of signs of target organ damage.
• Target organ damage: Structural or functional damage to organs, such as the heart, brain,
kidneys, or eyes, that can result from chronic hypertension.
• Left ventricular hypertrophy: An enlargement of the left ventricle of the heart that can occur
as a result of chronic hypertension.
• ECG: Electrocardiogram, a noninvasive test that records the electrical activity of the heart.
• Symptomatic hypertension: A type of secondary hypertension that is caused by an underlying
medical condition and is characterized by symptoms, such as headache and palpitations.
• Cardiomyopathy: A group of diseases that affect the heart muscle and can lead to heart failure
or arrhythmias.
• Ischemic heart disease: A condition in which there is reduced blood flow to the heart muscle,
typically due to coronary artery disease.
Findings:
• The patient's clinical findings include increased blood pressure up to 210/110 mm Hg for the
last 7 years, heart apex displaced to the left, and signs of left ventricular hypertrophy on ECG.

Therapy
cardiology

48 Hypertension

A 74 y.o. patient has been suffering from hypertension for 20 years. He complains of
frequent headache, dizziness, he takes enalapril. Objectively: accent of the SII above aorta,
Ps- 84 bpm, rhythmic, AP- 180/120 mm Hg. What group of hypotensive medications could
be additionally prescribed under consideration of the patient's age?
A Thiazide diuretics
B Loop diuretics
C beta-adrenoceptor blockers
D alpha-adrenoceptor blockers
E Central sympatholytics

Correct Answer: A. Thiazide diuretics.


Key Points:
• The patient is a 74-year-old with a long history of hypertension who is currently taking
enalapril.
• He is experiencing frequent headaches, dizziness, and has an accentuated S2 above the aorta.
• His blood pressure is very high (180/120 mm Hg) and he needs additional antihypertensive
medication.
• Thiazide diuretics are generally considered first-line therapy for hypertension in older adults.
Explanation:
• Hypertension is a common condition in older adults and is associated with an increased risk of

Therapy
cardiovascular disease and stroke.
• Treatment typically involves lifestyle modifications and medications to lower blood pressure.
• The patient is currently taking enalapril, an ACE inhibitor, but his blood pressure is still very
high and he is experiencing symptoms of hypertension.
• Thiazide diuretics are a class of medications that are commonly used as first-line therapy for
hypertension in older adults.
• They work by increasing the excretion of salt and water from the body, which reduces blood
volume and lowers blood pressure.
• Other classes of antihypertensive medications, such as beta-blockers and alpha-blockers, may
be less effective or have more side effects in older adults.
Other Options:
• Option B (Loop diuretics) are generally not used as first-line therapy for hypertension unless
there is significant fluid overload or renal dysfunction.
• Option C (Beta-adrenoceptor blockers) may be effective in treating hypertension in some older
adults, but are generally considered less effective than thiazide diuretics and may have more
side effects.
• Option D (Alpha-adrenoceptor blockers) are generally not recommended as first-line therapy
for hypertension, and may have more side effects in older adults than other classes of
medications.
• Option E (Central sympatholytics) are not commonly used as first-line therapy for
hypertension due to significant side effects such as sedation and depression.
Making Other Options Wrong:
• Options B, C, D, and E are generally less effective or have more side effects than thiazide
diuretics as first-line therapy for hypertension in older adults.
Medication:
• Thiazide diuretics are generally well-tolerated and effective in treating hypertension in older
adults.
• Commonly used thiazide diuretics include hydrochlorothiazide and chlorthalidone.
• The dosage and frequency of thiazide diuretics may need to be adjusted in older adults based
on renal function and other comorbidities.
Words:
cardiology

• Hypertension: A condition in which blood pressure is consistently elevated above normal


levels.
• Enalapril: A medication belonging to the class of ACE inhibitors that is commonly used to treat
hypertension.
• Thiazide diuretics: A class of medications that increase the excretion of salt and water from
the body, reducing blood volume and lowering blood pressure.
• Loop diuretics: A class of medications that work by blocking the reabsorption of salt and water
in the kidneys, leading to increased urine output and reduced blood volume.
• Beta-adrenoceptor blockers: A class of medications that block the effects of the hormone
adrenaline on the heart and blood vessels, reducing heart rate and blood pressure.
• Alpha-adrenoceptor blockers: A class of medications that block the effects of the hormone
norepinephrine on blood vessels, reducing blood pressure.
• Central sympatholytics: A class of medications that act on the central nervous system to
reduce sympathetic nervous system activity, leading to decreased heart rate and blood
pressure.
Findings:
• Thiazide diuretics are generally considered first-line therapy for hypertension in older adults.
• Loop diuretics, beta-blockers, alpha-blockers, and central sympatholytics may be less effective
or have more side effects in older adults than thiazide diuretics.
• The dosage and frequency of thiazide diuretics may need to be adjusted in older adults based
on renal function and other comorbidities.

Therapy
cardiology

49 Hypertension

A 58-year-old female patient complains about periodical headache, dizziness and ear
noise. She has been suffering from diabetes mellitus for 15 years. Objectively: heart
sounds are rhythmic, heart rate is 76/min, there is diastolic shock above aorta, AP is
180/110 mm Hg. In urine: OD- 1,014. Daily loss of protein with urine is 1,5 g. What drug
should be chosen for treatment of arterial hypertension?
A Ihibitor of angiotensin converting enzyme
B β-blocker
C Calcium channel antagonist
D Thiazide diuretic
E α-blocker

Correct Answer: A Inhibitor of angiotensin converting enzyme


Key Points:
• 58-year-old female patient with periodical headache, dizziness, and ear noise
• History of diabetes mellitus for 15 years
• Heart sounds are rhythmic, heart rate is 76/min, diastolic shock above aorta, AP is 180/110
mm Hg
• Urine analysis shows daily loss of protein of 1.5 g
• Drug of choice for treatment of arterial hypertension
Explanation:

Therapy
• The drug of choice for treatment of arterial hypertension in this case is an inhibitor of
angiotensin converting enzyme (ACE inhibitor), option A.
• Hypertension is a common complication of diabetes mellitus, and poorly controlled
hypertension can lead to diabetic nephropathy, which is characterized by proteinuria.
• The patient has symptoms of headache, dizziness, and ear noise, which are suggestive of
hypertensive encephalopathy, a serious complication of uncontrolled hypertension.
• The presence of diastolic shock above the aorta and elevated blood pressure (180/110 mm
Hg) indicate severe hypertension that requires immediate treatment.
• An ACE inhibitor is the drug of choice in patients with diabetes and hypertension with
proteinuria, as it can slow the progression of diabetic nephropathy and reduce proteinuria.
• ACE inhibitors work by inhibiting the conversion of angiotensin I to angiotensin II, a potent
vasoconstrictor, and by increasing the levels of bradykinin, a vasodilator.
Other Options:
• Option B, β-blocker, may be useful in patients with hypertension and a history of myocardial
infarction or heart failure, but it is not the drug of choice in this case. β-blockers can worsen
glycemic control in patients with diabetes and may also exacerbate bronchospasm in patients
with asthma or chronic obstructive pulmonary disease (COPD).
• Option C, calcium channel antagonist, can be used to treat hypertension, but it may not be the
best choice in this case. Calcium channel antagonists are generally less effective in reducing
proteinuria than ACE inhibitors or angiotensin receptor blockers (ARBs). They can also cause
peripheral edema and exacerbate constipation in some patients.
• Option D, thiazide diuretic, can be used to treat hypertension, but it may not be the best
choice in this case. Thiazide diuretics can worsen glycemic control in patients with diabetes and
may also exacerbate electrolyte disturbances, such as hypokalemia and hyponatremia.
• Option E, α-blocker, may be useful in patients with hypertension and benign prostatic
hyperplasia, but it is not the drug of choice in this case. α-blockers can cause orthostatic
hypotension and dizziness in some patients.
Making Other Options Right:
• Option B could be made correct if the question stated that the patient had a history of
myocardial infarction or heart failure.
cardiology

• Option C could be made correct if the question stated that the patient had contraindications or
intolerance to ACE inhibitors or ARBs.
• Option D could be made correct if the question stated that the patient had normal glucose
tolerance and no history of electrolyte disturbances.
• Option E could be made correct if the question stated that the patient had benign prostatic
hyperplasia.
Medication:
• ACE inhibitors are typically started at a low dose and gradually titrated to achieve a target
blood pressure.
• Common side effects of ACE inhibitors include cough, hypotension, hyperkalemia, and acute
kidney injury.
• Patients on ACE inhibitors should have their serum creatinine and electrolyte levels monitored
regularly.
• ACE inhibitors are contraindicated in patients with bilateral renal artery stenosis or a history of
angioedema.
Findings:
• The patient has a history of diabetes mellitus for 15 years, with associated proteinuria.
• The urine analysis shows a daily loss of protein of 1.5 g, which is indicative of significant
proteinuria.
• The blood pressure is severely elevated at 180/110 mm Hg, with diastolic shock above the
aorta, indicating severe hypertension.

Therapy
cardiology

50 Hypertension

A 18 y.o. female student complains of dyspnea during the intensive exertion. The condition
became worse half a year ago. On examination: pulse rate is 88 bpm, accelerated, AP180/20
mm Hg, pale skin, heart borders are dilated to the left and up. There is
systolic-diastolic murmur in the 2hd intercostal space, S2 at pulmonary artery is
accentuated. ECG has revealed both ventricles hypertrophy. Thoracic X-ray has revealed
pulsation and protrusion of the left ventricle, lung trunk. What doctor's tactics should be?
A Cardiosurgeon consultation
B Dispensary observation
C Administration of therapeutic treatment
D Continuation of investigation
E Exemption from physical exercises

Correct Answer: A Cardiosurgeon consultation


Key Points:
• 18-year-old female with dyspnea during intensive exertion
• Symptoms have been present for half a year
• Elevated blood pressure, pale skin, and dilated heart borders on examination
• Systolic-diastolic murmur in the 2nd intercostal space and accentuated S2 at the pulmonary
artery
• ECG shows both ventricles hypertrophy, and thoracic X-ray shows pulsation and protrusion of

Therapy
the left ventricle and lung trunk
Explanation:
The patient in the question is presenting with symptoms of heart failure, such as dyspnea during
exertion, and objective findings of hypertension, left ventricular hypertrophy, and cardiomegaly.
These are consistent with hypertensive heart disease, a condition caused by chronic high blood
pressure that leads to hypertrophy and dilation of the heart chambers.
The presence of a systolic-diastolic murmur in the 2nd intercostal space and an accentuated S2
at the pulmonary artery suggests the presence of pulmonary hypertension, which can occur in
patients with hypertensive heart disease. The pulsation and protrusion of the left ventricle and
lung trunk on thoracic X-ray are also indicative of hypertensive heart disease.
Cardiosurgeon consultation is necessary to determine the appropriate management for the
patient. Depending on the severity of the condition, treatment options may include medications
to lower blood pressure and manage heart failure symptoms, such as diuretics, angiotensin-
converting enzyme inhibitors, and beta-blockers. In some cases, surgery may be necessary to
correct structural abnormalities or improve blood flow to the heart.
Other Options:
B) Dispensary observation may be an appropriate option for patients with mild or stable
hypertension, but the patient in the question is presenting with symptoms of heart failure and
requires more urgent management.
C) Administration of therapeutic treatment is necessary, but the specific treatment should be
determined by a cardiosurgeon after further evaluation.
D) Continuation of investigation may be necessary to rule out other underlying conditions, but
the patient's symptoms and objective findings are highly suggestive of hypertensive heart
disease and require prompt management.
E) Exemption from physical exercises may be necessary in some cases, but it is not sufficient
management for hypertensive heart disease.
Making Other Options Right:
B) The correct management for a patient with stable hypertension may include dispensary
observation and lifestyle modifications, such as dietary changes and exercise.
C) The administration of therapeutic treatment is necessary, but the specific treatment should
be determined by a cardiosurgeon after further evaluation.
cardiology

D) Continuation of investigation may be necessary to rule out other underlying conditions, but
the patient's symptoms and objective findings are highly suggestive of hypertensive heart
disease and require prompt management.
E) Exemption from physical exercises may be necessary in some cases, but it is not sufficient
management for hypertensive heart disease.
Medication:
The medication for hypertensive heart disease includes antihypertensive drugs like diuretics,
beta-blockers, ACE inhibitors, and calcium channel blockers that lower blood pressure to prevent
further damage to the heart.
Words:
Hypertrophy - an increase in the size of an organ or tissue due to an increase in the size of its
cells.
Findings:
• Normal pulse rate: 60-100 bpm
• Normal blood pressure: 120/80 mm Hg
• Pathological findings: elevated blood pressure, dilated heart borders, systolic-diastolic
murmur, left ventricular hypertrophy, and cardiomegaly.

Therapy
cardiology

51 Hypertension

A 67-year-old female patient suffering from the essential hypertension suddenly at night
developed headache, dyspnea that quickly progressed to asphyxia. Objectively: the patient
is pale, with sweaty forehead, AP- 210/140 mm Hg, heart rate - 120/min, auscultation
revealed solitary dry rales and moist rales in the lower parts. The shins are pastose. What
kind of emergency aid would be the most efficient in this case?
A Nitroglycerin and furosemide intravenously
B Enalapril and furosemide intravenously
C Digoxin and nitroglycerin intravenously
D Labetalol and furosemide intravenously
E Nitroglycerin intravenously and capoten internally

Correct Answer: option A, Nitroglycerin and furosemide intravenously, would be the most
efficient emergency aid in this case.
The patient is experiencing a hypertensive emergency, which is a severe elevation in blood
pressure that can lead to organ damage. The symptoms, such as headache, dyspnea, asphyxia,
pale skin, sweating, and pulmonary edema, are all consistent with a hypertensive emergency.
Nitroglycerin is a vasodilator that can help to rapidly lower blood pressure and relieve chest
pain. Furosemide is a diuretic that can help to reduce fluid overload and pulmonary edema.
Together, these medications can help to lower blood pressure and improve the patient's
breathing.

Therapy
Other Options:
B) Enalapril is an angiotensin-converting enzyme (ACE) inhibitor that can help to lower blood
pressure. Furosemide is a diuretic that can help to reduce fluid overload and pulmonary edema.
While these medications can be useful in managing hypertension and heart failure, they may not
be the most efficient emergency aid in this case.
C) Digoxin is a medication that can help to improve heart function and reduce heart rate.
Nitroglycerin is a vasodilator that can help to lower blood pressure and relieve chest pain. While
these medications can be useful in managing heart failure and angina, they may not be the
most efficient emergency aid in this case.
D) Labetalol is a combined alpha and beta blocker that can be used to rapidly lower blood
pressure in a hypertensive emergency. Furosemide is a loop diuretic that can help to reduce
fluid overload and pulmonary edema. While these medications can be useful in managing
hypertension and heart failure, they may not be the most efficient emergency aid in this case.
E) Nitroglycerin is a vasodilator that can help to lower blood pressure and relieve chest pain.
Capoten (captopril) is an ACE inhibitor that can help to lower blood pressure. While these
medications can be useful in managing hypertension and heart failure, they may not be the
most efficient emergency aid in this case.
Making Other Options Wrong:
B) To make enalapril and furosemide intravenously a wrong option, the question would need to
describe symptoms that are more consistent with heart failure, such as decreased exercise
tolerance, fatigue, and orthopnea.
C) To make digoxin and nitroglycerin intravenously a wrong option, the question would need to
describe symptoms that are more consistent with heart failure, such as decreased exercise
tolerance, fatigue, and orthopnea.
D) To make Labetalol and furosemide intravenously a wrong option, the question would need to
describe symptoms that are more consistent with chest pain, such as angina, rather than
pulmonary edema.
E) To make nitroglycerin intravenously and capoten internally a wrong option, the question
would need to describe symptoms that are more consistent with angina or hypertension, such as
chest pain, palpitations, or headache.
Words:
cardiology

• Essential hypertension: high blood pressure without an identifiable cause


• Hypertensive emergency: a severe elevation in blood pressure that can lead to organ damage
• Headache: a pain or discomfort in the head or neck region
• Dyspnea: difficulty breathing or shortness of breath
• Asphyxia: a condition of severely deficient oxygen supply to the body that can lead to
unconsciousness or death
• Pale: having reduced or absent skin color
• Sweaty: producing or exuding sweat
• Blood pressure: the pressure exerted by the blood on the walls of blood vessels
• Tachycardia: a fast heart rate above the normal range
• Pulmonary edema: accumulation of fluid in the lungs
• Rales: abnormal breath sounds heard on auscultation of the lungs
• Edema: swelling caused by excess fluid in the tissues
• Loop diuretic: a type of diuretic that acts on the loop of Henle in the kidney to promote
diuresis
• Vasodilator: a medication that dilates blood vessels, reducing blood pressure
Findings:
• The patient is a 67-year-old female with essential hypertension, which is a risk factor for
hypertensive emergencies.
• The symptoms of headache, dyspnea, and asphyxia are consistent with pulmonary edema,
which can occur as a result of severe hypertension.
• The objective findings of pale skin, sweating, elevated blood pressure, tachycardia, and rales
are all consistent with a hypertensive emergency.
• The presence of edema in the shins suggests that the patient may have underlying heart or
kidney disease.
• Nitroglycerin and furosemide are commonly used medications in the treatment of hypertensive

Therapy
emergencies, as they can help to rapidly lower blood pressure and reduce fluid overload.
cardiology

52 Hypertension

A 46 year old woman who has been suffering from hypertension for 5 years was diagnosed
with hypertensive crisis. She complains about palpitation, sense of head pulsation; heart
rate is 100/min, AP is 190/100 mm Hg (haemodynamics is of hyperkinetic type). What
medication should be the medication of choice?
A β-adrenoceptor blocker
B Adenosine pyrophosphate inhibitor
C Diuretic
D α-adrenoceptor blocker
E Dihydropyridine calcium antagonist

Correct Answer: A β-adrenoceptor blocker


Key Points:
• 46-year-old woman with a history of hypertension
• Presenting with hypertensive crisis, palpitation, and sense of head pulsation
• Heart rate is 100/min, AP is 190/100 mm Hg (haemodynamics is of hyperkinetic type)
Explanation:
The medication of choice for hypertensive crisis with symptoms such as palpitation, sense of
head pulsation, and tachycardia is a β-adrenoceptor blocker. β-blockers reduce heart rate and
cardiac output, which can help lower blood pressure. They also have antiarrhythmic and
anxiolytic effects that can be beneficial in hypertensive crisis.

Therapy
Other Options:
B) Adenosine pyrophosphate inhibitor: Adenosine pyrophosphate inhibitors, such as ticlopidine
and clopidogrel, are used to prevent blood clots in patients with coronary artery disease or
peripheral vascular disease. They are not indicated for the treatment of hypertensive crisis.
C) Diuretic: Diuretics are commonly used for the treatment of hypertension, but they are not
the first-line medication for hypertensive crisis. Diuretics can take several hours to lower blood
pressure, which may not be fast enough in the setting of hypertensive crisis.
D) α-adrenoceptor blocker: α-blockers, such as doxazosin and prazosin, are used to treat
hypertension by relaxing blood vessels and reducing peripheral vascular resistance. They are not
the first-line medication for hypertensive crisis because they can cause reflex tachycardia and
worsen symptoms such as palpitation and sense of head pulsation.
E) Dihydropyridine calcium antagonist: Calcium channel blockers, such as amlodipine and
nifedipine, are used to treat hypertension by relaxing blood vessels and reducing peripheral
vascular resistance. They can also have a negative inotropic effect, which may worsen
symptoms such as palpitation and sense of head pulsation in the setting of hypertensive crisis.
Making Other Options Right:
B) Adenosine pyrophosphate inhibitor: The medication of choice for preventing blood clots in
patients with coronary artery disease or peripheral vascular disease.
C) Diuretic: A second-line medication for hypertensive crisis when β-blockers are
contraindicated or ineffective.
D) α-adrenoceptor blocker: A medication used to treat hypertension by relaxing blood vessels
and reducing peripheral vascular resistance, but not the first-line medication for hypertensive
crisis.
E) Dihydropyridine calcium antagonist: A medication used to treat hypertension by relaxing
blood vessels and reducing peripheral vascular resistance, but not the first-line medication for
hypertensive crisis.
Medication:
The medication of choice for hypertensive crisis with symptoms such as palpitation, sense of
head pulsation, and tachycardia is a β-adrenoceptor blocker. Examples of β-blockers used in the
treatment of hypertensive crisis include propranolol, esmolol, and labetalol. Propranolol is a
nonselective β-blocker that can cause bronchospasm and is contraindicated in patients with
cardiology

asthma or chronic obstructive pulmonary disease. Esmolol is a short-acting β-blocker that can
be given intravenously and is useful in the acute setting of hypertensive crisis. Labetalol is a
combined α- and β-blocker that can be given intravenously and is useful in patients with
hypertensive crisis complicated by tachycardia or angina.
Words:
• β-adrenoceptor blocker: a medication that blocks the action of catecholamines on β-adrenergic
receptors, resulting in decreased heart rate and cardiac output
• Adenosine pyrophosphate inhibitor: a medication that inhibits the activation of platelets by
adenosine diphosphate (ADP) and prevents blood clot formation
• Diuretic: a medication that increases urine output and reduces fluid volume in the body
• α-adrenoceptor blocker: a medication that blocks the action of catecholamines on α-adrenergic
receptors, resulting in relaxation of blood vessels and reduced peripheral vascular resistance
• Dihydropyridine calcium antagonist: a medication that blocks the influx of calcium ions into
vascular smooth muscle cells, resulting in relaxation of blood vessels and reduced peripheral
vascular resistance
Findings:
• Heart rate: Normal heart rate is between 60-100 beats per minute. A heart rate of 100 beats
per minute is considered tachycardia.
• Blood pressure: Normal blood pressure is less than 120/80 mm Hg. A blood pressure of
190/100 mm Hg is considered hypertensive crisis.

Therapy
cardiology

53 Infective Endocarditis

A 33-year-old man with a history of rheumatic fever complains of fever up to 38-39oC,


abdominal pain, dyspnea, tachycardia. Heart borders are displaced to the left by 2 cm,
systolic and diastolic murmurs above aorta, BP of 160/30 mm Hg. Petechial rash occurs
after measurement of blood pressure. Liver is enlarged by 3 cm, spleen is palpable. Urine
is brown-yellow. What is the most likely diagnosis?
A Infectious endocarditis
B Rheumatic fever
C Acute hepatitis
D Acute nephritis
E Aortic regurgitation

Correct Answer: Option A


The patient's clinical presentation is consistent with Infective Endocarditis, which is an infection
of the endothelial surface of the heart, including the heart valves. The most likely diagnosis is
option A, Infectious endocarditis.
Key Points:
• 33-year-old man with a history of rheumatic fever
• Fever up to 38-39oC, abdominal pain, dyspnea, tachycardia
• Heart borders displaced to the left by 2 cm, systolic and diastolic murmurs above aorta
• BP of 160/30 mm Hg, petechial rash after measurement of blood pressure

Therapy
• Enlarged liver by 3 cm, palpable spleen
• Brown-yellow urine
• Most likely diagnosis
Explanation:The patient's clinical presentation is consistent with Infective Endocarditis, which is
an infection of the endothelial surface of the heart, including the heart valves. The patient has a
history of rheumatic fever, which can predispose him to developing endocarditis. The presence
of fever, abdominal pain, dyspnea, and tachycardia suggests an acute infective process. The
physical examination findings of displaced heart borders, systolic and diastolic murmurs above
the aorta, and petechial rash after measurement of blood pressure are all suggestive of infective
endocarditis. The presence of an enlarged liver, palpable spleen, and brown-yellow urine
suggest the presence of systemic involvement.
Among the given options, Infectious endocarditis is the most likely diagnosis, given the patient's
clinical findings. Rheumatic fever, option B, can also result in valvular heart disease and
increase the risk of infective endocarditis, but the patient's presentation is more consistent with
an acute infective process. Acute hepatitis, option C, and Acute nephritis, option D, are less
likely given the absence of characteristic findings such as jaundice, liver tenderness, and
proteinuria. Aortic regurgitation, option E, can lead to a diastolic murmur but is less likely given
the presence of a systolic murmur above the aorta and the other systemic findings.
Other Options:
• Option A, Infectious endocarditis, is the most likely diagnosis.
• Option B, Rheumatic fever, can result in valvular heart disease and increase the risk of
infective endocarditis, but the patient's presentation is more consistent with an acute infective
process.
• Option C, Acute hepatitis, is less likely given the absence of characteristic findings such as
jaundice and liver tenderness.
• Option D, Acute nephritis, is less likely given the absence of characteristic findings such as
proteinuria and hematuria.
• Option E, Aortic regurgitation, is less likely given the presence of a systolic and diastolic
murmur above the aorta and the other systemic findings.
Medication:
cardiology

• The treatment for Infective Endocarditis involves a combination of antibiotics and, in severe
cases, surgical intervention to repair or replace the affected heart valve.
Words:
• Infective Endocarditis: An infection of the endothelial surface of the heart, including the heart
valves.
• Rheumatic fever: A systemic inflammatory disease that can result in valvular heart disease
and increase the risk of infective endocarditis.
• Petechial rash: A rash characterized by small, pinpoint, flat, round red spots on the skin, which
can occur in the setting of infective endocarditis and other conditions that cause vascular
damage.
• Systolic murmur: A heart murmur that occurs during systole, when the heart is contracting.
• Diastolic murmur: A heart murmur that occurs during diastole, when the heart is filling with
blood.
• Enlarged liver: A sign of liver involvement, which can occur in the setting of infective
endocarditis and other conditions that cause systemic inflammation and damage.
• Palpable spleen: A sign of spleen involvement, which can occur in the setting of infective
endocarditis and other conditions that cause systemic inflammation and damage.
• Brown-yellow urine: A sign of hemoglobinuria, which can occur in the setting of infective
endocarditis and other conditions that cause hemolysis.
Findings:
• The patient's clinical findings include fever up to 38-39oC, abdominal pain, dyspnea,
tachycardia, displaced heart borders, systolic and diastolic murmurs above aorta, BP of 160/30
mm Hg, petechial rash after measurement of blood pressure, enlarged liver by 3 cm, palpable
spleen, and brown-yellow urine.

Therapy
cardiology

54 Ischemic Heart Disease

A 42-year-old patient applied to hospital with complaints of pain behind the sternum with
irradiation to the left scapula. The pain appears during significant physical work, this lasts
for 5-10 minutes and is over on rest. The patient is sick for 3 weeks. What is the
preliminary diagnosis?
A IHD:First established angina pectoris
B IHD:Variant angina pectoris (Prinzmetal's)
C IHD:Stable angina pectoris of effort I FC
D IHD:Stable angina pectoris of effort IV FC
E IHD:Progressive angina pectoris

Correct Answer: A. IHD: First established angina pectoris.


Key Points:
• 42-year-old patient
• Pain behind the sternum with irradiation to the left scapula
• Pain appears during significant physical work
• Lasts for 5-10 minutes
• Goes away on rest
• Complaints for 3 weeks
Explanation:The patient's symptoms are indicative of angina pectoris, which is chest pain or
discomfort that occurs when the heart muscle does not get enough blood. The preliminary

Therapy
diagnosis is IHD: First established angina pectoris because the patient's symptoms are typical of
this type of angina.
Other Options:B. IHD: Variant angina pectoris (Prinzmetal's) is caused by coronary artery
spasm, which can occur at rest and is not typically associated with physical exertion.C. IHD:
Stable angina pectoris of effort I FC is characterized by chest pain that occurs during physical
activity, but the symptoms are mild and do not limit daily activities.D. IHD: Stable angina
pectoris of effort IV FC is characterized by severe chest pain that occurs at low levels of physical
activity or at rest and significantly limits daily activities.E. IHD: Progressive angina pectoris is
characterized by increasing frequency, duration, or severity of chest pain, and it is not specific
to physical activity.
Making Other Options Right:B. IHD: Variant angina pectoris (Prinzmetal's) would be the correct
option if the patient's symptoms included chest pain at rest, without physical exertion.C. IHD:
Stable angina pectoris of effort I FC would be the correct option if the patient's symptoms were
mild and did not limit daily activities.D. IHD: Stable angina pectoris of effort IV FC would be the
correct option if the patient's symptoms were severe and significantly limited daily activities.E.
IHD: Progressive angina pectoris would be the correct option if the patient's symptoms were
increasing in frequency, duration, or severity.
Medication:The treatment for angina pectoris may include medications such as nitrates, beta-
blockers, or calcium channel blockers, depending on the type and severity of the angina.
Words:IHD: Ischemic Heart Disease.
cardiology

55 Mitral Stenosis

A 40 y.o. woman is ill with rheumatic disease with composite mitral disease with prevalence
of the stenosis of left venous foramen.Complains of palpitation, fatigability, progressing
dyspnea, attacks of dyspnea and hemoptysis. Now she cannot be engaged even in the
easy activities. What tactics is the most expedient?
A Mitral comissurotomia
B Conduction of current bicilino-prophilaxis
C Prescription of anticoagulants
D Prescription of venous vasodilatators
E-

Correct Answer: A. Mitral commissurotomy is the most expedient treatment in this situation.
Key Points: 40-year-old woman, rheumatic disease, composite mitral disease with prevalence of
left venous foramen stenosis, dyspnea, hemoptysis, palpitations, fatigue, inability to engage in
even easy activities.
Explanation: The patient has significant symptoms of dyspnea, fatigue, hemoptysis, and
palpitations, which indicate severe mitral stenosis. Mitral commissurotomy is a percutaneous
intervention that can relieve the stenosis of the mitral valve and improve symptoms in patients
with rheumatic mitral stenosis. It involves inserting a balloon catheter into the narrowed valve
and inflating it to open up the narrowed area. This procedure is effective in reducing symptoms
and improving the quality of life in most patients with rheumatic mitral stenosis.

Therapy
Other Options:
C. Prescription of anticoagulants is not a primary treatment for mitral stenosis. Anticoagulants
may be prescribed to prevent thromboembolism in patients with atrial fibrillation, which can
occur in patients with mitral stenosis, but it is not the primary treatment for the stenosis itself.
D. Prescription of venous vasodilators is not a primary treatment for mitral stenosis. Venous
vasodilators may be used to relieve symptoms of pulmonary congestion, which can occur in
patients with mitral stenosis, but it is not the primary treatment for the stenosis itself.
Making Other Options Right:
C. The option is incorrect because it is not the primary treatment for mitral stenosis. It may be
used for the prevention of thromboembolism in patients with atrial fibrillation.
D. The option is incorrect because it is not the primary treatment for mitral stenosis. It may be
used to relieve symptoms of pulmonary congestion in patients with mitral stenosis.
Medication:
No medications were mentioned in the question. However, after mitral commissurotomy,
anticoagulants may be prescribed to prevent thromboembolism.
Words: Mitral commissurotomy - "Mitral" refers to the mitral valve in the heart, and
"commissurotomy" means a surgical incision or opening made in a commissure (a junction or
connection between two parts) of the mitral valve to relieve stenosis.
Findings:
The patient has dyspnea, hemoptysis, palpitations, and fatigue, which are clinical symptoms
that are indicative of severe mitral stenosis.
cardiology

56 Mitral Stenosis

A 42 year old woman complains of dyspnea, edema of the legs and tachycardia during
minor physical exertion. Heart borders are displaced to the left and S1 is accentuated,
there is diastolic murmur on apex. The liver is enlarged by 5 cm. What is the cause of heart
failure?
A Mitral stenosis
B Mitral regurgitation
C Tricuspid stenosis
D Tricuspid regurgitation
E Aortic stenosis

Correct Answer: Option A, Mitral stenosis, is the most likely cause of the patient's heart failure.
Key Points:
• 42-year-old woman
• Dyspnea, edema of the legs, tachycardia with minor physical exertion
• Heart borders displaced to the left, accentuated S1, diastolic murmur on apex
• Enlarged liver by 5 cm
• Cause of heart failure
Explanation:
Mitral stenosis is a valvular heart disease characterized by narrowing or obstruction of the mitral
valve, which can lead to impaired left ventricular filling and increased pressure in the left atrium

Therapy
and pulmonary circulation. The presence of dyspnea, edema of the legs, and tachycardia with
minor physical exertion suggests a decreased cardiac output and increased venous pressure,
which can occur in the setting of mitral stenosis. The physical examination findings of an
accentuated S1 and a diastolic rumble at the apex are characteristic of mitral stenosis.
Other Options:
• Option B, Mitral regurgitation, is less likely given the absence of a characteristic systolic
murmur and the presence of an accentuated S1 and a diastolic rumble, which are more
suggestive of mitral stenosis, so it is incorrect.
• Option C, Tricuspid stenosis, is less likely given the absence of a characteristic opening snap
and diastolic rumble, and the presence of an enlarged liver, which suggests right-sided heart
failure, so it is incorrect.
• Option D, Tricuspid regurgitation, is less likely given the absence of a characteristic systolic
murmur on the left lower sternal border, which is more suggestive of tricuspid regurgitation,
and the presence of an enlarged liver, which suggests right-sided heart failure, so it is incorrect.
• Option E, Aortic stenosis, is less likely given the absence of a characteristic systolic ejection
murmur and the presence of an accentuated S1 and a diastolic rumble, which are more
suggestive of mitral stenosis, so it is incorrect.
Making Other Options Right:
• Option B would be correct if the patient had evidence of mitral valve prolapse or other causes
of mitral regurgitation, such as infective endocarditis, and developed symptoms and signs of
left-sided heart failure.
• Option C would be correct if the patient had evidence of tricuspid stenosis, such as a
characteristic opening snap and diastolic rumble, and developed symptoms and signs of right-
sided heart failure.
• Option D would be correct if the patient had evidence of tricuspid regurgitation, such as a
characteristic systolic murmur on the left lower sternal border, and developed symptoms and
signs of right-sided heart failure.
• Option E would be correct if the patient had evidence of aortic stenosis, such as a
characteristic systolic ejection murmur, and developed symptoms and signs of left-sided heart
failure.
Medication:
cardiology

• The treatment for mitral stenosis involves monitoring for complications, such as atrial
fibrillation, pulmonary hypertension, and heart failure, and using medications, such as diuretics,
beta blockers, and calcium channel blockers, to control symptoms and maintain cardiac function.
In severe cases, percutaneous mitral balloon valvuloplasty or surgical mitral valve replacement
may be necessary.
Words:
• Mitral stenosis: A valvular heart disease characterized by narrowing or obstruction of the
mitral valve, which can lead to impaired left ventricular filling and increased pressure in the left
atrium and pulmonary circulation.
• Venous pressure: The pressure within the veins, which can increase in the presence of heart
failure or other conditions that impede venous return.
• Cardiac output: The amount of blood pumped by the heart per minute, which can decrease in
the presence of heart failure or other conditions that impair cardiac function.
• Left ventricular filling: The process of blood filling the left ventricle of the heart during diastole,
which can be impaired in the presence of mitral stenosis.
• Pulmonary circulation: The system of blood vessels that carries blood from the heart to the
lungs and back, which can be affected by mitral stenosis and other conditions that impair
cardiac function.
• Mitral regurgitation: A valvular heart disease characterized by the backflow of blood from the
left ventricle to the left atrium during systole, leading to chronic volume overload and left
ventricular dysfunction.
• Enlarged liver: A sign of right-sided heart failure, which can occur as a consequence of left-
sided heart failure.
• Diastolic murmur: A heart murmur that occurs during diastole, when the heart is filling with
blood.
Findings:

Therapy
• The patient's clinical findings include dyspnea, edema of the legs, tachycardia with minor
physical exertion, displaced heart borders, accentuated S1, and diastolic rumble on apex.
• The patient's physical examination findings include an enlarged liver by 5 cm.
cardiology

57 Mitral Stenosis

A female rheumatic patient experiences diastolic thoracic wall tremor (diastolic thrill),
accentuated S1 at apex, there is diastolic murmur with presystolic intensification,
opening snap, S2 accent at pulmonary artery. What rind of heart disorder is observed?
A Mitral stenosis
B Aortic valve insufficiency
C Pulmonary artery stenosis
D Mitral valve insufficiency
E Opened arterial duct

Correct Answer: Mitral stenosis is the observed heart disorder.


Key Points:
• Female rheumatic patient
• Diastolic thoracic wall tremor (diastolic thrill)
• Accentuated S1 at apex
• Diastolic murmur with presystolic intensification, opening snap
• S2 accent at pulmonary artery
Explanation:
• The patient's symptoms of diastolic thoracic wall tremor, accentuated S1 at the apex, and
diastolic murmur with presystolic intensification, opening snap, and S2 accent at the pulmonary
artery are all consistent with mitral stenosis.

Therapy
• Mitral stenosis is a valvular heart disease that occurs when the mitral valve, which separates
the left atrium and left ventricle, becomes narrowed, impeding blood flow from the left atrium to
the left ventricle.
• Other possible diagnoses, such as aortic valve insufficiency, pulmonary artery stenosis, mitral
valve insufficiency, and opened arterial duct, do not fit the patient's presentation as well as
mitral stenosis.
Other Options:
• Aortic valve insufficiency (option B) is a valvular heart disease that occurs when the aortic
valve, which separates the left ventricle and the aorta, does not close properly, allowing blood
to flow back into the left ventricle. It is not typically associated with a diastolic murmur or
opening snap.
• Pulmonary artery stenosis (option C) is a valvular heart disease that occurs when the
pulmonary valve, which separates the right ventricle and pulmonary artery, becomes narrowed,
impeding blood flow from the right ventricle to the lungs. It is not typically associated with a
diastolic murmur, presystolic intensification, or opening snap.
• Mitral valve insufficiency (option D) is a valvular heart disease that occurs when the mitral
valve does not close properly, allowing blood to flow back into the left atrium. It is not typically
associated with a diastolic murmur with presystolic intensification or an opening snap.
• Opened arterial duct (option E) refers to a congenital heart defect in which the ductus
arteriosus, a blood vessel that connects the pulmonary artery and aorta in the fetus, fails to
close after birth. It is not typically associated with a diastolic murmur, presystolic intensification,
or opening snap.
Making Other Options Right:
• Option B (Aortic valve insufficiency) could be correct if the patient had a regurgitation of the
aortic valve, but this is not suggested by the patient's presentation.
• Option C (Pulmonary artery stenosis) could be correct if the patient had a narrowing of the
pulmonary valve, but this is not the main diagnosis that is being presented.
• Option D (Mitral valve insufficiency) could be correct if the patient had a regurgitation of the
mitral valve, but this is not suggested by the patient's presentation.
• Option E (Opened arterial duct) could be correct if the patient had a congenital heart defect,
but this is not suggested by the patient's presentation.
cardiology

Medication:
• The treatment of mitral stenosis depends on the severity of the disease.
• In mild cases, no treatment may be necessary, and the patient may be monitored for signs of
progression.
• In moderate to severe cases, medications such as diuretics, beta-blockers, and calcium
channel blockers may be used to control symptoms.
• In severe cases, surgical interventions such as mitral valve repair or replacement may be
necessary.
Words:
• Mitral stenosis: A valvular heart disease that occurs when the mitral valve becomes narrowed,
impeding blood flow from the left atrium to the left ventricle.
Findings:
• Diastolic thoracic wall tremor (diastolic thrill): A diastolic thrill is a vibration or tremor that can
be felt over the chest wall during diastole, which is the relaxation phase of the cardiac cycle.
• Accentuated S1 at apex: S1 is the first heart sound, which is produced by the closure of the
mitral and tricuspid valves. It is accentuated at the apex of the heart, which is the point of
maximal impulse.
• Diastolic murmur with presystolic intensification, opening snap: A diastolic murmur is a sound
that can be heard during diastole, which is caused by the flow of blood through a narrowed
valve. In mitral stenosis, the murmur is typically preceded by an opening snap and is intensified
during presystole, which is the phase just before the contraction of the atria.
• S2 accent at pulmonary artery: S2 is the second heart sound, which is produced by the
closure of the aortic and pulmonary valves. In mitral stenosis, the pulmonary artery pressure
may be increased, leading to an accentuated S2 at the pulmonary artery.

Therapy
cardiology

58 Myocardial Infarction

After myocardial infarction, a 50-year-old patient had an attack of asthma. Objectively: bubbling
breathing with frequency of 32/min, cough with a lot of pink frothy sputum, acrocyanosis,
swelling
of the neck veins. Ps- 108/min, AP- 150/100 mm Hg. Heart sounds are muffled. Mixed moist
rales can be auscultated above the entire lung surface. What drug would be most effective in
this situation?
A Nitroglycerin intravenously
B Pentamin intravenously
C Strophanthin intravenously
D Dopamine intravenously
E Aminophylline intravenously

Correct Answer: A - Nitroglycerin intravenously


Key Points:
• 50-year-old patient with a history of myocardial infarction
• Currently experiencing an attack of asthma
• Bubbling breathing with frequency of 32/min, cough with pink frothy sputum, acrocyanosis,
swelling of the neck veins
• Elevated heart rate and blood pressure
• Muffled heart sounds and mixed moist rales throughout the lung surface

Therapy
Explanation:
The patient is presenting with symptoms of cardiogenic pulmonary edema, a complication of
heart failure. Nitroglycerin is a vasodilator that can help reduce the workload on the heart and
improve blood flow, which can help alleviate symptoms of pulmonary edema. It is the most
effective drug in this situation.
Nitroglycerin works by relaxing smooth muscle in blood vessels, which reduces blood pressure
and improves blood flow. It also reduces the workload on the heart, which can help improve
symptoms of heart failure and pulmonary edema.
Other Options:
B. Pentamin intravenously - Pentamin is a vasodilator that is used to treat hypertension. It is
not typically used to treat pulmonary edema.
C. Strophanthin intravenously - Strophanthin is a cardiac glycoside that is used to treat heart
failure. While it may be useful in some cases, it is not likely to be the most effective drug in this
situation.
D. Dopamine intravenously - Dopamine is a medication that can increase blood flow and
improve heart function. While it may be useful in some cases, it is not likely to be the most
effective drug in this situation.
E. Aminophylline intravenously - Aminophylline is a bronchodilator that is used to treat asthma.
While it may be helpful in treating the patient's asthma symptoms, it is not likely to be the most
effective drug in this situation.
Making Other Options Right:
B. Pentamin intravenously - If the patient had hypertension that was contributing to their
pulmonary edema, pentamin may be useful in reducing blood pressure and improving blood
flow.
C. Strophanthin intravenously - Strophanthin may be useful if the patient's heart failure is not
responsive to nitroglycerin or other treatments.
D. Dopamine intravenously - If the patient's blood pressure was low, dopamine may be useful in
increasing blood flow and improving heart function.
E. Aminophylline intravenously - Aminophylline may be useful in treating the patient's asthma
symptoms, but it is not likely to be the most effective drug for their pulmonary edema.
Medication:
cardiology

The patient with myocardial infarction and asthma should be treated with nitroglycerin to
manage symptoms of pulmonary edema. Bronchodilators such as albuterol or ipratropium can
also be used to manage asthma symptoms.
Words:
Acrocyanosis - A condition in which the hands and feet appear bluish due to poor circulation.
Findings:
Ps - 108/min (elevated)
AP - 150/100 mm Hg (elevated)
Heart sounds are muffled
Mixed moist rales can be auscultated above the entire lung surface.

Therapy
cardiology

59 Myocardial Infarction

A 45-year-old driver was admitted to the hospital with 5 hour substernal pain. Nitroglycerin
is not effective. He is pale, heart sounds are regular but weak. HR - 96 per minute, BP of
100/60 mm Hg. What is the most likely diagnosis?
A Acute myocardial infarction
B Stable angina
C Pulmonary embolism
D Acute myocarditis
E Acute left ventricular failure

Correct Answer: Option A, Acute myocardial infarction, is the most likely diagnosis.
Key Points:
• 45-year-old driver
• Admitted to the hospital with 5-hour substernal pain
• Nitroglycerin is not effective
• Pale appearance, weak heart sounds, HR - 96 per minute, BP of 100/60 mm Hg
• Most likely diagnosis
Explanation:The patient's symptoms and clinical presentation are consistent with acute
myocardial infarction (AMI), which is a medical emergency and requires immediate treatment.
The typical presentation of AMI includes substernal chest pain that may radiate to the left arm
or jaw, and is often associated with nausea, sweating, and shortness of breath. The patient's

Therapy
duration of symptoms, lack of response to nitroglycerin, and clinical findings of weak heart
sounds and low blood pressure are concerning for a severe and possibly complicated AMI.
Other Options:
• Option B, Stable angina, is less likely given the prolonged duration of the patient's symptoms
and lack of response to nitroglycerin, so it is incorrect.
• Option C, Pulmonary embolism, is less likely given the patient's presentation with chest pain
and lack of significant respiratory symptoms or risk factors for pulmonary embolism, so it is
incorrect.
• Option D, Acute myocarditis, is less likely given the patient's clinical findings of weak heart
sounds and low blood pressure, which are more suggestive of a severely compromised
myocardium, and lack of significant viral symptoms or history of recent infection, so it is
incorrect.
• Option E, Acute left ventricular failure, is less likely given the patient's presentation with chest
pain and lack of significant respiratory symptoms or signs of pulmonary congestion, so it is
incorrect.
Making Other Options Right:
• Option B would be correct if the patient had a history of stable angina and developed
symptoms that were typical for their condition and were relieved by nitroglycerin.
• Option C would be correct if the patient had significant risk factors for pulmonary embolism
and developed symptoms such as chest pain, shortness of breath, and hemoptysis.
• Option D would be correct if the patient had a recent viral illness and developed symptoms
such as chest pain, dyspnea, and arrhythmias.
• Option E would be correct if the patient had a history of heart failure and developed symptoms
such as chest pain, dyspnea, and signs of pulmonary congestion.
Medication:
• The treatment for acute myocardial infarction involves immediate reperfusion therapy, which
can be achieved through percutaneous coronary intervention (PCI) or fibrinolytic therapy. The
patient may also receive medications to manage pain, reduce myocardial oxygen demand, and
prevent complications.
Words:
cardiology

• Acute myocardial infarction: A medical emergency characterized by the sudden occlusion of a


coronary artery, leading to ischemia and necrosis of the myocardium.
• Nitroglycerin: A medication used to relieve chest pain associated with angina and acute
myocardial infarction.
• Reperfusion therapy: A treatment strategy aimed at restoring blood flow to an ischemic tissue,
such as the myocardium, to prevent irreversible damage.
Findings:
• The patient's clinical findings include a pale appearance, weak heart sounds, HR - 96 per
minute, and BP of 100/60 mm Hg.

Therapy
cardiology

60 Myocardial Infarction

A 64 y.o. patient has developed of squeering substernal pain which had appeared 2 hours
ago and irradiated to the left shoulder, marked weakness. On examination: pale skin, cold
sweat. Pulse- 108 bpm, AP- 70/50 mm Hg, heart sound are deaf, vesicular breathing, soft
abdomen, painless, varicouse vein on the left shin, ECG: synus rhythm, heart rate is 100
bmp, ST-segment is sharply elevated in II, III aVF leads. What is the most likely disorder?
A Cardiogenic shock
B Cardiac asthma
C Pulmonary artery thromboembolia
D Disquamative aortic aneurizm
E Cardiac tamponade

Correct Answer: A. Cardiogenic shock.


Key Points:
• The patient is a 64-year-old with substernal pain irradiating to the left shoulder, weakness,
pale skin, and cold sweat.
• On examination, the patient has a low blood pressure and a fast heart rate, and the ECG
shows ST-segment elevation.
• These symptoms suggest a possible acute coronary syndrome with cardiogenic shock.
Explanation:
• Acute coronary syndrome (ACS) is a medical emergency that occurs when there is a sudden

Therapy
reduction or blockage of blood flow to the heart muscle.
• ACS can cause a range of symptoms, including chest pain, shortness of breath, sweating, and
weakness.
• Cardiogenic shock is a severe complication of ACS, in which the heart is unable to pump
enough blood to meet the body's needs, causing low blood pressure and organ dysfunction.
• The patient's symptoms of substernal pain, weakness, pale skin, and cold sweat, in addition to
the low blood pressure and fast heart rate, suggest cardiogenic shock due to ACS.
• The ECG findings of ST-segment elevation in leads II, III, and aVF are consistent with an
inferior wall myocardial infarction (MI).
Other Options:
• Option B (Cardiac asthma) is unlikely as the patient does not have a history of heart failure,
and the symptoms are more consistent with ACS and cardiogenic shock.
• Option C (Pulmonary artery thromboembolism) is unlikely as the patient does not have
symptoms such as shortness of breath, cough, or hemoptysis, which are typical of pulmonary
embolism.
• Option D (Dissecting aortic aneurysm) is unlikely as the patient does not have symptoms such
as severe chest or back pain, and the ECG findings are not consistent with aortic dissection.
• Option E (Cardiac tamponade) is unlikely as the patient does not have symptoms such as
dyspnea, jugular venous distention, or pulsus paradoxus, which are typical of cardiac
tamponade.
Making Other Options Wrong:
• Options B, C, D, and E are not likely diagnoses for the patient's symptoms of substernal pain,
weakness, pale skin, and cold sweat, in addition to the low blood pressure and fast heart rate.
Medication:
• Treatment for ACS with cardiogenic shock typically involves hospitalization, oxygen therapy,
intravenous medications such as nitroglycerin and morphine, and reperfusion therapy such as
percutaneous coronary intervention (PCI) or thrombolytic therapy.
Words:
• Acute coronary syndrome (ACS): A medical emergency that occurs when there is a sudden
reduction or blockage of blood flow to the heart muscle.
cardiology

• Cardiogenic shock: A severe complication of ACS or other cardiac conditions, in which the
heart is unable to pump enough blood to meet the body's needs, causing low blood pressure
and organ dysfunction.
• Myocardial infarction (MI): A type of ACS that occurs when there is a prolonged reduction or
blockage of blood flow to the heart muscle, causing damage or death of the heart muscle cells.
• ST-segment elevation: A characteristic ECG finding in MI, in which the ST segment of the ECG
tracing is elevated above the baseline, indicating myocardial injury.
Findings:
• ACS with cardiogenic shock is a medical emergency that requires prompt diagnosis and
treatment to prevent complications such as heart failure, arrhythmias, and death.
• Reperfusion therapy with PCI or thrombolytic therapy is the preferred treatment for patients
with ST-segment elevation MI and cardiogenic shock.
• Patients with ACS and cardiogenic shock require close monitoring of their vital signs, fluid
balance, and organ function.

Therapy
cardiology

61 Myocardial Infarction

A 30 year old woman ill with influenza felt palpitation and dull cardiac pain during moderate
physical exercise. Objectively: Ps - 96 bpm, AP - 100/60 mm Hg. The first sound is quiet
above the apex, soft systolic murmur is present. What complication is indicated by these
clinical presentations?
A Acute viral myocarditis
B Acute allergic infectious myocarditis
C Idiopathic myocarditis
D Myocardiopathy
E Neurocirculatory dystonia

Correct Answer: A
Key Points:
• 30-year-old woman with influenza.
• Palpitations and dull cardiac pain during moderate physical exercise.
• Ps - 96 bpm and AP - 100/60 mm Hg.
• First heart sound is quiet above the apex, and soft systolic murmur is present.
Explanation:
The clinical presentations described in the question suggest acute viral myocarditis as the
complication of influenza. Myocarditis is an inflammatory disease of the myocardium, the heart
muscle, which can be caused by viral infections, such as influenza, coxsackievirus, and

Therapy
adenovirus. Acute viral myocarditis is characterized by symptoms such as palpitations, chest
pain, and shortness of breath, as well as abnormal cardiac findings, such as abnormal heart
sounds, murmurs, and arrhythmias.
The presence of a quiet first heart sound above the apex and a soft systolic murmur is indicative
of left ventricular dysfunction, which can occur in acute viral myocarditis due to inflammation,
necrosis, and fibrosis of the myocardium.
Other Options:
B. Acute allergic infectious myocarditis is not a recognized clinical entity and is not a likely
diagnosis.
C. Idiopathic myocarditis is myocarditis of unknown cause, and it is less likely than acute viral
myocarditis in a patient with influenza.
D. Myocardiopathy is a general term that refers to any disease of the heart muscle, including
myocarditis, but it is not a specific diagnosis.
E. Neurocirculatory dystonia is a term that is no longer used in modern medicine, and it is not a
recognized diagnosis.
Making Other Options Right:
B. Acute allergic myocarditis could be a possible diagnosis if the patient had a history of allergy
or anaphylaxis and had been exposed to an allergen.
C. Idiopathic myocarditis could be a possible diagnosis if the patient had no history of viral
infection and no other identifiable cause of myocarditis.
D. Myocardiopathy could be a possible diagnosis if the patient had a primary disease of the
heart muscle, such as dilated cardiomyopathy or hypertrophic cardiomyopathy.
E. Neurocirculatory dystonia is an outdated term for a group of symptoms, including
palpitations, chest pain, and fatigue, which are now recognized as part of various cardiac and
non-cardiac conditions.
Medication:
The treatment of acute viral myocarditis is supportive and symptomatic, and it includes rest,
avoidance of physical exertion, and management of arrhythmias and heart failure. Antiviral
therapy and immunosuppressive therapy may be considered in selected cases. The medication
options given in the question are not relevant to the patient's diagnosis.
Words:
cardiology

Palpitation - sensation of rapid or irregular heartbeats.


Myocarditis - inflammation of the heart muscle.
Murmur - abnormal heart sound caused by turbulent blood flow.
Neurocirculatory dystonia - an outdated term for a group of symptoms, including palpitations,
chest pain, and fatigue.
Findings:
The normal range of heart rate is 60-100 beats per minute, and the normal range of blood
pressure is 90/60 mm Hg to 120/80 mm Hg. The patient's heart rate of 96 bpm and blood
pressure of 100/60 mm Hg are within the normal range. The normal first heart sound is loud
and the normal second heart sound is soft. The presence of a quiet first heart sound above the
apex and a soft systolic murmur is abnormal and suggests left ventricular dysfunction due to
acute viral myocarditis.

Therapy
cardiology

62 Myocardial Infarction

A patient has got acute macrofocal myocardial infarction complicated by cardiogenic shock.
The latter is progressing under conditions of weak general peripheric resistance and
decreased cardiac output. What antihypotensive drug should be injected to the patient in
the first place?
A Dopamine
B Noradrenaline
C Adrenaline
D Mesatonum
E Prednisolone

Option A, Dopamine, is the correct answer because it is a first-line antihypotensive drug used in
the management of cardiogenic shock. It increases cardiac output by increasing the heart rate
and constricting blood vessels, improving blood flow to vital organs.
Key Points:
• Acute macrofocal myocardial infarction
• Complicated by cardiogenic shock
• Weak general peripheral resistance
• Decreased cardiac output
Explanation:
Dopamine is a catecholamine that acts as a potent vasoconstrictor and inotropic agent. It

Therapy
increases cardiac output by increasing the heart rate and constricting blood vessels, improving
blood flow to vital organs. In the setting of cardiogenic shock, dopamine is a first-line
antihypotensive drug used to improve cardiac output and increase blood pressure.
Other Options:
B) Noradrenaline is not the correct answer because it is a potent vasoconstrictor that can
increase peripheral resistance and worsen myocardial ischemia in the setting of acute
myocardial infarction.
C) Adrenaline is not the correct answer because it is a potent vasoconstrictor that can increase
peripheral resistance and worsen myocardial ischemia in the setting of acute myocardial
infarction.
D) Mesatonum is not the correct answer because it is a peripheral vasoconstrictor that may
cause reflex bradycardia and worsen myocardial ischemia.
E) Prednisolone is not the correct answer because it is a steroid medication with anti-
inflammatory properties and has no direct effect on blood pressure or cardiac output.
Making Other Options Right:
B) To make this option correct, the question could change the patient's condition to one with
adequate cardiac output and low blood pressure, where noradrenaline would be a suitable
choice for increasing blood pressure by vasoconstriction.
C) To make this option correct, the question could change the patient's condition to one with
adequate cardiac output and low blood pressure, where adrenaline would be a suitable choice
for increasing blood pressure and cardiac output.
D) To make this option correct, the question could change the patient's condition to one with
peripheral vasodilation and adequate cardiac output, where Mesatonum would be a suitable
choice for increasing peripheral resistance.
E) To make this option correct, the question could change the patient's condition to one with an
inflammatory condition or an allergic reaction, where Prednisolone would be a suitable choice for
reducing inflammation and allergic symptoms.
Medication:
Dopamine is administered via continuous intravenous infusion and requires careful monitoring of
blood pressure, heart rate, and cardiac rhythm. Side effects of dopamine include tachycardia,
arrhythmias, and peripheral ischemia.
cardiology

Words:
Macrofocal - refers to a large area of tissue, in this case, a large area of the myocardium
affected by the infarction.
Mesatonum - a brand name for methoxamine, a peripheral vasoconstrictor used to increase
blood pressure in cases of shock or hypotension.
Findings:
None mentioned in the question.

Therapy
cardiology

63 Myocardial Infarction

A patient had macrofocal myocardial infarction. He is overweight for 36%, AP is 150/90 mm


Hg, blood sugar- 5,9 mmol/L, general cholesterol- 4,9 mmol/L, uric acid- 0,211 mmol/L.
Which risk factor should be urgently eradicated during the secondary prevention?
A Obesity
B Arterial hypertension
C Hyperglycemia
D Hypercholesterolemia
E Hyperuricemia

Correct Answer: The correct answer is A, Obesity.


Key Points:
• The patient has macrofocal myocardial infarction.
• The patient is overweight for 36%
• The patient has arterial hypertension with a blood pressure of 150/90 mm Hg.
• The patient's blood sugar is 5.9 mmol/L.
• The patient's cholesterol level is 4.9 mmol/L.
• The patient's uric acid level is 0.211 mmol/L.
• The question is asking which risk factor should be urgently eradicated during secondary
prevention.
Explanation:

Therapy
• Obesity is a significant risk factor for cardiovascular disease, including myocardial infarction,
and is associated with increased morbidity and mortality.
• Weight loss is an essential component of secondary prevention in patients with myocardial
infarction who are overweight or obese.
• Arterial hypertension is also a significant risk factor for cardiovascular disease and is often
present in patients with myocardial infarction. However, it is not the urgent risk factor to be
eradicated in this case.
• Hyperglycemia, hypercholesterolemia, and hyperuricemia are also risk factors for
cardiovascular disease. However, in this case, obesity is the most critical risk factor to be
urgently eradicated during secondary prevention.
Other Options:
• Option B, arterial hypertension, is a significant risk factor for cardiovascular disease, but it is
not the most urgent risk factor to be eradicated in this case.
• Option C, hyperglycemia, is a risk factor for cardiovascular disease, but it is not the most
urgent risk factor to be eradicated in this case.
• Option D, hypercholesterolemia, is a risk factor for cardiovascular disease, but it is not the
most urgent risk factor to be eradicated in this case.
• Option E, hyperuricemia, is a risk factor for cardiovascular disease, but it is not the most
urgent risk factor to be eradicated in this case.
Making Other Options Right:
• Option B, arterial hypertension, could be the correct answer if the patient did not have
obesity.
• Option C, hyperglycemia, could be the correct answer if the patient had uncontrolled diabetes.
• Option D, hypercholesterolemia, could be the correct answer if the patient had familial
hypercholesterolemia or other genetic lipid disorders.
• Option E, hyperuricemia, could be the correct answer if the patient had gout or other medical
conditions associated with hyperuricemia.
Medication:
• For obesity, weight loss is the primary treatment. Diet and exercise are the first-line therapies,
but medications or bariatric surgery may be necessary for some patients.
cardiology

• For arterial hypertension, lifestyle modifications such as weight loss, exercise, and a healthy
diet are the first-line therapies. Medications such as ACE inhibitors, ARBs, beta-blockers, calcium
channel blockers, and diuretics may also be necessary.
• For hyperglycemia, lifestyle modifications such as weight loss, exercise, and a healthy diet are
the first-line therapies. Medications such as metformin, sulfonylureas, DPP-4 inhibitors, GLP-1
receptor agonists, SGLT2 inhibitors, and insulin may also be necessary.
• For hypercholesterolemia, lifestyle modifications such as weight loss, exercise, and a healthy
diet are the first-line therapies. Medications such as statins, ezetimibe, PCSK9 inhibitors, and
bile acid sequestrants may also be necessary.
• For hyperuricemia, lifestyle modifications such as weight loss, a healthy diet, and avoiding
alcohol and purine-rich foods are the first-line therapies. Medications such as allopurinol and
febuxostat may also be necessary.
Words:
• Macrofocal: a term used to describe a large area of tissue, such as a myocardial infarction.
• Uric acid: a waste product formed by the breakdown of purines in the body. High levels of uric
acid can lead to gout and other medical conditions.
Findings:
• Blood sugar: The normal range for fasting blood glucose is 3.9-5.6 mmol/L. The patient's
blood sugar level of 5.9 mmol/L is slightly elevated but not diagnostic of diabetes.
• Cholesterol: The normal range for total cholesterol is <5.2 mmol/L. The patient's cholesterol
level of 4.9 mmol/L is within the normal range.
• Uric acid: The normal range for uric acid is 0.12-0.42 mmol/L. The patient's uric acid level of
0.211 mmol/L is within the normal range.

Therapy
cardiology

64 Myocardial Infarction

Thrombosis of the coronary artery caused myocardial infarction. What mechanisms of


injury will be the dominating ones in this disease?
A Calcium mechanisms
B Electrolytoosmotic mechanisms
C Acidotic mechanisms
D Protein mechanisms
E Lipid mechanism

Correct Answer: Option A (Calcium mechanisms) is the correct answer because calcium plays a
critical role in the pathophysiology of myocardial infarction.
Key Points:
• Thrombosis of the coronary artery causing myocardial infarction.
• Dominating mechanisms of injury in myocardial infarction.
Explanation:
Myocardial infarction (MI), commonly known as a heart attack, occurs when the blood supply to
a part of the heart is interrupted, leading to ischemia and subsequent tissue damage.
Thrombosis of the coronary artery is a common cause of MI. The mechanisms of injury in MI are
complex and involve multiple pathways, but calcium plays a critical role in the pathophysiology
of the disease.
During ischemia, the lack of oxygen and nutrients leads to a disruption of cellular metabolism,

Therapy
including the accumulation of intracellular calcium. The influx of calcium into cells triggers a
cascade of events that leads to cell death and tissue damage. Calcium overload can activate
enzymes, such as proteases and lipases, that degrade cellular components and contribute to
tissue injury. Calcium can also lead to the opening of mitochondrial permeability transition
pores, which can trigger apoptosis and necrosis.
Other mechanisms of injury in MI include oxidative stress, inflammation, and the production of
reactive oxygen species. These mechanisms can contribute to cell death and tissue damage
through the activation of pro-inflammatory pathways, such as the release of cytokines and
chemokines.
Other Options:
• Option B (Electrolytoosmotic mechanisms) is incorrect because electrolyte imbalances, such as
hyperkalemia or hypokalemia, can contribute to the development of arrhythmias in MI, but they
are not the primary mechanisms of injury.
• Option C (Acidotic mechanisms) is incorrect because acidosis can occur during ischemia and
contribute to cell injury, but it is not the primary mechanism of injury in MI.
• Option D (Protein mechanisms) is incorrect because protein degradation can occur during
ischemia and contribute to cell injury, but it is not the primary mechanism of injury in MI.
• Option E (Lipid mechanism) is incorrect because lipid accumulation, such as cholesterol
deposition, can contribute to the development of atherosclerosis and the formation of coronary
thrombi, but it is not the primary mechanism of injury in MI.
Making Other Options Right:
• Option B (Electrolytoosmotic mechanisms) could be made correct if the question was asking
about the mechanisms of arrhythmias in MI, rather than the mechanisms of injury.
• Option C (Acidotic mechanisms) could be made correct if the question was asking about the
effects of acidosis on cellular metabolism during ischemia, rather than the primary mechanisms
of injury.
• Option D (Protein mechanisms) could be made correct if the question was asking about the
effects of protein degradation on cellular metabolism during ischemia, rather than the primary
mechanisms of injury.
cardiology

• Option E (Lipid mechanism) could be made correct if the question was asking about the
mechanisms of atherosclerosis and thrombosis, rather than the primary mechanisms of injury in
MI.
Medication:
Treatment for myocardial infarction typically involves restoring blood flow to the affected area of
the heart as quickly as possible through interventions such as percutaneous coronary
intervention (PCI) or coronary artery bypass graft surgery (CABG). Medications such as
antiplatelet agents, anticoagulants, beta-blockers, and ace inhibitors may also be used to
prevent further thrombosis and reduce the risk of complications.
Words:
• Ischemia: a lack of blood flow to a part of the body, often caused by a blockage in a blood
vessel.
• Pathophysiology: the study of the physiological processes that occur in disease.

Therapy
cardiology

65 Myocarditis

A 43 y.o. woman complains of shooting heart pain, dyspnea, irregularities in the heart
activity, progressive fatigue during 3 weeks. She had acute respiratory disease a month
ago. On examination: AP- 120/80 mm Hg, heart rate 98 bpm, heart boarders +1,5 cm left
side, sounds are muffled, soft systolic murmur at apex and Botkin's area; sporadic
extrasystoles. Liver isn't palpated, there are no edema. Blood test: WBC- 6,7*109
/L,
sedimentation rate- 21 mm/hour. What is the most probable diagnosis?
A Acute myocarditis
B Climacteric myocardiodystrophia
C Ichemic heart disease, angina pectoris
D Rheumatism, mitral insufficiency
E Hypertrophic cardiomyopathy

Correct Answer: A Acute myocarditis.


Key Points:
• 43-year-old woman with shooting heart pain, dyspnea, irregularities in heart activity, and
progressive fatigue for 3 weeks.
• The patient had an acute respiratory disease a month ago.
• On examination, the patient has a heart rate of 98 bpm, heart boarders +1,5 cm left side,
sounds are muffled, soft systolic murmur at apex and Botkin's area, and sporadic extrasystoles.

Therapy
• Blood test shows WBC- 6,7*109/L, sedimentation rate- 21 mm/hour.
• The question asks for the most probable diagnosis.
Explanation:
The patient's symptoms and examination findings suggest a cardiac problem. The shooting heart
pain, dyspnea, irregularities in heart activity, and progressive fatigue could be indicative of
myocarditis, which is an inflammation of the heart muscle.
The patient had an acute respiratory disease a month ago, which could have predisposed her to
develop myocarditis. On examination, the patient has a heart rate of 98 bpm, heart boarders
+1,5 cm left side, sounds are muffled, soft systolic murmur at apex and Botkin's area, and
sporadic extrasystoles. These findings are consistent with myocarditis.
The blood test shows a slightly elevated sedimentation rate, which is also indicative of an
inflammatory process. However, the WBC count is within the normal range.
Other Options:
B) Climacteric myocardiodystrophia is not the correct answer because the patient's symptoms
and examination findings are not indicative of this condition.
C) Ichemic heart disease, angina pectoris is not the correct answer because the patient's
symptoms and examination findings are not indicative of this condition.
D) Rheumatism, mitral insufficiency is not the correct answer because the patient's symptoms
and examination findings are not indicative of this condition.
E) Hypertrophic cardiomyopathy is not the correct answer because the patient's symptoms and
examination findings are not indicative of this condition.
Making Other Options Right:
B) Climacteric myocardiodystrophia would be a correct answer if the patient had symptoms and
examination findings consistent with this condition, such as chest discomfort, palpitations,
fatigue, and shortness of breath.
C) Ichemic heart disease, angina pectoris would be a correct answer if the patient had
symptoms and examination findings consistent with this condition, such as chest pain, shortness
of breath, and fatigue.
D) Rheumatism, mitral insufficiency would be a correct answer if the patient had symptoms and
examination findings consistent with this condition, such as fever, joint pain, and heart murmur.
cardiology

E) Hypertrophic cardiomyopathy would be a correct answer if the patient had symptoms and
examination findings consistent with this condition, such as chest pain, shortness of breath, and
palpitations.
Medication:
The treatment for acute myocarditis includes rest, supportive care, and treatment of the
underlying cause. In some cases, medications may be used to manage symptoms, such as pain,
inflammation, and arrhythmias.
Words:
Myocarditis is a condition characterized by inflammation of the heart muscle.
Findings:
The blood test shows a normal WBC count of 6,7*109/L, and a slightly elevated sedimentation
rate of 21 mm/hour. The normal range for sedimentation rate is up to 20 mm/hour for women.
Sporadic extrasystoles are also present on examination.

Therapy
cardiology

66 Pericarditis

A 43-year-old female patient complains of dyspnea, swelling of legs, abdomen enlargement,


pricking heart pain. She has a history of tuberculous bronchadenitis, quinsies. The patient's
condition deteriorated 6 months ago. Objectively: cyanosis, bulging neck veins, vesicular
breathing. Heart borders are not displaced. Heart sounds are muffled, Ps is 106/min, liver is +4
cm, ascites is present. Low voltage on the ECG has been revealed. Radiograph shows a thin
layer of calcium deposits along the left contour of heart. What treatment should be
recommended to the patient?
A Treatment by a cardiac surgeon
B Digitalis preparations
C Anti-TB drugs
D Diuretics
E Vasodilators, nitrates

Correct Answer: A Treatment by a cardiac surgeon - correct.


Key Points:
• 43-year-old female patient with dyspnea, leg swelling, abdominal enlargement, and pricking
heart pain.
• History of tuberculous bronchadenitis and quinsies.
• Objectively, cyanosis, bulging neck veins, vesicular breathing, muffled heart sounds, Ps of
106/min, +4 cm liver, and ascites.

Therapy
• Low voltage on ECG, with thin layer of calcium deposits along the left contour of heart on
radiograph.
Explanation:
The patient's symptoms and physical examination findings are consistent with constrictive
pericarditis, a condition in which the pericardium (the sac surrounding the heart) becomes
thickened and restricts the heart's ability to expand during diastole. The thin layer of calcium
deposits along the left contour of the heart on radiograph is also suggestive of this diagnosis.
Treatment for constrictive pericarditis typically involves surgical removal of the pericardium, also
known as pericardiectomy. This is the most appropriate treatment recommendation for this
patient.
Other Options:
B. Digitalis preparations, such as digoxin, are not typically used as first-line treatment for
constrictive pericarditis, as they do not address the underlying pathology.
C. Anti-TB drugs would be indicated if the patient had active tuberculosis, but do not address
the underlying condition of constrictive pericarditis.
D. Diuretics may be useful for managing the patient's symptoms of leg swelling and ascites, but
do not address the underlying pathology of constrictive pericarditis.
E. Vasodilators and nitrates may be useful in treating heart failure with preserved ejection
fraction, but are not typically used as first-line treatment for constrictive pericarditis.
Making Other Options Right:
B. To make digitalis preparations the correct answer, the question would need to specify that
the patient has atrial fibrillation or another arrhythmia that requires treatment with these drugs.
C. To make anti-TB drugs the correct answer, the question would need to specify that the
patient has active tuberculosis that is contributing to their symptoms.
D. To make diuretics the correct answer, the question would need to specify that the patient's
symptoms of leg swelling and ascites are the primary concern, rather than the underlying
pathology of constrictive pericarditis.
E. To make vasodilators and nitrates the correct answer, the question would need to specify that
the patient has heart failure with preserved ejection fraction rather than constrictive pericarditis.
Medication:
cardiology

• Treatment for constrictive pericarditis typically involves surgical removal of the pericardium
(pericardiectomy).
Words:
• Constrictive pericarditis: a condition in which the pericardium becomes thickened and restricts
the heart's ability to expand during diastole.
• Ps: pulse rate.
• Ascites: accumulation of fluid in the abdomen.
Findings:
• Normal heart borders: the borders of the heart on chest radiograph, which should be within
normal limits.
• Low voltage on ECG: a finding that may be indicative of pericardial effusion or pericarditis.

Therapy
cardiology

67 Pericarditis

A 60-year-old patient has been admitted to a hospital with complaints of dyspnea, tightness
in the right subcostal area, abdomen enlargement. These presentations have been
progressing for a year. Heart auscultation reveals presystolic gallop rhythm. Objectively:
swelling of the neck veins, ascites, palpable liver and spleen. What disease requires
differential diagnostics?
A Constrictive pericarditis
B Hepatocirrhosis
C Lung cancer with invasion to the pleura
D Chronic pulmonary heart
E Pulmonary embolism

Correct Answer: A Constrictive pericarditis - Constrictive pericarditis is the disease that requires
differential diagnosis in this case. The symptoms of dyspnea, tightness in the right subcostal
area, abdomen enlargement, and presystolic gallop rhythm, along with the objective findings of
swelling of the neck veins, ascites, and palpable liver and spleen, are consistent with
constrictive pericarditis.
Key Points:
• 60-year-old patient with dyspnea, tightness in the right subcostal area, and abdomen
enlargement
• Symptoms have been progressing for a year

Therapy
• Presystolic gallop rhythm on heart auscultation
• Swelling of the neck veins, ascites, palpable liver and spleen
• Disease requiring differential diagnosis
Explanation:
Constrictive pericarditis is a condition in which the pericardium, the sac that surrounds the
heart, becomes thickened and rigid, leading to impaired heart function. It is a rare condition
that can be caused by a variety of factors, including infections, autoimmune disorders, and
previous cardiac surgery.
In this case, the patient presents with symptoms of dyspnea, tightness in the right subcostal
area, and abdomen enlargement that have been progressing for a year. Heart auscultation
reveals a presystolic gallop rhythm, which is a sign of impaired diastolic function. Objective
findings include swelling of the neck veins, ascites, and palpable liver and spleen, which are
consistent with constrictive pericarditis.
Differential diagnosis for constrictive pericarditis includes other conditions that can cause similar
symptoms and objective findings. Hepatocirrhosis (option B) can cause liver enlargement and
ascites, but would not explain the other symptoms or the presystolic gallop rhythm. Lung cancer
with invasion to the pleura (option C) may cause dyspnea and chest pain, but would not explain
the other symptoms or the presystolic gallop rhythm. Chronic pulmonary heart (option D) can
cause dyspnea and heart failure symptoms, but would not explain the other symptoms or the
presystolic gallop rhythm. Pulmonary embolism (option E) can cause dyspnea, but would not
explain the other symptoms or the presystolic gallop rhythm.
Other Options:
B Hepatocirrhosis - Hepatocirrhosis can cause liver enlargement and ascites, but would not
explain the other symptoms or the presystolic gallop rhythm.
C Lung cancer with invasion to the pleura - Lung cancer with invasion to the pleura may cause
dyspnea and chest pain, but would not explain the other symptoms or the presystolic gallop
rhythm.
D Chronic pulmonary heart - Chronic pulmonary heart can cause dyspnea and heart failure
symptoms, but would not explain the other symptoms or the presystolic gallop rhythm.
E Pulmonary embolism - Pulmonary embolism can cause dyspnea, but would not explain the
other symptoms or the presystolic gallop rhythm.
cardiology

Making Other Options Right:


B Hepatocirrhosis - This option could be made right if the patient had a history of alcohol abuse
or hepatitis, which could lead to liver cirrhosis.
C Lung cancer with invasion to the pleura - This option could be made right if the patient had a
history of smoking or exposure to carcinogens and had evidence of lung cancer on imaging
studies.
D Chronic pulmonary heart - This option could be made right if the patient had a history of
chronic lung disease such as COPD or pulmonary fibrosis and evidence of pulmonary
hypertension on echocardiogram.
E Pulmonary embolism - This option could be made right if the patient had a history of recent
surgery, cancer, or immobilization, and had evidence of pulmonary embolism on imaging
studies.
Medication:
No medication is mentioned in the question.
Words:
• Constrictive pericarditis: a condition in which the pericardium becomes thickened and rigid,
leading to impaired heart function
• Presystolic gallop rhythm: an abnormal heart sound heard on auscultation that indicates
impaired diastolic function
• Ascites: accumulation of fluid in the peritoneal cavity
• COPD: chronic obstructive pulmonary disease, a chronic lung disease characterized by airflow
obstruction
• Echocardiogram: an imaging test that uses sound waves to create pictures of the heart
Findings:
• No lab findings are mentioned in the question.

Therapy
cardiology

68 Pericarditis

A 50 year old woman complains about dull cardiac pain, asphyxia, body temperature rise
up to 38oC. She had influenza a week ago. Objectively: Ps - 100 bpm, dropped-beat
pulse during inspiration. AP - 100/70 mm Hg, heart sounds are muffled. ECG: reduced
voltage, ST segment is above the isoline in all leads. X-ray picture shows extensively
enlarged cardiac silhouette. Palmus is of small amplitude. What is the most probable
diagnosis?
A Exudative pericarditis
B Myocardium infarction
C Dilatation cardiomyopathy
D Myocarditis
E Stenocardia

Correct Answer: Option A, Exudative pericarditis, is the most probable diagnosis.


Key Points:
• A 50-year-old woman complains of dull cardiac pain, asphyxia, and body temperature rise up
to 38oC, one week after having influenza.
• On examination, the patient has a dropped-beat pulse during inspiration, muffled heart
sounds, and a small-amplitude palmus.
• The electrocardiogram (ECG) shows reduced voltage and ST segment elevation in all leads.
• The X-ray shows an extensively enlarged cardiac silhouette.

Therapy
• The question asks for the most probable diagnosis.
Explanation:
• Pericarditis is an inflammation of the pericardium, the double-layered sac that surrounds the
heart.
• Exudative pericarditis is a type of pericarditis in which there is an accumulation of fluid in the
pericardial sac.
• Symptoms of pericarditis may include chest pain, shortness of breath, and fever, and may
occur after a viral infection, such as influenza.
• Physical examination findings in pericarditis may include muffled heart sounds, a small-
amplitude pulse, and a dropped-beat pulse during inspiration, known as pulsus paradoxus.
• Electrocardiogram (ECG) findings in pericarditis may include reduced voltage and ST segment
elevation in all leads.
• X-ray findings in pericarditis may include an enlarged cardiac silhouette due to the
accumulation of fluid in the pericardial sac.
Other Options:
• Option B, Myocardial infarction, is incorrect because the symptoms and physical examination
findings are not typical of this condition. Myocardial infarction, also known as a heart attack, is
characterized by severe chest pain, diaphoresis, and ST segment elevation in the affected leads
on ECG.
• Option C, Dilated cardiomyopathy, is incorrect because the symptoms and physical
examination findings are not typical of this condition. Dilated cardiomyopathy is characterized
by symptoms of heart failure, such as dyspnea, orthopnea, and peripheral edema, and may be
associated with an enlarged cardiac silhouette on X-ray.
• Option D, Myocarditis, is incorrect because the symptoms and physical examination findings
are not typical of this condition. Myocarditis is characterized by symptoms of heart failure, such
as dyspnea, orthopnea, and peripheral edema, and may be associated with ST segment changes
on ECG and elevated cardiac enzymes.
• Option E, Stenocardia, is incorrect because the symptoms and physical examination findings
are not typical of this condition. Stenocardia, also known as angina pectoris, is characterized by
chest pain or discomfort that is typically triggered by physical or emotional stress and relieved
by rest or nitroglycerin.
cardiology

Making Other Options Right:


• Option B would be correct if the question described symptoms of myocardial infarction, such
as severe chest pain, diaphoresis, and ST segment elevation in the affected leads on ECG.
• Option C would be correct if the question described symptoms of dilated cardiomyopathy, such
as dyspnea, orthopnea, and peripheral edema, and the X-ray showed an enlarged cardiac
silhouette.
• Option D would be correct if the question described symptoms of myocarditis, such as
dyspnea, orthopnea, and peripheral edema, and laboratory findings showed elevated cardiac
enzymes.
• Option E would be correct if the question described symptoms of stenocardia, such as chest
pain or discomfort that is typically triggered by physical or emotional stress and relieved by rest
or nitroglycerin.
Treatment:
• The treatment of pericarditis depends on the underlying cause and may include nonsteroidal
anti-inflammatory drugs (NSAIDs), colchicine, and corticosteroids.
• In severe cases of exudative pericarditis with significant fluid accumulation, pericardiocentesis,
or surgical drainage of the pericardial fluid, may be necessary.
Words:
• Pericarditis: Inflammation of the pericardium, the double-layered sac that surrounds the heart.
• Exudative: Referring to fluid that has leaked out of blood vessels into surrounding tissues or
body cavities.
• Palmus: A strong, bounding pulse.
Findings:
• The patient has a dropped-beat pulse during inspiration, which is known as pulsus paradoxus
and is a characteristic finding in pericarditis.
• Heart sounds are muffled, which may suggest the presence of fluid in the pericardial sac.

Therapy
• The electrocardiogram (ECG) shows reduced voltage and ST segment elevation in all leads,
which are characteristic findings in pericarditis.
• The X-ray shows an extensively enlarged cardiac silhouette, which may suggest the
accumulation of fluid in the pericardial sac.
• The palmus is of small amplitude, which may indicate reduced cardiac output.
• The patient complains of dull cardiac pain, asphyxia, and fever, which are common symptoms
of pericarditis.
cardiology

69 Pericarditis

After a long periode of subfebrility a patient registered increase of dyspnea, pain in the
right hypochondrium, leg edemata. Objectively: neck veins are edematic. Ps is 120 bpm,
sometimes it disappears during inspiration. Heart sounds are very weakened. ECG showed
low-voltage waves of ventricular complex. A month ago there was raise of ST V1-V4
segment. Cardiac silhouette is enlarged, roundish. What is the most probable diagnosis?
A Exudative pericarditis
B Small-focal myocardial infarction
C Postinfarction cardiosclerosis
D Metabolic postinfection myocardiopathy
E Primary rheumatic carditis

Correct Answer: A) Exudative pericarditis.


Explanation:
• The patient's symptoms and objective findings suggest acute pericarditis, which is
inflammation of the pericardium, the sac surrounding the heart.
• The history of subfebrility and recent ST segment elevation on ECG are consistent with an
inflammatory process affecting the heart.
• The presence of neck vein distension, tachycardia, weakened heart sounds, and an enlarged
cardiac silhouette are all suggestive of acute pericarditis with pericardial effusion, which is the
accumulation of fluid within the pericardial sac.

Therapy
• Exudative pericarditis is a type of acute pericarditis characterized by the accumulation of
inflammatory fluid within the pericardial sac.
• This condition can cause symptoms of heart failure, such as dyspnea, leg edema, and
hepatomegaly.
Other options:
B) Small-focal myocardial infarction: The patient's symptoms and objective findings are not
consistent with myocardial infarction, which typically presents with chest pain and ECG changes
indicative of myocardial ischemia or injury.
C) Postinfarction cardiosclerosis: The patient has no history of myocardial infarction and the
objective findings are not consistent with postinfarction cardiosclerosis.
D) Metabolic postinfection myocardiopathy: The patient's symptoms and objective findings are
not consistent with metabolic postinfection myocardiopathy, which typically presents with a
history of recent infection and more diffuse involvement of the heart muscle.
E) Primary rheumatic carditis: The patient's symptoms and objective findings are not consistent
with primary rheumatic carditis, which typically presents with a history of streptococcal infection
and specific clinical and laboratory criteria for the diagnosis.
Making other options wrong:
B) Small-focal myocardial infarction: The patient's symptoms and objective findings are not
consistent with myocardial infarction, which typically presents with chest pain and ECG changes
indicative of myocardial ischemia or injury.
C) Postinfarction cardiosclerosis: The patient has no history of myocardial infarction and the
objective findings are not consistent with postinfarction cardiosclerosis.
D) Metabolic postinfection myocardiopathy: The patient's symptoms and objective findings are
not consistent with metabolic postinfection myocardiopathy, which typically presents with a
history of recent infection and more diffuse involvement of the heart muscle.
E) Primary rheumatic carditis: The patient's symptoms and objective findings are not consistent
with primary rheumatic carditis, which typically presents with a history of streptococcal infection
and specific clinical and laboratory criteria for the diagnosis.
Words:
• Acute pericarditis: Inflammation of the pericardium, the sac surrounding the heart.
• Pericardial effusion: The accumulation of fluid within the pericardial sac.
cardiology

• Exudative pericarditis: A type of acute pericarditis characterized by the accumulation of


inflammatory fluid within the pericardial sac.
• Heart failure: A condition in which the heart is unable to pump blood effectively.
• Dyspnea: Difficulty breathing.
• Tachycardia: Abnormally fast heart rate.
• Hepatomegaly: Enlargement of the liver.
• Electrocardiogram (ECG): A test that records the electrical activity of the heart.
• Myocardial infarction: Death of heart muscle tissue due to a blockage in a coronary artery.
• Cardiosclerosis: Scar tissue that forms in the heart muscle following injury or disease.
• Primary rheumatic carditis: A type of heart disease caused by rheumatic fever, which is a
complication of untreated streptococcal infections.
Findings:
• The patient presents with a long period of subfebrility, followed by dyspnea, right
hypochondrial pain, and leg edema, suggestive of heart failure.
• Objective findings include edematous neck veins, tachycardia, weakened heart sounds, low-
voltage ventricular complex on ECG, and an enlarged, round cardiac silhouette.
• The ECG also shows ST segment elevation in V1-V4 from a month ago.

Therapy
cardiology

70 Prevention

A healthy 75 year old woman who leads a moderately active way of life went through a
preventive examination that revealed serum concentration of common cholesterol at the
rate of 5,1 millimole/l and HDL (high-density lipoproteins) cholesterol at the rate of 70
mg/dl. ECG reveals no pathology. What dietary recommendation is the most adequate?
A Any dietary changes are necessary
B Decrease of cholesterol consumption
C Decrease of saturated fats consumption
D Decrease of carbohydrates consumption
E Increase of cellulose consumption

Correct Answer: A. Any dietary changes are necessary.


Key Points:
• The patient is a 75-year-old woman with moderately active lifestyle and normal ECG.
• The serum cholesterol level is within the normal range, and the HDL level is high.
• No specific dietary changes are necessary, but a healthy and balanced diet should be
encouraged.
Explanation:
• Elevated levels of serum cholesterol, particularly LDL (low-density lipoprotein) cholesterol, are
a major risk factor for cardiovascular disease.
• HDL cholesterol, on the other hand, is considered "good" cholesterol as it helps to remove

Therapy
excess cholesterol from the bloodstream and reduce the risk of heart disease.
• In this case, the patient's serum cholesterol level is within the normal range, and the HDL
level is high, indicating a low risk of cardiovascular disease.
• While no specific dietary changes are necessary, a healthy and balanced diet should be
encouraged to maintain optimal cardiovascular health.
• A healthy diet should include a variety of fruits, vegetables, whole grains, lean proteins, and
healthy fats, and should limit intake of processed and high-fat foods.
Other Options:
• Option B (Decrease of cholesterol consumption) is not necessary as the patient's serum
cholesterol level is already within the normal range.
• Option C (Decrease of saturated fats consumption) may be beneficial for patients with high
LDL cholesterol levels, but is not necessary for this patient with normal serum cholesterol levels.
• Option D (Decrease of carbohydrates consumption) may be beneficial for patients with insulin
resistance or diabetes, but is not necessary in this case.
• Option E (Increase of cellulose consumption) may be beneficial for overall health and
gastrointestinal function, but is not necessary for cholesterol management in this patient.
Making Other Options Wrong:
• Options B, C, D, and E are not necessary or appropriate for this patient with normal serum
cholesterol levels and high HDL cholesterol levels.
Medication:
• In some cases, medications such as statins or other cholesterol-lowering drugs may be
recommended to manage high LDL cholesterol levels.
• However, in this case, medication is not necessary as the patient's serum cholesterol levels
are within the normal range.
Words:
• Cardiovascular disease: A group of disorders that affect the heart and blood vessels, including
coronary artery disease, heart failure, and stroke.
• Serum cholesterol: The level of cholesterol in the bloodstream, which can be measured by a
blood test.
cardiology

• HDL cholesterol: High-density lipoprotein cholesterol, commonly known as "good" cholesterol,


which helps to remove excess cholesterol from the bloodstream and reduce the risk of heart
disease.
• LDL cholesterol: Low-density lipoprotein cholesterol, commonly known as "bad" cholesterol,
which can accumulate in the arteries and increase the risk of heart disease.
• Statins: A class of medications used to lower LDL cholesterol levels and reduce the risk of
cardiovascular disease.
Findings:
• Maintaining normal serum cholesterol levels and high HDL cholesterol levels is important for
cardiovascular health.
• A healthy and balanced diet, including a variety of fruits, vegetables, whole grains, lean
proteins, and healthy fats, is recommended for optimal cardiovascular health.
• Dietary changes may be recommended for patients with high LDL cholesterol levels, but are
not necessary for this patient with normal serum cholesterol levels and high HDL cholesterol
levels.

Therapy
cardiology

71 Pulmonary Edema

A 58 y.o. man complaines of severe inspiratory dyspnea and expectoration of frothy and
blood-tinged sputum. He has been suffering from essential hypertension and ischemic
heart disease. On examination: acrocyanosis, "bubbling" breathing, Ps- 30/min, BP230/130 mm
Hg, bilateral rales. Choose medicines for treatment.
A Morphine, furosemide, nitroprusside sodium
B Theophylline, prednisolon
C Albuterol, atropine, papaverine
D Strophanthine, potassium chloride, plathyphylline
E Cordiamine, isoproterenol

Correct Answer: A Morphine, furosemide, nitroprusside sodium


Key Points: A 58-year-old man with a history of essential hypertension and ischemic heart
disease presents with severe inspiratory dyspnea, expectoration of frothy and blood-tinged
sputum, acrocyanosis, "bubbling" breathing, Ps- 30/min, BP- 230/130 mm Hg, bilateral rales.
Explanation: The patient is exhibiting signs of acute pulmonary edema, a condition where
excess fluid accumulates in the lungs due to impaired pumping function of the heart. Morphine
is an effective treatment for reducing anxiety and dilating the blood vessels in the lungs, making
it easier for the patient to breathe. Furosemide is a loop diuretic that helps to eliminate excess
fluid from the body, including the lungs. Nitroprusside sodium is a vasodilator that can reduce
blood pressure and improve blood flow to the lungs. These three drugs are often used in

Therapy
combination for the management of acute pulmonary edema.
Other Options:
B. Theophylline and prednisolone are not appropriate for the treatment of acute pulmonary
edema. Theophylline is a bronchodilator that is used for asthma and chronic obstructive
pulmonary disease, and prednisolone is a steroid used to treat inflammation.
C. Albuterol is a bronchodilator that can help to relieve shortness of breath, but it is not a first-
line treatment for pulmonary edema. Atropine is used to treat bradycardia, and papaverine is a
vasodilator that is used to treat vascular spasms.
D. Strophanthin is a cardiac glycoside that is used to treat heart failure. Potassium chloride is a
supplement that is used to correct low potassium levels. Plathyphylline is a bronchodilator that
can help to relieve shortness of breath, but it is not a first-line treatment for pulmonary edema.
E. Cordiamine is a medication used to treat low blood pressure, and isoproterenol is a
bronchodilator used for asthma and chronic obstructive pulmonary disease. Neither of these
medications are appropriate for the treatment of acute pulmonary edema.
Medication:
• Morphine is an opioid analgesic that works by binding to opioid receptors in the brain and
spinal cord, reducing anxiety, and causing vasodilation.
• Furosemide is a loop diuretic that works by inhibiting the reabsorption of sodium and chloride
in the kidneys, resulting in increased urine output and reduced fluid volume.
• Nitroprusside sodium is a vasodilator that works by releasing nitric oxide in the smooth muscle
of blood vessels, causing relaxation and dilation of the blood vessels.
Words:
• Essential hypertension: High blood pressure that does not have a known cause.
• Ischemic heart disease: A condition where there is reduced blood flow to the heart due to
blockages in the coronary arteries.
Findings:
• Ps- 30/min: The patient's heart rate is 30 beats per minute.
• BP- 230/130 mm Hg: The patient's blood pressure is 230/130 millimeters of mercury, which is
very high.
• Bilateral rales: Abnormal lung sounds heard on auscultation, indicating fluid accumulation in
the lungs.
cardiology

72 pulmonary embolism

A 55-year-old male had been treated at the surgical department for acute lower-extremity
thrombophlebitis. On the 7th day of treatment he suddenly developed pain in the left part of
chest, dyspnea and cough. Body temperature was $36,1^oC$, respiratory rate - 36/min. The
patient was also found to have diminished breath sounds without wheezing. Ps- 140/min,
thready. AP- 70/50 mm Hg. The ECG shows QІІІ-S1 syndrome. What is the most likely
diagnosis?
A Pulmonary embolism
B Myocardial infarction
C Cardiac asthma
D Bronchial asthma
E Pneumothorax

Correct Answer: A Pulmonary embolism - This is the most likely diagnosis.


Key Points:
• A 55-year-old male had been treated for acute lower-extremity thrombophlebitis.
• On the 7th day of treatment, he suddenly developed chest pain, dyspnea, and cough.
• Physical examination reveals diminished breath sounds without wheezing, tachycardia, and
hypotension.
• The ECG shows QІІІ-S1 syndrome.
• The question asks for the most likely diagnosis.

Therapy
Explanation:
The patient's history of acute lower-extremity thrombophlebitis puts him at increased risk for
developing a pulmonary embolism (PE), which is a potentially life-threatening condition caused
by the obstruction of a pulmonary artery by a blood clot that has traveled from another part of
the body, usually the legs. The sudden onset of chest pain, dyspnea, and cough, along with
tachycardia, hypotension, and diminished breath sounds, are all consistent with a diagnosis of
PE.
The ECG finding of QІІІ-S1 syndrome, also known as S1QІІІT3 pattern, is a specific but not
sensitive finding for PE. It is characterized by a deep S wave in lead I and a deep Q wave in lead
III, along with an inverted T wave in lead III. While this pattern is not present in all cases of PE,
its presence in this patient supports the diagnosis.
Other Options:
• B Myocardial infarction: Myocardial infarction (MI) is a blockage of blood flow to the heart
muscle, which can cause chest pain, dyspnea, and cough. However, the patient's history of
lower-extremity thrombophlebitis and the presence of QІІІ-S1 syndrome on ECG make PE a
more likely diagnosis.
• C Cardiac asthma: Cardiac asthma is a condition in which heart failure causes respiratory
symptoms such as dyspnea and cough. However, the patient's hypotension and diminished
breath sounds without wheezing make PE a more likely diagnosis.
• D Bronchial asthma: Bronchial asthma is a chronic respiratory disease characterized by
recurrent episodes of wheezing, dyspnea, and cough. The absence of wheezing and the sudden
onset of symptoms make PE a more likely diagnosis.
• E Pneumothorax: Pneumothorax is a condition in which air leaks into the pleural cavity,
causing lung collapse and respiratory symptoms such as dyspnea and chest pain. However, the
absence of lung sounds and the sudden onset of symptoms make PE a more likely diagnosis.
Making Other Options Right:
• B Myocardial infarction: A history of coronary artery disease and the presence of typical ECG
changes, such as ST-segment elevation or depression, would support a diagnosis of MI.
• C Cardiac asthma: Symptoms of cardiac asthma are often worse at night and may be relieved
by sitting up or standing. The presence of rales or wheezing on auscultation would support a
diagnosis of cardiac asthma.
cardiology

• D Bronchial asthma: A history of recurrent episodes of wheezing and the presence of wheezing
on auscultation would support a diagnosis of bronchial asthma.
• E Pneumothorax: A chest X-ray showing a collapsed lung and a history of trauma or
underlying lung disease would support a diagnosis of pneumothorax.
Medication:
• The question does not provide any information about medications or treatments for the
patient's condition.
Words:
• Thrombophlebitis: inflammation of a vein with the formation of a blood clot.
• Pulmonary embolism: obstruction of a pulmonary artery by a blood clot.
• Q$_ІІІ$-S$_1$ syndrome: a specific ECG finding characterized by a deep S wave in lead I and
a deep Q wave in lead III, along with an inverted T wave in lead III.
Findings:
• Body temperature is 36,1^oC$.
• Respiratory rate is 36/min.
• Ps is 140/min, thready.
• AP is 70/50 mm Hg.
• The patient has tachycardia and hypotension.
• Physical examination reveals diminished breath sounds without wheezing.
• The ECG shows QІІІ-S1 syndrome.

Therapy
cardiology

73 pulmonary embolism

A 45-year-old man was brought to clinic with complaints of the pain that started suddenly in
the left chest part and epigastric area, shortness of breath, nausea, one-time vomiting.
The acute pain started after weight-lifting. On physical exam: shallow breathing, RR -
38/min, left chest part is behind during respiration, by percussion - tympanitic sound,
respiration is not ausculated. Ps - 110 bpm, of weak filling. BP - 100/60 mm Hg, insignificant
displacement of heart to the right, sounds are dull. What examination is the most expedient to
do
first?
A Roentgenography
B Electrocardiography
C Bronchoscopy
D Esophagogastroscopy
E Ultrasound of the abdominal cavit

Correct Answer: A
Key Points:
• 45-year-old man with sudden onset of chest pain and shortness of breath after weight-lifting.
• Shallow breathing, left chest part behind during respiration, tympanitic sound on percussion,
and absent breath sounds on auscultation.
• Weak pulse, low blood pressure, and dull heart sounds.

Therapy
Explanation:
The most expedient examination to perform in this patient is roentgenography, also known as
chest X-ray. The patient's clinical presentation is consistent with pulmonary embolism, which is
a potentially life-threatening condition that occurs when a blood clot travels to the lungs and
obstructs blood flow.
The chest X-ray can help confirm the diagnosis of pulmonary embolism by demonstrating a
wedge-shaped area of decreased density in the lung tissue, known as a Hampton's hump. This
finding is not specific to pulmonary embolism, but it can be suggestive of this condition in the
appropriate clinical context.
Electrocardiography (ECG) is a useful diagnostic tool in patients with chest pain, but it may not
be specific for pulmonary embolism. ECG changes in pulmonary embolism can include sinus
tachycardia, right ventricular strain pattern, and ST-segment changes, but these findings are
not always present.
Bronchoscopy and esophagogastroscopy are invasive procedures that are not indicated in the
initial evaluation of a patient with suspected pulmonary embolism.
Ultrasound of the abdominal cavity is not relevant to the diagnosis of pulmonary embolism.
Other Options:
B. Electrocardiography is a useful diagnostic tool in patients with chest pain, but it may not be
specific for pulmonary embolism.
C. Bronchoscopy is an invasive procedure that is not indicated in the initial evaluation of a
patient with suspected pulmonary embolism.
D. Esophagogastroscopy is an invasive procedure that is not indicated in the initial evaluation of
a patient with suspected pulmonary embolism.
E. Ultrasound of the abdominal cavity is not relevant to the diagnosis of pulmonary embolism.
Making Other Options Right:
B. Electrocardiography could be useful in detecting cardiac involvement in pulmonary embolism,
but it may not be specific for this condition.
C. Bronchoscopy could be useful in ruling out other causes of chest pain and shortness of
breath, such as airway obstruction, but it is not indicated in the initial evaluation of a patient
with suspected pulmonary embolism.
cardiology

D. Esophagogastroscopy could be useful in ruling out other causes of chest pain and nausea,
such as gastroesophageal reflux disease or peptic ulcer disease, but it is not indicated in the
initial evaluation of a patient with suspected pulmonary embolism.
E. Ultrasound of the abdominal cavity is not relevant to the diagnosis of pulmonary embolism.
Medication:
The treatment of pulmonary embolism includes anticoagulation therapy with heparin, followed
by oral anticoagulants such as warfarin or direct oral anticoagulants. Thrombolytic therapy may
be considered in patients with massive pulmonary embolism and hemodynamic instability. The
medication options given in the question are not relevant to the initial management of
pulmonary embolism.
Words:
Pulmonary embolism - a potentially life-threatening condition that occurs when a blood clot
travels to the lungs and obstructs blood flow.
Hampton's hump - a wedge-shaped area of decreased density in the lung tissue, suggestive of
pulmonary embolism.
Sinus tachycardia - a fast heart rate that originates from the sinus node.
Right ventricular strain pattern - a pattern of ECG changes that reflects the strain on the right
ventricle in response to increased pulmonary artery pressure.
ST-segment changes - abnormal changes in the ST segment of the ECG, which can indicate
myocardial ischemia or injury.
Thrombolytic therapy - the administration of drugs that dissolve blood clots.
Findings:
The patient's clinical presentation is consistent with pulmonary embolism, which is a potentially
life-threatening condition that requires prompt intervention. The most expedient examination to
perform in this patient is roentgenography, which can help confirm the diagnosis of pulmonary
embolism by demonstrating a Hampton's hump. Initial management should focus on

Therapy
anticoagulation therapy with heparin, followed by oral anticoagulants such as warfarin or direct
oral anticoagulants. Thrombolytic therapy may be considered in patients with massive
pulmonary embolism and hemodynamic instability.
cardiology

74 Rheumatic Fever

A 25 year old patient had pharyngitis 2 weeks ago. Now he complains about body
temperature rise up to 38oC, general weakness, dyspnea during walking, swelling and
shifting pain in the articulations. Objectively: cyanosis of lips, rhythmic pulse of poor volume
- 100 bpm. Left cardiac border deviates outwards from the mediaclavicular line by 1 cm.
The first heart sound is weakened on the apex, auscultation revealed systolic souffle. What
is the most probable aetiological factor that caused this pathological process?
A β-haemolytic streptococcus
B Staphylococcus
C Pneumococcus
D Virus
E Fung

Correct Answer: A β-hemolytic streptococcus


Key Points:
• 25-year-old patient with a history of pharyngitis two weeks ago
• Presents with fever, weakness, dyspnea, joint pain and swelling, and cardiac symptoms
• Cyanosis of lips, poor volume pulse at 100 bpm, left cardiac border deviates outwards by 1
cm, weakened S1 at the apex, and a systolic murmur
• Most probable etiological factor
Explanation:

Therapy
• The most probable etiological factor that caused this pathological process is β-hemolytic
streptococcus, option A.
• The patient's history of pharyngitis two weeks ago suggests a possible streptococcal infection,
which can lead to rheumatic fever.
• Rheumatic fever is a complication of untreated or inadequately treated streptococcal
pharyngitis, and is characterized by inflammatory lesions that can affect the joints, heart, skin,
and central nervous system.
• The symptoms of fever, weakness, dyspnea, joint pain and swelling, and cardiac symptoms
are all consistent with rheumatic fever.
• The presence of a systolic murmur and weakened S1 at the apex, as well as deviation of the
left cardiac border, suggest involvement of the heart, specifically mitral valve regurgitation.
• Rheumatic fever is caused by an autoimmune reaction to streptococcal antigens, rather than
direct infection with the bacteria.
Other Options:
• Option B, Staphylococcus, is not a common cause of rheumatic fever. Staphylococcal
infections can cause endocarditis, but this typically occurs in patients with preexisting heart
valve disease or intravenous drug use.
• Option C, Pneumococcus, is also not a common cause of rheumatic fever. Pneumococcal
infections can cause pneumonia, meningitis, and sepsis, but are not associated with rheumatic
fever.
• Option D, Virus, is not a cause of rheumatic fever. Viral infections can cause myocarditis, but
this typically presents with symptoms of heart failure rather than mitral valve regurgitation and
a systolic murmur.
• Option E, Fungus, is not a cause of rheumatic fever. Fungal infections can cause endocarditis,
but this is rare and typically occurs in immunocompromised patients.
Making Other Options Right:
• Options B, C, D, and E cannot be made correct in this case, as they are not associated with
rheumatic fever.
Treatment:
• The treatment for rheumatic fever involves antibiotics to eradicate the streptococcal infection,
as well as anti-inflammatory therapy to control the autoimmune reaction.
cardiology

• Antibiotics are typically given for 10 days, and may include penicillin, amoxicillin, or
erythromycin, depending on the patient's allergy status and other factors.
• Anti-inflammatory therapy may include nonsteroidal anti-inflammatory drugs (NSAIDs) or, in
severe cases, corticosteroids.
• Patients with cardiac involvement may require treatment with diuretics, angiotensin-
converting enzyme (ACE) inhibitors, or other medications to manage heart failure.
• Patients with severe mitral valve regurgitation may require surgical intervention, such as
mitral valve repair or replacement.
Findings:
• The patient has a history of pharyngitis two weeks ago, which suggests a possible
streptococcal infection.
• The symptoms of fever, weakness, dyspnea, joint pain and swelling, and cardiac symptoms
are all consistent with rheumatic fever.
• The presence of a systolic murmur and weakened S1 at the apex, as well as deviation of the
left cardiac border, suggest involvement of the heart, specifically mitral valve regurgitation.

Therapy
cardiology

75 Rheumatic Fever

A 18 y.o. male patient complains of pain in knee and ankle joints, temperature elevation to
39,50C. He had a respiratory disease 1,5 week ago. On examination: temperature38,50C,
swollen knee and ankle joints, pulse- 106 bpm, rhythmic, AP- 90/60 mm Hg,
heart borders without changes, sounds are weakened, soft systolic apical murmur. What
indicator is connected with possible etiology of the process?
A Antistreptolysine-0
B 1-antitrypsine
C Creatinkinase
D Rheumatic factor
E Seromucoid

Correct Answer: A. Antistreptolysin-O (ASO) is the indicator connected with the possible etiology
of the process.
Key Points:
• 18-year-old male with joint pain, fever, and a recent respiratory illness
• Swollen knee and ankle joints, soft systolic apical murmur on examination
• The question asks for the indicator connected with the possible etiology of the process
Explanation:
The presentation in this patient is consistent with rheumatic fever, which is a systemic
inflammatory disease that can occur after a streptococcal infection, such as pharyngitis.

Therapy
Rheumatic fever typically presents with fever, joint pain, and cardiac involvement, which can
manifest as murmurs, valve damage, or other cardiac complications.
The soft systolic apical murmur in this patient suggests mitral regurgitation, which is a common
complication of rheumatic fever. The presence of swollen knee and ankle joints further supports
the diagnosis of rheumatic fever, as joint involvement is a common feature.
The indicator connected with the possible etiology of the process is the antistreptolysin-O (ASO)
titer, which is a laboratory test that measures the level of antibodies against streptolysin-O, a
toxin produced by group A streptococcus. The ASO titer is elevated in patients with recent
streptococcal infections, and can be used to diagnose rheumatic fever.
Other Options:
• B. 1-antitrypsin is not directly related to the etiology of rheumatic fever.
• C. Creatinkinase (CK) is a nonspecific marker of muscle damage, and is not directly related to
the etiology of rheumatic fever.
• D. Rheumatic factor is an antibody that is associated with rheumatoid arthritis, and is not
directly related to the etiology of rheumatic fever.
• E. Seromucoid is an acute-phase protein that is elevated in response to inflammation, but is
not directly related to the etiology of rheumatic fever.
Making Other Options Right:
• B. 1-antitrypsin would be the correct answer if the question asked for a laboratory marker of
lung disease, as 1-antitrypsin deficiency is a genetic disorder that predisposes to emphysema.
• C. Creatinkinase (CK) would be the correct answer if the patient had evidence of muscle
damage or rhabdomyolysis, which can manifest as elevated CK levels in the blood.
• D. Rheumatic factor would be the correct answer if the patient had symptoms of rheumatoid
arthritis, such as joint pain and swelling, and positive rheumatoid factor in the blood.
• E. Seromucoid would be the correct answer if the patient had evidence of acute-phase
response, such as fever, elevated erythrocyte sedimentation rate (ESR) or C-reactive protein
(CRP), and elevated seromucoid levels in the blood.
Medication:
The treatment for rheumatic fever typically involves antibiotics to eradicate the streptococcal
infection, and anti-inflammatory medications to manage symptoms and prevent cardiac
cardiology

complications. In severe cases, hospitalization may be necessary for heart monitoring and
intravenous medications.
Words:
Rheumatic fever - a systemic inflammatory disease that can occur after a streptococcal
infection, and can affect the joints, heart, skin, and nervous system.
Findings:
• Normal pulse rate in adults is between 60-100 bpm.
• Mitral regurgitation is a condition in which the mitral valve does not close properly, allowing
blood to leak back into the left atrium, which can cause a heart murmur and other cardiac
symptoms.
• Antistreptolysin-O (ASO) titer is a laboratory test that measures the level of antibodies against
streptolysin-O, a toxin produced by group A streptococcus.

Therapy
cardiology

76 Rheumatic heart disease

Examination of a 35-year-old patient with rheumatism revealed that the right heart border was
1
cm displaced outwards from the right parasternal line, the upper border was on the level with
inferior margin of the 1st rib, the left border was 1 cm in from the left midclavicular line.
Auscultation revealed atrial fibrillation, loud apical first sound, diastolic shock above the
pulmonary artery. Echocardiocopy revealed abnormal pattern of the mitral valve motion. What
heart disease is characterized by these symptoms?
A Mitral stenosis
B Mitral valve prolapse
C Mitral valve insufficiency
D Aortic stenosis
E Tricuspid valve insufficienc

Correct Answer: The heart disease characterized by these symptoms is Mitral Stenosis.
Key Points:
• 35-year-old patient with rheumatism.
• Displaced right heart border, upper border on the level with inferior margin of the 1st rib, and
left border 1 cm in from the left midclavicular line.
• Auscultation revealed atrial fibrillation, loud apical first sound, diastolic shock above the
pulmonary artery.

Therapy
• Echocardiography revealed an abnormal pattern of the mitral valve motion.
Explanation:
Rheumatic heart disease is a complication of rheumatic fever that can occur several years after
an untreated streptococcal infection. It can lead to valvular heart disease, including mitral
stenosis, which is narrowing of the mitral valve opening that impedes blood flow from the left
atrium to the left ventricle.
The displacement of the right heart border, upper border on the level with inferior margin of the
1st rib, and left border 1 cm in from the left midclavicular line are consistent with the physical
findings of mitral stenosis. The loud apical first sound and diastolic shock above the pulmonary
artery are also typical findings in mitral stenosis. Atrial fibrillation is a common arrhythmia in
patients with mitral stenosis due to the enlargement of the left atrium.
Echocardiography is a non-invasive imaging technique that can be used to diagnose various
heart diseases, including mitral stenosis. The abnormal pattern of the mitral valve motion seen
on echocardiography is consistent with the diagnosis of mitral stenosis.
Other Options:
• Option B: Mitral valve prolapse is a condition in which the mitral valve leaflets bulge back into
the left atrium during systole. It is not consistent with the physical findings of displaced heart
border and diastolic shock above the pulmonary artery.
• Option C: Mitral valve insufficiency, also known as mitral regurgitation, is a condition in which
the mitral valve does not close properly, allowing blood to flow back into the left atrium during
systole. It is not consistent with the physical findings of displaced heart border and diastolic
shock above the pulmonary artery.
• Option D: Aortic stenosis is a condition in which the aortic valve opening is narrowed,
impeding blood flow from the left ventricle to the aorta. It is not consistent with the physical
findings of displaced heart border and diastolic shock above the pulmonary artery.
• Option E: Tricuspid valve insufficiency, also known as tricuspid regurgitation, is a condition in
which the tricuspid valve does not close properly, allowing blood to flow back into the right
atrium during systole. It is not consistent with the physical findings of displaced heart border
and diastolic shock above the pulmonary artery.
Making Other Options Right:
cardiology

• Option B: To make mitral valve prolapse the correct answer, the physical findings would need
to include a mid-systolic click and a late systolic murmur, without displaced heart border or
diastolic shock above the pulmonary artery.
• Option C: To make mitral valve insufficiency the correct answer, the physical findings would
need to include a holosystolic murmur radiating to the axilla, without displaced heart border or
diastolic shock above the pulmonary artery.
• Option D: To make aortic stenosis the correct answer, the physical findings would need to
include a systolic ejection murmur radiating to the carotids, without displaced heart border or
diastolic shock above the pulmonary artery.
• Option E: To make tricuspid valve insufficiency the correct answer, the physical findings would
need to include a holosystolic murmur heard best at the lower left sternal border, without
displaced heart border or diastolic shock above the pulmonary artery.
Medication:
• The treatment of mitral stenosis depends on the severity of the condition and the presence of
symptoms.
• In mild cases, no specific treatment may be necessary.
• In more severe cases, medications may be used to control symptoms, such as diuretics to
reduce fluid overload and beta-blockers to control the heart rate.
• Anticoagulants may also be used to prevent blood clots from forming in the left atrium due to
atrial fibrillation.
• In some cases, surgical intervention may be necessary, such as mitral valve repair or
replacement.
Words:
• Rheumatic fever: A complication of untreated streptococcal infection that can lead to various
manifestations, including rheumatic heart disease.
• Valvular heart disease: A condition in which one or more heart valves are damaged or not

Therapy
functioning properly.
• Mitral stenosis: A condition in which the mitral valve opening is narrowed, impeding blood flow
from the left atrium to the left ventricle.
• Atrial fibrillation: An arrhythmia in which the atria contract rapidly and irregularly, leading to
inefficient blood flow and an increased risk of blood clots.
• Diastolic shock: A palpable sensation felt over the pulmonary artery during diastole, indicating
increased pressure in the pulmonary circulation.
• Echocardiography: A non-invasive imaging technique that uses sound waves to visualize the
heart and its structures.
• Mitral valve leaflets: The flaps of tissue that make up the mitral valve and control the flow of
blood from the left atrium to the left ventricle.
• Mitral valve prolapse: A condition in which the mitral valve leaflets bulge back into the left
atrium during systole.
• Mitral valve insufficiency: A condition in which the mitral valve does not close properly,
allowing blood to flow back into the left atrium during systole.
• Aortic stenosis: A condition in which the aortic valve opening is narrowed, impeding blood flow
from the left ventricle to the aorta.
• Tricuspid valve insufficiency: A condition in which the tricuspid valve does not close properly,
allowing blood to flow back into the right atrium during systole.
Findings:
• Normal values for heart borders and positions vary based on age, sex, and body habitus.
• The normal range for heart rate is 60-100 beats per minute.
• The normal range for the mitral valve area is 4-6 cm2.
• The normal range for left atrial size is less than 4 cm in diameter.
cardiology

77 Rheumatic heart disease

A 40 y.o. patient with rheumatic heart disease complains of anorexia, weakness and loss of
weight, breathlessness and swelling of feet. The patient had tooth extraction one month
ago. On examination: t0
- 390C, Ps- 100/min. Auscultation: diastolic murmur in the
mitral area. Petechial lesion around the clavicle; spleen was palpable.
A Subacute bacteria endocarditis
B Recurrence of rheumatic fever
C Thrombocytopenia purpura
D Mitral stenosis
E Aortic stenosis

Correct Answer: A) Subacute bacterial endocarditis.


Key Points:
• Patient has a history of rheumatic heart disease.
• Complaints of anorexia, weakness, loss of weight, breathlessness, and swelling of feet.
• Recent tooth extraction one month ago.
• Physical exam shows diastolic murmur in the mitral area, petechial lesion around the clavicle,
and palpable spleen.
Explanation:
Subacute bacterial endocarditis is a bacterial infection of the endocardium, which is the inner

Therapy
lining of the heart chambers and valves. It typically occurs in patients with pre-existing heart
disease or valve abnormalities, as in this case with rheumatic heart disease. Dental procedures,
such as tooth extraction, can increase the risk of bacterial infection in the bloodstream and
subsequent infection of the heart valves.
The clinical presentation of subacute bacterial endocarditis includes nonspecific symptoms such
as anorexia, weakness, and weight loss, as well as more specific symptoms such as fever, new
murmur, and petechial lesions. The presence of a diastolic murmur in the mitral area suggests
involvement of the mitral valve, which is common in subacute bacterial endocarditis.
Other Options:
B) Recurrence of rheumatic fever is unlikely since the patient has a diastolic murmur in the
mitral area, which is not a typical feature of rheumatic fever.
C) Thrombocytopenic purpura is unlikely since the petechial lesion would be more widespread.
D) Mitral stenosis is a possible complication of rheumatic heart disease, but the diastolic
murmur and other findings suggest a bacterial infection.
E) Aortic stenosis is not consistent with the findings in this case.
Making Other Options Right:
B) Recurrence of rheumatic fever could be a possibility if the patient had fever and joint pain in
addition to the current findings.
C) Thrombocytopenic purpura could be a possibility if the petechial lesion was more widespread
and the platelet count was low.
Medication:
No medication is mentioned in the question.
Words:
Petechial lesion - a small, pinpoint skin rash that is red or purple in color and is caused by
bleeding under the skin.
Findings:
t0- 39°C (fever); Ps- 100/min (tachycardia); palpable spleen (splenomegaly). Normal values
are: t0- 36.5-37.5°C; Ps- 60-100/min. Palpable spleen can be a sign of a systemic disease or an
infection.
Rheumatic fever is an inflammatory condition that can develop after a group A streptococcal
infection, such as strep throat. It typically affects children between the ages of 5 and 15, and it
cardiology

can cause symptoms such as fever, joint pain, and a characteristic rash called erythema
marginatum. If left untreated, rheumatic fever can lead to damage of the heart valves and
result in rheumatic heart disease.
Rheumatic heart disease is a complication of rheumatic fever that affects the heart valves. It
can lead to stenosis (narrowing) or regurgitation (leakage) of the valves, which can cause
symptoms such as shortness of breath, fatigue, and swelling of the legs and feet. Rheumatic
heart disease can be prevented by promptly treating strep throat infections with antibiotics.

Therapy
cardiology

78 Shock

A 62-year-old male has been hospitalized in the intensive care unit with a continuous attack
of retrosternal pain that cannot be relieved by nitroglycerin. Objectively: AP- 80/60 mm Hg,
heart rate - 106/min, breathing rate - 22/min. Heart sounds are muffled, a gallop rhythm is
present. How would you explain the AP drop?
A Reduction in cardiac output
B Reduction in peripheral resistance
C Blood depositing in the abdominal cavity
D Adrenergic receptor block
E Internal haemorrhage

Correct Answer: A Reduction in cardiac output - The most likely cause of the drop in AP in this
scenario is a reduction in cardiac output.
Key Points:
• A 62-year-old male with retrosternal pain that cannot be relieved by nitroglycerin has been
hospitalized in the intensive care unit.
• Objectively, the patient has an AP of 80/60 mm Hg, heart rate of 106/min, breathing rate of
22/min, muffled heart sounds, and a gallop rhythm.
• The question asks for an explanation for the drop in AP.
Explanation:
• The patient's symptoms suggest that he is experiencing cardiogenic shock, which is a type of

Therapy
shock that occurs when the heart is unable to pump enough blood to meet the body's needs.
• The muffled heart sounds and gallop rhythm suggest that the patient may have fluid
accumulation in the pericardium, which is called cardiac tamponade.
• Cardiac tamponade can cause the heart to compress, reducing its ability to pump blood and
leading to a reduction in cardiac output.
• A reduction in cardiac output can cause a drop in AP, which is the pressure in the arteries
when the heart is contracting (systolic pressure) and when the heart is relaxed (diastolic
pressure).
• Nitroglycerin is a vasodilator that can help to relieve chest pain by decreasing cardiac
workload and increasing blood flow to the heart. However, in this case, the patient's symptoms
suggest that he may have a mechanical problem with the heart rather than a blockage in the
coronary arteries.
Other Options:
• B) Reduction in peripheral resistance - Peripheral resistance refers to the resistance to blood
flow in the blood vessels outside of the heart. While a reduction in peripheral resistance could
cause a drop in AP, it is unlikely to be the cause of the patient's symptoms in this case.
• C) Blood depositing in the abdominal cavity - This option suggests that the patient may be
experiencing abdominal bleeding, which could cause a drop in AP. However, there is no evidence
in the question to support this as a likely cause.
• D) Adrenergic receptor block - Adrenergic receptors are receptors that bind to adrenaline and
other catecholamines, which can increase heart rate and contractility. A block in these receptors
could reduce cardiac output and cause a drop in AP, but this is not a likely cause of the patient's
symptoms.
• E) Internal haemorrhage - Hemorrhage could cause a drop in AP, but there is no evidence in
the question to suggest that this is the cause of the patient's symptoms.
Medication:
• Nitroglycerin is a vasodilator that can help to relieve chest pain by decreasing cardiac
workload and increasing blood flow to the heart.
• Treatment for cardiac tamponade may involve pericardiocentesis, which is a procedure to
remove fluid from the pericardium.
cardiology

• Inotropic agents such as dobutamine or milrinone can be used to increase cardiac output and
improve blood pressure.
Words:
• Retrosternal pain - Pain behind the breastbone.
• AP - Arterial pressure, which is the pressure in the arteries when the heart is contracting
(systolic pressure) and when the heart is relaxed (diastolic pressure).
• Gallop rhythm - An abnormal heart rhythm that sounds like a galloping horse.
Findings:
• AP: 80/60 mm Hg (diastolic pressure is low)
• Heart rate: 106/min (high)
• Breathing rate: 22/min (high)
• Muffled heart sounds and gallop rhythm (suggestive of cardiac tamponade)

Therapy
cardiology

79 Stable Angina

A 47-year-old male patient has been lately complaining of compressing chest pain that occurs
when he walks a distane of 700-800 m. Once a week, he drinks 2 liters of beer. Rise in arterial
pressure has been observed for the last 7 years. Objectively: Ps - 74/min, AP - 120/80 mm Hg.
The bicycle ergometry performed at workload of 75 watts shows 2 mm ST-segment
depression in V4-V6 leads. What is the most likely diagnosis ?
A Exertional stenocardia, II functional class
B Exertional stenocardia, III functional class
C Exertional stenocardia, IV functional class
D Vegetative-vascular dystonia of hypertensive type
E Alcoholic cardiomyopathy

Correct Answer: A Exertional stenocardia, II functional class - correct.


Key Points:
• 47-year-old male patient with compressing chest pain that occurs with exertion, specifically
walking a distance of 700-800 m.
• History of alcohol use, specifically drinking 2 liters of beer once a week.
• History of hypertension for the past 7 years.
• Objective findings of normal heart rate and blood pressure, and ST-segment depression on
bicycle ergometry.
• Diagnosis is being asked for.

Therapy
Explanation:
The patient's symptoms and objective findings suggest the possibility of ischemic heart disease,
specifically stable angina. The chest pain that occurs with exertion and is relieved with rest is a
characteristic symptom of stable angina. The patient's history of alcohol use and hypertension
are also risk factors for the development of ischemic heart disease. The presence of ST-segment
depression on bicycle ergometry is a diagnostic finding consistent with myocardial ischemia.
Based on the patient's symptoms and objective findings, the most likely diagnosis is exertional
stenocardia, also known as stable angina, in the second functional class, which indicates that
the patient experiences symptoms with moderate exertion.
Other Options:
B. Exertional stenocardia in the third functional class indicates that the patient experiences
symptoms with mild exertion.
C. Exertional stenocardia in the fourth functional class indicates that the patient experiences
symptoms with minimal exertion or at rest.
D. Vegetative-vascular dystonia of hypertensive type is a condition characterized by autonomic
nervous system dysfunction and hypertension, but does not typically present with chest pain or
ST-segment changes on ECG.
E. Alcoholic cardiomyopathy is a condition characterized by heart muscle damage due to long-
term alcohol use, but does not typically present with chest pain or ST-segment changes on ECG.
Making Other Options Right:
B. To make exertional stenocardia in the third functional class the correct answer, the question
would need to specify that the patient experiences symptoms with mild exertion, rather than
moderate exertion.
C. To make exertional stenocardia in the fourth functional class the correct answer, the question
would need to specify that the patient experiences symptoms with minimal exertion or at rest,
rather than moderate exertion.
D. To make vegetative-vascular dystonia of hypertensive type the correct answer, the question
would need to specify additional symptoms that are consistent with this diagnosis, such as
dizziness, fainting, or headache.
cardiology

E. To make alcoholic cardiomyopathy the correct answer, the question would need to specify
additional symptoms that are consistent with this diagnosis, such as shortness of breath,
fatigue, or leg swelling.
Medication:
• The question does not provide information about the patient's current or past medication use.
Words:
• Exertional stenocardia: chest pain or discomfort that occurs with exertion due to reduced
blood flow to the heart muscle.
• Functional class: a grading system used to describe the severity of symptoms in patients with
cardiovascular disease, with class I indicating no symptoms and class IV indicating severe
symptoms.
• Hypertension: a condition in which there is persistently elevated blood pressure.
• ST-segment depression: a finding on electrocardiogram (ECG) that indicates myocardial
ischemia, or reduced blood flow to the heart muscle.
• Bicycle ergometry: a test in which the patient exercises on a stationary bicycle while
connected to an ECG machine to measure heart function.
Findings:
• Normal heart rate: 60-100 beats per minute.
• Normal blood pressure: <120/80 mmHg.
• Normal ST-segment on ECG: no ST-segment depression or elevation.
• Normal workload on bicycle ergometry: varies depending on patient age, sex, and physical
fitness level.
Functional class is a grading system used to describe the severity of symptoms in patients with
cardiovascular disease. The functional classification system is used to categorize patients into
different classes based on their symptoms, physical activities, and limitations. The most
commonly used functional classification system for cardiovascular disease is the New York Heart

Therapy
Association (NYHA) functional classification system.
The NYHA functional classification system consists of four classes, which are as follows:
1. Class I: No limitation of physical activity. Ordinary physical activity does not cause undue
fatigue, palpitation, or dyspnea (shortness of breath).
2. Class II: Slight limitation of physical activity. Comfortable at rest, but ordinary physical
activity results in fatigue, palpitation, or dyspnea.
3. Class III: Marked limitation of physical activity. Comfortable at rest, but less than ordinary
physical activity causes fatigue, palpitation, or dyspnea.
4. Class IV: Unable to carry out any physical activity without discomfort. Symptoms of heart
failure at rest. If any physical activity is undertaken, discomfort is increased.
cardiology

80 Stable Angina

A 56-year-old scientist experiences constricting retrosternal pain several times a day while
walking for 100-150 m. The pain lasts for up to 10 minutes and can be relieved by
nitroglycerine. Objectively: the patient is overweight, heart borders exhibit no abnormalities,
heart sounds are rhythmic, Ps- 78 bpm, AP- 130/80 mm Hg. ECG contains low amplitude
of $T$ wave in $V_{4-5}$. What disease might be suspected?
A Stable FC III stenocardia
B Instable stenocardia
C Stable FC I stenocardia
D Stable FC II stenocardia
E Stable FC IV stenocardia

Correct Answer: A Stable FC III stenocardia - Option A is the correct answer because the
patient's symptoms of constricting retrosternal pain that is relieved by nitroglycerin, along with
the ECG finding of low amplitude of T wave in V4-5, are consistent with stable angina, a type of
chest pain that occurs during physical activity or emotional stress and is caused by reduced
blood flow to the heart.
Key Points:
• 56-year-old scientist
• Constricting retrosternal pain several times a day while walking for 100-150 m
• Pain lasts up to 10 minutes and can be relieved by nitroglycerin

Therapy
• Overweight
• Heart borders exhibit no abnormalities
• Heart sounds are rhythmic, Ps- 78 bpm, AP- 130/80 mm Hg
• ECG contains low amplitude of T wave in V4-5
• Requires a suspected diagnosis
Explanation:
• Stable angina is a type of chest pain that occurs during physical activity or emotional stress
and is caused by reduced blood flow to the heart due to coronary artery disease.
• The patient's symptoms of constricting retrosternal pain that is relieved by nitroglycerin, along
with the ECG finding of low amplitude of T wave in V4-5, are consistent with stable angina.
• The patient's overweight status is also a risk factor for coronary artery disease.
• The absence of abnormalities in heart borders and heart sounds suggest no significant
structural or functional abnormalities in the heart.
• FC stands for functional class and is a term used to describe the severity of angina or heart
failure based on the patient's symptoms and limitations in daily activities. The functional class is
usually determined by the New York Heart Association (NYHA) or Canadian Cardiovascular
Society (CCS) classification systems. In the context of the given question, Stable angina FC III
refers to the third functional class of stable angina, which is characterized by chest pain or
discomfort that occurs with moderate exertion, such as walking for 100-150 meters.
Other Options:
• Unstable angina (Option B) is a type of chest pain that occurs at rest or with minimal exertion
and is considered a medical emergency, but the patient's symptoms of pain during physical
activity and relief with nitroglycerin suggest stable angina.
• Stable angina FC I (Option C) is a milder form of stable angina that occurs with strenuous or
prolonged exertion, but the patient's symptoms of pain with minimal exertion suggest stable
angina FC III.
• Stable angina FC II (Option D) is a more severe form of stable angina that occurs with
moderate exertion, but the patient's symptoms of pain with minimal exertion suggest stable
angina FC III.
cardiology

• Stable angina FC IV (Option E) is the most severe form of stable angina that occurs with
minimal or no exertion, but the patient's symptoms of pain with minimal exertion suggest stable
angina FC III.
Making Other Options Right:
• To make Option B (Unstable angina) right, the question could describe a patient with chest
pain that occurs at rest or with minimal exertion, with ECG findings of ST-segment depression or
transient ST-segment elevation.
• To make Option C (Stable angina FC I) right, the question could describe a patient with chest
pain that occurs with strenuous or prolonged exertion, but is relieved with rest or nitroglycerin.
• To make Option D (Stable angina FC II) right, the question could describe a patient with chest
pain that occurs with moderate exertion, but is relieved with rest or nitroglycerin.
• To make Option E (Stable angina FC IV) right, the question could describe a patient with chest
pain that occurs with minimal or no exertion, and is not relieved with rest or nitroglycerin.
Medication:
• Treatment for stable angina may include medications to reduce the workload on the heart and
improve blood flow, such as nitroglycerin, beta blockers, calcium channel blockers, and nitrates.
Words:
• Retrosternal pain: Pain located behind the sternum (breastbone).
• Nitroglycerin: A medication that relaxes the smooth muscle in blood vessels, dilating them and
increasing blood flow.
• T wave: A wave on the electrocardiogram (ECG) that represents ventricular repolarization.
Findings:
• Ps of 78 bpm is within the normal range of 60-100 bpm.
• AP of 130/80 mm Hg is within the normal range of 90/60-140/90 mm Hg.
• Low amplitude of T wave in V4-5 may be seen in patients with coronary artery disease or
myocardial ischemia.

Therapy
Functional Classes -
The New York Heart Association (NYHA) classification system is a commonly used system to
describe the functional class of heart failure. It categorizes heart failure into four classes based
on the severity of the patient's symptoms and limitations in daily activities. The four classes are:
Class I:
• Patients with no limitation of physical activity. Ordinary physical activity does not cause undue
fatigue, palpitation, or dyspnea (shortness of breath).
Class II:
• Patients with slight limitation of physical activity. They are comfortable at rest. Ordinary
physical activity results in fatigue, palpitation, or dyspnea.
Class III:
• Patients with marked limitation of physical activity. They are comfortable at rest. Less than
ordinary activity causes fatigue, palpitation, or dyspnea.
Class IV:
• Patients who are unable to carry out any physical activity without discomfort. Symptoms of
heart failure are present at rest, and physical activity increases discomfort.
cardiology

81 syncope

During the ultrasound study of carotid and vertebral arteries a 74-year-old patient developed a
condition manifested by dizziness, weakness, nausea, transient loss of consciousness.
Objectively: pale skin, AP- 80/60 mm Hg, Ps- 96/min of poor volume. ECG shows sinus
tachycardia, left ventricular hypertrophy. Focal neurological symptoms were not found. What is
the provisional diagnosis?
A Carotid sinus syncope
B Orthostatic syncope
C Morgagni-Adams-Stokes attack
D Complete atrioventricular block
E Acute cerebrovascular accident

Correct Answer: Carotid sinus syncope - The patient's symptoms and objective findings are
consistent with carotid sinus syncope, which is a condition characterized by a reflex response to
carotid sinus stimulation, leading to a sudden decrease in heart rate and blood pressure.
Key Points:
• Patient with symptoms of dizziness, weakness, nausea, and transient loss of consciousness
during an ultrasound study of carotid and vertebral arteries.
• Objective findings of pale skin, low blood pressure, and sinus tachycardia with left ventricular
hypertrophy.
• No focal neurological symptoms were found.

Therapy
Explanation:
Syncope is a sudden, transient loss of consciousness and postural tone due to decreased blood
flow to the brain. Carotid sinus syncope is a type of syncope caused by a reflex response to
carotid sinus stimulation, leading to a sudden decrease in heart rate and blood pressure. The
patient's symptoms and objective findings, including dizziness, weakness, nausea, and transient
loss of consciousness, are consistent with carotid sinus syncope.
Carotid sinus syncope is more common in older patients with underlying cardiac disease, such
as left ventricular hypertrophy. The patient's ECG findings of sinus tachycardia and left
ventricular hypertrophy support this diagnosis.
Other Options:
B) Orthostatic syncope: Orthostatic syncope is a type of syncope caused by a sudden decrease
in blood pressure upon standing up. The patient's objective findings do not suggest orthostatic
syncope, as the blood pressure is low in the supine position.
C) Morgagni-Adams-Stokes attack: Morgagni-Adams-Stokes attack is a type of syncope caused
by a sudden decrease in heart rate and blood pressure due to complete heart block. The
patient's ECG does not suggest complete heart block, as there is sinus tachycardia.
D) Complete atrioventricular block: Complete atrioventricular block is a type of heart block
characterized by a complete dissociation between the atria and the ventricles, leading to a slow
heart rate and decreased cardiac output. The patient's ECG does not suggest complete
atrioventricular block, as there is sinus tachycardia.
E) Acute cerebrovascular accident: Acute cerebrovascular accident, also known as stroke, is a
neurological emergency characterized by sudden onset of focal neurological symptoms due to a
disruption of blood flow to the brain. The patient's symptoms and objective findings do not
suggest acute cerebrovascular accident.
Medication:
• In the acute setting, treatment for carotid sinus syncope includes placing the patient in a
supine position with the legs elevated, administering oxygen, and monitoring vital signs.
• Long-term management may include medications to control blood pressure and heart rate,
such as beta blockers or calcium channel blockers.
Words:
cardiology

• Syncope: A sudden, transient loss of consciousness and postural tone due to decreased blood
flow to the brain.
• Carotid sinus: A baroreceptor located in the carotid artery that senses changes in blood
pressure and regulates heart rate and blood pressure.
• Left ventricular hypertrophy: An increase in the size and thickness of the left ventricle of the
heart, often due to underlying cardiac disease.
Findings:
• Normal values for laboratory tests are not mentioned in the question.
• The patient's objective findings, including pale skin, low blood pressure, and sinus tachycardia
with left ventricular hypertrophy, are abnormal findings indicative of a cardiac or vascular
disorder.

Therapy
cardiology

82 Unstable Angina Pectoris

A patient with unstable angina pectoris was given the following complex treatment:
anticoagulants, nitrates, α-adrenoblockers. However on the third day of treatment
the pain still romains. Which in vestigation shoud be carried out to establish diagnosis?
A Coronarography
B Stress-echocardiogram
C Test with dosed physical exercises
D Esophageal electrocardiac stimulator
E Myocardial scintigraphy

Correct Answer: A Coronarography


Key Points: The patient has unstable angina pectoris and is not responding to initial treatment
with anticoagulants, nitrates, and α-adrenoblockers.
Explanation: Coronarography is a diagnostic procedure used to visualize the coronary arteries
and identify any stenoses or occlusions that may be causing the patient's angina symptoms. It
involves the injection of contrast dye into the coronary arteries and imaging with X-rays.
Coronarography is the gold standard for diagnosing coronary artery disease and is
recommended for patients with unstable angina who do not respond to initial medical treatment.
Other Options:
B) Stress-echocardiogram: This test may be used to evaluate the heart's response to stress, but
it is not as specific for diagnosing coronary artery disease as coronarography.

Therapy
C) Test with dosed physical exercises: This test may be used to evaluate the heart's response to
exercise, but it is not as specific for diagnosing coronary artery disease as coronarography.
D) Esophageal electrocardiac stimulator: This test is used to evaluate the electrical activity of
the heart and is not specific for diagnosing coronary artery disease.
E) Myocardial scintigraphy: This test may be used to evaluate the blood flow to the heart, but it
is not as specific for diagnosing coronary artery disease as coronarography.
Making Other Options Right:
B) To make stress-echocardiogram the correct answer, the patient must have stable angina
pectoris instead of unstable, and initial treatment with anticoagulants, nitrates, and α-
adrenoblockers must have been successful.
C) To make test with dosed physical exercises the correct answer, the patient must have stable
angina pectoris instead of unstable, and initial treatment with anticoagulants, nitrates, and α-
adrenoblockers must have been successful.
D) To make esophageal electrocardiac stimulator the correct answer, the patient must have
arrhythmias, and initial treatment with anticoagulants, nitrates, and α-adrenoblockers must
have been unsuccessful.
E) To make myocardial scintigraphy the correct answer, the patient must have stable angina
pectoris instead of unstable, and initial treatment with anticoagulants, nitrates, and α-
adrenoblockers must have been successful.
Medication:
Anticoagulants: These medications prevent blood clots from forming by interfering with the
blood clotting process. Common anticoagulants used in the treatment of unstable angina include
heparin and low molecular weight heparins.
Nitrates: These medications relax the smooth muscles of the blood vessels, which can improve
blood flow to the heart and reduce angina symptoms. Common nitrates used in the treatment of
unstable angina include nitroglycerin and isosorbide dinitrate.
α-Adrenoblockers: These medications block the effects of adrenaline on the heart and blood
vessels, which can reduce blood pressure and heart rate and improve blood flow to the heart.
Common α-adrenoblockers used in the treatment of unstable angina include metoprolol and
carvedilol.
cardiology

83 Unstable Angina Pectoris

A 52 year old patient with history of functional Class II angina complains of having intense
and prolonged retrosternal pains, decreased exercise tolerance for 5 days. Angina is less
responsive to nitroglycerine. What is the most probable diagnosis?
A IHD. Unstable angina
B Cardialgia due to spine problem
C IHD. Functional Class II angina
D Myocarditis
E Myocardial dystrophy

Correct Answer: A IHD. Unstable angina.


Key Points:
• A 52-year-old patient with a history of functional Class II angina
• Prolonged and intense retrosternal pain
• Decreased exercise tolerance for 5 days
• Angina less responsive to nitroglycerin.
Explanation:
The clinical presentation of a 52-year-old patient with a history of functional Class II angina,
prolonged and intense retrosternal pain, decreased exercise tolerance for 5 days, and angina
that is less responsive to nitroglycerin indicates unstable angina, which is a manifestation of
acute coronary syndrome (ACS) in patients with underlying ischemic heart disease (IHD). ACS is

Therapy
caused by the rupture of an atherosclerotic plaque that leads to the formation of a thrombus,
which partially or completely occludes the coronary artery. Unstable angina is diagnosed in
patients with new-onset angina or a significant increase in the frequency, duration, or intensity
of angina. Unstable angina is differentiated from stable angina by its unpredictability, its
prolonged and intense character, and its less responsiveness to nitroglycerin. Prompt
recognition and treatment of unstable angina are essential to prevent the progression to
myocardial infarction.
Other Options:
B Cardialgia due to spine problem - There is no indication of a spinal problem that would cause
the described symptoms.
C IHD. Functional Class II angina - The patient has symptoms of unstable angina, which is a
more severe form of IHD than functional Class II angina.
D Myocarditis - Myocarditis is an inflammation of the myocardium that presents with symptoms
similar to ACS, but it is not caused by atherosclerotic plaque rupture. The described symptoms
are not typical of myocarditis.
E Myocardial dystrophy - Myocardial dystrophy is a rare condition that is not typically associated
with acute onset symptoms of chest pain and decreased exercise tolerance.
Making Other Options Right:
B - Cardialgia due to spine problem could be made correct if there was any indication of a spinal
issue causing the described symptoms.
C - IHD. Functional Class II angina could be made correct if the patient had no new or
worsening symptoms, and their angina was still classified as functional Class II.
D - Myocarditis could be made correct if the patient's symptoms were consistent with the clinical
presentation of myocarditis.
E - Myocardial dystrophy could be made correct if there were any indications of the patient
having this rare condition.
Medication: Nitroglycerin - Nitroglycerin is a vasodilator that relaxes smooth muscles in blood
vessels and is used to treat angina by improving blood flow to the heart. Its side effects may
include headaches, dizziness, and hypotension.
cardiology

84 Unstable Angina Pectoris

A 60 year old man with unstable angina pectoris fails to respond to heparin, nitroglycerin,
beta adrenegic blockers and calcium channel antagonist. The best management includes:
A Coronary artery bypass grafting
B Intravenous streptokinase
C Excercise testing
D Oral aspirin
E Antihypertensive therapy

Correct Answer: A Coronary artery bypass grafting.


Key Points: The patient has unstable angina pectoris and is unresponsive to heparin,
nitroglycerin, beta adrenegic blockers, and calcium channel antagonists.
Explanation: Unstable angina pectoris is a condition in which there is a reduction in blood flow to
the heart due to partial or complete occlusion of a coronary artery. The treatment of unstable
angina pectoris initially involves heparin, nitroglycerin, beta-adrenergic blockers, and calcium
channel antagonists. However, if these treatments fail to provide relief, the best management
option is coronary artery bypass grafting (CABG). CABG involves taking a healthy blood vessel
from another part of the body and grafting it onto the occluded coronary artery to bypass the
blockage and restore blood flow to the heart muscle.
Other Options:
B) Intravenous streptokinase is not the preferred treatment for unstable angina pectoris. It is

Therapy
used for the treatment of acute myocardial infarction (MI) with ST-segment elevation.
C) Exercise testing is not the best management option for unstable angina pectoris as it can
cause further myocardial ischemia and worsen the patient's symptoms.
D) Oral aspirin is part of the initial management of unstable angina pectoris but is not the best
management option for a patient who fails to respond to other treatments.
E) Antihypertensive therapy is not the best management option for unstable angina pectoris.
While hypertension is a risk factor for developing coronary artery disease, lowering blood
pressure alone will not restore blood flow to the occluded coronary artery.
Medication: The medications mentioned in the options are heparin, nitroglycerin, beta-
adrenergic blockers, and calcium channel antagonists. Heparin is an anticoagulant that is used
to prevent the formation of blood clots in the coronary arteries. Nitroglycerin is a vasodilator
that is used to relieve angina by increasing blood flow to the heart. Beta-adrenergic blockers
and calcium channel antagonists are also used to relieve angina by decreasing the workload of
the heart and reducing the demand for oxygen.
Beta-adrenergic blockers are contraindicated in patients with heart failure, bradycardia, or heart
block. Calcium channel antagonists are contraindicated in patients with heart block,
hypotension, or heart failure.
cardiology

85 Unstable Angina Pectoris

A 61 y.o. man complained of sneezing and substernal pain on exertion. In the last 2 weeks
such pain appeared at rest, with increased frequency, and couldn't be suppressed by 1
tablet of nitroglycerin. What is the most likely diagnosis?
A Unstable angina pectoris
B Angina pectoris of a new onset
C Myocarditis
D Radiculitis
E Stable angina pectoris of the III functional class

Correct Answer: A - Unstable angina pectoris.


Key Points:
• Patient has a history of substernal pain on exertion.
• Pain has increased in frequency, intensity, and duration over the last 2 weeks.
• Pain now occurs at rest and is not relieved by nitroglycerin.
Explanation:
Unstable angina pectoris is a type of acute coronary syndrome that is characterized by angina at
rest, new-onset severe angina, or an increase in the frequency, intensity, and duration of
previously stable angina. It is caused by the rupture or erosion of an atherosclerotic plaque,
which leads to the formation of a thrombus and subsequent partial or complete occlusion of the
coronary artery. Unstable angina pectoris requires urgent evaluation and treatment to prevent

Therapy
myocardial infarction or death.
Other Options:
B - Angina pectoris of a new onset: This option is incorrect because the patient has a history of
chest pain on exertion, which is suggestive of stable angina pectoris. However, the new-onset
chest pain at rest is suggestive of unstable angina pectoris.
C - Myocarditis: This option is incorrect because myocarditis is an inflammation of the heart
muscle, which typically presents with symptoms such as fever, fatigue, chest pain, and dyspnea.
However, the patient's symptoms are more suggestive of coronary artery disease.
D - Radiculitis: This option is incorrect because radiculitis is a condition that involves
inflammation or irritation of the spinal nerve roots, which typically presents with symptoms such
as back pain, radiating pain, and sensory deficits. However, the patient's symptoms are more
suggestive of coronary artery disease.
E - Stable angina pectoris of the III functional class: This option is incorrect because the
patient's symptoms have progressed beyond stable angina pectoris and are now suggestive of
unstable angina pectoris.
Making Other Options Right:
B - Angina pectoris of a new onset: This option would be correct if the patient only presented
with new-onset chest pain at rest without any history of chest pain on exertion.
C - Myocarditis: This option would be correct if the patient presented with symptoms such as
fever, fatigue, and chest pain, in addition to cardiac-specific biomarkers such as elevated
troponin levels.
D - Radiculitis: This option would be correct if the patient presented with symptoms such as
back pain, radiating pain, and sensory deficits, in addition to a negative cardiac workup.
Medication:
The treatment for unstable angina pectoris includes antiplatelet therapy (aspirin), nitroglycerin,
beta-blockers, and heparin. Nitroglycerin works by dilating the coronary arteries and increasing
blood flow to the heart. Side effects of nitroglycerin include headache, hypotension, and
flushing.
Words:
Angina pectoris - chest pain or discomfort that occurs when the heart muscle does not receive
enough blood and oxygen.
cardiology

86 Valvular Disorders

A 67-year-old male complains of dyspnea on exertion, attacks of retrosternal pain, dizziness.


He has no history of rheumatism. Objectively: pale skin, acrocyanosis. There are rales in the
lower parts of lungs. There is systolic thrill in the II intercostal space on the right, coarse
systolic
murmur conducted to the vessels of neck. AP- 130/90 mm Hg, heart rate - 90/min, regular
rhythm. The liver extends 5 cm under the edge of costal arch, shin edemata are present.
Specify the assumed valvular defect:
A Aortic stenosis
B Pulmonary artery stenosis
C Mitral insufficiency
D Ventricular septal defect
E Tricuspid regurgitation

Correct Answer: A Aortic stenosis - Aortic stenosis is the assumed valvular defect.
Key Points:
• A 67-year-old male complains of dyspnea on exertion, attacks of retrosternal pain, and
dizziness.
• Objectively, the patient has pale skin, acrocyanosis, rales in the lower parts of lungs, a systolic
thrill in the II intercostal space on the right, coarse systolic murmur conducted to the vessels of
the neck, and shin edema.

Therapy
• The liver extends 5 cm under the edge of costal arch.
• The question asks to specify the assumed valvular defect.
Explanation:
The patient's symptoms of dyspnea on exertion, attacks of retrosternal pain, and dizziness,
along with the objective findings of pale skin, acrocyanosis, rales in the lower parts of lungs, and
a coarse systolic murmur conducted to the vessels of the neck, are all indicative of a valvular
heart disease.
The presence of a systolic thrill in the II intercostal space on the right, along with a coarse
systolic murmur, suggests aortic stenosis. Aortic stenosis is a valvular heart disease
characterized by narrowing of the aortic valve, which can lead to symptoms such as dyspnea on
exertion, angina, and syncope. The murmur of aortic stenosis is typically heard best in the right
second intercostal space and may radiate to the carotid arteries. The presence of rales in the
lower parts of lungs and shin edema suggests that the patient also has congestive heart failure,
which can occur as a complication of aortic stenosis.
Other Options:
• B Pulmonary artery stenosis: Pulmonary artery stenosis is a valvular heart disease
characterized by narrowing of the pulmonary valve. This condition typically presents with
symptoms such as dyspnea on exertion, fatigue, and syncope. The presence of a systolic thrill in
the II intercostal space on the right and a coarse systolic murmur conducted to the vessels of
the neck suggests aortic stenosis rather than pulmonary artery stenosis.
• C Mitral insufficiency: Mitral insufficiency, also known as mitral regurgitation, is a valvular
heart disease characterized by backflow of blood from the left ventricle into the left atrium
during systole. This condition typically presents with symptoms such as dyspnea on exertion,
fatigue, and palpitations. The absence of a holosystolic murmur and the presence of a systolic
thrill in the II intercostal space on the right suggest aortic stenosis rather than mitral
insufficiency.
• D Ventricular septal defect: Ventricular septal defect is a congenital heart defect characterized
by an opening in the septum between the left and right ventricles. This condition typically
presents with symptoms such as dyspnea on exertion, fatigue, and poor growth in infants. The
presence of a systolic thrill in the II intercostal space on the right and a coarse systolic murmur
cardiology

conducted to the vessels of the neck suggest aortic stenosis rather than ventricular septal
defect.
• E Tricuspid regurgitation: Tricuspid regurgitation is a valvular heart disease characterized by
backflow of blood from the right ventricle into the right atrium during systole. This condition
typically presents with symptoms such as fatigue, edema, and ascites. The presence of a
systolic thrill in the II intercostal space on the right and a coarse systolic murmur conducted to
the vessels of the neck suggest aortic stenosis rather than tricuspid regurgitation.
Making Other Options Right:
• B Pulmonary artery stenosis: If the patient had a systolic murmur heard best in the left second
intercostal space, pulmonary artery stenosis may be a more likely diagnosis.
• C Mitral insufficiency: If the patient had a holosystolic murmur heard best at the apex, mitral
insufficiency may be a more likely diagnosis.
• D Ventricular septal defect: If the patient had a harsh holosystolic murmur heard best at the
left lower sternal border, ventricular septal defect may be a more likely diagnosis.
• E Tricuspid regurgitation: If the patient had a holosystolic murmur heard best at the left lower
sternal border, tricuspid regurgitation may be a more likely diagnosis.
Medication:
• Treatment of aortic stenosis depends on the severity of the disease and the presence of
symptoms. In severe cases, surgical valve replacement is the most effective treatment. In
milder cases, medications such as diuretics and vasodilators may be used to manage symptoms.
Words:
• Valvular heart disease: a group of conditions characterized by dysfunction of one or more of
the heart's valves.
• Aortic stenosis: a valvular heart disease characterized by narrowing of the aortic valve.
• Systolic thrill: a vibration felt on the chest wall during systole.
• Coarse systolic murmur: a type of murmur heard during systole that has a rough, rasping

Therapy
quality.
• Rales: abnormal breath sounds heard on auscultation of the lungs.
• Acrocyanosis: bluish discoloration of the extremities.
• Shin edema: swelling of the lower legs and ankles.
• Congestive heart failure: a condition in which the heart is unable to pump blood efficiently,
leading to fluid accumulation in the lungs and other parts of the body.
Findings:
• AP (blood pressure): 130/90 mmHg (normal range: <120/80 mmHg)
• Heart rate: 90/min (normal range: 60-100/min)
• Liver extends 5 cm under the edge of costal arch (normal liver span is 6-12 cm)
• Normal values for acrocyanosis and shin edema are not applicable as they are physical
findings and not laboratory values.
cardiology

87 Valvular Disorders

A patient, aged 49, complains of fever of 37,5oC, heart pain, dyspnea. S1 is


clapping; S2 is accentuated in the aortic area; opening snap, presystolic murmur can
be auscultated. What is the most efficient examination for valvular disorder assessment?
A Echocardiography+Doppler-Echocardiography
B Phonocardiography
C Ballistocardiogram
D Chest X-ray
E ECG

Correct Answer: A) Echocardiography+Doppler-Echocardiography


Key Points: A 49-year-old patient with fever, heart pain, and dyspnea. Auscultation reveals
clapping S1, accentuated S2 in the aortic area, opening snap, and presystolic murmur. The
question is asking for the most efficient examination to assess valvular disorder.
Explanation: The most efficient examination to assess valvular disorders is echocardiography
with Doppler-Echocardiography. This non-invasive test is used to visualize the heart structures
and blood flow through the heart. Echocardiography can evaluate valve morphology, assess
valve stenosis, and regurgitation. The Doppler mode can measure blood flow velocity across the
valve, which helps to quantify the severity of the valvular lesion. The opening snap and
presystolic murmur are characteristic of mitral stenosis. The accentuated S2 in the aortic area
may indicate aortic stenosis.

Therapy
Other Options:
B) Phonocardiography is a non-invasive technique that records heart sounds and murmurs. It
can help to identify the timing and intensity of heart sounds, but it does not provide information
about the valve structure or blood flow through the heart. Therefore, it is not the most efficient
examination to assess valvular disorders.
C) Ballistocardiogram measures the movement of the body in response to the cardiac
contraction. This technique has limited clinical application and is not commonly used to assess
valvular disorders.
D) Chest X-ray can reveal cardiomegaly and pulmonary congestion but is not specific to valvular
disorders.
E) ECG can detect arrhythmias and conduction abnormalities but does not provide direct
information about valve structure or function.
Making Other Options Right:
B) Phonocardiography can be a useful adjunct to the diagnosis of valvular disorders, but it is not
the most efficient examination for the assessment of valvular disorders.
C) Ballistocardiography is not useful for the assessment of valvular disorders.
D) Chest X-ray is not the most efficient examination for the assessment of valvular disorders.
E) ECG is not the most efficient examination for the assessment of valvular disorders.
Medication: Not applicable.
Words:
• Clapping S1: Loud and sharp first heart sound
• Opening snap: A sharp sound heard in early diastole, often associated with mitral stenosis.
• Presystolic murmur: A murmur heard just before S1, often associated with mitral stenosis.
Findings:
• Fever: Body temperature above the normal range of 36.5-37.5°C
• Accentuated S2 in the aortic area: A loud S2 sound heard best at the second intercostal space
to the right of the sternum, often associated with aortic stenosis.
• Mitral stenosis: Narrowing of the mitral valve orifice, resulting in obstruction to blood flow
from the left atrium to the left ventricle.
cardiology, pulmonology

88 Euler-Liljestrand reflex

A 49-year-old male patient complains of dyspnea of combined nature, cough, shin edemata,
abdomen enlargement due to ascites. He has a 20-year history of chronic bronchitis. For the last
3 years he has been disabled (group II) because of cardiac changes. Objectively: mixed
cyanosis, edemata. Ps - 92/min, rhythmic, AP - 120/70 mm Hg, respiration rate - 24/min. There
is accentuation of the second sound above the pulmonary artery. Auscultation reveals the box
resonance above the lungs. There are also dry rales
over the entire surface of lungs. What is the mechanism of heart changes development in
this patient?
A Euler-Liljestrand reflex
B Kitaev's reflex
C Bainbridge reflex
D Cardiovascular reflex
E Respiratory reflex

Correct Answer: A Euler-Liljestrand reflex - The mechanism of heart changes development in


this patient is likely due to the Euler-Liljestrand reflex.
Key Points:
• A 49-year-old male patient complains of dyspnea, cough, shin edema, and ascites.
• The patient has a 20-year history of chronic bronchitis and has been disabled due to cardiac
changes for the last 3 years.

Therapy
• Objectively, the patient has mixed cyanosis, edema, and abnormal heart sounds and lung
sounds.
• The question asks for the mechanism of heart changes development in this patient.
Explanation:
• The patient's symptoms of dyspnea, cough, and edema suggest heart failure, which is a
condition where the heart is unable to pump blood effectively.
• Heart failure can be caused by a variety of factors, including coronary artery disease,
hypertension, and valvular disease.
• In this case, the patient's history of chronic bronchitis and cardiac disability suggests that the
heart changes may be due to pulmonary hypertension, which is a common complication of
chronic lung disease.
• Pulmonary hypertension can cause right ventricular hypertrophy and heart failure through
several mechanisms, including the Euler-Liljestrand reflex.
• The Euler-Liljestrand reflex is a mechanism by which the pulmonary arterioles constrict in
response to low oxygen levels in the alveoli, which increases pulmonary vascular resistance and
can lead to right ventricular hypertrophy and failure.
• The accentuation of the second sound above the pulmonary artery and the box resonance
above the lungs are consistent with right ventricular hypertrophy, while the dry rales over the
lungs suggest pulmonary congestion.
Other Options:
B - Kitaev's reflex is a reflex where an increase in venous pressure causes an increase in the
heart rate. However, it is not relevant to the heart changes observed in this patient.
C) Bainbridge reflex - The Bainbridge reflex is a reflex that causes an increase in heart rate in
response to an increase in venous return. While it is involved in the regulation of cardiac output,
it is not directly related to the development of heart changes in this patient.
D) Cardiovascular reflex - This is a broad term that includes many different reflexes involved in
the regulation of cardiovascular function. Without further information, it is impossible to
determine which specific reflex is involved in the development of heart changes in this patient.
E) Respiratory reflex - This is a broad term that includes many different reflexes involved in the
regulation of respiration.
cardiology, pulmonology

While pulmonary hypertension can be caused by chronic lung disease, the specific mechanism
involved in the development of heart changes is likely the Euler-Liljestrand reflex.
Medication:
• Treatment for chronic bronchitis includes bronchodilators, corticosteroids, and antibiotics.
• Treatment for pulmonary hypertension includes calcium channel blockers, endothelin receptor
antagonists, phosphodiesterase inhibitors, and prostacyclin analogs.
• Treatment for right-sided heart failure includes diuretics, ACE inhibitors, beta-blockers, and in
severe cases, heart transplantation.
Words:
• Euler-Liljestrand reflex - A reflex by which the pulmonary arterioles constrict in response to
low oxygen levels in the alveoli, increasing pulmonary vascular resistance and leading to right
ventricular hypertrophy and failure.
• Cyanosis - A bluish discoloration of the skin and mucous membranes due to a lack of oxygen
in the blood.
• Pulmonary hypertension - High blood pressure in the pulmonary arteries, which can lead to
right ventricular hypertrophy and heart failure.
• Edema - Swelling caused by the accumulation of fluid in the tissues.
Findings:
• Ps (pulse) - 92/min (normal < 100/min)
• AP (arterial pressure) - 120/70 mm Hg (normal < 120/80 mm Hg)
• Respiration rate - 24/min (normal 12-20/min)
• Accentuation of the second sound above the pulmonary artery (indicative of pulmonary
hypertension)
• Box resonance above the lungs (indicative of right-sided heart failure)
• Dry rales over the entire surface of lungs (indicative of right-sided heart failure)

Therapy
dermatology

89 Alopecia

A 32 y.o. man is divorced, has an irregular sexual life. He complains of falling out of hair in
the region of eyelashes, eyebrows, scalp. Objectively: diffuse alopecia is observed,
eyebrow margin is absent, eyelashes are stair-like (Pinkus' sign). What examination should
be carried out first of all?
A Wasserman test, IFT
B T.pallidum Immobilization Test (TPI)
C Detection of the nasal mucous membrane for Micobacterium Leprae Hansen
D Consultation of neuropathist
E CBC

Correct Answer: A Wasserman test, IFT should be carried out first of all.
Key Points: 32 y.o. man, divorced, diffuse alopecia, Pinkus' sign, irregular sexual life.
Explanation: The given case represents a manifestation of syphilis, a sexually transmitted
disease caused by the bacterium Treponema pallidum. The primary stage of syphilis is
characterized by the appearance of a painless chancre at the site of infection, which can be
unnoticed or mistaken for a benign skin lesion. The secondary stage of syphilis typically
presents with a variety of skin and mucosal manifestations, including diffuse alopecia, eyebrow
and eyelash loss, and the stair-step appearance of eyelashes (Pinkus' sign). Therefore, in this
case, the first examination that should be carried out is a Wasserman test or an IFT
(Immunofluorescence test), which are screening tests for syphilis. Further investigations,

Therapy
including a T.pallidum Immobilization Test (TPI) or dark-field microscopy, may be required to
confirm the diagnosis.
Other Options:
B. T.pallidum Immobilization Test (TPI): This test is used to confirm the diagnosis of syphilis,
but it is not a screening test.
C. Detection of the nasal mucous membrane for Micobacterium Leprae Hansen: This option is
incorrect because it suggests the possibility of leprosy, which is not associated with the clinical
features described in the case.
D. Consultation of neuropathist: This option is incorrect because the clinical features described
are indicative of a dermatological condition, and a neuropathologist is not the appropriate
specialist to consult in this case.
E. CBC: This option is incorrect because a complete blood count is not relevant for the diagnosis
of syphilis or alopecia.
Words: Wasserman test - a blood test used to screen for syphilis.
Findings: Pinkus' sign - the stair-step appearance of eyelashes seen in secondary syphilis.
dermatology

90 atopic dermatitis

A 35 year old woman consulted a doctor about affection of arm skin and lower third of
forearm in form of a large edema, hyperemia, vesiculation and maceration. The disease
developed after using a laundry detergent "Lotos". The patient has been using it for a
month. She hasn't suffered from dermatological diseases before. What is the most
probable diagnosis?
A Allergic dermatitis
B Dermatitis simplex
C Toxicoallergic dermatitis
D Microbial eczema
E Localized neurodermatitis

Correct Answer: option A.


Explanation:
• Allergic dermatitis is a type of contact dermatitis that results from exposure to an allergen.
• The patient's use of the laundry detergent "Lotos" for a month and subsequent development of
skin symptoms is highly suggestive of a contact dermatitis due to an allergen in the detergent.
• The symptoms of edema, hyperemia, vesiculation, and maceration are consistent with a
severe reaction to an allergen in the detergent.
• The absence of a previous history of dermatological diseases makes it less likely that the
patient has a pre-existing skin condition that is exacerbating the reaction.

Therapy
Other Options:
• Option B, dermatitis simplex, is a non-specific term that refers to a type of skin inflammation
that is not caused by a specific allergen or irritant. This is not a likely diagnosis in this case.
• Option C, toxicoallergic dermatitis, is a type of contact dermatitis that results from exposure to
a chemical irritant or allergen. While this is a possibility, the more severe presentation of skin
symptoms and the absence of a previous history of dermatological diseases make allergic
dermatitis more likely.
• Option D, microbial eczema, is a type of skin inflammation that results from a bacterial or
fungal infection. This is not a likely diagnosis in this case, as the patient's symptoms are
consistent with a reaction to an allergen in the laundry detergent rather than an infection.
• Option E, localized neurodermatitis, is a type of skin inflammation that results from chronic
scratching or rubbing of the skin. This is not a likely diagnosis in this case, as the patient's
symptoms developed after exposure to a laundry detergent rather than prolonged scratching or
rubbing of the skin.
Making Other Options Right:
• Option B, dermatitis simplex, could be made correct if the patient's symptoms are found to be
non-specific and not due to a specific allergen or irritant.
• Option C, toxicoallergic dermatitis, could be made correct if the patient is found to have a
positive patch test to a chemical irritant or allergen in the laundry detergent or if the symptoms
do not improve after cessation of exposure to the detergent.
• Option D, microbial eczema, could be made correct if the patient's symptoms are found to be
due to a secondary bacterial or fungal infection of the skin.
• Option E, localized neurodermatitis, could be made correct if the patient's symptoms are found
to be due to chronic scratching or rubbing of the skin.
Treatment:
• The treatment for allergic dermatitis involves identifying and avoiding the offending allergen.
• Topical or oral corticosteroids can be used to reduce inflammation and pruritus.
• Cool compresses and emollients can be used to soothe the affected skin.
• In severe cases, oral corticosteroids or immunomodulators may be necessary.
• If the patient's symptoms are due to an infection, topical or oral antibiotics or antifungals may
be necessary.
dermatology

Findings:
• The patient has arm skin and lower third of forearm affected by edema, hyperemia,
vesiculation, and maceration after using a laundry detergent "Lotos" for a month.
• The symptoms are consistent with a severe reaction to an allergen in the detergent,
suggesting allergic dermatitis.
• The absence of a previous history of dermatological diseases makes it less likely that the
patient has a pre-existing skin condition that is exacerbating the reaction.

Therapy
dermatology

91 Diagnostic Method

A male, 50 y.o., has a black flat mole on the skin of the leg for 10 years. Since 4 months
ago the shapes of the mole become irregular. What diagnostic methods should be used?
A Excision biopsy
B Fine needle biopsy
C Smear for microscopic examination
D Incision biopsy
E Thermography

Correct Answer: Option A


The patient in the case presented has a black flat mole on the skin of the leg for 10 years, which
has recently developed irregular shapes. The irregularity in shape is concerning for the
possibility of a malignant transformation of the mole, which would require further evaluation.
Therefore, the diagnostic method that should be used in this case is Option A, Excision biopsy.
Key Points:
• Male, 50 y.o. with a black flat mole on the skin of the leg for 10 years, with irregular shapes
recently developed.
• The irregularity in shape is concerning for the possibility of a malignant transformation of the
mole.
• The diagnostic method that should be used in this case is excision biopsy.
Explanation:The presence of a black flat mole on the skin of the leg for 10 years is concerning

Therapy
for the possibility of melanoma, which is a type of skin cancer that can develop from pre-
existing moles. The recent development of irregular shapes in the mole is particularly
concerning for the possibility of a malignant transformation of the mole. Therefore, further
evaluation of the mole is necessary.
Option A, Excision biopsy, is the diagnostic method of choice for evaluating suspicious skin
lesions, particularly those that are concerning for melanoma. Excision biopsy involves removing
the entire lesion, along with a margin of normal skin around it, and sending it for pathological
examination. It is the most accurate method for diagnosing melanoma and can also provide
information on the depth and extent of the tumor, which is important for determining the
appropriate treatment.
Other options, such as fine needle biopsy (Option B) and incision biopsy (Option D), may not
provide enough tissue for accurate pathological examination and may miss the diagnosis of
melanoma. Smear for microscopic examination (Option C) may be used for diagnosing certain
skin conditions, such as fungal infections or inflammatory disorders, but is not useful for
diagnosing melanoma. Thermography (Option E) is a non-invasive imaging technique that can
detect changes in skin temperature, but is not useful for diagnosing melanoma.
Other Options:
• Option A, Excision biopsy, is the correct answer.
• Option B, Fine needle biopsy, may not provide enough tissue for accurate pathological
examination and may miss the diagnosis of melanoma.
• Option C, Smear for microscopic examination, may be used for diagnosing certain skin
conditions, but is not useful for diagnosing melanoma.
• Option D, Incision biopsy, may not provide enough tissue for accurate pathological
examination and may miss the diagnosis of melanoma.
• Option E, Thermography, is not useful for diagnosing melanoma.
Treatment:
• The treatment of melanoma depends on the stage of the tumor and may involve surgical
excision, radiation therapy, or chemotherapy.
• Early detection and treatment are essential for improving the prognosis of melanoma.
Words:
• Melanoma: A type of skin cancer that can develop from pre-existing moles.
dermatology

• Excision biopsy: A diagnostic method that involves removing the entire lesion, along with a
margin of normal skin around it, and sending it for pathological examination.
• Fine needle biopsy: A diagnostic method that involves inserting a thin needle into the lesion
and aspirating a small sample of tissue for pathological examination.
• Incision biopsy: A diagnostic method that involves removing a portion of the lesion for
pathological examination.
• Smear for microscopic examination: A diagnostic method that involves obtaining a sample of
cells from the lesion and examining them under a microscope.
• Thermography: A non-invasive imaging technique that can detect changes in skin
temperature.
Findings:
• The patient has a black flat mole on the skin of the leg for 10 years, which has recently
developed irregular shapes.
• The irregularity in shape is concerning for the possibility of a malignant transformation of the
mole.
• The diagnostic method that should be used in this case is excision biopsy.

Therapy
dermatology

92 Drug reactions

A 32-year-old female patient consulted a doctor about a skin affection of her body, face
and extremities which turned up after peroral intake of biseptol. The patient was diagnosed
with drug toxicoderma. What treatment methods would be the most effective for this
patient?
A Extracorporal therapy
B Anti-inflammatory therapy
C Antihistamine therapy
D Disintoxication therapy
E Immunotherapy

Correct Answer: Option A (Extracorporal therapy) is the correct answer because extracorporeal
therapies, such as hemodialysis or plasmapheresis, may be used in severe cases of drug-
induced toxicoderma.
Key Points:
• 32-year-old female with skin affection after peroral intake of biseptol.
• Diagnosis of drug toxicoderma.
• Treatment methods for drug-induced toxicoderma.
Explanation:
Toxicoderma is a term used to describe a group of skin eruptions that are caused by
medications. The primary treatment for drug-induced toxicoderma involves discontinuing the

Therapy
offending drug and providing supportive care to manage symptoms and remove the drug from
the body. In severe cases of toxicoderma, extracorporeal therapies, such as hemodialysis or
plasmapheresis, may be used to remove the offending drug from the body and manage
symptoms.
Anti-inflammatory therapy, such as corticosteroids, may also be used in some cases to manage
symptoms of toxicoderma, but it is not the primary treatment. Antihistamine therapy may also
be used to manage itching and other symptoms, but it is not typically effective for severe cases
of toxicoderma. Immunotherapy is not typically used in the treatment of toxicoderma.
Other Options:
• Option B (Anti-inflammatory therapy) is incorrect because although anti-inflammatory
therapy, such as corticosteroids, may be used in some cases of drug-induced toxicoderma, it is
not the primary treatment.
• Option C (Antihistamine therapy) is incorrect because although antihistamines may be used to
manage itching and other symptoms in some cases of drug-induced toxicoderma, they are not
typically effective for severe cases.
• Option D (Disintoxication therapy) is incorrect because disintoxication therapy is not a specific
treatment, but rather a general term for the removal of toxins from the body, which can be
achieved through various methods, such as intravenous fluids, diuresis, or hemodialysis.
Extracorporeal therapies, such as hemodialysis or plasmapheresis, are a type of disintoxication
therapy and may be used in severe cases of toxicoderma, but they are not the primary
treatment.
• Option E (Immunotherapy) is incorrect because immunotherapy is not typically used in the
treatment of toxicoderma.
Making Other Options Right:
• Option B (Anti-inflammatory therapy) could be made correct if the patient had severe
inflammation or swelling associated with the toxicoderma.
• Option C (Antihistamine therapy) could be made correct if the patient had mild symptoms and
itching associated with the toxicoderma.
• Option D (Disintoxication therapy) could be made correct if the patient had severe toxicity and
required extracorporeal therapy to remove the offending drug from the body.
dermatology

• Option E (Immunotherapy) could be made correct if the patient had an immune-mediated


form of toxicoderma, such as Stevens-Johnson syndrome or toxic epidermal necrolysis, but this
is not typically the case for drug-induced toxicoderma.
Medication:
There is no specific medication used to treat drug-induced toxicoderma. Discontinuing the
offending drug and providing supportive care, such as extracorporeal therapy or anti-
inflammatory drugs, is the primary treatment. Symptomatic relief may be provided with
antihistamines for itching.
Words:
• Toxicoderma: a group of skin eruptions caused by medications.

Therapy
dermatology

93 eczema

A welder at work got the first-degree burns of the middle third of his right shin. 5 days later
the skin around the burn became edematic and itchy. Objectively: on a background of a
well-defined erythema there is polymorphic rash in form of papules, vesicles, pustules,
erosions with serous discharge. What is the most likely diagnosis?
A Microbal eczema
B True eczema
C Toxicoderma
D Occupational eczema
E Streptococcal impetigo

Correct Answer: A Microbial eczema - The patient's symptoms of first-degree burns followed by
edema, erythema, and polymorphic rash with serous discharge suggest a diagnosis of microbial
eczema.
Key Points:
• Welder with first-degree burns of right shin
• 5 days later, skin around the burn becomes edematous and itchy
• Polymorphic rash in form of papules, vesicles, pustules, erosions with serous discharge
Explanation:
Microbial eczema, also known as infectious eczematoid dermatitis, is a type of eczema that
occurs as a secondary infection on pre-existing skin lesions. In this case, the patient's first-

Therapy
degree burns may have provided a portal of entry for microorganisms to infect the skin, leading
to the development of microbial eczema. Common causative microorganisms include
Staphylococcus aureus and Streptococcus pyogenes.
The symptoms of microbial eczema typically include erythema, edema, and a polymorphic rash
with papules, vesicles, pustules, and erosions with serous discharge. The rash is often itchy and
may be accompanied by fever and lymphadenopathy.
Diagnosis of microbial eczema is typically made through clinical evaluation and laboratory tests,
such as bacterial cultures and sensitivity testing. Treatment may include topical or systemic
antibiotics, as well as management of the underlying skin condition.
Other Options:
B True eczema - True eczema, also known as atopic dermatitis, is a chronic inflammatory skin
condition that typically presents with erythema, pruritus, and eczematous lesions. The patient's
history of first-degree burns and subsequent infection is not typical of true eczema.
C Toxicoderma - Toxicoderma refers to a group of skin conditions caused by exposure to drugs,
chemicals, or other environmental agents. The patient's history of burns and subsequent
infection is not typical of toxicoderma.
D Occupational eczema - Occupational eczema refers to a type of eczema that is caused by
exposure to irritants or allergens in the workplace. The patient's history of burns and
subsequent infection is not typical of occupational eczema.
E Streptococcal impetigo - Streptococcal impetigo is a skin infection caused by Streptococcus
pyogenes, which typically presents with honey-colored crusts on the skin. The patient's
symptoms of polymorphic rash with serous discharge are not typical of impetigo.
Making Other Options Right:
B True eczema - This option could be made right if the patient had a history of eczematous
lesions prior to the burns and subsequent infection.
C Toxicoderma - This option could be made right if the patient had a history of exposure to a
drug, chemical, or other environmental agent that could have caused the skin reaction.
D Occupational eczema - This option could be made right if the patient had a history of exposure
to irritants or allergens in the workplace that could have caused the skin reaction.
E Streptococcal impetigo - This option could be made right if the patient had honey-colored
crusts on the skin, which are typical of streptococcal impetigo.
dermatology

Medication:
• Treatment for microbial eczema typically involves topical or systemic antibiotics, as well as
management of the underlying skin condition.
Words:
• Microbial eczema: a type of eczema that occurs as a secondary infection on pre-existing skin
lesions, typically caused by Staphylococcus aureus or Streptococcus pyogenes.
Findings:
• First-degree burns of the middle third of right shin
• 5 days later, skin around the burn becomes edematous and itchy
• Polymorphic rash in form of papules, vesicles, pustules, erosions with serous discharge.

Therapy
dermatology

94 eczema

A 72-year-old male patient complains about itch in his left shin, especially around a trophic
ulcer. Skin is reddened and edematic, there are some oozing lesions, single yellowish
crusts. The focus of affection is well-defined. What is the most likely diagnosis?
A Microbial eczema
B Allergic dermatitis
C Seborrheic eczema
D Cutaneous tuberculosis
E Streptococcal impetigo

Correct Answer: A Microbial eczema


Key Points:
• 72-year-old male patient with itching in his left shin, especially around a trophic ulcer
• Skin is reddened and edematous, with oozing lesions and yellowish crusts
• The focus of affection is well-defined
• Need to determine the most likely diagnosis
Explanation:
• The clinical presentation is consistent with microbial eczema, also known as infected eczema
or secondary infected eczema.
• Microbial eczema occurs when bacteria, such as Staphylococcus aureus or Streptococcus
pyogenes, infect an existing eczematous lesion.

Therapy
• The symptoms of microbial eczema include itching, redness, edema, oozing lesions, and
yellowish crusts.
• The focus of affection is usually well-defined, and the surrounding skin may be eczematous.
• Trophic ulcers are chronic, non-healing ulcers that develop in areas of skin damage, such as
eczema or venous stasis dermatitis.
Other Options:
• B Allergic dermatitis: Allergic dermatitis is a type of eczema that is caused by exposure to
allergens, such as poison ivy, nickel, or fragrances. It typically presents as a rash with redness,
itching, and vesicles or bullae. However, the presence of a trophic ulcer and yellowish crusts is
not typical of allergic dermatitis.
• C Seborrheic eczema: Seborrheic eczema is a type of eczema that affects areas of the body
with a high density of sebaceous glands, such as the scalp, face, and trunk. It typically presents
as red, scaly patches with itching and burning. However, the location of the lesion in the left
shin and the presence of a trophic ulcer are not typical of seborrheic eczema.
• D Cutaneous tuberculosis: Cutaneous tuberculosis is a rare form of tuberculosis that affects
the skin. It typically presents as nodules or ulcers with a raised border and a central crater.
However, the presentation of microbial eczema with a trophic ulcer and yellowish crusts is not
typical of cutaneous tuberculosis.
• E Streptococcal impetigo: Streptococcal impetigo is a bacterial skin infection that is caused by
Streptococcus pyogenes. It typically presents as small, fluid-filled blisters or pustules with a
honey-colored crust. However, the presentation of microbial eczema with a trophic ulcer and
yellowish crusts is not typical of streptococcal impetigo.
Words:
• Trophic ulcer: A chronic, non-healing ulcer that develops in areas of skin damage, such as
eczema or venous stasis dermatitis.
Findings:
• Microbial eczema is characterized by infection of an existing eczematous lesion with bacteria,
such as Staphylococcus aureus or Streptococcus pyogenes.
• The symptoms of microbial eczema include itching, redness, edema, oozing lesions, and
yellowish crusts.
• The focus of affection is usually well-defined, and the surrounding skin may be eczematous.
dermatology

• Trophic ulcers are chronic, non-healing ulcers that develop in areas of skin damage, such as
eczema or venous stasis dermatitis.

Therapy
dermatology

95 eczema

A 35 y.o. patient experienced a strong nervous stress that resulted in formation of


reddened and edematic areas on the back surface of her hands with further formation of
small inflammated nodules, vesicles and then erosions accompanied by profuse discharge
of serous liquid. The process is also accompanied by intense itching. What is the most
probable diagnosis?
A Common eczema
B Allergic dermatitis
C Microbial eczema
D Common contact dermatitis
E Toxicodermia

Correct Answer: A) Common eczema is the most probable diagnosis.


Key Points:
• 35-year-old patient with reddened and edematous areas on the back surface of her hands.
• Formation of small inflamed nodules, vesicles, and erosions with profuse discharge of serous
liquid.
• Accompanied by intense itching.
• History of strong nervous stress.
Explanation:
Common eczema, also known as atopic dermatitis, is a chronic inflammatory skin condition that

Therapy
is characterized by red, itchy, and inflamed skin. It is often triggered by environmental factors,
such as stress, allergens, or irritants. Symptoms of common eczema include redness,
inflammation, itching, and the formation of small bumps, blisters, or scales. The diagnosis is
based on the patient's history and clinical examination. In some cases, a skin biopsy may be
necessary to confirm the diagnosis.
Other Options:
B) Allergic dermatitis is a type of skin inflammation that occurs as a result of exposure to an
allergen. It typically causes redness, itching, and the formation of small bumps or blisters.
However, the patient's history of strong nervous stress makes this diagnosis less likely.
C) Microbial eczema is a type of eczema that is caused by a bacterial or fungal infection. It
typically causes redness, inflammation, and the formation of small bumps or blisters. However,
the patient's symptoms do not suggest an infection.
D) Common contact dermatitis is a type of skin inflammation that occurs as a result of exposure
to an irritant or allergen. It typically causes redness, itching, and the formation of small bumps
or blisters. However, the patient's history of strong nervous stress makes this diagnosis less
likely.
E) Toxicodermia is a skin reaction that occurs as a result of exposure to a medication or
chemical. It typically causes redness, itching, and the formation of small bumps or blisters.
However, the patient does not report exposure to any medications or chemicals.
Making Other Options Right:
B) Allergic dermatitis would be the correct option if the patient had a known exposure to an
allergen that triggered the skin inflammation.
C) Microbial eczema would be the correct option if the patient had a bacterial or fungal infection
that was causing the skin inflammation.
D) Common contact dermatitis would be the correct option if the patient had a known exposure
to an irritant or allergen that triggered the skin inflammation.
E) Toxicodermia would be the correct option if the patient had a known exposure to a
medication or chemical that triggered the skin inflammation.
Medication:
dermatology

Treatment for common eczema may include topical corticosteroids to reduce inflammation and
itching, moisturizers to keep the skin hydrated, and antihistamines to relieve itching. In severe
cases, oral corticosteroids or immunosuppressants may be necessary.
Words:
• Eczema: a group of conditions that cause inflammation of the skin, characterized by redness,
itching, and the formation of small bumps, blisters, or scales.
Findings:
• No specific findings mentioned, other than the symptoms described in the question.

Therapy
dermatology

96 Erysipelas

A patient complains of skin painfullness and reddness of the right gastrocnemius muscle.
Objectively: body temperature is 38,50С, enlarged and painful inguinal lymph nodes on
the right. Skin of extremity is edematic, hyperemic, covered with eruption in form of vesicles
containing dark fluid; its palpation is painful. There is distinct border between normal and
hyperemic skin. What is the most probable diagnosis?
A Erysipelas, hemorrhagic form
B Anthrax, dermal form
C Herpetic infection
D Chickenpox
E Crus phlegmon

Correct Answer: A) Erysipelas, hemorrhagic form is the most probable diagnosis.


Key Points:
• Patient with skin painfulness and redness of the right gastrocnemius muscle.
• Enlarged and painful inguinal lymph nodes on the right.
• Skin of the extremity is edematous, hyperemic, and covered with vesicles containing dark
fluid.
• Palpation of the skin is painful.
• Distinct border between normal and hyperemic skin.
Explanation:

Therapy
Erysipelas is a bacterial skin infection caused by Streptococcus bacteria that affects the upper
dermis and superficial lymphatics. The infection is usually characterized by a raised, well-
defined, and fiery-red rash with a sharply demarcated border. The affected area is typically
painful, tender, and warm to the touch. In severe cases, the infection may cause blistering and
hemorrhage, leading to the hemorrhagic form of erysipelas. The diagnosis is usually based on
the patient's history and clinical examination. In some cases, a skin biopsy or bacterial culture
may be necessary to confirm the diagnosis.
Other Options:
B) Anthrax, dermal form is a bacterial infection caused by Bacillus anthracis that affects the
skin. It typically causes a raised, itchy, and painless lesion with a black center. However, the
patient's symptoms are not consistent with anthrax.
C) Herpetic infection is a viral infection caused by the herpes simplex virus that typically causes
a painful rash with fluid-filled blisters. However, the patient's symptoms are not consistent with
a herpetic infection.
D) Chickenpox is a viral infection caused by the varicella-zoster virus that typically causes a
rash with fluid-filled blisters. However, the patient's symptoms are not consistent with
chickenpox.
E) Crus phlegmon is a severe bacterial infection that affects the soft tissue of the leg. It typically
causes redness, swelling, and pain. However, the patient's symptoms are not consistent with a
crus phlegmon.
Making Other Options Right:
B) Anthrax, dermal form would be the correct option if the patient had a known exposure to
Bacillus anthracis and presented with a skin lesion with a black center.
C) Herpetic infection would be the correct option if the patient had a history of recurrent
herpetic infections and presented with a painful rash with fluid-filled blisters.
D) Chickenpox would be the correct option if the patient had a history of not being vaccinated
against varicella-zoster virus and presented with a rash with fluid-filled blisters.
E) Crus phlegmon would be the correct option if the patient had a severe bacterial infection
affecting the soft tissue of the leg.
Medication:
dermatology

Treatment for erysipelas typically involves antibiotics, such as penicillin or erythromycin, to


eliminate the bacterial infection. Pain relief medications, such as acetaminophen or nonsteroidal
anti-inflammatory drugs (NSAIDs), may also be used to relieve pain and fever. In severe cases,
hospitalization and intravenous antibiotics may be necessary.
Words:
• Hemorrhagic form of erysipelas: a severe form of erysipelas characterized by blistering and
hemorrhage.

Therapy
dermatology

97 Erysipelas

A 43-year-old female patiet complains of eruption on her right leg skin, pain, weakness,
body temperature rise up to 38oC. The disease is acute. Objectively: there is an edema
on the right leg skin in the region of foot, a well-defined bright red spot in form of flame tips
which feels hot. There are isolated vesicles in focus. What is your provisional diagnosis?
A Erysipelas
B Microbial eczema
C Contact dermatitis
D Toxicoderma
E Haemorrhagic vasculitis

Correct Answer: A Erysipelas is the correct answer based on the patient's symptoms of an acute
eruption on the right leg skin with pain, weakness, fever, and edema, along with a well-defined
bright red spot in the form of flame tips with isolated vesicles in the focus.
Key Points:
• 43-year-old female patient
• Complaints of an eruption on the right leg skin, pain, weakness, and fever
• Acute disease
• Objective findings of edema on the right leg skin in the region of the foot, a well-defined bright
red spot in the form of flame tips, and isolated vesicles in the focus
Explanation:

Therapy
Erysipelas is a bacterial skin infection that affects the upper dermis and superficial lymphatics,
caused primarily by Streptococcus pyogenes. The infection usually starts with a small area of
skin that becomes red, swollen, and painful, and then spreads rapidly to involve a larger area.
The affected area is typically warm to the touch and may be associated with fever, chills, and
general malaise.
The diagnosis of erysipelas is usually made based on the clinical presentation and physical
examination findings. The typical features include a well-defined, raised, bright red spot in the
form of flame tips with surrounding edema, and the presence of isolated vesicles within the
focus.
The treatment of erysipelas involves prompt initiation of antibiotic therapy with penicillin,
amoxicillin, or other beta-lactam antibiotics, and supportive measures to manage the fever and
pain.
Other Options:
B) Microbial eczema: Microbial eczema is not the correct answer because it typically presents
with a chronic, relapsing course, and does not cause the acute symptoms seen in erysipelas.
C) Contact dermatitis: Contact dermatitis is not the correct answer because it typically presents
with a rash in areas of skin that have come into contact with an allergen or irritant, and does
not cause the acute symptoms seen in erysipelas.
D) Toxicoderma: Toxicoderma is not the correct answer because it refers to a group of skin
disorders caused by exposure to drugs, chemicals, or toxins, and does not cause the acute
symptoms seen in erysipelas.
E) Haemorrhagic vasculitis: Haemorrhagic vasculitis is not the correct answer because it
typically presents with palpable purpura, joint pain, and abdominal pain, and does not cause the
acute symptoms seen in erysipelas.
Making Other Options Right:
B) Microbial eczema: Microbial eczema would be a correct answer if the patient presented with a
chronic, relapsing rash that was associated with bacterial infection.
C) Contact dermatitis: Contact dermatitis would be a correct answer if the patient had a history
of exposure to an allergen or irritant that was associated with the rash.
D) Toxicoderma: Toxicoderma would be a correct answer if the patient had a history of exposure
to a drug, chemical, or toxin that was associated with the rash.
dermatology

E) Haemorrhagic vasculitis: Haemorrhagic vasculitis would be a correct answer if the patient


presented with palpable purpura, joint pain, and abdominal pain.
Medication:
The treatment of erysipelas involves prompt initiation of antibiotic therapy with penicillin,
amoxicillin, or other beta-lactam antibiotics, and supportive measures to manage the fever and
pain.
Words:
Erysipelas - a bacterial skin infection that affects the upper dermis and superficial lymphatics
Edema - the abnormal accumulation of fluid in the interstitial spaces of tissues
Findings:
There are isolated vesicles within the focus of the erysipelas, which are small fluid-filled blisters
that can be seen in various skin conditions.

Therapy
dermatology

98 Furuncle

A patient at a doctor complains of temperature rise up to 38,20С, edema in the region


of his upper lip. Objectively: upper lip is evidently edematic, in the middle of edema there is
a cone-shaped swelling. Skin and mucous membrane over it are dark-red. Diagnosis: labial
furuncle. A surgeon cut the furuncle, treated the wound with hydrogen peroxide solution
and applied a bandage with hypertensive solution. What therapeutic regimen should be
recommended?
A In-patient treatment with common regimen
B First out-patient treatment, then out-patient treatment
C Out-patient treatment
D In-patient treatment with bed rest
E-

Correct Answer: A. In-patient treatment with common regimen.


Key Points:
• The patient has a labial furuncle, which is a common skin infection caused by bacteria entering
a hair follicle.
• The furuncle has been surgically drained and treated with topical antiseptic.
• In-patient treatment may be necessary for patients with severe or complicated furuncles, or
for those with underlying medical conditions that increase the risk of complications.
Explanation:

Therapy
• A furuncle, also known as a boil, is a skin infection caused by the bacterium Staphylococcus
aureus.
• Furuncles typically develop as a painful, red, swollen bump on the skin, and may contain pus.
• Treatment for furuncles involves surgical drainage and/or antibiotics, depending on the
severity of the infection.
• In this case, the furuncle has been surgically drained and treated with hydrogen peroxide
solution and hypertonic bandage.
• In-patient treatment may be necessary for patients with severe or complicated furuncles, or
for those with underlying medical conditions that increase the risk of complications.
• In-patient treatment may involve intravenous antibiotics, wound care, and monitoring for
signs of systemic infection or complications.
Other Options:
• Option B (First out-patient treatment, then out-patient treatment) is not a practical option as
it does not provide a clear plan for follow-up or management, and may not be appropriate for
severe or complicated furuncles.
• Option C (Out-patient treatment) is appropriate for uncomplicated furuncles, but may not be
sufficient for severe or complicated cases.
• Option D (In-patient treatment with bed rest) may be appropriate for some patients with
severe or complicated furuncles, but is not necessary for all cases.
Making Other Options Wrong:
• Options B and C may not be appropriate for severe or complicated furuncles, which may
require more intensive in-patient treatment.
• Option D is not necessary for all cases of furuncles, and may not be appropriate for patients
with underlying medical conditions that limit mobility.
Medication:
• The use of antibiotics for furuncles is controversial, and is typically reserved for cases with
significant surrounding cellulitis or systemic symptoms.
• Topical antiseptics such as hydrogen peroxide or povidone-iodine may be used to clean the
wound and prevent infection.
Words:
dermatology

• Furuncle: A skin infection caused by bacteria entering a hair follicle, resulting in a painful, red,
swollen bump on the skin.
• Staphylococcus aureus: A type of bacteria that commonly causes skin infections such as
furuncles and cellulitis.
• Surgical drainage: A medical procedure in which a healthcare provider makes an incision in the
skin to allow for the release of pus and other fluids from an infected area.
• Topical antiseptics: Medications applied to the skin to prevent or treat infection.
Findings:
• Furuncles are a common and usually uncomplicated skin infection that can be managed on an
out-patient basis in most cases.
• Surgical drainage is the preferred treatment for furuncles, with antibiotics reserved for cases
with significant surrounding cellulitis or systemic symptoms.
• In-patient treatment may be necessary for patients with severe or complicated furuncles, or
for those with underlying medical conditions that increase the risk of complications.

Therapy
dermatology

99 Geographic Tongue

Medical examination of a man revealed "geographic tongue". This microsymptom is the


evidence of the following vitamin deficiency:
A Vitamins of B group
B Vitamin A
C Vitamin C
D Vitamin D
E Vitamin PP

Correct Answer: Option A, Vitamins of B group, is the correct answer.


Key Points:
• Medical examination of a man revealed "geographic tongue."
• The microsymptom is the evidence of a vitamin deficiency.
• The question is asking for the specific vitamin deficiency.
Explanation:
• Geographic tongue is a benign condition that affects the tongue's surface, making it appear
like a map or geographic area.
• The exact cause of geographic tongue is unknown, but it has been associated with multiple
factors, including genetics, immune system dysfunction, and environmental factors.
• One of the possible causes of geographic tongue is a deficiency in B vitamins, specifically B2
(riboflavin), B3 (niacin), B6 (pyridoxine), and B12 (cobalamin).

Therapy
• B vitamins are essential for the proper functioning of the body, including the maintenance of
healthy skin, nerves, and red blood cells.
• Vitamin deficiencies can cause a range of symptoms, including glossitis (inflammation of the
tongue), dry and cracked lips, and mouth ulcers.
• In the case of geographic tongue, a deficiency in B vitamins can lead to inflammation of the
tongue's surface and the characteristic pattern of the condition.
Other Options:
• Option B, Vitamin A, is incorrect because a deficiency in this vitamin can cause a range of
symptoms, including dry and rough skin, night blindness, and impaired immune function, but it
is not associated with geographic tongue.
• Option C, Vitamin C, is incorrect because a deficiency in this vitamin can lead to scurvy, which
causes bleeding gums, loose teeth, and joint pain, but it is not associated with geographic
tongue.
• Option D, Vitamin D, is incorrect because a deficiency in this vitamin can lead to rickets, which
causes soft and weak bones, but it is not associated with geographic tongue.
• Option E, Vitamin PP (Niacin), is incorrect because it is a B vitamin and is already included in
option A.
Making Other Options Right:
• Option B could be made correct by changing the microsymptom to "dry and rough skin" or
"night blindness."
• Option C could be made correct by changing the microsymptom to "bleeding gums" or "joint
pain."
• Option D could be made correct by changing the microsymptom to "soft and weak bones" or
"muscle weakness."
• Option E is a B vitamin and is already included in option A.
Medication:
• The treatment for a deficiency in B vitamins depends on the severity of the deficiency and the
underlying cause.
• Mild deficiencies can be treated with dietary changes or oral supplements, while severe
deficiencies may require intravenous administration of the deficient vitamin.
dermatology

• Common dietary sources of B vitamins include whole grains, meat, fish, eggs, dairy products,
and leafy green vegetables.
Words:
• Geographic tongue: The term "geographic tongue" comes from the tongue's appearance,
which resembles a map or a geographic area.

Therapy
dermatology

100 Herpetic Vesicular Dermatitis

A 26-year-old male patient complains of a rash on the upper lip skin, which arose on a
background of influenza with high-grade fever and is accompanied by pain and burning. The
rash has been present for 3 days. Objectively: the skin of the upper lip is edematic and
erythematous, grouped vesicles are filled with serous fluid and have a rough surface. What is
the most likely diagnosis?
A Herpetic vesicular dermatitis
B Eczema
C Contact dermatitis
D Dermatitis herpetiformis
E Erythema multiforme

Correct Answer: Option A (Herpetic vesicular dermatitis) is the correct answer because the
patient's symptoms and findings, including a vesicular rash on the upper lip that arose on a
background of influenza with high-grade fever, are consistent with a diagnosis of herpes simplex
virus (HSV) infection, which commonly causes recurrent vesicular eruptions in the orofacial
region.
Key Points:
• 26-year-old male patient with a vesicular rash on the upper lip that arose on a background of
influenza with high-grade fever and is accompanied by pain and burning
• Rash has been present for 3 days and consists of edematous and erythematous skin with

Therapy
grouped vesicles filled with serous fluid and a rough surface
• Most likely diagnosis is herpetic vesicular dermatitis
Explanation:
• Herpetic vesicular dermatitis refers to a viral infection caused by the herpes simplex virus
(HSV), which commonly causes recurrent blister-like eruptions in the orofacial region.
• Symptoms of herpetic vesicular dermatitis may include pain, burning, and itching, as well as
systemic symptoms such as fever and malaise.
• Diagnosis of herpetic vesicular dermatitis may involve visual inspection of the rash, as well as
viral culture or polymerase chain reaction (PCR) testing to confirm the presence of the herpes
simplex virus.
• Treatment for herpetic vesicular dermatitis may involve antiviral medications such as
acyclovir, valacyclovir, or famciclovir, as well as supportive care such as pain relief and
hydration.
Other Options:
• Option B (Eczema) is less likely given the acute onset of symptoms and the presence of
grouped vesicles, which are not typically seen in eczematous eruptions.
• Option C (Contact dermatitis) is less likely given the absence of a clear history of contact with
an irritant or allergen, as well as the presence of systemic symptoms such as fever and malaise.
• Option D (Dermatitis herpetiformis) is less likely given the absence of other signs or symptoms
of celiac disease or gluten intolerance, as well as the lack of a clear association between
dermatitis herpetiformis and influenza.
• Option E (Erythema multiforme) is less likely given the absence of other signs or symptoms of
erythema multiforme, as well as the presence of grouped vesicles that are not typically seen in
erythema multiforme.
Making Other Options Right:
• Option B (Eczema) could be made correct if the question specified additional details that were
consistent with eczematous eruptions, such as a history of atopic dermatitis or a characteristic
distribution of the rash.
• Option C (Contact dermatitis) could be made correct if the question provided additional details
that were consistent with a history of exposure to an irritant or allergen, such as a recent
change in cosmetics or personal care products.
dermatology

• Option D (Dermatitis herpetiformis) could be made correct if the question specified additional
signs or symptoms of celiac disease or gluten intolerance, such as gastrointestinal symptoms or
laboratory abnormalities.
• Option E (Erythema multiforme) could be made correct if the question specified additional
signs or symptoms of erythema multiforme, such as target lesions or mucosal involvement.
Medication:
• Treatment for herpetic vesicular dermatitis may involve antiviral medications such as
acyclovir, valacyclovir, or famciclovir, which work by inhibiting viral replication and reducing the
duration and severity of symptoms.
• Supportive care for herpetic vesicular dermatitis may include pain relief with analgesics such
as acetaminophen or nonsteroidal anti-inflammatory drugs (NSAIDs), as well as hydration with
oral fluids or intravenous fluids if necessary.
Words:
• Herpetic vesicular dermatitis: a viral infection caused by the herpes simplex virus (HSV),
which commonly causes recurrent blister-like eruptions in the orofacial region
• Vesicle: a blister-like lesion filled with serous fluid
• Erythema: redness of the skin or mucous membranes, typically due to inflammation
• Influenza: a viral respiratory infection characterized by fever, cough, and muscle aches
• Polymerase chain reaction (PCR) testing: a laboratory technique used to amplify and detect
specific segments of DNA or RNA, often used to diagnose viral infections
Findings:
• Normal values for this question are not applicable, as laboratory or other objective findings
were not provided.

Therapy
dermatology

101 Hydradenitis

A 23 year old patient fell ill 3 weeks ago when she noticed a very painful induration in her
axillary crease. 4-5 days later it burst and discharged a lot of pus. After that some new
infiltrations appeared around the affected area. The patient has never suffered from skin
diseases before. What is the most probable diagnosis?
A Hydradenitis
B Furuncle
C Mycosis
D Herpes zoster
E Streptococcal impetigo

Correct Answer: Option A (Hydradenitis) is the correct answer because the patient's symptoms
of a painful induration in the axillary crease that burst and discharged pus, followed by the
appearance of new infiltrations, are consistent with the diagnosis of hydradenitis, also known as
hidradenitis suppurativa.
Key Points:
• 23-year-old female patient
• Symptoms of a painful induration in the axillary crease that burst and discharged pus, followed
by the appearance of new infiltrations
• No history of skin diseases
Explanation:

Therapy
• Hydradenitis is a chronic inflammatory skin condition characterized by recurrent abscesses and
sinus tracts in the apocrine gland-bearing areas of the body, such as the axillae, groin, and
perianal region.
• The etiology of hydradenitis is not fully understood, but it is believed to be caused by a
combination of genetic, immunological, and environmental factors.
• Symptoms include painful, inflamed nodules or abscesses that may rupture and discharge pus,
as well as the development of sinus tracts and scarring.
• Diagnosis is based on clinical presentation and biopsy if necessary.
• Treatment includes topical and/or systemic antibiotics to control bacterial infection, as well as
surgical interventions such as incision and drainage, excision, or laser therapy.
Other Options:
• Option B (Furuncle) is incorrect because furuncles, also known as boils, are localized skin
infections that involve a hair follicle and its surrounding tissue. They typically present as tender,
erythematous nodules that progress to a pustule and eventually drain. However, furuncles are
usually solitary lesions, whereas hydradenitis involves multiple abscesses and sinus tracts.
• Option C (Mycosis) is incorrect because mycosis, or fungal infection, does not typically present
with painful nodules or abscesses. Fungal infections of the skin may present with scaling,
itching, and erythema.
• Option D (Herpes zoster) is incorrect because herpes zoster, or shingles, presents with a
painful, unilateral vesicular rash in a dermatomal distribution. It is typically preceded by
prodromal symptoms such as burning, tingling, or itching. The patient's symptoms of a painful
induration in the axillary crease that burst and discharged pus are not consistent with herpes
zoster.
• Option E (Streptococcal impetigo) is incorrect because streptococcal impetigo presents with a
superficial, crusted rash that may be pruritic. It is caused by group A streptococcus and typically
affects children. The patient's symptoms of a painful induration in the axillary crease that burst
and discharged pus are not consistent with streptococcal impetigo.
Making Other Options Right:
• Option B (Furuncle) would be correct if the question mentioned a solitary lesion rather than
multiple infiltrations.
dermatology

• Option C (Mycosis) would be correct if the question mentioned symptoms of scaling, itching, or
erythema rather than painful nodules or abscesses.
• Option D (Herpes zoster) would be correct if the question mentioned a painful, unilateral
vesicular rash in a dermatomal distribution rather than a painful induration in the axillary
crease.
• Option E (Streptococcal impetigo) would be correct if the question mentioned a superficial,
crusted rash that may be pruritic, rather than a painful induration in the axillary crease.
Medication:
• The treatment for hydradenitis involves antibiotics to control bacterial infection. Topical or
systemic antibiotics such as clindamycin, tetracycline, or a combination of rifampicin and
clindamycin may be used.
• The mechanism of action of clindamycin is to inhibit bacterial protein synthesis, leading to
bacterial cell death. Common side effects include diarrhea, nausea, and vomiting.
• The mechanism of action of tetracycline is to inhibit bacterial protein synthesis by binding to
the bacterial ribosome. Common side effects include nausea, vomiting, and photosensitivity.
• The mechanism of action of rifampicin is to inhibit bacterial RNA synthesis, leading to bacterial
cell death. Common side effects include nausea, vomiting, and hepatotoxicity.
Words:
• Hydradenitis: hydra- (sweat gland) + adenitis (inflammation of a gland)
• Hidradenitis suppurativa: alternative name for hydradenitis
• Mycosis: a fungal infection

Therapy
dermatology

102 Pigment nevi

A 28 y.o. woman consulted a doctor with the complaints of enlargening in size of an inborn
pigment nevus, it was also wetting and itching. What test should not be used for
diagnostics in this case?
A Incision biopsy
B Yaks's reaction
C Radioisotope diagnostics
D Termography
E Glass-print

Correct Answer: Option A, Incision biopsy, should not be used for diagnostics in this case.
Key Points: A 28-year-old woman complains of enlargening in size of an inborn pigment nevus
that is also wetting and itching. The question asks what test should not be used for diagnosis.
Explanation: An incision biopsy should not be used to diagnose a pigmented nevus that is
suspected of being melanoma. An incision biopsy can disturb the structure of the nevus and
interfere with further diagnosis. In this case, other diagnostic methods should be used, such as
dermatoscopy, which is a non-invasive imaging technique used to examine pigmented skin
lesions.
Other Options:
Option B, Yaks's reaction, is a skin test that can be used to diagnose tuberculosis, so it is not
relevant to this case.

Therapy
Option C, radioisotope diagnostics, is a technique that uses radioactive isotopes to diagnose
disease. It is not commonly used in the diagnosis of pigmented nevi or melanoma.
Option D, termography, is a technique that uses infrared radiation to detect heat patterns in the
body. It is not commonly used in the diagnosis of pigmented nevi or melanoma.
Option E, glass-print, is a method of diagnosing skin diseases by taking a print of the skin on
glass and examining it under a microscope. It is not commonly used in the diagnosis of
pigmented nevi or melanoma.
Making Other Options Right:
Option B, Yaks's reaction, is a skin test that can be used to diagnose tuberculosis, so it is not
relevant to this case.
Option C, radioisotope diagnostics, can be used to diagnose certain types of cancer, but it is not
commonly used in the diagnosis of pigmented nevi or melanoma.
Option D, termography, is not commonly used in the diagnosis of pigmented nevi or melanoma.
Option E, glass-print, is not commonly used in the diagnosis of pigmented nevi or melanoma.
Words:
• Pigment nevus: A pigmented skin lesion also known as a mole.
• Dermatoscopy: A non-invasive imaging technique used to examine pigmented skin lesions.
dermatology

103 Psoriasis

A 45-year-old patient complains of some painless nodular elements tending to peripheral


growth and fusion. He has a 2-year history of this disease. Aggravation takes place mainly
in spring. In anamnesis: the patient's father had similar skin lesions. Objectively:
pathological elements looke like guttate and nummular nodules, plaques covered with white
scales. What is your provisional diagnosis?
A Psoriasis
B Lichen ruber planus
C Neurodermitis
D Pityriasis rosea
E Seborrheic eczema

Correct Answer: A Psoriasis - The patient's symptoms of painless nodular elements with
peripheral growth and fusion, covered with white scales, suggest a diagnosis of psoriasis.
Key Points:
• 45-year-old patient with a 2-year history of painless nodular elements
• Lesions tend to peripheral growth and fusion
• Aggravation in spring
• Family history of similar skin lesions
• Pathological elements look like guttate and nummular nodules, plaques covered with white
scales

Therapy
Explanation:
Psoriasis is a chronic autoimmune disorder that affects the skin, characterized by the
development of thick, scaly patches of skin. The exact cause of psoriasis is unknown, but it is
thought to be related to an overactive immune system and genetic predisposition.
The symptoms of psoriasis typically include painless, red, raised patches of skin that are
covered with silvery-white scales. These patches may appear on any part of the body, but are
most commonly found on the scalp, elbows, knees, and lower back. In some cases, psoriasis
may also present as small, guttate or nummular nodules that tend to peripheral growth and
fusion.
Psoriasis is a chronic condition that may have periods of exacerbation and remission.
Exacerbations may be triggered by factors such as stress, infections, and changes in climate or
weather.
Diagnosis of psoriasis is typically made through clinical evaluation and biopsy. Treatment may
include topical or systemic medications, as well as phototherapy or other forms of light therapy.
Other Options:
B Lichen ruber planus - Lichen ruber planus is a chronic inflammatory skin condition that
typically presents as itchy, flat-topped, pink or purple bumps on the skin. The patient's
symptoms of painless nodular elements with white scales are not typical of lichen ruber planus.
C Neurodermitis - Neurodermitis, also known as atopic dermatitis, is a chronic inflammatory skin
condition that typically presents with itching, redness, and eczematous lesions. The patient's
symptoms of painless nodular elements with white scales are not typical of neurodermitis.
D Pityriasis rosea - Pityriasis rosea is a relatively common skin condition that presents as a rash
of oval-shaped, pink or red patches with fine scales. The patient's symptoms of painless nodular
elements with white scales are not typical of pityriasis rosea.
E Seborrheic eczema - Seborrheic eczema is a common skin condition that presents with red,
scaly patches on the skin, usually on the scalp, face, and chest. The patient's symptoms of
painless nodular elements with white scales are not typical of seborrheic eczema.
Making Other Options Right:
B Lichen ruber planus - This option could be made right if the patient's symptoms were more
consistent with lichen ruber planus, such as the presence of flat-topped, pink or purple bumps
on the skin that are itchy and scaly.
dermatology

C Neurodermitis - This option could be made right if the patient's symptoms were more
consistent with neurodermitis, such as the presence of eczematous lesions that are itchy and
red.
D Pityriasis rosea - This option could be made right if the patient's symptoms were more
consistent with pityriasis rosea, such as the presence of oval-shaped, pink or red patches with
fine scales.
E Seborrheic eczema - This option could be made right if the patient's symptoms were more
consistent with seborrheic eczema, such as the presence of red, scaly patches on the skin that
are itchy and located on the scalp, face, and chest.
Medication:
• Treatment of psoriasis may include topical or systemic medications, such as topical
corticosteroids, vitamin D analogues, and retinoids. Systemic medications such as methotrexate,
cyclosporine, and biologic agents may also be used in more severe cases.
• Other treatment options for psoriasis may include phototherapy or light therapy, which
involves exposing the skin to ultraviolet light.
Words:
• Psoriasis: a chronic autoimmune disorder that affects the skin, characterized by the
development of thick, scaly patches of skin.
• Guttate: small, drop-like lesions.
• Nummular: coin-shaped lesions.
• Lichen ruber planus: a chronic inflammatory skin condition that typically presents as itchy,
flat-topped, pink or purple bumps on the skin.
• Neurodermitis: also known as atopic dermatitis, a chronic inflammatory skin condition that
typically presents with itching, redness, and eczematous lesions.
• Pityriasis rosea: a relatively common skin condition that presents as a rash of oval-shaped,
pink or red patches with fine scales.

Therapy
• Seborrheic eczema: a common skin condition that presents with red, scaly patches on the
skin, usually on the scalp, face, and chest.
Findings:
• 45-year-old patient with a 2-year history of painless nodular elements
• Lesions tend to peripheral growth and fusion
• Aggravation in spring
• Family history of similar skin lesions
• Pathological elements look like guttate and nummular nodules, plaques covered with white
scales
dermatology

104 Psoriasis

A triad of symptoms ("stearing spot", "terminal film", "blood dew") has been revealed on
examination of a patient. What disease should you think about?
A Psoriasis
B Lichen ruber planus
C Vasculitis
D Seborrhea
E Ritter's disease

Correct Answer: A - Psoriasis


Key Points: Triad of symptoms - "steering spot", "terminal film", "blood dew"
Explanation: The triad of symptoms, "steering spot", "terminal film", and "blood dew" is
characteristic of psoriasis, a chronic autoimmune disorder that affects the skin. The "steering
spot" is a small, red, scaly patch that appears on the skin and eventually expands into the
characteristic psoriatic plaque with a silvery-white surface and well-defined edges. The "terminal
film" refers to the formation of a film of white, silvery scales that cover the plaque. Finally, the
"blood dew" refers to the punctate bleeding that can occur when the scales are removed.
Psoriasis is a chronic condition that tends to flare up periodically, and while it can affect any part
of the body, it most commonly affects the elbows, knees, scalp, and lower back. Psoriasis is a
chronic condition with periods of remission and flare-ups.
Other Options:

Therapy
B - Lichen ruber planus: Lichen planus is a non-infectious inflammatory condition that affects
the skin and mucous membranes. It typically presents with a flat, itchy, purplish rash on the
skin or white, lacy patches on the mucous membranes. There is no triad of symptoms
associated with lichen planus.
C - Vasculitis: Vasculitis is a group of diseases characterized by inflammation and damage to
blood vessels. It can present with a wide range of symptoms depending on the type and location
of the affected blood vessels, but there is no specific triad of symptoms associated with
vasculitis.
D - Seborrhea: Seborrhea is a common skin condition that affects the scalp and face,
characterized by excessive oiliness and scaling. It does not present with the triad of symptoms
described in the question.
E - Ritter's disease: Ritter's disease, also known as staphylococcal scalded skin syndrome, is a
bacterial skin infection that primarily affects infants and young children. It presents with
blistering and peeling of the skin, but does not present with the triad of symptoms described in
the question.
Making Other Options Right:
B) To make Lichen ruber planus the right answer, the triad of symptoms should be changed to
itchy, flat-topped, polygonal papules and plaques.
C) To make Vasculitis the right answer, the triad of symptoms should be changed to
inflammation and damage to blood vessels, fever, and joint pain.
D) To make Seborrhea the right answer, the triad of symptoms should be changed to red, scaly
patches, dandruff, and itching.
E) To make Ritter's disease the right answer, the triad of symptoms should be changed to
blisters, skin peeling, and Staphylococcus aureus infection.
Words:
• "Steering spot": a term used to describe the characteristic erythematous plaque with well-
defined borders seen in psoriasis.
• "Terminal film": a term used to describe the silvery-white scale that can be seen on the
surface of the psoriatic plaque.
• "Blood dew": a term used to describe the small droplets of blood that can sometimes be seen
on the surface of the psoriatic plaque when the scale is removed.
dermatology

105 Rubromycosis

A 41-year-old patient cosulted a dermatologist about discoloration, thickening, brittleness of


toenails. These symptoms have been present for about five years. Objectively: nail plates in all
toes are thickened, of dirty yellow color, lustreless, crumble over the edge. Microscopy of the
nail plate material treated with alkali revealed mycelial filaments. Material inoculation onto
Sabouraud medium resulted in growth of the Trichophyton rubrum colony. What is the most
likely
diagnosis?
A Rubromycosis of toenails
B Candidal onychia
C Psoriasis of the nails
D Nail dystrophy
E Epidermophytosis of nails

Correct Answer: A Rubromycosis of toenails - The presence of mycelial filaments and growth of
Trichophyton rubrum colony on Sabouraud medium are indicative of rubromycosis, which is a
common fungal infection of the toenails.
Key Points:
• Symptoms of discoloration, thickening, and brittleness of toenails for 5 years
• Objective findings of thickened, yellow, lustreless, crumbly nail plates
• Microscopy reveals mycelial filaments and Sabouraud medium culture growth of Trichophyton

Therapy
rubrum
Explanation:
Rubromycosis, also known as tinea unguium or onychomycosis, is a fungal infection of the
toenails caused by dermatophytes, especially Trichophyton rubrum. It is characterized by
thickening, discoloration, and brittleness of the nails. The infection can be acquired from
contaminated surfaces or from other people with the infection. Risk factors for developing
onychomycosis include diabetes, immunosuppression, and peripheral vascular disease.
Microscopic examination of the nail plate material treated with alkali may reveal mycelial
filaments, and the fungal species can be cultured on Sabouraud medium. Treatment depends on
the severity of the infection and may include topical or oral antifungal medications.
Other Options:
B) Candidal onychia: Candidal onychia is a fungal infection of the nails caused by Candida
species. It is less common than rubromycosis and is usually seen in people with
immunosuppression or chronic paronychia. The clinical presentation is similar to rubromycosis,
but the microscopic appearance and culture results are different.
C) Psoriasis of the nails: Psoriasis can involve the nails and cause similar symptoms of
thickening, discoloration, and crumbly nails. However, the presence of mycelial filaments and
culture growth of Trichophyton rubrum rule out psoriasis as a diagnosis.
D) Nail dystrophy: Nail dystrophy is a term used to describe any condition that affects the
normal growth and appearance of the nails. It is a nonspecific term and does not provide a
specific diagnosis.
E) Epidermophytosis of nails: Epidermophytosis is another term for tinea pedis, a fungal
infection of the feet. It can spread to the toenails and cause onychomycosis, but the presence of
mycelial filaments and culture growth of Trichophyton rubrum make rubromycosis a more likely
diagnosis.
Making Other Options Right:
B) Candidal onychia could be a possible diagnosis if the culture results showed growth of
Candida species instead of Trichophyton rubrum.
C) Psoriasis of the nails could be a possible diagnosis if the microscopic examination and culture
results were negative for fungal elements and the patient had a history of psoriasis.
dermatology

D) Nail dystrophy could be a possible diagnosis if the clinical presentation was less specific and
there were no positive findings on microscopic examination or culture.
E) Epidermophytosis of nails could be a possible diagnosis if the culture results showed growth
of Epidermophyton floccosum instead of Trichophyton rubrum.
Medication:
• Oral or topical antifungal medications are the mainstay of treatment for rubromycosis.
• Commonly used medications include terbinafine, itraconazole, fluconazole, and ciclopirox.
Words:
• Rubromycosis: A fungal infection of the nails caused by dermatophytes, especially
Trichophyton rubrum.
• Mycelial filaments: The branching, thread-like structures of fungi.
• Sabouraud medium: A culture medium used for the isolation and identification of fungi.
Findings:
• Normal values for nail appearance and microscopy are not mentioned in the question.
• The presence of mycelial filaments and growth of Trichophyton rubrum colony on Sabouraud
medium are abnormal findings indicative of rubromycosis.
Antifungal medications can be classified into several categories based on their mechanism of
action, chemical structure, and spectrum of activity. The main categories of antifungal
medications include:
1. Polyenes: Polyenes bind to the fungal cell membrane and disrupt its integrity, leading to
leakage of intracellular contents and eventual cell death. Examples include amphotericin B and
nystatin. These medications are broad-spectrum and effective against most fungal infections,
but they can have significant side effects and require careful monitoring.
2. Azoles: Azoles inhibit the synthesis of ergosterol, a key component of the fungal cell
membrane, leading to membrane dysfunction and cell death. Examples include fluconazole,
itraconazole, and voriconazole. Azoles are effective against a wide range of fungal infections and

Therapy
are generally well-tolerated, but they can interact with other medications and have some
potential side effects.
3. Echinocandins: Echinocandins inhibit the synthesis of beta-glucan, a key component of the
fungal cell wall, leading to weakening and eventual lysis of the cell. Examples include
caspofungin, micafungin, and anidulafungin. Echinocandins are effective against many Candida
species and some Aspergillus species, but they are not active against other types of fungi.
4. Allylamines: Allylamines inhibit the synthesis of ergosterol by blocking the enzyme squalene
epoxidase. Examples include terbinafine and naftifine. Allylamines are effective against
dermatophyte infections and some Candida species.
The choice of antifungal medication and dose depends on the type and severity of the fungal
infection, the patient's medical history and comorbidities, and other factors. Some common
indications for antifungal medications include:
• Candidiasis: Fluconazole, itraconazole, voriconazole, caspofungin, micafungin, and
anidulafungin are all effective against Candida species.
• Aspergillosis: Voriconazole, isavuconazole, and echinocandins are effective against most
Aspergillus species.
• Cryptococcosis: Amphotericin B, fluconazole, and echinocandins are all options for treating
cryptococcal infections.
• Dermatophytosis: Terbinafine, itraconazole, and fluconazole are all effective against
dermatophytes.
• Histoplasmosis: Amphotericin B and itraconazole are options for treating histoplasmosis.
dermatology

106 Rubromycosis

A patient is being prepared for the operation on account of varix dilatation of lower
extremities veins. Examination of the patient's soles revealed flour-like desquamation along
the skin folds. All the toenails are greyish-yellow, thickened and partially decayed. What
dermatosis should be suspected?
A Rubromycosis
B Pityriasis versicolor
C Candidosis
D Microsporia
E Microbial eczema

Correct Answer: A Rubromycosis should be suspected.


Key Points: Varix dilatation of lower extremities veins, flour-like desquamation along the skin
folds, greyish-yellow, thickened, and partially decayed toenails
Explanation: Rubromycosis, also known as tinea corporis, is a dermatomycosis caused by the
fungi Trichophyton rubrum or Trichophyton mentagrophytes. It commonly affects the skin of the
feet, especially in the interdigital spaces. In addition to skin lesions, patients may also have
onychomycosis, which is an infection of the nails. The symptoms of rubromycosis include
scaling, erythema, and itching. It is often diagnosed by clinical appearance and confirmed by
microscopic examination and culture of the affected area. Treatment includes topical or systemic
antifungal agents, depending on the severity of the infection.

Therapy
Other Options:
B Pityriasis versicolor is a superficial fungal infection caused by Malassezia furfur. It is
characterized by hypopigmented or hyperpigmented macules that are typically found on the
trunk. It is not associated with desquamation along the skin folds or toenail abnormalities.
C Candidosis is a fungal infection caused by Candida albicans. It can occur in the oral cavity,
gastrointestinal tract, and genitourinary system. Skin manifestations of candidiasis are typically
erythematous with satellite lesions. It is not associated with desquamation along the skin folds
or toenail abnormalities.
D Microsporia is a fungal infection caused by Microsporum species. It typically affects the scalp,
but it can also involve the skin and nails. Skin lesions are typically circular with raised borders
and central clearing. It is not associated with desquamation along the skin folds or toenail
abnormalities.
E Microbial eczema is a skin inflammation that is caused by bacterial infection. It is
characterized by erythema, oozing, crusting, and scaling. It is not associated with desquamation
along the skin folds or toenail abnormalities.
Making Other Options Right:
B Pityriasis versicolor could be a possible diagnosis if the patient had hypopigmented or
hyperpigmented macules on the trunk.
C Candidosis could be a possible diagnosis if the patient had erythematous skin lesions with
satellite lesions in the affected area.
D Microsporia could be a possible diagnosis if the patient had circular skin lesions with raised
borders and central clearing in the affected area.
E Microbial eczema could be a possible diagnosis if the patient had erythema, oozing, crusting,
and scaling in the affected area.
Medication: Treatment for rubromycosis includes topical or systemic antifungal agents such as
terbinafine, itraconazole, and fluconazole. These agents work by inhibiting the synthesis of
ergosterol, which is an essential component of the fungal cell membrane. Possible side effects of
these medications include gastrointestinal symptoms, hepatotoxicity, and skin rash.
Words: Rubromycosis is derived from the Latin word "rubor," which means redness, and
"mycosis," which means fungal infection.
dermatology

107 Scabies

A female patient consulted a dermatologist about the rash on the trunk and extremities.
Objectively: interdigital folds, flexor surfaces of wrists and navel region are affected with pairs
of
nodulo-cystic eruptions and crusts. The rash is accompanied by skin itch that is getting stronger
at night. What external treatment should be administered?
A 20¥% benzyl benzoate emulsion
B 5¥% sulfuric ointment
C 2¥% sulfuric paste
D 5¥% naphthalan ointment
E 5¥% tetracycline ointment

Correct Answer: A - 20% benzyl benzoate emulsion - is the correct answer.


Key Points:
• Female patient with rash on the trunk and extremities.
• Interdigital folds, flexor surfaces of wrists, and navel region affected with pairs of nodulo-
cystic eruptions and crusts.
• Rash accompanied by skin itch that is getting stronger at night.
• External treatment needed.
Explanation:
• The clinical findings of nodulo-cystic eruptions and crusts on interdigital folds, flexor surfaces

Therapy
of wrists, and navel region with intense pruritus that worsens at night suggest scabies.
• Scabies is a parasitic infection caused by the mite Sarcoptes scabiei that is typically treated
with topical medications.
• 20% benzyl benzoate emulsion is an effective topical treatment for scabies, as it penetrates
the skin and kills the mites.
• The emulsion should be applied to the entire body, including the scalp and face, and left on for
at least 24 hours before being washed off.
Other Options:
• B: 5% sulfuric ointment is another topical treatment for scabies, but it is less effective than
benzyl benzoate and may cause skin irritation.
• C: 2% sulfuric paste is another topical treatment for scabies, but it is less effective than
benzyl benzoate and may cause skin irritation.
• D: 5% naphthalan ointment is not an effective treatment for scabies and is typically used for
other skin conditions such as psoriasis and eczema.
• E: 5% tetracycline ointment is not an effective treatment for scabies and is typically used for
bacterial infections.
Making Other Options Right:
• To make option B, 5% sulfuric ointment, right, the question would need to be changed to
describe a patient with a milder case of scabies, for whom sulfuric ointment would be an
appropriate treatment option.
• To make option C, 2% sulfuric paste, right, the question would need to be changed to describe
a patient with a milder case of scabies, for whom sulfuric paste would be an appropriate
treatment option.
• To make option D, 5% naphthalan ointment, right, the question would need to be changed to
describe a patient with psoriasis or eczema, for whom naphthalan ointment would be an
appropriate treatment option.
• To make option E, 5% tetracycline ointment, right, the question would need to be changed to
describe a patient with a bacterial skin infection, for whom tetracycline ointment would be an
appropriate treatment option.
Medication:
dermatology

• 20% benzyl benzoate emulsion - a topical treatment for scabies that penetrates the skin and
kills the mites.
Words:
• Scabies: a parasitic infection caused by the mite Sarcoptes scabiei, characterized by intense
pruritus and nodulo-cystic eruptions and crusts on the skin.
• Emulsion: a mixture of two immiscible liquids, such as oil and water, stabilized by an
emulsifying agent.
Findings:
• Nodulo-cystic eruptions and crusts on interdigital folds, flexor surfaces of wrists, and navel
region, indicating a rash suggestive of scabies.
• Intense pruritus that worsens at night is a characteristic symptom of scabies.

Therapy
dermatology

108 Sexually Transmitted diseases

The 28 y.o. woman applied to doctor because of limited loss of the hair. In the anamnesis -
she had frequent headache indisposition, arthromyalgia, fever, irregular casual sexual life,
drug user. RW is negative. What examination must be done first?
A Examination for HIV
B Examination for neuropathology
C Examination for gonorrhea
D Examination for fungi
E Examination for trichomoniasis

Correct Answer: A. Examination for HIV.


Key Points: The patient presents with limited hair loss and a history of headache, indisposition,
arthromyalgia, fever, irregular casual sexual life, and drug use. RW (Rapid Plasma Reagin test
for syphilis) is negative.
Explanation: The patient's history suggests a possible infectious cause of hair loss. Sexually
transmitted infections, particularly HIV, can cause hair loss, along with other symptoms like
fever and muscle aches. HIV-positive individuals are more susceptible to hair loss due to a
weakened immune system. Therefore, the first examination to be done is for HIV, followed by
other STIs like gonorrhea and trichomoniasis.
Other Options:
B. Examination for neuropathology - This option is incorrect because there are no indications of

Therapy
neuropathological symptoms or conditions mentioned in the patient's history.
C. Examination for gonorrhea - This option is incorrect because gonorrhea is not known to cause
hair loss, and other symptoms like discharge from the genitals or pain while urinating are not
mentioned in the patient's history.
D. Examination for fungi - This option is incorrect because fungal infections can cause hair loss,
but there are no indications of fungal infection in the patient's history.
E. Examination for trichomoniasis - This option is incorrect because although trichomoniasis is a
sexually transmitted infection that can cause itching and irritation, it is not typically associated
with hair loss.
Medication: None mentioned.
Words: RW - Rapid Plasma Reagin test for syphilis.
Findings: None mentioned.
dermatology

109 tinea corporis

A 30 y.o. patient has got multiple body skin rash consisting of small paired elements that
are scattered on the skin diorderly and mostly focally, they are accompanied by itch. The
rash appeared a few days after attending sport centre and sauna. What is the most
probable diagnosis?
A Scab
B Eczema
C Contact dermatitis
D Allergic dermatitis
E Neurodermitis

Correct Answer:
A - Scab
Key Points:
• Multiple body skin rash consisting of small paired elements
• Scattered on the skin disorderly, mostly focally
• Accompanied by itch
• Developed a few days after attending a sport center and sauna
Explanation:
Scab, or tinea corporis, is a dermatophyte fungal infection of the skin that typically presents as
a circular or oval-shaped rash with raised edges and a clear center. The rash may be

Therapy
accompanied by itching or burning and can occur on any part of the body.
In this case, the patient's symptoms and history of attending a sport center and sauna are
consistent with scab. The rash consisting of small paired elements scattered on the skin
disorderly and mostly focally is typical of scab. The itching also suggests a fungal infection.
Other Options:
B - Eczema:
Eczema is a chronic inflammatory skin condition that can cause a rash consisting of red, scaly
patches on the skin. The rash is typically not circular or oval-shaped, and the itching is usually
more severe than that seen in scab.
C - Contact dermatitis:
Contact dermatitis is a type of skin rash that occurs when the skin comes into contact with an
irritant or allergen. The rash is typically not circular or oval-shaped and usually occurs in areas
of skin that have come into contact with the irritant or allergen.
D - Allergic dermatitis:
Allergic dermatitis is a type of skin rash that occurs when the skin comes into contact with an
allergen. The rash is typically not circular or oval-shaped and usually occurs in areas of skin that
have come into contact with the allergen.
E - Neurodermitis:
Neurodermatitis, also known as lichen simplex chronicus, is a skin condition that causes
thickened, scaly patches of skin that are often accompanied by itching. The rash is typically not
circular or oval-shaped and usually occurs in areas of skin that have been repeatedly scratched.
Making Other Options Right:
B - Eczema:
If the patient had a chronic inflammatory skin condition with red, scaly patches on the skin,
then eczema would be the correct answer.
C - Contact dermatitis:
If the patient had a rash that occurred in areas of skin that had come into contact with an
irritant or allergen, then contact dermatitis would be the correct answer.
D - Allergic dermatitis:
If the patient had a rash that occurred in areas of skin that had come into contact with an
allergen, then allergic dermatitis would be the correct answer.
dermatology

E - Neurodermitis:
If the patient had thickened, scaly patches of skin that were often accompanied by itching and
appeared in areas of the skin repeatedly scratched, then neurodermatitis would be the correct
answer.
Medication:
Treatment for scab typically involves the use of topical antifungal medications, such as
clotrimazole, miconazole, or terbinafine. These medications can help to clear the infection and
relieve symptoms. In some cases, oral antifungal medications may be necessary for more
severe or widespread infections.
Words:
• Dermatophyte: A type of fungus that causes skin infections, including scab (tinea corporis).
Findings:
• Circular or oval-shaped rash with raised edges and a clear center: This is a characteristic
finding of scab.
• Paired elements scattered on the skin disorderly and mostly focally: This is also a
characteristic finding of scab.
• Itching: This is a common symptom of scab and other skin infections.

Therapy
dermatology

110 Toxicodermatosis

A 38-year-old female suddenly developed acute inflammatory rash in form of roseolas, papules,
vesicles that are scattered on the skin of trunk in irregular and predominantly focal manner. The
rash appeared a few hours after visiting a restaurant. The patient complains of itching skin.
What is the most likely diagnosis?
A Toxicodermatosis
B Atopic dermatitis
C Contact dermatitis
D Eczema
E-

Correct Answer: A - Toxicodermatosis is the most likely diagnosis.


Key Points:
• 38-year-old female
• Acute inflammatory rash
• Roseolas, papules, and vesicles
• Scattered on the skin of the trunk
• Appeared a few hours after visiting a restaurant
• Complaints of itching skin
Explanation:
Toxicodermatosis is a skin reaction that occurs due to exposure to certain toxins or drugs. In

Therapy
this case, the patient developed an acute inflammatory rash after visiting a restaurant, which
suggests exposure to some toxic substance or allergen.
The rash is characterized by roseolas, papules, and vesicles that are scattered on the skin of the
trunk in an irregular and predominantly focal manner. Itching is a common symptom of
toxicodermatosis.
Other Options:
B. Atopic dermatitis is a chronic inflammatory skin disease that is characterized by itching,
redness, and scaling of the skin. It does not present with acute inflammatory rash, roseolas, or
vesicles.
C. Contact dermatitis is an inflammatory skin condition that occurs due to exposure to an
irritant or allergen. It typically presents with a localized rash that is in contact with the offending
substance. The rash in this case is not localized and is not in contact with any specific
substance.
D. Eczema is a chronic inflammatory skin condition that is characterized by dry, itchy, and scaly
skin. It does not present with acute inflammatory rash, roseolas, or vesicles.
Making Other Options Right:
B. Atopic dermatitis can be made correct if the question mentions that the patient has a history
of chronic eczema or atopy.
C. Contact dermatitis can be made correct if the question specifies that the rash is localized and
in contact with a specific substance.
Medication:
The treatment for toxicodermatosis depends on the underlying cause and severity of the
reaction. In mild cases, the rash may resolve on its own without any treatment. However, in
severe cases, the following medications may be used:
• Antihistamines to relieve itching and allergic symptoms
• Corticosteroids to reduce inflammation and itching
• Topical creams or ointments to soothe the skin
Words:
• Toxicodermatosis: A skin reaction that occurs due to exposure to certain toxins or drugs.
• Roseolas: Small, pink, and slightly raised spots on the skin.
• Papules: Small, raised bumps on the skin.
dermatology

• Vesicles: Small, fluid-filled blisters on the skin.


Findings:
There are no lab findings mentioned in the question, but the rash is described as acute
inflammatory with roseolas, papules, and vesicles scattered on the skin of the trunk in an
irregular and predominantly focal manner.

Therapy
Emergency Medicine

111 Anaphylaxis

A 27-year-old patient with a history of ronchial asthma was stung by a bee. He had a
sensation of chest compression, breath shortage, difficult expiration, sense of heat in the
upper half of body, dizziness, apparent itch, convulsions. Objectively: noisy wheezing
breath, AP - 90/60 mm Hg, Ps- 110 bpm. Auscultation revealed weak rhythmic heart
sounds, rough respiration above lungs, sibilant rales. What drug group should be
administered in the first place?
A Glucocorticoids
B Methylxanthines
C Cardiac glycosides
D Anticonvulsive
E Analgetics

Correct Answer: A Glucocorticoids


Key Points:
• 27-year-old patient with a history of bronchial asthma was stung by a bee.
• The patient has symptoms of anaphylaxis, including chest compression, breath shortage,
difficult expiration, sense of heat in the upper half of the body, dizziness, apparent itch, and
convulsions.
• Objectively, the patient has noisy wheezing breath, low blood pressure, and a rapid heart rate.
• Auscultation reveals weak rhythmic heart sounds, rough respiration above the lungs, and

Therapy
sibilant rales.
Explanation:
• The patient's symptoms are consistent with anaphylaxis, a severe and potentially life-
threatening allergic reaction.
• Anaphylaxis can be triggered by insect stings, and patients with a history of asthma are at
increased risk of developing anaphylaxis.
• The patient's symptoms indicate respiratory distress, which is a medical emergency that
requires immediate intervention.
• Glucocorticoids, such as prednisone or methylprednisolone, are the first-line treatment for
anaphylaxis and should be administered as soon as possible to reduce inflammation and
stabilize the patient's condition.
• Glucocorticoids work by suppressing the immune system's response to allergens and reducing
inflammation in the airways and other organs.
• Other medications, such as epinephrine, may also be used in the treatment of anaphylaxis,
but glucocorticoids are the first-line treatment.
Other Options:B) Methylxanthines are a group of medications that are used to treat asthma and
other respiratory conditions by relaxing the smooth muscles in the airways. However, they are
not the first-line treatment for anaphylaxis, and glucocorticoids would be a more appropriate
choice in this scenario.C) Cardiac glycosides, such as digoxin, are medications that are used to
treat heart failure and certain arrhythmias. They are not indicated for the treatment of
anaphylaxis and would not be an appropriate choice in this scenario.D) Anticonvulsant
medications, such as benzodiazepines, are used to treat seizures and convulsions. While the
patient is experiencing convulsions, the underlying cause of the convulsions is anaphylaxis, and
the first-line treatment for anaphylaxis is glucocorticoids.E) Analgesics are medications used to
relieve pain. While the patient may be experiencing itching, which can be uncomfortable, the
primary concern is the respiratory distress caused by anaphylaxis, and the first-line treatment is
glucocorticoids.
Therefore, the correct answer is A) Glucocorticoids.
Medication: Glucocorticoids, such as prednisone or methylprednisolone, are the first-line
treatment for anaphylaxis. Other medications, such as epinephrine, may also be used in the
treatment of anaphylaxis, but glucocorticoids are the first-line treatment.
Emergency Medicine

Words: None of the words used in the question are uncommon.


Findings: The findings mentioned in the question are the patient's symptoms, which are
consistent with anaphylaxis, a severe and potentially life-threatening allergic reaction, as well as
objective findings, such as noisy wheezing breath, low blood pressure, and a rapid heart rate.

Therapy
Emergency Medicine

112 cardiac arrest

A 48 y.o. patient has been staying in the emergency department for 2 days on account of
acute anteroseptal myocardial infarction. In the course of examination he suddenly
"snored". There was a tonic contraction of skeletal muscles; eye pupils dilatated. Pulse on
a.carotis is absent. What is the immediate tactics?
A Electric defibrillation
B Saphar's triple airway maneuver
C ECG record
D Intracardiac introduction of adrenalin with atropine
E Precardiac stroke

Correct Answer: A
Key Points:
• 48-year-old patient with acute anteroseptal myocardial infarction.
• Sudden "snoring," tonic contraction of skeletal muscles, dilated pupils, and absent carotid
pulse.
Explanation:
The patient is experiencing cardiac arrest, which is a medical emergency that requires
immediate intervention to restore circulation and prevent brain injury. The sudden onset of
"snoring," tonic contraction of skeletal muscles, dilated pupils, and absent carotid pulse suggests
ventricular fibrillation, a type of cardiac arrhythmia that can lead to sudden cardiac arrest.

Therapy
The immediate tactic in this situation is electric defibrillation, which is the administration of an
electric shock to the heart to restore normal rhythm. Defibrillation is the most effective
treatment for ventricular fibrillation, and it should be performed as soon as possible to improve
the chances of survival.
Other Options:
B. Saphar's triple airway maneuver is a technique used to clear the airway in patients with
respiratory distress, but it is not relevant to a patient in cardiac arrest.
C. ECG record is a diagnostic tool that can help identify the type of arrhythmia and guide
therapy, but it is not an immediate tactic in a patient in cardiac arrest.
D. Intracardiac introduction of adrenalin with atropine is a treatment option for certain types of
cardiac arrhythmias, but it is not an immediate tactic in a patient in cardiac arrest.
E. Precardiac stroke, or cardiac massage, is a technique used to manually compress the heart to
restore circulation in a patient in cardiac arrest, but it is not the first-line treatment for
ventricular fibrillation.
Making Other Options Right:
B. Saphar's triple airway maneuver could be useful in a patient with respiratory distress who is
not in cardiac arrest.
C. ECG record is an important diagnostic tool in the management of cardiac arrhythmias, and it
should be performed as soon as possible in a stable patient.
D. Intracardiac introduction of adrenalin with atropine could be considered as a treatment option
for certain types of cardiac arrhythmias, but it is not the first-line treatment for ventricular
fibrillation.
E. Precardiac stroke, or cardiac massage, is a technique used to manually compress the heart to
restore circulation in a patient in cardiac arrest, and it can be used if defibrillation is not
immediately available.
Medication:
The medications used in the treatment of cardiac arrest include epinephrine, amiodarone, and
lidocaine, which can be administered before or after defibrillation. The medication options given
in the question are not relevant to the immediate management of ventricular fibrillation.
Words:
Emergency Medicine

Anteroseptal myocardial infarction - a type of heart attack that affects the front wall of the heart
and the septum.
Ventricular fibrillation - a type of cardiac arrhythmia characterized by rapid and irregular
contractions of the heart muscle.
Defibrillation - the administration of an electric shock to the heart to restore normal rhythm.
Arrhythmia - an abnormal heart rhythm.
Findings:
The absence of carotid pulse and the sudden onset of "snoring" and skeletal muscle contraction
suggest ventricular fibrillation, a type of cardiac arrhythmia. The absence of blood pressure and
altered mental status may indicate brain injury due to prolonged cardiac arrest. Immediate
intervention with defibrillation is necessary to restore circulation and prevent further organ
damage.

Therapy
Emergency Medicine

113 cardiac tamponade

A 20 y.o. man has a stab knife wound in the left half of thorax close to nipple. AP is 90/60
mm Hg, Ps- 130/min, BR- 32/min. During inspiration there is increase of pulse wave in the
region of jugular vein, decrease of peripheral arterial pulse and reduction of AP.
Respiratory murmurs are unchanged. X-ray pattern of thorax organs has no pecularities.
After introduction of 2 l of isotonic solution the AP stayed low, CVP raised up to 32 cm of
water column. The first step in further treatment of the patient will be:
A Echocardiogram
B Catheterization of left pleural cavity while the outer end of catheter is submerged in water
C Continued parenteral introduction of fliud in order to raise AP
D Introduction of peripheral vasodilatators in order to reduce CVP
E Introduction of loop diuretics in order to reduce CVP

Correct Answer: The correct option is A, which is to perform an echocardiogram.


Key Points: A 20-year-old man with a stab knife wound in the left half of his thorax close to the
nipple has a low AP, high Ps, and high BR. During inspiration, there is an increase in the pulse
wave in the jugular vein, a decrease in peripheral arterial pulse, and a reduction of AP. X-ray of
thorax organs shows no peculiarities. After the introduction of 2 liters of isotonic solution, the AP
remained low, and CVP increased up to 32 cm of water column.
Explanation: The clinical presentation of this patient suggests cardiac tamponade. Cardiac
tamponade is a medical emergency caused by the accumulation of fluid, pus, or blood in the

Therapy
pericardial sac, leading to an increase in intrapericardial pressure that impairs cardiac filling,
leading to reduced cardiac output and shock. In this case, the stab wound could have caused
bleeding into the pericardial sac, leading to cardiac tamponade.
Echocardiogram is the gold standard test to diagnose cardiac tamponade. It can show the
accumulation of fluid in the pericardial sac, signs of decreased cardiac filling, and right atrial and
ventricular collapse during diastole. Therefore, option A is the most reasonable measure to take
in this situation.
Other Options:
Option B: Catheterization of the left pleural cavity while the outer end of the catheter is
submerged in water is incorrect because this procedure is used to confirm the correct placement
of the pleural catheter in the pleural cavity during drainage of pleural effusions or
pneumothorax, which is not the case here.
Option C: Continued parenteral introduction of fluid in order to raise AP is incorrect because, in
this situation, the patient already received 2 liters of isotonic solution, and the AP remained low,
suggesting that the problem is not due to volume depletion.
Option D: Introduction of peripheral vasodilators in order to reduce CVP is incorrect because
peripheral vasodilators reduce systemic vascular resistance, which could lead to a further
decrease in AP and shock.
Option E: Introduction of loop diuretics in order to reduce CVP is incorrect because loop diuretics
can cause hypovolemia, which could exacerbate the patient's condition.
Words:
AP - arterial pressure
Ps - pulse
BR - breathing rate
CVP - central venous pressure
Findings:
The normal range of CVP is 2-8 cm H2O. In this case, the CVP raised up to 32 cm of water
column, indicating elevated intrapericardial pressure.
Emergency Medicine

114 Disaster

Explosion of a tank with benzene at a chemical plant has killed and wounded a large number of
people. There are over 50 victims with burns, mechanical injuries and intoxication. Specify the
main elements of medical care and evacuation of population in this situation:
A Sorting, medical assistance, evacuation
B Sorting, evacuation, treatment
C Medical assistance, evacuation, isolation
D Isolation, rescue activity, recovery
E Sorting, recovery, rescue activity

Correct Answer: A - Sorting, medical assistance, evacuation - is the correct answer as it includes
the essential elements of medical care and evacuation in a situation involving a chemical
explosion. The first priority is to perform a triage or sorting of the victims according to the
severity of their injuries and to provide medical assistance to those who need it urgently. The
next step is to evacuate the victims to safe locations while also ensuring the safety of the
rescuers and other personnel involved in the rescue efforts.
Key Points:
• Explosion of a tank with benzene at a chemical plant has resulted in over 50 victims with
burns, mechanical injuries, and intoxication.
• Main elements of medical care and evacuation are required in this situation.
Explanation:

Therapy
In a situation involving a chemical explosion, the first priority is to ensure the safety of the
rescuers and other personnel involved in the rescue efforts. This involves assessing the risk of
further explosions or chemical releases and taking appropriate measures to minimize the risk.
Once the area has been secured, the focus shifts to providing medical care and evacuation of
the victims.
Sorting or triage is an important first step in medical care and evacuation. It involves a rapid
assessment of the victims to determine the severity of their injuries and the urgency of their
need for medical care. The victims are then categorized into different groups based on the
severity of their injuries, and the most severely injured are given priority for medical care and
evacuation.
Medical assistance should be provided to the victims as quickly as possible, and this may involve
basic first aid, wound care, or more advanced medical interventions, depending on the severity
of their injuries. Victims with burns, mechanical injuries, and intoxication may require different
types of medical care, and the appropriate interventions should be provided based on the
specific needs of each victim.
Evacuation of the victims should be carried out as quickly and efficiently as possible, while also
ensuring the safety of the rescuers and other personnel involved in the rescue efforts. The
victims should be transported to safe locations, such as hospitals or other medical facilities,
where they can receive further medical care as needed.
Other Options:
B) Sorting, evacuation, treatment - This option includes two of the main elements of medical
care and evacuation (sorting and evacuation) but does not include medical assistance, which is
an essential component of disaster medicine.
C) Medical assistance, evacuation, isolation - While medical assistance and evacuation are
important elements of disaster medicine, isolation is not typically required in a situation
involving a chemical explosion unless there is a risk of further chemical releases or
contamination.
D) Isolation, rescue activity, recovery - Isolation and rescue activity may be necessary in a
situation involving a chemical explosion, but they are not the main elements of medical care and
evacuation. Recovery is also an important component of disaster medicine, but it is not one of
the main elements listed in the question.
Emergency Medicine

E) Sorting, recovery, rescue activity - Recovery and rescue activity are important components of
disaster medicine, but they are not the main elements of medical care and evacuation. Sorting
is included in this option, but medical assistance and evacuation, which are essential
components of disaster medicine, are not included.
Medication:
• The initial focus in a situation involving a chemical explosion is on providing medical care and
evacuation, rather than medication. However, depending on the specific injuries sustained by
the victims, they may require pain management, wound care, or other medications as part of
their treatment.
Words:
• Triage: the process of sorting patients based on the severity of their injuries or illness
• Benzene: a highly flammable and toxic chemical that is commonly used in the production of
plastics, synthetic fibers, and other materials
Findings:
• No laboratory or physical findings were mentioned in the question.

Therapy
Emergency Medicine

115 Drowning

A 14-year-old victim was drawn out of the water in winter after 15 minutes of being in the
water. The victim shows no vital signs. What measures are to be taken?
A To release respiratory tract from water, to create drain position and to take on measures
to restore respiration and blood circulation
B Not to waste time on the release of respiratory tract from water, to take on
cardiopulmonary reanimation
C To transport the victim to the nearest hospital to carry out reanimation measures
D Тo transport the victim to the nearest warm room to carry out reanimation measures
E Not to carry out reanimation measure

Correct Answer: A) To release respiratory tract from water, to create drain position and to take
on measures to restore respiration and blood circulation.
Key Points:
• A 14-year-old victim has been drawn out of the water in winter after being in the water for 15
minutes and is showing no vital signs.
• The question asks what measures are to be taken.
Explanation:
• Drowning is a medical emergency that requires immediate intervention to restore breathing
and circulation.
• The first step is to remove the victim from the water and release the respiratory tract from

Therapy
water, such as by tilting the head back and lifting the chin to open the airway.
• A drain position, with the victim lying on their side with their head downward, can help to clear
water from the airway and prevent aspiration.
• Once the airway is clear, measures should be taken to restore respiration and blood
circulation, such as by performing cardiopulmonary resuscitation (CPR).
• CPR involves chest compressions to circulate blood and rescue breaths to provide oxygen.
• CPR should be continued until emergency medical services (EMS) arrive or until the victim
shows signs of life.
Other Options:
B) Not to waste time on the release of respiratory tract from water, to take on cardiopulmonary
reanimation: This option is incorrect because the first step should be to release the respiratory
tract from water to prevent aspiration and allow for effective ventilation during CPR.
C) To transport the victim to the nearest hospital to carry out reanimation measures: This
option is incorrect because time is critical in cases of drowning, and immediate intervention is
necessary to prevent brain damage and improve outcomes.
D) To transport the victim to the nearest warm room to carry out reanimation measures: This
option is incorrect because the priority should be to restore breathing and circulation, which
requires immediate intervention and may be more effective outside in the cold air.
E) Not to carry out reanimation measure: This option is incorrect because drowning is a medical
emergency that requires immediate intervention to restore breathing and circulation.
Making Other Options Wrong:
B) Not to waste time on the release of respiratory tract from water, to take on cardiopulmonary
reanimation: This option is incorrect because the first step should be to release the respiratory
tract from water to prevent aspiration and allow for effective ventilation during CPR.
C) To transport the victim to the nearest hospital to carry out reanimation measures: This
option is incorrect because time is critical in cases of drowning, and immediate intervention is
necessary to prevent brain damage and improve outcomes.
D) To transport the victim to the nearest warm room to carry out reanimation measures: This
option is incorrect because the priority should be to restore breathing and circulation, which
requires immediate intervention and may be more effective outside in the cold air.
Emergency Medicine

E) Not to carry out reanimation measure: This option is incorrect because drowning is a medical
emergency that requires immediate intervention to restore breathing and circulation.
Words:
• Drowning: A medical emergency that occurs when an individual's airway becomes blocked by
water, preventing effective ventilation and leading to hypoxemia and, potentially, cardiac arrest.
• Cardiopulmonary resuscitation (CPR): A technique used to restore breathing and circulation in
individuals experiencing cardiac arrest or other medical emergencies.
• Aspiration: The inhalation of foreign material, such as water, into the airway or lungs.
• Vital signs: Indicators of life, including heart rate, respiratory rate, blood pressure, and body
temperature.
• Hypoxemia: A low level of oxygen in the blood.
Findings:
• The victim has been in the water for 15 minutes and is showing no vital signs, indicating a
severe medical emergency.

Therapy
Emergency Medicine

116 Epilepsy

A 51 y.o. women was taken to the emergency department in convulsive status epilepticus.
The first means of medical management should be:
A Ensuring that the airway is open and the patient is oxygenating
B Inserting a tongue blade
C Administering an instravenous bolus of 50% dextrose
D Injecting 5 mg of diazepam followed by a loading dose of phenytoin
E Inducing pentobarbital coma

Correct Answer: A. Ensuring that the airway is open and the patient is oxygenating.
Key Points: A 51-year-old woman is brought to the emergency department in convulsive status
epilepticus.
Explanation: Status epilepticus is a medical emergency characterized by prolonged or repeated
seizures that occur without full recovery of consciousness in between. The first step in
management is to ensure the patient's airway is open and that they are receiving oxygen. This
step is crucial because seizures can lead to respiratory depression and hypoxia, which can result
in brain damage or even death. After securing the airway, further interventions may be
necessary, including the administration of intravenous medications such as benzodiazepines or
antiepileptic drugs.
Other Options:
B. Inserting a tongue blade - Incorrect. Inserting a tongue blade can cause injury and is not

Therapy
recommended as a first-line intervention for convulsive status epilepticus.
C. Administering an intravenous bolus of 50% dextrose - Incorrect. This intervention is not
indicated for convulsive status epilepticus, as hypoglycemia is not a common cause of seizures
in adults.
D. Injecting 5 mg of diazepam followed by a loading dose of phenytoin - Incorrect. While this
may be a subsequent step in the management of convulsive status epilepticus, it is not the first-
line intervention.
E. Inducing pentobarbital coma - Incorrect. This is an extreme intervention reserved for
refractory cases of status epilepticus that do not respond to initial treatment.
Making Other Options Right:
B. Inserting an oropharyngeal airway or nasopharyngeal airway - This intervention may be
necessary if the patient's airway is obstructed or compromised by the convulsions.
C. Administering intravenous glucose - This intervention may be necessary if the patient's blood
glucose level is low, which can lead to seizures in patients with diabetes or other metabolic
disorders.
D. Injecting 5 mg of diazepam followed by a loading dose of phenytoin - This is a subsequent
step in the management of convulsive status epilepticus and may be necessary if the initial
airway management and benzodiazepine intervention are ineffective.
Words:
Convulsive status epilepticus - a medical emergency characterized by prolonged or repeated
seizures that occur without full recovery of consciousness in between.
Benzodiazepines - a class of medications that are commonly used to treat seizures and anxiety.
Antiepileptic drugs - a class of medications that are used to treat epilepsy and other seizure
disorders.
findings mentioned are as follows:
• Consciousness is confused: Confusion is a state of disorientation where an individual is unable
to think clearly or understand their surroundings. It is often caused by various medical
conditions affecting the brain, such as head injury, stroke, seizures, or infections.
• Left pupil is mydriatic: Mydriasis is the medical term used to describe dilated pupils. Dilated
pupils can be caused by several factors, such as head injury, medications, or brain damage.
Emergency Medicine

• Photophobia: Photophobia is a condition where an individual experiences sensitivity to light. It


can be a symptom of several medical conditions, such as migraine, meningitis, or eye diseases.
• Tension of neck muscles: Neck stiffness is a common symptom of meningitis and other
infections affecting the central nervous system. It can also be caused by injuries, such as
whiplash or muscle strain.
• Left-side hemiparesis with increased muscle tonus and reflexes: Hemiparesis refers to
weakness on one side of the body. It can be caused by several medical conditions affecting the
brain, such as stroke, brain injury, or infections. Increased muscle tone and reflexes are often
associated with neurological conditions affecting the motor system.
• Body temperature is low: Hypothermia is a condition where the body temperature drops below
normal levels. It can be caused by several factors, such as exposure to cold weather, certain
medications, or medical conditions affecting the thyroid gland.
• Rash is absent: A rash is an outbreak of red or swollen skin that can be caused by several
factors, such as infections, allergies, or medications. Its absence suggests that the cause of the
patient's symptoms is not an allergic reaction or an infectious disease that typically presents
with a rash.

Therapy
Emergency Medicine

117 Gastrointestinal Bleeding

The family doctor examined a patient and diagnosed an acute bleeding of an intestine.
What is the doctor's professional tactics in this situation?
A The urgent hospitalization in to the surgical department
B Aminocapronic acid intravenously
C The urgent hospitalization in to tthe herapeutic department
D Treatment at a day time hospital
E Treatment at home

Correct Answer: A
Key Points: Acute bleeding of the intestine
Explanation: Acute bleeding of the intestine is a medical emergency that requires urgent
hospitalization in the surgical department. The treatment approach will depend on the cause and
severity of the bleeding. Initial treatment may include supportive measures such as intravenous
fluids, blood transfusions, and medications to stabilize the patient. Diagnostic tests, such as
endoscopy or angiography, may be performed to locate the source of bleeding and to guide
further management. In some cases, surgery may be necessary to stop the bleeding.
Other Options:
B. Aminocaproic acid is a medication that helps to prevent the breakdown of clots in the body.
While it may be used in some cases of bleeding, it is not the first-line treatment for acute
bleeding of the intestine.

Therapy
C. Urgent hospitalization in the therapeutic department is not appropriate in this situation, as
acute bleeding of the intestine requires urgent surgical management.
D. Treatment at a day time hospital or home is not appropriate for a medical emergency such as
acute bleeding of the intestine.
Making Other Options Right:
B. Aminocaproic acid can be used in some cases of bleeding, but it is not the appropriate
treatment for acute bleeding of the intestine.
C. Urgent hospitalization is required for acute bleeding of the intestine, but the patient should be
admitted to the surgical department, not the therapeutic department.
Medication:
Aminocaproic acid is an antifibrinolytic medication that works by inhibiting the breakdown of
clots in the body. It is used to prevent or treat excessive bleeding in conditions such as
hemophilia, surgery, or trauma. Common side effects include nausea, vomiting, diarrhea,
headache, and muscle pain.
Emergency Medicine

118 Hypothermia

In a cold weather, the emergency room admitted a patient pulled out of the open water. There
was no respiratory contact with the water. The patient is excited, pale, complains of pain,
numbness of hands and feet, cold shiver. Respiratory rate is 22/min, AP - 120/90 mm Hg, Ps -
110/min, rectal temperature is 34,5oC. What kind of warming is indicated for this patient?
A Passive warming
B Infusion of 37oC solutions
C Hot compresses
D Warm bath
E Hemodialysis with blood warming

Correct Answer: A Passive warming - correct.


Key Points:
• Patient pulled out of open water in cold weather.
• No respiratory contact with water.
• Excited, pale, complains of pain, numbness of hands and feet, cold shiver.
• Respiratory rate is 22/min, AP - 120/90 mm Hg, Ps - 110/min, rectal temperature is 34,5oC.
• Warming method indicated is being asked for.
Explanation:
The patient in this case is exhibiting signs of hypothermia, which is a medical emergency that
occurs when the body's core temperature drops below normal. In this case, the patient's rectal

Therapy
temperature is 34.5°C, which is significantly below the normal range of 36.5-37.5°C. The
primary goal of treatment for hypothermia is to rewarm the patient, but this must be done
carefully to avoid causing harm.
Passive warming is the most appropriate method of warming for this patient. Passive warming
involves removing wet clothing and covering the patient with dry blankets or clothing to
minimize heat loss. This method is generally safe and effective for patients with mild to
moderate hypothermia, and can help to prevent further heat loss while the patient is being
transported to a medical facility for further treatment.
Other Options:
B. Infusion of 37°C solutions is not a recommended method of warming for hypothermia, as it
can cause peripheral vasodilation and lead to further heat loss.
C. Hot compresses are not a recommended method of warming for hypothermia, as they can
cause burns or localized tissue damage.
D. Warm baths are not a recommended method of warming for hypothermia, as they can cause
peripheral vasodilation and lead to further heat loss. Additionally, the patient in this case has
not been cleared for respiratory contact with water.
E. Hemodialysis with blood warming is not a method of warming that is typically used for
hypothermia, as it is an invasive procedure that is primarily used for patients with renal failure.
Making Other Options Right:
B. To make infusion of 37°C solutions the correct answer, the question would need to specify
additional clinical findings that are consistent with severe hypothermia and shock, such as
altered mental status or signs of cardiovascular collapse.
C. To make hot compresses the correct answer, the question would need to specify a specific
area of the body that is affected by hypothermia and requires localized warming, such as
frostbite.
D. To make warm baths the correct answer, the question would need to specify a specific type
of hypothermia that is amenable to this treatment, such as immersion hypothermia.
E. To make hemodialysis with blood warming the correct answer, the question would need to
specify additional clinical findings that are consistent with renal failure or another condition that
requires hemodialysis.
Medication:
Emergency Medicine

• No medication is mentioned in the question.


Words:
• Hypothermia: A medical condition in which the body's core temperature drops below normal
levels, typically as a result of exposure to cold temperatures or immersion in cold water.
• Passive warming: A method of warming that involves removing wet clothing and covering the
patient with dry blankets or clothing to minimize heat loss.
• Peripheral vasodilation: Widening of the blood vessels in the extremities, which can increase
blood flow to these areas but also lead to further heat loss.
• Hemodialysis: A medical procedure that involves using a machine to filter waste products and
excess fluid from the blood in patients with kidney failure.
• Frostbite: A medical condition that occurs when the skin and underlying tissues freeze due to
exposure to cold temperatures.
Findings:
• Open water immersion in cold weather with no respiratory contact with water.
• Excited, pale, complains of pain, numbness of hands and feet, cold shiver.
• Respiratory rate is 22/min, AP - 120/90 mm Hg, Ps - 110/min, rectal temperature is 34.5°C.
• These findings are consistent with hypothermia, which is a medical emergency that requires
urgent treatment to prevent further heat loss and rewarm the body.

Therapy
Emergency Medicine

119 Overdose

A patient who takes diuretics has developed arrhythmia as a result of cardiac glycoside
overdose. What is the treatment tactics in this case?
A Increased potassium concentration in blood
B Increased sodium consentration in blood
C Reduced magnesium concentration in blood
D Increased calcium concentration in blood
E-

Correct Answer: A Increased potassium concentration in blood.


Key Points: The patient who takes diuretics and has developed arrhythmia as a result of cardiac
glycoside overdose.
Explanation: The treatment of cardiac glycoside overdose should be started immediately to
prevent life-threatening complications. The primary treatment is the administration of activated
charcoal to absorb any unabsorbed drug in the gut. In addition, potassium replacement should
be given as hypokalemia is common in patients taking diuretics and can exacerbate the toxicity
of cardiac glycosides.
Cardiac glycosides such as digoxin inhibit the sodium-potassium ATPase pump, leading to an
increase in intracellular calcium, which results in positive inotropy and chronotropy. However, in
cases of overdose, these effects can become exaggerated, leading to bradyarrhythmias,
tachyarrhythmias, and potentially fatal cardiac toxicity. Diuretics can lead to hypokalemia, which

Therapy
increases the risk of cardiac toxicity from cardiac glycosides.
Other Options:
B. Increased sodium concentration in blood is incorrect because sodium has no role in treating
cardiac glycoside toxicity.
C. Reduced magnesium concentration in blood is incorrect because magnesium has no role in
treating cardiac glycoside toxicity.
D. Increased calcium concentration in blood is incorrect because calcium can exacerbate the
toxic effects of cardiac glycosides and should be avoided.
E. The option is not given.
Medication:
Diuretics are medications that increase urine production, leading to increased sodium and water
excretion. They are often used to treat hypertension, heart failure, and other conditions where
fluid retention is a problem. However, diuretics can lead to electrolyte imbalances, particularly
hypokalemia, which can exacerbate the toxicity of cardiac glycosides.
Cardiac glycosides, such as digoxin, are medications used to treat heart failure and atrial
fibrillation. They work by inhibiting the sodium-potassium ATPase pump, leading to increased
intracellular calcium, which results in positive inotropy and chronotropy.
Emergency Medicine

120 Shock

A 50 y.o. male patient was taken to the emergency department with diffuse abdominal pain
and signs of cardiovascular collapse. On arrival he had BP- 95/60 mm Hg, Ps- 120/min,
diuresis - 20 ml/h, HgB- 100 g/L, RBC- 2,1*1012/L. The patient needs introduction of:
A Crystalloid and colloid
B Crystalloid and red blood cells
C Crystalloid and 5% dextrose
D 5% dextrose and red blood cells
E 5% dextrose and colloid

orrect Answer: A Crystalloid and colloid


Key Points:
• A 50-year-old male patient is brought to the emergency department with diffuse abdominal
pain and signs of cardiovascular collapse.
• The patient has low blood pressure, high heart rate, low urine output, and anemia.
• The patient needs fluid therapy to treat shock.
• The correct option for fluid therapy is Crystalloid and colloid.
Explanation:
The patient's presentation suggests hypovolemic shock, which requires immediate fluid
resuscitation to restore the circulating blood volume.
Crystalloid and colloid are the best fluids to use in the treatment of shock because they have

Therapy
different properties that can improve blood volume and organ perfusion.
The initial choice for fluid resuscitation in hypovolemic shock is crystalloids such as normal
saline or lactated Ringer's solution. Crystalloids increase the intravascular volume and help
restore organ perfusion. However, in severe hypovolemic shock, crystalloid infusion alone may
not be enough to restore circulating blood volume. In this case, colloid solutions such as
albumin, dextran, or hydroxyethyl starch can be added to the fluid regimen. Colloids have a
higher oncotic pressure than crystalloids, which enables them to remain in the intravascular
space for longer periods, increasing the plasma volume and improving organ perfusion.
Other Options:
B) Crystalloid and red blood cells - Red blood cells are indicated in patients with severe anemia
or massive blood loss, which is not evident in this case. This option does not address the
immediate need for fluid resuscitation in hypovolemic shock.
C) Crystalloid and 5% dextrose - 5% dextrose is a hypotonic solution that provides calories but
not volume expansion. It is not indicated in hypovolemic shock, where the primary goal is to
restore circulating blood volume.
D) 5% dextrose and red blood cells - This option provides calories and red blood cells, but not
volume expansion. It is not indicated in hypovolemic shock, where the primary goal is to restore
circulating blood volume.
E) 5% dextrose and colloid - This option provides calories and a colloid solution, but not volume
expansion. It is not the best initial therapy for hypovolemic shock.
Medication: The patient needs fluid resuscitation with a crystalloid and a colloid solution. The
choice of specific fluid and its rate of administration depends on the severity of hypovolemia and
the patient's response to therapy. Albumin is a common colloid used in fluid resuscitation, which
works by increasing the plasma oncotic pressure and expanding the circulating blood volume.
Potential side effects of albumin include anaphylaxis, fluid overload, and interference with
coagulation.
Words:
• Colloid: a substance composed of particles suspended in another substance
• Crystalloid: a substance that forms a clear solution when dissolved in water or saline
Findings:
• BP: 95/60 mm Hg (normal range: 90/60 - 120/80 mm Hg)
Emergency Medicine

• Ps: 120/min (normal range: 60-100/min)


• Diuresis: 20 ml/h (normal range: 30-50 ml/h)
• HgB: 100 g/L (normal range: 130-160 g/L)
• RBC: 2,110^12/L (normal range: 4.5-5.510^12/L)

Therapy
Emergency Medicine

121 Varicose Veins

A 58 y.o. woman had voluminous bleeding from the ruptured varicous node on the left crus.
What first aid should be provided?
A Heightened position of extremity, compressive sterile bandage
B Distal tourniquet
C Proximal turniquet
D Troyanov-Trendelenburg operation
E Z-shaped stitch on the raptured varicose node

Correct Answer: A Heightened position of extremity, compressive sterile bandage is the correct
answer because the patient has a ruptured varicose vein, which is causing voluminous bleeding
from the left crus. The first aid should include elevating the affected extremity above the level of
the heart to reduce blood flow to the area and applying a compressive sterile bandage to help
control the bleeding.
Key Points:
• 58-year-old woman with voluminous bleeding from a ruptured varicose node on the left crus
• First aid to be provided
Explanation:
• Varicose veins are enlarged, twisted veins that can occur anywhere in the body but are most
commonly found in the legs. They are caused by weakened valves and veins that allow blood to
flow backward and accumulate, leading to swelling and increased pressure.

Therapy
• Ruptured varicose veins can cause significant bleeding, and prompt first aid is necessary to
control the bleeding and prevent further complications.
• Heightening the position of the extremity above the level of the heart can help reduce blood
flow to the area and slow down the bleeding.
• Applying a compressive sterile bandage can help control the bleeding and prevent further
blood loss.
Other Options:
• B Distal tourniquet is not the correct answer because it can cut off blood flow to the affected
area completely, leading to tissue damage, and is not recommended for the treatment of
varicose vein bleeding.
• C Proximal tourniquet is not the correct answer because it can also cut off blood flow to the
affected area completely, leading to tissue damage, and is not recommended for the treatment
of varicose vein bleeding.
• D Troyanov-Trendelenburg operation is not the correct answer because it is a surgical
procedure that involves tying off the affected vein and redirecting blood flow to healthy veins. It
is not a first-aid option for the treatment of varicose vein bleeding.
• E Z-shaped stitch on the ruptured varicose node is not the correct answer because it is a
surgical procedure that involves suturing the ruptured vein. It is not a first-aid option for the
treatment of varicose vein bleeding.
Making Other Options Right:
• B Distal tourniquet could be the correct answer if the patient is experiencing severe, life-
threatening bleeding from an artery, where the use of a tourniquet is necessary to control the
bleeding until medical help arrives.
• C Proximal tourniquet could be the correct answer if the patient is experiencing severe, life-
threatening bleeding from an artery, where the use of a tourniquet is necessary to control the
bleeding until medical help arrives.
• D Troyanov-Trendelenburg operation could be the correct answer if the patient is experiencing
recurrent, severe bleeding from varicose veins despite other treatments, and surgery is
necessary to remove or tie off the affected veins.
Emergency Medicine

• E Z-shaped stitch on the ruptured varicose node could be the correct answer if the patient is
experiencing severe, life-threatening bleeding from a ruptured varicose vein, and surgical
intervention is necessary to control the bleeding.
Words:
• Varicose: Refers to veins that are enlarged, twisted, and bulging, often seen in the legs and
caused by weakened valves and veins.

Therapy
endocrinology

122 Addison's disease.

Which of the following IS NOT TYPICAL for Addison's disease?


A High serum Na+
B High serum K+
C Elevated BUN
D Dilute urine
E Elevated hematocrit

Correct Answer: High serum Na+ is NOT TYPICAL for Addison's disease.
Key Points: Addison's disease is a rare endocrine disorder characterized by the deficiency of
cortisol and aldosterone hormones produced by the adrenal gland. The most common causes
are autoimmune, infections (e.g., tuberculosis, HIV), and cancer. Symptoms include fatigue,
weight loss, hypotension, hyperpigmentation, and salt craving. Laboratory findings include
hyponatremia, hyperkalemia, hypoglycemia, and metabolic acidosis.
Explanation: High serum Na+ is not typical for Addison's disease. In Addison's disease, the lack
of aldosterone hormone leads to hyponatremia, a decrease in serum sodium levels. Other
electrolyte abnormalities seen in Addison's disease are hyperkalemia and metabolic acidosis.
Addison's disease can also cause a decrease in blood glucose levels, leading to hypoglycemia.
Moreover, it can lead to an increase in blood urea nitrogen (BUN), which is a marker of impaired
kidney function. Dilute urine is a characteristic finding in diabetes insipidus, not Addison's
disease. Lastly, Addison's disease can cause a decrease in hematocrit levels due to the loss of

Therapy
red blood cells.
Other Options: High serum Na+ is not typical for Addison's disease, so option A is the correct
answer.
Making Other Options Right: Option B is typical for Addison's disease because the lack of
aldosterone hormone causes hyperkalemia. Option C is typical for Addison's disease because it
can lead to an increase in BUN levels.
Medication: In the case of Addison's disease, hormone replacement therapy with glucocorticoids
and mineralocorticoids is the mainstay of treatment. Hydrocortisone is a commonly used
glucocorticoid medication, and fludrocortisone is a mineralocorticoid medication.
Words: None.
Findings: Hyponatremia, hyperkalemia, hypoglycemia, metabolic acidosis, and an increase in
BUN are laboratory findings typical of Addison's disease.
endocrinology

123 Addison's disease.

A 38 y.o. patient was urgently admitted to the hospital with complaints of sudden weakness,
dizziness, loss of consciousness, body weight loss, nausea, vomiting, severe pain in
epigastric area, diarrhea, skin hyperpigmentation. What is the most probable diagnosis?
A Addisonic crisis
B Acute gastroenteritis
C Meningoencephalitis
D Scleroderma
E Pellagra

Correct Answer: A. Addisonic Crisis is the most probable diagnosis based on the clinical features
described.
Key Points: Sudden weakness, dizziness, loss of consciousness, weight loss, nausea, vomiting,
severe pain in epigastric area, diarrhea, skin hyperpigmentation.
Explanation: Addisonic crisis is a life-threatening condition caused by a deficiency of cortisol and
aldosterone hormones, which are produced by the adrenal glands. The symptoms of Addisonic
crisis include sudden weakness, dizziness, loss of consciousness, weight loss, nausea, vomiting,
severe pain in epigastric area, diarrhea, skin hyperpigmentation, and fever. The skin
hyperpigmentation is usually seen in sun-exposed areas and pressure points as a result of
increased production of ACTH.
Other Options:

Therapy
B. Acute gastroenteritis is less likely as it typically presents with fever, abdominal cramps, and
diarrhea.
C. Meningoencephalitis presents with headache, fever, and neurological symptoms, such as
altered mental status, seizures, and focal deficits.
D. Scleroderma is a connective tissue disorder that typically presents with skin thickening,
Raynaud's phenomenon, and involvement of other organ systems.
E. Pellagra is caused by niacin deficiency and presents with dermatitis, diarrhea, and dementia.
Making Other Options Right:
B. To make acute gastroenteritis right, the question should include fever, abdominal cramps,
and a positive stool culture for a pathogen.
C. To make meningoencephalitis right, the question should include headache, fever, and
neurological symptoms such as altered mental status, seizures, and focal deficits.
D. To make scleroderma right, the question should include skin thickening, Raynaud's
phenomenon, and involvement of other organ systems.
E. To make pellagra right, the question should include niacin deficiency and dermatitis, diarrhea,
and dementia.
Medication: Treatment for Addisonic crisis includes intravenous administration of hydrocortisone,
saline solution, and glucose.
Words:
Addisonic crisis: Named after Thomas Addison, a British physician who first described the
condition in 1855.
Epigastric: Relating to the upper central region of the abdomen.
Hyperpigmentation: Excessive pigmentation of the skin.
endocrinology

124 adrenal gland disorder

A 45-year-old female patient complaining of general weakness, nausea and vomiting hass been
delivered to a hospital by the ambulance. Recently there has been a lack of appetite, weight
loss. Objectively: hyperpigmentation of skin, blood pressure at the rate of 70/45 mm Hg,
bradycardia. Additional studies revealed the reduced concentration of aldosterone and cortisol
in blood, decreased excretion of 17-ketosteroids and 17-oxyketosteroids in the urine,
hyponatremia, chloropenia, hypokalemia. What therapeutic measures are required?
A To administer glucocorticoids, mineralocorticoids, and a diet with a high content of cooking
salt
B To prescribe a diet with a high content of cooking salt
C To administer prednisolone
D To administer aldosterone
E To administer insulin

Correct Answer: A - To administer glucocorticoids, mineralocorticoids, and a diet with a high


content of cooking salt
Key Points:
• 45-year-old female patient with complaints of weakness, nausea, and vomiting
• Hyperpigmentation of skin, low blood pressure, bradycardia, weight loss, and lack of appetite
• Blood tests show low levels of aldosterone and cortisol
• Urine tests show decreased excretion of 17-ketosteroids and 17-oxyketosteroids

Therapy
• Hyponatremia, chloropenia, and hypokalemia
Explanation:
The patient is presenting with symptoms of adrenal insufficiency, also known as Addison's
disease. In this condition, the adrenal glands do not produce enough cortisol and aldosterone,
which can lead to a variety of symptoms including weakness, hypotension, and electrolyte
imbalances.
The most appropriate treatment for adrenal insufficiency is to administer glucocorticoids (such
as prednisolone) and mineralocorticoids (such as aldosterone) to replace the hormones that the
adrenal glands are not producing. In addition, a diet with a high content of cooking salt is
recommended to help address the electrolyte imbalances seen in this condition.
Other Options:
B. To prescribe a diet with a high content of cooking salt - While a high-salt diet is
recommended in the treatment of adrenal insufficiency, it is not sufficient on its own and should
be combined with hormone replacement therapy.
C. To administer prednisolone - Prednisolone is a glucocorticoid that is used to replace cortisol in
the treatment of adrenal insufficiency. It may be part of the appropriate treatment regimen, but
mineralocorticoid replacement is also necessary.
D. To administer aldosterone - Aldosterone is a mineralocorticoid that is used to replace
aldosterone in the treatment of adrenal insufficiency. It may be part of the appropriate
treatment regimen, but glucocorticoid replacement is also necessary.
E. To administer insulin - Insulin is not indicated in the treatment of adrenal insufficiency.
Making Other Options Right:
B. To prescribe a diet with a high content of cooking salt - A high-salt diet is recommended in
the treatment of adrenal insufficiency, but it should be combined with hormone replacement
therapy.
C. To administer prednisolone - Prednisolone may be part of the appropriate treatment regimen,
but mineralocorticoid replacement is also necessary.
D. To administer aldosterone - Aldosterone may be part of the appropriate treatment regimen,
but glucocorticoid replacement is also necessary.
E. To administer insulin - Insulin is not indicated in the treatment of adrenal insufficiency.
Medication:
endocrinology

In addition to hormone replacement therapy and a high-salt diet, treatment for adrenal
insufficiency may include management of any other underlying conditions and monitoring of
electrolyte levels.
Words:
Hyperpigmentation - Darkening of the skin due to increased melanin production.
Hyponatremia - Low sodium levels in the blood.
Chloropenia - Low chloride levels in the blood.
Hypokalemia - Low potassium levels in the blood.
Findings:
Blood pressure - 70/45 mm Hg (low)
Bradycardia - Slow heart rate
Reduced concentration of aldosterone and cortisol in blood
Decreased excretion of 17-ketosteroids and 17-oxyketosteroids in urine
Hyponatremia, chloropenia, and hypokalemia.

Therapy
endocrinology

125 adrenal gland disorder

A 38-year-old woman experiences episodic increases in arterial pressure up to 240/120


mm Hg, which is accompanied by nausea, vomiting, tachycardia, increased sweating,
hyperglycemia. The attack is usually followed by the excessive urination. Renal sonography
reveals an additional formation adjacent to the upper pole of the right kidney and possibly
belonging to the adrenal gland. What laboratory test will allow to clarify the diagnosis?
A Determination of urinary excretion of catecholamines and vanillylmandelic acid
B Blood test for insulin and C-peptide
C Estimation of glomerular filtration rate by measuring endogenous creatinine clearance
D Blood test for thyroxine and thyrotrophic hormone
E Blood test for renin level

Correct Answer: A Determination of urinary excretion of catecholamines and vanillylmandelic


acid - The patient's symptoms of episodic hypertension, nausea, vomiting, tachycardia,
sweating, hyperglycemia, and excessive urination, along with the presence of an adrenal gland
tumor, are consistent with pheochromocytoma. Determination of urinary excretion of
catecholamines and vanillylmandelic acid is the laboratory test of choice for diagnosing
pheochromocytoma.
Key Points:
• 38-year-old woman with episodic increases in arterial pressure up to 240/120 mm Hg
• Accompanied by nausea, vomiting, tachycardia, increased sweating, hyperglycemia

Therapy
• Excessive urination after attack
• Renal sonography reveals additional formation adjacent to upper pole of right kidney, possibly
adrenal gland tumor
• Laboratory test of choice to clarify diagnosis is determination of urinary excretion of
catecholamines and vanillylmandelic acid
Explanation:
Pheochromocytoma is a rare type of adrenal gland tumor that produces excessive amounts of
catecholamines, such as epinephrine and norepinephrine. The excess catecholamines can cause
episodic hypertension, along with other symptoms such as nausea, vomiting, tachycardia,
sweating, and hyperglycemia. Excessive urination after an attack is also a characteristic feature
of pheochromocytoma.
In this case, the patient's symptoms of episodic hypertension, along with the presence of an
adrenal gland tumor on renal sonography, suggest a possible diagnosis of pheochromocytoma.
The laboratory test of choice for diagnosing pheochromocytoma is determination of urinary
excretion of catecholamines and their metabolites, such as vanillylmandelic acid (VMA). Elevated
levels of urinary catecholamines and VMA are indicative of pheochromocytoma.
Other Options:
B Blood test for insulin and C-peptide - This option is not relevant to the patient's symptoms and
findings, and is not useful in diagnosing pheochromocytoma.
C Estimation of glomerular filtration rate by measuring endogenous creatinine clearance - This
option is not relevant to the patient's symptoms and findings, and is not useful in diagnosing
pheochromocytoma.
D Blood test for thyroxine and thyrotrophic hormone - This option is not relevant to the patient's
symptoms and findings, and is not useful in diagnosing pheochromocytoma.
E Blood test for renin level - This option is not relevant to the patient's symptoms and findings,
and is not useful in diagnosing pheochromocytoma.
Making Other Options Right:
B Blood test for insulin and C-peptide - This option could be made right if the patient had
symptoms suggestive of insulinoma, such as fasting hypoglycemia or neuroglycopenic
symptoms, or if there was evidence of insulinoma on imaging studies such as CT or MRI.
endocrinology

C Estimation of glomerular filtration rate by measuring endogenous creatinine clearance - This


option could be made right if the patient had symptoms suggestive of renal dysfunction, such as
edema, proteinuria, or oliguria, or if there was evidence of renal dysfunction on other laboratory
tests such as serum creatinine or BUN.
D Blood test for thyroxine and thyrotrophic hormone - This option could be made right if the
patient had symptoms suggestive of thyroid dysfunction, such as weight changes, fatigue, or
goiter, or if there was evidence of thyroid dysfunction on other laboratory tests such as TSH or
free T4.
E Blood test for renin level - This option could be made right if the patient had symptoms
suggestive of renovascular disease, such as hypertension with unilateral renal artery stenosis, or
if there was evidence of renovascular disease on imaging studies such as renal angiography or
Doppler ultrasound.
Medication:
• Treatment for pheochromocytoma typically involves surgical removal of the tumor, followed by
medical management to control hypertension and other symptoms.
Words:
• Catecholamines: a class of neurotransmitters and hormones, including epinephrine and
norepinephrine, that are produced by the adrenal glands and other sympathetic nervous system
tissues.
• Vanillylmandelic acid (VMA): a metabolite of catecholamines that is excreted in urine and can
be used as a marker of pheochromocytoma.
Findings:
• Episodic increases in arterial pressure up to 240/120 mm Hg
• Nausea, vomiting, tachycardia, increased sweating, hyperglycemia
• Excessive urination after attack
• Presence of an adrenal gland tumor on renal sonography.

Therapy
endocrinology

126 adrenal gland disorder

A 38 y.o. woman suffers from paroxysmal AP rises up to 240/120 mm Hg accompanied by


nausea, vomiting, tachycardia, excessive sweating. During the onset blood is
hyperglycemic. After the onset there is voluminous urination. Kidneys sonography revealed
accessory mass bordering upon the upper pole of the right kidney, presumably it belongs
to the adrenal gland. What laboratory test will allow to make a more precise diagnosis?
A Estimation of catecholamine and vanillylmandelic acid excretion with urine
B Estimation of insulin and C-peptide content in blood
C Estimation of glomerular filtration rate
D Estimation of thyroxin and thyrotropic hormon in blood
E Estimation of renin content in blood

Correct Answer: A) Estimation of catecholamine and vanillylmandelic acid excretion with urine
will allow to make a more precise diagnosis.
Key Points:
• 38 y.o. woman with paroxysmal AP rises up to 240/120 mm Hg accompanied by nausea,
vomiting, tachycardia, excessive sweating.
• Hyperglycemia during the onset and voluminous urination after the onset.
• Kidneys sonography revealed an accessory mass bordering upon the upper pole of the right
kidney, presumably belonging to the adrenal gland.
Explanation:

Therapy
The patient's symptoms and physical examination findings suggest a pheochromocytoma, which
is a rare neuroendocrine tumor that arises from chromaffin cells in the adrenal gland or
sympathetic ganglia. Pheochromocytoma can cause episodic hypertension, tachycardia,
sweating, and other symptoms due to the excessive secretion of catecholamines, such as
epinephrine and norepinephrine. The main laboratory test that can help to confirm the diagnosis
of pheochromocytoma is the estimation of catecholamine and vanillylmandelic acid (VMA)
excretion with urine. Elevated levels of catecholamines and their metabolites, such as VMA,
indicate excessive secretion of catecholamines and support the diagnosis of pheochromocytoma.
Other tests, such as imaging studies, may also be necessary to locate the tumor and assess its
size and characteristics.
Other Options:
B) Estimation of insulin and C-peptide content in blood is a test that is used to evaluate insulin
secretion and assess the presence of insulin resistance. However, this test is not specific for
pheochromocytoma and is unlikely to assist in the diagnosis of this condition.
C) Estimation of glomerular filtration rate is a test that is used to assess kidney function and
diagnose renal disease. However, this test is not specific for pheochromocytoma and is unlikely
to assist in the diagnosis of this condition.
D) Estimation of thyroxin and thyrotropic hormone in blood is a test that is used to evaluate
thyroid function and diagnose thyroid disorders. However, this test is not specific for
pheochromocytoma and is unlikely to assist in the diagnosis of this condition.
E) Estimation of renin content in blood is a test that is used to evaluate the renin-angiotensin-
aldosterone system and diagnose hypertension and related disorders. However, this test is not
specific for pheochromocytoma and is unlikely to assist in the diagnosis of this condition.
Making Other Options Wrong:
B) Estimation of insulin and C-peptide content in blood would be the correct option if the patient
had diabetes or insulin resistance.
C) Estimation of glomerular filtration rate would be the correct option if the patient had renal
disease.
D) Estimation of thyroxin and thyrotropic hormone in blood would be the correct option if the
patient had thyroid disorders.
endocrinology

E) Estimation of renin content in blood would be the correct option if the patient had
hypertension or related disorders.
Words:
• Pheochromocytoma: a rare neuroendocrine tumor that arises from chromaffin cells in the
adrenal gland or sympathetic ganglia.
• Catecholamines: hormones that are released by the adrenal gland and sympathetic nervous
system in response to stress, exercise, or other stimuli.
• Vanillylmandelic acid (VMA): a metabolite of catecholamines that is excreted in urine.
Findings:
• Paroxysmal AP rises up to 240/120 mm Hg accompanied by nausea, vomiting, tachycardia,
excessive sweating.
• Hyperglycemia during the onset and voluminous urination after the onset.
• Kidneys sonography revealed an accessory mass bordering upon the upper pole of the right
kidney, presumably belonging to the adrenal gland.

Therapy
endocrinology

127 adrenal gland disorder

A 43-year-old female patient was delivered to the hospital in grave condition. She suffers
from Addison's disease. The patient had been regularly taking prednisolone but a week before
she stopped taking this drug. Objectively: sopor, skin and visible mucous
membranes are pigmented, skin and muscle turgor is decreased. Heart sounds are
muffled, rapid. AP- 60/40 mm Hg, heart rate - 96/min. In blood: Na - 120 millimole/l, K - 5,8
millimole/l. Development of this complication is primarily caused by the deficit of the
following hormone:
A Cortisol
B Corticotropin (ACTH)
C Adrenaline
D Noradrenaline
E Adrostendion

Correct Answer: A Cortisol.


Key Points:
• 43-year-old female patient with Addison's disease.
• Stopped taking prednisolone a week before admission.
• Sopor, pigmented skin and visible mucous membranes, decreased skin and muscle turgor.
• Muffled, rapid heart sounds, low blood pressure, and high heart rate.
• Low sodium and high potassium levels in the blood.

Therapy
Explanation:
• Addison's disease is a condition caused by the chronic insufficiency of the adrenal cortex,
which leads to a deficiency of cortisol and aldosterone.
• Cortisol is a glucocorticoid hormone that regulates carbohydrate, protein, and fat metabolism,
suppresses the immune system, and helps the body cope with stress.
• Patients with Addison's disease require lifelong replacement therapy with glucocorticoids, such
as prednisolone, to prevent adrenal crisis and other complications.
• Abrupt discontinuation of glucocorticoids can cause adrenal crisis, a life-threatening condition
characterized by severe hypotension, dehydration, electrolyte imbalances, and cardiovascular
collapse.
• The patient's clinical presentation is consistent with adrenal crisis due to the sudden
withdrawal of prednisolone.
• The deficit of cortisol is primarily responsible for the development of this complication.
Other Options:
• B Corticotropin (ACTH) is a hormone produced by the anterior pituitary gland that stimulates
the secretion of cortisol by the adrenal cortex. The deficit of ACTH can cause secondary adrenal
insufficiency, but it is not the primary cause of adrenal crisis in patients with Addison's disease.
• C Adrenaline, also called epinephrine, is a hormone produced by the adrenal medulla that
increases heart rate, blood pressure, and cardiac output. Adrenaline does not play a significant
role in the pathophysiology of adrenal crisis in patients with Addison's disease.
• D Noradrenaline, also called norepinephrine, is a hormone produced by the adrenal medulla
that increases peripheral vasoconstriction and blood pressure. Noradrenaline does not play a
significant role in the pathophysiology of adrenal crisis in patients with Addison's disease.
• E Adrostendion, also called androstenedione, is a weak androgen hormone produced by the
adrenal cortex and the gonads. Adrostendion does not play a significant role in the
pathophysiology of adrenal crisis in patients with Addison's disease.
Making Other Options Right:
• B would be correct if the question described a patient with secondary adrenal insufficiency due
to the deficit of ACTH.
• C and D would be correct if the question described a patient with pheochromocytoma or other
adrenal medulla tumors secreting excessive amounts of catecholamines.
endocrinology

• E would be correct if the question described a patient with congenital adrenal hyperplasia or
other disorders of androgen synthesis.
Medication:
• Patients with Addison's disease require lifelong replacement therapy with glucocorticoids, such
as hydrocortisone, prednisolone, or dexamethasone, to prevent adrenal crisis and other
complications.
• Glucocorticoids work by replacing the deficient cortisol and suppressing the immune system.
• The dose and timing of glucocorticoid therapy should be individualized based on the patient's
clinical status and needs.
• Abrupt discontinuation of glucocorticoids should be avoided, and gradual tapering may be
necessary to prevent adrenal crisis.
Words:
• Sopor: from Latin "sopor", meaning deep sleep or coma.
• Turgor: from Latin "turgere", meaning to swell or be swollen.
• Glucocorticoid: from Greek "glykys", meaning sweet, and "kortiko-", meaning cortex.
Findings:
• Low blood pressure: 60/40 mm Hg.
• High heart rate: 96/min.
• Low sodium: 120 millimole/l.
• High potassium: 5.8 millimole/l.

Therapy
endocrinology

128 Adrenal Insufficiency

After having the flu, a 39-year-old male patient with a history of Addison's disease developed a
condition manifested by weakness, depression, nausea, vomiting, diarrhea, hypoglycemia.
AP75/50 mm Hg. Blood test results: low corticosterone and cortisol, 13-oxycorticosteroids,
17-oxycorticosteroids levels. What condition developed in the patient?
A Acute adrenal insufficiency
B Acute gastritis
C Acute enterocolitis
D Collapse
E Diabetes mellitus

Correct Answer: Option A (Acute adrenal insufficiency) is the correct answer because the
patient's symptoms and laboratory findings are consistent with a deficiency of cortisol and other
adrenal hormones, which can occur in acute adrenal insufficiency. The history of Addison's
disease, which is a chronic condition characterized by adrenal insufficiency, also increases the
likelihood of acute adrenal crisis.
Key Points:
• 39-year-old male patient with a history of Addison's disease
• Developed weakness, depression, nausea, vomiting, diarrhea, hypoglycemia after the flu
• Blood pressure: 75/50 mm Hg
• Low levels of corticosterone, cortisol, 13-oxycorticosteroids, 17-oxycorticosteroids

Therapy
• What condition developed in the patient?
Explanation:
• Addison's disease is a chronic condition characterized by adrenal insufficiency, which can lead
to a deficiency of cortisol and other adrenal hormones.
• Acute adrenal insufficiency (also known as adrenal crisis) can occur in patients with Addison's
disease who experience a stressful event, such as an infection or surgery, that increases the
demand for cortisol and other hormones.
• The patient's symptoms of weakness, depression, nausea, vomiting, diarrhea, and
hypoglycemia are consistent with acute adrenal crisis.
• The low levels of corticosterone, cortisol, 13-oxycorticosteroids, and 17-oxycorticosteroids in
the blood test results support the diagnosis of adrenal insufficiency.
• Treatment of acute adrenal crisis typically involves administration of intravenous
hydrocortisone and saline, as well as correction of hypoglycemia and electrolyte imbalances.
Other Options:
• Option B (Acute gastritis) is not the correct answer because the patient's symptoms and
laboratory findings are not consistent with gastritis.
• Option C (Acute enterocolitis) is not the correct answer because the patient's symptoms and
laboratory findings are not consistent with enterocolitis.
• Option D (Collapse) is not the correct answer because it is a nonspecific term that does not
describe the underlying pathophysiology of the patient's condition.
• Option E (Diabetes mellitus) is not the correct answer because the patient's symptoms and
laboratory findings are not consistent with diabetes mellitus.
Medication:
• Treatment of acute adrenal crisis typically involves administration of intravenous
hydrocortisone and saline to replace the deficient cortisol and correct hypotension.
• Glucose should also be administered to correct hypoglycemia.
• Electrolyte imbalances, such as hyponatremia and hyperkalemia, should be corrected as well.
Words:
• Addison's disease: A chronic condition characterized by adrenal insufficiency, which can lead to
a deficiency of cortisol and other adrenal hormones
endocrinology

• Corticosterone: A steroid hormone produced by the adrenal cortex that is involved in the
regulation of metabolism and immune function
• Cortisol: A steroid hormone produced by the adrenal cortex that is involved in the regulation
of metabolism, stress response, and immune function
• 13-oxycorticosteroids: Metabolites of cortisol and other adrenal hormones that are excreted in
urine and can be measured to assess adrenal function
• 17-oxycorticosteroids: Metabolites of cortisol and other adrenal hormones that are excreted in
urine and can be measured to assess adrenal function
• Acute adrenal insufficiency: A condition in which the adrenal glands fail to produce enough
cortisol and other hormones, which can lead to hypotension, hypoglycemia, and electrolyte
imbalances
Findings:
• Weakness, depression, nausea, vomiting, diarrhea, hypoglycemia (patient history)
• Blood pressure: 75/50 mm Hg (abnormal finding)
• Low levels of corticosterone, cortisol, 13-oxycorticosteroids, and 17-oxycorticosteroids
(abnormal findings)

Therapy
endocrinology

129 Adrenal Insufficiency

After a holiday in the Crimea, a 49-year-old male patient with a history of lung tuberculosis felt
increased weakness, periodic dizziness, easing bowel movements with abdominal pain, the
need for additional salting his meals. The patient has noted that his condition improves after
some sweet tea and validol taken sublingually. Objectively: there is an intense darkening of
skin,
AP- 70/50 mm Hg, glycemia is 3,0 mmol/l. What is the possible cause of health deterioration:
A Chronic adrenal insufficiency
B Diabetes mellitus
C Coronary artery disease
D Chronic pancreatitis
E Pulmonary tuberculosis

Correct Answer: A - Chronic adrenal insufficiency.


Key Points:
• 49-year-old male patient with a history of lung tuberculosis
• Increased weakness, periodic dizziness, easing bowel movements with abdominal pain, the
need for additional salting his meals
• Condition improves after some sweet tea and validol taken sublingually
• Intense darkening of skin, AP- 70/50 mm Hg, glycemia is 3,0 mmol/l
Explanation:

Therapy
Chronic Adrenal Insufficiency (Addison's Disease) is a condition where the adrenal glands do not
produce enough hormones, mainly cortisol and aldosterone. The symptoms of chronic adrenal
insufficiency include fatigue, weakness, dizziness, abdominal pain, and salt craving. The
darkening of the skin is also a characteristic feature of chronic adrenal insufficiency known as
hyperpigmentation.
The patient's history of lung tuberculosis is significant as it may have caused damage to the
adrenal glands. The patient's symptoms of weakness, dizziness, and salt craving indicate a
deficiency in aldosterone. The low blood pressure reading of 70/50 mm Hg is also suggestive of
adrenal insufficiency. The patient's glycemia of 3.0 mmol/l indicates hypoglycemia, which is a
common finding in adrenal insufficiency.
Other Options:
B) Diabetes Mellitus - Diabetes mellitus is a metabolic disorder characterized by high blood
glucose levels. The patient's glycemia of 3.0 mmol/l is low, which rules out diabetes mellitus.
C) Coronary Artery Disease - There is no mention of any symptoms suggestive of coronary
artery disease in the patient's history or physical examination findings.
D) Chronic Pancreatitis - Chronic pancreatitis is a condition characterized by inflammation of the
pancreas. The patient's symptoms and physical examination findings are not suggestive of
chronic pancreatitis.
E) Pulmonary Tuberculosis - The patient's history suggests that he has already had pulmonary
tuberculosis, but it is not the cause of his current symptoms.
Making Other Options Right:
B) Diabetes Mellitus - If the patient's glycemia was elevated instead of low, diabetes mellitus
would be a possible cause of his symptoms.
C) Coronary Artery Disease - If the patient had chest pain or other symptoms suggestive of
coronary artery disease, this would be a possible cause of his symptoms.
D) Chronic Pancreatitis - If the patient had abdominal pain, nausea, and vomiting, which are
characteristic symptoms of chronic pancreatitis, this would be a possible cause of his symptoms.
Medication:
• Glucocorticoids (Hydrocortisone) and Mineralocorticoids (Fludrocortisone) are the mainstay of
treatment for chronic adrenal insufficiency.
endocrinology

• In addition to medication, patients with chronic adrenal insufficiency should be educated on


the importance of stress management and the need for increased steroid replacement during
periods of stress.
• The patient should also be advised to carry an emergency card or bracelet that identifies their
condition and the need for increased steroid replacement during periods of stress.
• Surgery may be indicated if there is an underlying tumor causing the adrenal insufficiency.
Words:
• Chronic Adrenal Insufficiency: A condition where the adrenal glands do not produce enough
hormones, mainly cortisol and aldosterone.
• Hyperpigmentation: Darkening of the skin, which is a characteristic feature of chronic adrenal
insufficiency.
• Glycemia: The concentration of glucose in the blood.
Findings:
• Normal blood pressure is around 120/80 mm Hg, but the patient's blood pressure reading is
low at 70/50 mm Hg.
• Normal glycemia is between 3.9 and 5.5 mmol/l, but the patient's glycemia is low at 3.0
mmol/l.
Hormones of the Adrenal Gland:
The adrenal gland produces several hormones, including:
1. Cortisol - A glucocorticoid hormone that plays a role in the body's stress response and helps
regulate metabolism, immune function, and blood sugar levels.
2. Aldosterone - A mineralocorticoid hormone that regulates blood pressure by controlling the
body's salt and water balance.
3. Androgens - Male sex hormones that are produced in small amounts in both males and
females.
4. Epinephrine and Norepinephrine - Also known as adrenaline and noradrenaline, these

Therapy
hormones are produced by the adrenal medulla and play a role in the body's stress response.
Disorders of the Adrenal Gland:
1. Adrenal Insufficiency - A condition where the adrenal gland does not produce enough
hormones. It can be caused by autoimmune diseases, infections, or genetic disorders.
2. Cushing's Syndrome - A condition where the body produces too much cortisol, which can be
caused by tumors, medication, or pituitary gland disorders.
3. Conn's Syndrome - A condition where there is too much aldosterone production, which can
lead to high blood pressure and low potassium levels.
Treatment of Adrenal Gland Disorders:
1. Adrenal Insufficiency - Treatment typically involves hormone replacement therapy with
glucocorticoids (e.g., hydrocortisone) and mineralocorticoids (e.g., fludrocortisone).
2. Cushing's Syndrome - Treatment options include surgery to remove the tumor, medication to
lower cortisol levels, or radiation therapy.
3. Conn's Syndrome - Treatment typically involves medication to lower aldosterone levels, such
as spironolactone.
endocrinology

130 Adrenal Insufficiency

A 34 y.o. patient has been suffering from pulmonary tuberculosis for 7 years; he complains
of muscle feebleness, weight loss, diarrheas, increased frequency of urination. Objectively:
hyperpigmentation of skin, gums, internal cheek surfaces. AP is 90/58 mm Hg. Blood
count: erythrocutes - 3,11012/L, Hb- 95 g/L, C.I.- 0,92; leukocytes -
9,4109
/L, eosinophils - 7, segmentonuclear leukocytes - 45, stab neutrophils - 1,
lymphocytes - 40, monocytes - 7, Na+
- 115 mmole/L, К+
- 7,3 mmole/L. What is the
preliminary diagnosis?
A Primary insufficiency of adrenal cortex
B Pheochromocytoma
C Primary hyperaldosteronism
D Congenital adrenal hyperplasia
E Diabetes insipidus

Correct Answer: A Primary insufficiency of adrenal cortex


Key Points:
• 34-year-old patient with a history of pulmonary tuberculosis for 7 years and symptoms of
muscle weakness, weight loss, diarrhea, increased frequency of urination, and

Therapy
hyperpigmentation of the skin and mucous membranes
• Low blood pressure (90/58 mm Hg), low red blood cell count (3.1*1012/L), low hemoglobin
(95 g/L), low serum sodium (115 mmol/L), and high serum potassium (7.3 mmol/L)
• Preliminary diagnosis: A) Primary insufficiency of adrenal cortex (also known as Addison's
disease)
Explanation:
• The most probable diagnosis for this patient with a history of pulmonary tuberculosis for 7
years and symptoms of muscle weakness, weight loss, diarrhea, increased frequency of
urination, and hyperpigmentation of the skin and mucous membranes is A) Primary insufficiency
of adrenal cortex (also known as Addison's disease).
• Addison's disease is a rare endocrine disorder characterized by chronic adrenal insufficiency,
which is the inability of the adrenal gland to produce sufficient amounts of cortisol and
aldosterone.
• The clinical features of Addison's disease include fatigue, muscle weakness, weight loss,
gastrointestinal symptoms such as nausea, vomiting, and diarrhea, and hyperpigmentation of
the skin and mucous membranes.
• The low blood pressure (90/58 mm Hg), low red blood cell count (3.1*1012/L), low
hemoglobin (95 g/L), and low serum sodium (115 mmol/L) seen in this patient are consistent
with adrenal insufficiency.
• The high serum potassium (7.3 mmol/L) seen in this patient is also consistent with adrenal
insufficiency, as aldosterone deficiency can lead to hyperkalemia.
• Other diagnoses such as pheochromocytoma, primary hyperaldosteronism, congenital adrenal
hyperplasia, or diabetes insipidus are less likely based on the clinical presentation.
Other Options:
• B) Pheochromocytoma is less likely in this case because it typically presents with paroxysmal
hypertension, palpitations, and headaches, rather than low blood pressure and gastrointestinal
symptoms.
• C) Primary hyperaldosteronism is less likely in this case because it typically presents with
hypertension, hypokalemia, and metabolic alkalosis, rather than low blood pressure and
hyperkalemia.
endocrinology

• D) Congenital adrenal hyperplasia is less likely in this case because it typically presents in
childhood with ambiguous genitalia in females or precocious puberty in males, rather than
gastrointestinal symptoms and hyperpigmentation.
• E) Diabetes insipidus is less likely in this case because it typically presents with polydipsia and
polyuria, rather than gastrointestinal symptoms and hyperpigmentation.
Making Other Options Right:
• B) Pheochromocytoma can occur in patients with a history of hypertension or paroxysmal
symptoms, with the clinical features typically involving paroxysmal hypertension, palpitations,
and headaches. Changing the question to ask about the most probable diagnosis in a patient
with paroxysmal symptoms and palpitations would make this option correct.
• C) Primary hyperaldosteronism can occur in patients with a history of hypertension or
hypokalemia, with the clinical features typically involving hypertension, hypokalemia, and
metabolic alkalosis. Changing the question to ask about the most probable diagnosis in a patient
with hypertension and hypokalemia would make this option correct.
• D) Congenital adrenal hyperplasia can occur in patients with a history of ambiguous genitalia
in females or precocious puberty in males, with the clinical features typically involving virilization
in females or premature development of secondary sexual characteristics in males. Changing
the question to ask about the most probable diagnosis in a patient with ambiguous genitalia or
precocious puberty would make this option correct.
• E) Diabetes insipidus can occur in patients with a history of polydipsia and polyuria, with the
clinical features typically involving excessive thirst and urine output. Changing the question to
ask about the most probable diagnosis in a patient with excessive thirst and urine output would
make this option correct.
Words:
• Adrenal insufficiency: the inability of the adrenal gland to produce sufficient amounts of
cortisol and aldosterone, leading to fatigue, muscle weakness, weight loss, gastrointestinal

Therapy
symptoms, and hyperpigmentation
• Addison's disease: a rare endocrine disorder characterized by chronic adrenal insufficiency,
which is the inability of the adrenal gland to produce sufficient amounts of cortisol and
aldosterone
• Cortisol: a steroid hormone produced by the adrenal gland that helps regulate metabolism,
immune function, and stress response
• Aldosterone: a steroid hormone produced by the adrenal gland that helps regulate blood
pressure and electrolyte balance
• Pheochromocytoma: a rare tumor of the adrenal gland or sympathetic nervous system that
produces excess amounts of catecholamines, leading to hypertension, palpitations, headaches,
and other symptoms
• Hyperaldosteronism: a condition in which the adrenal gland produces excess amounts of
aldosterone, leading to hypertension, hypokalemia, and metabolic alkalosis
• Congenital adrenal hyperplasia: a group of genetic disorders that affect the adrenal gland's
ability to produce cortisol and aldosterone, leading to a range of symptoms including ambiguous
genitalia in females and precocious puberty in males
• Diabetes insipidus: a rare disorder characterized by excessive thirst and urine output due to a
deficiency of antidiuretic hormone (ADH) or a resistance to its effects
• Hyperpigmentation: darkening of the skin or mucous membranes due to increased melanin
production
Findings:
• The patient has a history of pulmonary tuberculosis for 7 years and symptoms of muscle
weakness, weight loss, diarrhea, increased frequency of urination, and hyperpigmentation of the
skin and mucous membranes.
• The patient has low blood pressure (90/58 mm Hg), low red blood cell count (3.1*1012/L),
low hemoglobin (95 g/L), and low serum sodium (115 mmol/L), which are consistent with
adrenal insufficiency.
• The patient has high serum potassium (7.3 mmol/L), which is also consistent with adrenal
insufficiency.
• These findings suggest a preliminary diagnosis of primary insufficiency of adrenal cortex
(Addison's disease), which is a rare endocrine disorder characterized by chronic adrenal
endocrinology

insufficiency and can present with fatigue, muscle weakness, weight loss, gastrointestinal
symptoms, and hyperpigmentation.

Therapy
endocrinology

131 Adrenal Insufficiency

A 26-year-old patient complains about considerable muscle weakness, dizziness, extended


abdominal pain, nausea and vomiting giving no relief. The disease has been gradually
developing within 6 months. There was progress of general weakness, skin darkening. The
patient fell into grave condition after an ARD: there appeared abdominal pain and frequent
vomiting. Objectively: the skin is dry with diffuse pigmentation. Heart sounds are significantly
weakened, heart rate - 60/min, AP- 80/40 mm Hg. The abdomen is slightly painful in the
epigastrial
region. In blood: WBCs - 8,1*109
/l, glucose - 3,0 millimole/l. What is the most likely diagnosis?
A Chronic adrenal insufficiency. Addisonian crisis
B Acute pancreatitis
C Toxic infectious shock
D Acute food poisoning
E Acute cholecystitis

Correct Answer: A Chronic adrenal insufficiency. Addisonian crisis


Key Points:
• 26-year-old patient with muscle weakness, dizziness, abdominal pain, nausea, and vomiting
• Gradual development over 6 months, with progress of weakness and skin darkening
• Fell into grave condition after an acute respiratory disease, with abdominal pain and frequent

Therapy
vomiting
• Diffuse pigmentation of the skin, weakened heart sounds, low blood pressure, slightly painful
epigastric region
• Laboratory findings include low glucose and normal white blood cell count
Explanation:
The patient's symptoms and laboratory findings are consistent with chronic adrenal insufficiency,
also known as Addison's disease. The disease occurs due to a deficiency of cortisol and
aldosterone, which are hormones produced by the adrenal glands.
The gradual onset of symptoms over 6 months, including muscle weakness, dizziness, and
abdominal pain, is typical of chronic adrenal insufficiency. The skin darkening, or
hyperpigmentation, is also a characteristic feature of the disease. The acute worsening of
symptoms, including abdominal pain and vomiting, may indicate an Addisonian crisis, which is a
potentially life-threatening complication of chronic adrenal insufficiency.
The weakened heart sounds, low heart rate and blood pressure, and slightly painful epigastric
region are also consistent with chronic adrenal insufficiency. The low glucose level in the blood
may indicate adrenal insufficiency, as cortisol plays a role in maintaining normal blood glucose
levels.
Other Options:
B) Acute pancreatitis is characterized by sudden onset of severe abdominal pain, nausea, and
vomiting. It is not associated with skin darkening or adrenal insufficiency.
C) Toxic infectious shock is a severe and potentially life-threatening condition caused by a
bacterial infection. It typically presents with fever, low blood pressure, and organ failure. It is
not associated with skin darkening or adrenal insufficiency.
D) Acute food poisoning can cause nausea, vomiting, and abdominal pain. It is not associated
with skin darkening or adrenal insufficiency.
E) Acute cholecystitis is inflammation of the gallbladder and is typically characterized by severe
abdominal pain, fever, and nausea. It is not associated with skin darkening or adrenal
insufficiency.
Making Other Options Wrong:
endocrinology

B) To make acute pancreatitis a wrong option, the question would need to describe symptoms
that are less consistent with chronic adrenal insufficiency, such as absence of skin darkening
and normal blood glucose levels.
C) To make toxic infectious shock a wrong option, the question would need to describe
symptoms that are less consistent with chronic adrenal insufficiency, such as absence of skin
darkening and low blood pressure.
D) To make acute food poisoning a wrong option, the question would need to describe
symptoms that are less consistent with chronic adrenal insufficiency, such as absence of skin
darkening and low blood pressure.
E) To make acute cholecystitis a wrong option, the question would need to describe symptoms
that are less consistent with chronic adrenal insufficiency, such as absence of skin darkening
and low blood pressure.
Words:
• Muscle weakness: a condition in which muscles become weak and fatigued
• Dizziness: a feeling of lightheadedness or unsteadiness
• Abdominal pain: pain or discomfort in the abdomen
• Nausea: a feeling of sickness or discomfort in the stomach
• Vomiting: the act of forcefully expelling the contents of the stomach through the mouth
• Skin darkening: hyperpigmentation of the skin
• Adrenal insufficiency: a condition in which the adrenal glands do not produce enough cortisol
and aldosterone
• Addison's disease: another name for chronic adrenal insufficiency
• Addisonian crisis: a potentially life-threatening complication of chronic adrenal insufficiency
characterized by acute worsening of symptoms
• Low blood pressure: a condition in which blood pressure is lower than normal
• Epigastric region: the upper central region of the abdomen

Therapy
• Laboratory findings: results of blood tests and other diagnostic tests
• Glucose: a type of sugar found in the blood
• Acute pancreatitis: sudden inflammation of the pancreas
• Toxic infectious shock: a severe and potentially life-threatening condition caused by a bacterial
infection
• Acute food poisoning: illness caused by consuming contaminated food or water
• Acute cholecystitis: inflammation of the gallbladder
Findings:
• 26-year-old patient with muscle weakness, dizziness, abdominal pain, nausea, and vomiting
• Gradual development over 6 months, with progress of weakness and skin darkening
• Fell into grave condition after an acute respiratory disease, with abdominal pain and frequent
vomiting
• Diffuse pigmentation of the skin, weakened heart sounds, low blood pressure, slightly painful
epigastric region
• Laboratory findings include low glucose and normal white blood cell count
• Consistent with chronic adrenal insufficiency, also known as Addison's disease
endocrinology

132 Coma

A 23 y.o. woman who suffers from insulin-dependent diabetes was admitted to the acute
care department with mental confusion, inadequate anxious behaviour, hyperhidrosis,
excessive salivation, tachycardia. What examination will be a primary task?
A Blood test for sugar
B Clinical blood analysis
C Plasma electrolytes test
D Gaseous composition of arterial blood
E Blood urea and creatinine test

Correct Answer: A blood test for sugar is the primary examination that should be done in a
patient with insulin-dependent diabetes admitted with the symptoms of mental confusion,
inadequate anxious behavior, hyperhidrosis, excessive salivation, and tachycardia.
Key Points: 23 y.o. woman, insulin-dependent diabetes, acute care department, mental
confusion, inadequate anxious behavior, hyperhidrosis, excessive salivation, tachycardia.
Explanation: The patient is presenting with symptoms of diabetic ketoacidosis (DKA). DKA is a
severe complication of diabetes that occurs when the body produces high levels of ketones and
acids, leading to metabolic acidosis. The primary cause of DKA is an absolute or relative insulin
deficiency, which leads to an increase in glucagon levels, stimulating lipolysis, and ketone body
formation. The most important examination in a patient with DKA is a blood test for sugar,
which will help to confirm hyperglycemia.

Therapy
Other Options:
B. Clinical blood analysis is not the primary examination in a patient with DKA.
C. A plasma electrolytes test can be helpful in the diagnosis of DKA, but it is not the primary
examination.
D. Gaseous composition of arterial blood may be done to determine the degree of acidosis, but
it is not the primary examination.
E. A blood urea and creatinine test can be done to evaluate kidney function, but it is not the
primary examination.
Making Other Options Right:
B. Clinical blood analysis can be useful to evaluate other parameters, such as leukocyte count,
that may be altered in patients with DKA.
C. A plasma electrolytes test can be helpful in the diagnosis of other electrolyte imbalances that
may occur in DKA, such as hyponatremia and hypokalemia.
D. Gaseous composition of arterial blood may be done to determine the degree of acidosis in
DKA patients.
E. A blood urea and creatinine test can be done to evaluate kidney function, which may be
affected in severe cases of DKA.
Medication: Insulin is the primary medication used to treat DKA. It helps to lower blood sugar
levels and decrease ketone production. Electrolyte imbalances, such as hypokalemia and
hyponatremia, may also require correction.
Words: Diabetic ketoacidosis (DKA) is a medical emergency that requires prompt treatment.
Findings: In a blood test for sugar, normal values for fasting blood glucose are between 70 and
100 mg/dL, while random blood glucose levels should be less than 200 mg/dL. In a patient with
DKA, blood glucose levels are typically greater than 250 mg/dL.
endocrinology

133 Coma

A 13 year old patient is suffering from an acute disease with the following symptoms: thirst,
polyuria, weakness. Objectively: his general condition is satisfactory, there is no smell of
acetone. Glucose concentration in blood on an empty stomach is 32 micromole/l, in urine -
6%, acetone +. What treatment should be administered?
A Short-acting insulin
B Long-acting insulin
C Biguanides
D Sulfonylurea
E Diet

Correct Answer: A Short-acting insulin is the correct answer because the patient is presenting
with symptoms of diabetic ketoacidosis, which is a life-threatening complication of diabetes, and
requires prompt treatment with insulin.
Key Points:
• 13-year-old patient
• Acute disease with symptoms of thirst, polyuria, and weakness
• Normal general condition, no smell of acetone
• Glucose concentration in blood is 32 micromole/l and in urine, it is 6%
• Acetone present
• Treatment is required

Therapy
Explanation:
Diabetic ketoacidosis (DKA) is a life-threatening complication of diabetes that occurs when there
is a lack of insulin in the body. The body starts breaking down fat for energy, producing ketones
that build up in the blood and urine. Symptoms of DKA include thirst, polyuria, weakness, and
fruity breath odor. The laboratory findings include hyperglycemia, ketonemia, and metabolic
acidosis.
The treatment for DKA requires immediate administration of short-acting insulin to reduce blood
glucose levels, suppress ketone production, and halt lipolysis. Insulin therapy should be started
with an intravenous infusion, followed by subcutaneous injections to maintain glucose control.
Additionally, fluid and electrolyte replacement is crucial to correct dehydration and electrolyte
imbalances.
Other Options:
B) Long-acting insulin: Long-acting insulin is not the correct answer because it would not be
effective in treating DKA, which requires immediate and aggressive treatment with short-acting
insulin.
C) Biguanides: Biguanides are not the correct answer because they are not used to treat DKA,
which requires insulin therapy.
D) Sulfonylurea: Sulfonylureas are not the correct answer because they stimulate insulin
secretion from the pancreas and are not effective in treating DKA.
E) Diet: Diet is not the correct answer because it cannot treat DKA, which requires insulin
therapy.
Making Other Options Right:
B) Long-acting insulin: Long-acting insulin would be a correct answer if the patient had type 1 or
type 2 diabetes and was not presenting with symptoms of DKA.
C) Biguanides: Biguanides would be a correct answer if the patient had type 2 diabetes and was
not presenting with symptoms of DKA.
D) Sulfonylurea: Sulfonylureas would be a correct answer if the patient had type 2 diabetes and
was not presenting with symptoms of DKA.
E) Diet: Diet would be a correct answer if the patient had type 2 diabetes and was not
presenting with symptoms of DKA.
Medication:
endocrinology

The treatment for DKA requires immediate administration of short-acting insulin to reduce blood
glucose levels, suppress ketone production, and halt lipolysis.
Words:
Polyuria - the production of abnormally large volumes of dilute urine
Acetone - a ketone body produced during fat metabolism
Findings:
The normal glucose concentration in blood is 4-6 mmol/L, and the normal glucose concentration
in urine is less than 0.1%. The finding of 6% glucose in urine indicates hyperglycemia and is
consistent with the diagnosis of diabetes. The presence of acetone in the urine indicates ketosis
and is consistent with the diagnosis of diabetic ketoacidosis.

Therapy
endocrinology

134 Coma

During examination a patient is unconscious, his skin is dry and hot, face hyperemia is
present. The patient has Kussmaul's respiration, there is also smell of acetone in the air.
Symptoms of peritoneum irritation are positive. Blood sugar is at the rate of 33 millimole/l.
What emergency actions should be taken?
A Intravenous infusion of short-acting insulin
B Intravenous infusion of glucose along with insulin
C Introduction of long-acting insulin
D Intravenous infusion of neohaemodesum along with glutamic acid
E Intravenous infusion of sodium chloride saline

Correct Answer: intravenous infusion of short-acting insulin.


Key Points:
• The patient is unconscious with dry and hot skin, hyperemic face, and positive symptoms of
peritoneal irritation.
• The patient has Kussmaul's respiration and the smell of acetone in the air.
• Blood sugar is 33 millimole/l, which is significantly elevated.
Explanation:
• The patient's symptoms and blood sugar level suggest that they are in a state of diabetic
ketoacidosis (DKA), which is a life-threatening complication of uncontrolled diabetes.
• DKA is characterized by hyperglycemia, ketosis, and metabolic acidosis, which can lead to

Therapy
dehydration, electrolyte imbalances, and organ failure.
• Intravenous infusion of short-acting insulin is the cornerstone of DKA management. Insulin is
given to promote glucose uptake by cells and inhibit ketone production.
• Intravenous infusion of short-acting insulin is preferred over long-acting insulin, as it can
rapidly lower blood sugar levels and correct the acidosis.
• Intravenous infusion of glucose should also be given to prevent hypoglycemia.
• Intravenous infusion of neohaemodesum and glutamic acid, as well as intravenous infusion of
sodium chloride saline, are not appropriate treatments for DKA.
Other Options:
B) Intravenous infusion of glucose along with insulin: This option is partially correct, but insulin
should be given first to prevent hypoglycemia.
C) Introduction of long-acting insulin: This option is not indicated in the acute management of
DKA.
D) Intravenous infusion of neohaemodesum along with glutamic acid: This option is not
appropriate for the management of DKA.
E) Intravenous infusion of sodium chloride saline: This option is not appropriate for the
management of DKA.
Making Other Options Right:
B) Intravenous infusion of glucose along with insulin: This option may be correct if insulin is
given first to prevent hypoglycemia.
C) Introduction of long-acting insulin: This option may be correct if the patient has stabilized
and is transitioning to subcutaneous insulin therapy.
D) Intravenous infusion of neohaemodesum along with glutamic acid: This option may be
correct if the patient has other medical conditions that require correction of fluid and electrolyte
imbalances.
E) Intravenous infusion of sodium chloride saline: This option may be correct if the patient is
dehydrated and requires fluid resuscitation.
Medication: In addition to insulin and glucose, the patient may require intravenous fluids,
electrolyte replacement, and treatment for any underlying medical conditions.
Words:
endocrinology

• Unconscious: the state of being unaware of one's surroundings and unable to respond to
stimuli.
• Kussmaul's respiration: a type of deep, rapid breathing that occurs in response to metabolic
acidosis.
• Acetone: a ketone body that can be detected on the breath of patients with DKA.
• Peritoneal irritation: inflammation of the peritoneum, the lining of the abdominal cavity.
• Blood sugar: the concentration of glucose in the blood, measured in millimoles per liter
(mmol/L).

Therapy
endocrinology

135 Conn's syndrome

A 40-year-old female patient complain of headache, dizziness, muscle weakness,


sometimes - cramps in the extremities. She has been taking antihypertensive medications for
10 years. AP- 180/100 mm Hg. Blood potassium - 1,8 millimole/l, sodium - 4,8 millimole/l. In
urine: alkaline reaction, the relative density - 1012, protein and sugar are not found, WBCs -
3-4 in the field of vision, RBCs - 1-2 in the field of vision. Conn's syndrome is suspected.
Which drug should be chosen for the treatment of arterial hypertension?
A Spironolactone
B Propanolol
C Enalapril
D Hydrochlorothiazide
E Clonidine

Correct Answer: A - Spironolactone is the correct answer because it is a potassium-sparing


diuretic that inhibits the action of aldosterone in the distal tubules of the kidney. In Conn's
syndrome, there is an excess production of aldosterone leading to hypokalemia and
hypertension. Spironolactone helps in controlling hypertension by blocking the action of
aldosterone, which leads to increased excretion of sodium and water and retention of potassium.
Key Points:
• 40-year-old female patient with headache, dizziness, muscle weakness, and cramps in the
extremities

Therapy
• Hypertension for 10 years
• Blood potassium - 1,8 millimole/l, sodium - 4,8 millimole/l
• Urine: alkaline reaction, the relative density - 1012, protein and sugar are not found, WBCs -
3-4 in the field of vision, RBCs - 1-2 in the field of vision
• Conn's syndrome is suspected
Explanation:
Conn's syndrome is a condition in which there is an excess production of aldosterone by the
adrenal glands, leading to hypertension and hypokalemia. The patient's symptoms and lab
findings suggest that she might be suffering from Conn's syndrome. Spironolactone is a
potassium-sparing diuretic that helps in controlling hypertension in Conn's syndrome by blocking
the action of aldosterone in the distal tubules of the kidney. This leads to increased excretion of
sodium and water and retention of potassium.
Other Options:
• B Propanolol is a beta-blocker that is used in the treatment of hypertension. It works by
blocking the effects of adrenaline, which leads to decreased heart rate and blood pressure.
However, it is not the drug of choice for the treatment of Conn's syndrome.
• C Enalapril is an ACE inhibitor that is used in the treatment of hypertension. It works by
blocking the production of angiotensin II, which leads to decreased blood pressure. However, it
is not the drug of choice for the treatment of Conn's syndrome.
• D Hydrochlorothiazide is a thiazide diuretic that is used in the treatment of hypertension. It
works by increasing the excretion of sodium and water, leading to decreased blood volume and
blood pressure. However, it is not the drug of choice for the treatment of Conn's syndrome.
• E Clonidine is an alpha-2 agonist that is used in the treatment of hypertension. It works by
decreasing sympathetic outflow, leading to decreased heart rate and blood pressure. However, it
is not the drug of choice for the treatment of Conn's syndrome.
Making Other Options Right:
• Option B Propanolol: If the question mentioned that the patient had tachycardia or
palpitations, propranolol would be the correct answer as it is a beta-blocker that helps in
controlling both hypertension and tachycardia.
endocrinology

• Option C Enalapril: If the question mentioned that the patient had heart failure, enalapril
would be the correct answer as it is an ACE inhibitor that helps in controlling hypertension and
heart failure.
• Option D Hydrochlorothiazide: If the question mentioned that the patient had mild
hypertension without hypokalemia, hydrochlorothiazide would be the correct answer as it is a
thiazide diuretic that helps in controlling mild hypertension.
• Option E Clonidine: If the question mentioned that the patient had hypertension with anxiety
or withdrawal symptoms, clonidine would be the correct answer as it helps in controlling both
hypertension and anxiety/withdrawal symptoms.
Medication:
• Spironolactone: The usual starting dose for spironolactone is 25 mg once daily. The maximum
dose is 400 mg/day. Side effects include hyperkalemia, electrolyte disturbances, and
gynecomastia.
• Propanolol: The usual starting dose for propranolol is 40-80 mg twice daily. The maximum
dose is 640 mg/day. Side effects include bradycardia, hypotension, bronchospasm, and fatigue.
• Enalapril: The usual starting dose for enalapril is 5-10 mg once daily. The maximum dose is 40
mg/day. Side effects include hypotension, dry cough, and hyperkalemia.
• Hydrochlorothiazide: The usual starting dose for hydrochlorothiazide is 12.5-25 mg once daily.
The maximum dose is 100 mg/day. Side effects include hypokalemia, hyponatremia, and
hyperuricemia.
• Clonidine: The usual starting dose for clonidine is 0.1 mg twice daily. The maximum dose is
2.4 mg/day. Side effects include bradycardia, hypotension, and dry mouth.
Words:
• Conn's syndrome: a condition in which there is an excess production of aldosterone by the
adrenal glands, leading to hypertension and hypokalemia.
• Hypokalemia: a condition in which there is a low level of potassium in the blood.

Therapy
• Diuretic: a medication that increases the excretion of urine, leading to increased excretion of
sodium and water and decreased blood volume and blood pressure.
Findings:
• Blood potassium - normal range is 3.6-5.2 millimole/l
• Blood sodium - normal range is 135-145 millimole/l
• Urine pH - normal range is 5-7
• Urine specific gravity - normal range is 1.010-1.030
• WBCs in urine - normal range is 0-5 in the field of vision
• RBCs in urine - normal range is 0-2 in the field of vision
endocrinology

136 Diabetes

A 49-year-old female patient has type 1 diabetes of moderate severity. The disease is
complicated by retinopathy and polyneuropathy. Besides that, repeated analyses of the daily
urinary excretion of albumin revealed microalbuminuria (200-300 mg/day). Glomerular filtration
rate is 105 ml/min. Blood pressure is within normal range. Normalization of the following
indicator should be the first-priority task in the secondary prevention of diabetic nephropathy:
A Glycosylated hemoglobin
B C-peptide
C Blood insulin
D Fasting glucose
E Glycemia 2 hours after a meal

Correct Answer: Option A (Glycosylated hemoglobin) is the correct answer because it is a


measure of long-term glycemic control and is a predictor of the risk of developing diabetic
complications, including nephropathy. In patients with diabetes and microalbuminuria,
maintaining tight glycemic control through lifestyle modification and/or medication therapy is
the first-priority task in the secondary prevention of diabetic nephropathy.
Key Points:
• 49-year-old female patient with type 1 diabetes of moderate severity
• Complicated by retinopathy and polyneuropathy
• Repeated analyses of daily urinary albumin excretion revealed microalbuminuria (200-300

Therapy
mg/day)
• Glomerular filtration rate is 105 ml/min
• Blood pressure is within normal range
• What is the first-priority task in the secondary prevention of diabetic nephropathy?
Explanation:
• Diabetic nephropathy is a complication of diabetes that affects the kidneys and can lead to
kidney failure if left untreated.
• Microalbuminuria is an early sign of diabetic nephropathy and is characterized by a urinary
excretion of 30-299 mg of albumin per day.
• Glycosylated hemoglobin (HbA1c) is a measure of long-term glycemic control and reflects the
average blood glucose level over the past 2-3 months. In patients with diabetes and
microalbuminuria, maintaining tight glycemic control through lifestyle modification and/or
medication therapy is the first-priority task in the secondary prevention of diabetic nephropathy.
• C-peptide and blood insulin levels are measures of insulin secretion and are not directly
related to the development of diabetic nephropathy.
• Fasting glucose and glycemia 2 hours after a meal are measures of short-term glycemic
control and are not as reliable as HbA1c in predicting the risk of developing diabetic
complications.
Other Options:
• Option B (C-peptide) is not the correct answer because it is a measure of endogenous insulin
secretion and is not directly related to the development of diabetic nephropathy.
• Option C (Blood insulin) is not the correct answer because it is also a measure of insulin
secretion and is not directly related to the development of diabetic nephropathy.
• Option D (Fasting glucose) is not the correct answer because it is a measure of short-term
glycemic control and is not as reliable as HbA1c in predicting the risk of developing diabetic
complications.
• Option E (Glycemia 2 hours after a meal) is not the correct answer because it is also a
measure of short-term glycemic control and is not as reliable as HbA1c in predicting the risk of
developing diabetic complications.
Medication:
endocrinology

• Medications that may be used to achieve tight glycemic control in patients with diabetes
include insulin, oral hypoglycemic agents, and glucagon-like peptide-1 (GLP-1) receptor
agonists.
• Other medications that may be used in the management of diabetic nephropathy include
angiotensin-converting enzyme inhibitors (ACE inhibitors) and angiotensin receptor blockers
(ARBs), which can help to reduce proteinuria and slow the progression of kidney damage.
Words:
• Type 1 diabetes: A form of diabetes in which the body's immune system attacks and destroys
the insulin-producing cells in the pancreas
• Retinopathy: A complication of diabetes that affects the blood vessels in the retina and can
lead to vision loss
• Polyneuropathy: A type of nerve damage that can cause numbness, tingling, and pain in the
hands and feet
• Microalbuminuria: A urinary excretion of 30-299 mg of albumin per day, which is an early sign
of diabetic nephropathy
• Glomerular filtration rate (GFR): A measure of kidney function that estimates how much blood
passes through the glomeruli (small blood vessels in the kidneys) each minute
• Blood pressure: The force of blood against the walls of the arteries
• Secondary prevention: Measures taken to prevent the recurrence or progression of a disease
or condition
Findings:
• Type 1 diabetes of moderate severity (patient history)
• Retinopathy and polyneuropathy (patient history)
• Microalbuminuria: 200-300 mg/day (abnormal finding)
• Glomerular filtration rate: 105 ml/min (normal range: 90-120 ml/min)
• Blood pressure: within normal range (normal finding)

Therapy
• Glycosylated hemoglobin (HbA1c): not mentioned in the question (normal range: <6.5% for
most adults with diabetes)
endocrinology

137 Diabetes

A 54-year-old patient complains of weakness, weight loss despite the unchanged appetite,
frequent urination, skin itch for six months. Some time ago the patient underwent treatment for
furunculosis. She hasn't been examined recently. Objectively: malnutrition, dry skin with signs
of
scratching. Small lymph nodes can be palpated in the axillary regions. Changes in the internal
organs are absenr. What testing must be administered in the first place?
A Blood sugar test on an empty stomach
B Complete blood count
C Endoscopy of stomach
D Lymph node biopsy
E Blood sterility testing

Correct Answer: A Blood sugar test on an empty stomach - This is the test that must be
administered in the first place.
Key Points:
• A 54-year-old patient complains of weakness, weight loss despite unchanged appetite,
frequent urination, and skin itch for six months.
• The patient underwent treatment for furunculosis in the past.
• Objectively, the patient has malnutrition, dry skin with signs of scratching, and palpable lymph
nodes in the axillary regions.

Therapy
• The question asks which test should be administered first.
Explanation:
The patient's symptoms of weakness, weight loss, frequent urination, and skin itch are
suggestive of hyperglycemia, which can occur as a result of uncontrolled diabetes mellitus. The
presence of palpable lymph nodes may also be suggestive of an underlying infection, which can
be more common in patients with poorly controlled diabetes.
A blood sugar test on an empty stomach (also known as a fasting blood glucose test) is a simple
and non-invasive test that can help to diagnose diabetes mellitus. The test involves measuring
the patient's blood glucose level after an overnight fast, which can help to identify
hyperglycemia and confirm the diagnosis of diabetes.
Other Options:
• B Complete blood count: A complete blood count (CBC) is a blood test that measures the
number and types of cells in the blood. While a CBC may provide some information about the
patient's overall health, it is not specific to any particular condition and is unlikely to be helpful
in this case.
• C Endoscopy of stomach: Endoscopy of the stomach may be useful in patients with symptoms
such as abdominal pain or nausea, but it is unlikely to be helpful in this case, where the
patient's symptoms are suggestive of hyperglycemia and an underlying infection.
• D Lymph node biopsy: Lymph node biopsy may be useful in patients with palpable lymph
nodes that are suspicious for cancer or other systemic diseases, but it is unlikely to be helpful in
this case, where the patient's symptoms are suggestive of hyperglycemia and an underlying
infection.
• E Blood sterility testing: Blood sterility testing may be useful in patients with symptoms of
infection, but it is unlikely to be helpful in this case, where the patient's symptoms are
suggestive of hyperglycemia and an underlying infection.
Making Other Options Right:
• B Complete blood count: If the patient had symptoms such as fever or anemia, a CBC may be
helpful in identifying the underlying cause.
• C Endoscopy of stomach: If the patient had symptoms such as abdominal pain or nausea,
endoscopy of the stomach may be useful in identifying the underlying cause.
endocrinology

• D Lymph node biopsy: If the patient had palpable lymph nodes that were suspicious for cancer
or other systemic diseases, lymph node biopsy may be helpful in making a diagnosis.
• E Blood sterility testing: If the patient had symptoms of infection, blood sterility testing may
be helpful in identifying the underlying cause.
Medication:
• Treatment for diabetes mellitus typically involves lifestyle modifications (such as diet and
exercise) and medication to control blood glucose levels. Medications may include oral
hypoglycemic agents (such as metformin) or insulin injections.
• Treatment for an underlying infection may involve antibiotics or other antimicrobial agents.
Words:
• Hyperglycemia: high blood glucose levels.
• Diabetes mellitus: a chronic metabolic disorder characterized by hyperglycemia and impaired
insulin production or action.
• Fasting blood glucose test: a blood test that measures blood glucose levels after an overnight
fast.
• Palpable lymph nodes: lymph nodes that can be felt on physical examination.
• Furunculosis: a skin condition characterized by the formation of boils or abscesses.
Findings:
• No normal values are mentioned in the question.

Therapy
endocrinology

138 Diabetes

A 58-year-old patient has a 3-year history diabetes mellitus type II. He has been keeping to
a diet and regularly taking glyburide. He has been delivered to a hospital on an
emergency basis for acute abdomen. Objectively: the patient is of supernutrition type. The
skin is dry. In the lungs vesicular breathing can be auscultated. Heart sounds are regular,
90/min. AP- 130/70 mm Hg. The symptom of "wooden belly" is visible. Blood sugar - 9,8
millimole/l. The patients has indication for laparotomy. What is the most appropriate way of
further treatment of diabetes?
A To administer short insulin
B To continue taking glyburide
C To administer Semilong to be taken in the morning and insulin - in the evening
D To administer 1 tablet of Glurenorm three times a day
E To administer 1 tablet of Maninil three times a day

Correct Answer: A To administer short insulin - The patient's acute abdomen and high blood
sugar level of 9.8 mmol/L suggest a need for immediate control of blood glucose levels. The
most appropriate way of further treatment of diabetes is to administer short-acting insulin.
Key Points:
• 58-year-old patient with diabetes mellitus type II
• Regularly taking glyburide
• Admitted to hospital for acute abdomen

Therapy
• Blood sugar level of 9.8 mmol/L
• Indication for laparotomy
Explanation:
Diabetes mellitus type II is a chronic metabolic disorder characterized by high blood sugar levels
due to insulin resistance and/or insufficient insulin secretion. Treatment of diabetes mellitus
type II typically involves lifestyle modifications, such as diet and exercise, as well as
medications, including oral hypoglycemic agents and insulin.
In this case, the patient's acute abdomen and high blood sugar level of 9.8 mmol/L suggest a
need for immediate control of blood glucose levels. Glyburide is an oral hypoglycemic agent that
stimulates insulin secretion from the pancreas, but it may not be effective enough to control
blood glucose levels in an acute setting. The most appropriate way of further treatment of
diabetes in this case is to administer short-acting insulin to rapidly lower blood glucose levels.
Once the acute situation is resolved, the patient's diabetes management can be adjusted
accordingly. The use of oral hypoglycemic agents and/or insulin will depend on the patient's
individualized medical history, comorbidities, and response to treatment.
Other Options:
B To continue taking glyburide - Glyburide may not be effective enough to control blood glucose
levels in an acute setting, and the patient requires immediate management of high blood sugar
levels.
C To administer Semilong to be taken in the morning and insulin - in the evening - The patient
requires immediate management of high blood sugar levels, and the use of Semilong and insulin
may not be effective enough to rapidly lower blood glucose levels.
D To administer 1 tablet of Glurenorm three times a day - Glurenorm is an oral hypoglycemic
agent that stimulates insulin secretion from the pancreas, but it may not be effective enough to
control blood glucose levels in an acute setting.
E To administer 1 tablet of Maninil three times a day - Maninil is an oral hypoglycemic agent
that stimulates insulin secretion from the pancreas, but it may not be effective enough to
control blood glucose levels in an acute setting.
Making Other Options Right:
endocrinology

B To continue taking glyburide - This option could be made right if the patient's blood glucose
levels were stable and well-controlled with glyburide, and the acute abdomen was unrelated to
diabetes.
C To administer Semilong to be taken in the morning and insulin - in the evening - This option
could be made right if the patient's blood glucose levels were stable and well-controlled with
Semilong and insulin, and the acute abdomen was unrelated to diabetes.
D To administer 1 tablet of Glurenorm three times a day - This option could be made right if the
patient's blood glucose levels were stable and well-controlled with Glurenorm, and the acute
abdomen was unrelated to diabetes.
E To administer 1 tablet of Maninil three times a day - This option could be made right if the
patient's blood glucose levels were stable and well-controlled with Maninil, and the acute
abdomen was unrelated to diabetes.
Medication:
• Treatment for high blood glucose levels in an acute setting may require the administration of
short-acting insulin.
Words:
• Diabetes mellitus type II: a chronic metabolic disorder characterized by high blood sugar
levels due to insulin resistance and/or insufficient insulin secretion.
• Glyburide: an oral hypoglycemic agent that stimulates insulin secretion from the pancreas.
• Short-acting insulin: a type of insulin that is rapidly absorbed and has a short duration of
action.
Findings:
• 58-year-old patient with diabetes mellitus type II
• Regularly taking glyburide
• Admitted to hospital for acute abdomen
• Blood sugar level of 9.8 mmol/L

Therapy
• Supernutrition type
• Wooden belly
• Vesicular breathing in lungs
• Regular heart sounds, 90/min
• AP- 130/70 mm Hg
• Dry skin
• Blood sugar level: Normal fasting blood glucose levels should be below 5.6 mmol/L. A random
blood glucose level of 9.8 mmol/L is elevated and requires immediate management.
• Blood pressure: Normal blood pressure is generally considered to be 120/80 mm Hg or lower.
The patient's blood pressure of 130/70 mm Hg is slightly elevated but not necessarily
concerning in this context.
• Heart rate: Normal resting heart rate is generally considered to be between 60 and 100 beats
per minute. The patient's heart rate of 90/min is within the normal range.
• Physical examination findings: The patient's supernutrition type and dry skin are subjective
findings that do not have normal ranges. The presence of vesicular breathing in the lungs and
"wooden belly" are objective findings that suggest underlying medical conditions.
endocrinology

139 Diabetes

A 62 year old patient suffers from DM-2. Diabetes is being compensated by diet and
Maninilum. The patient has to undergo an operation on inguinal hernia. What tactics of
hypoglycemic therapy should be chosen?
A Prescribe fast-acting insulin
B Give Glurenorm instead of Maninilum
C Continue with the current therapy
D Prescribe long-acting insulin
E Prescribe guanyl guanidines

Correct Answer: The correct answer is to prescribe fast-acting insulin.


Key Points: The patient is a 62-year-old suffering from DM-2. Diabetes is controlled with diet
and Maninilum. The patient needs surgery on an inguinal hernia, which may cause changes in
blood glucose levels.
Explanation: Surgery is a stressful event that causes the release of stress hormones, such as
cortisol and adrenaline, which can lead to hyperglycemia. Therefore, patients with diabetes who
are scheduled for surgery require close monitoring and modification of their hypoglycemic
treatment to prevent complications. The current therapy of diet and Maninilum may not be
sufficient to maintain glycemic control during surgery, so the best option is to prescribe fast-
acting insulin. This will allow for rapid adjustment of blood glucose levels, and can be
discontinued as soon as the patient's condition stabilizes after the surgery.

Therapy
Other Options:
B) Giving Glurenorm instead of Maninilum is not recommended in this case as it is not as
effective as Maninilum in controlling blood sugar levels.
C) Continuing with the current therapy of diet and Maninilum is not the best option because
surgery is a stress factor that can cause changes in blood glucose levels.
D) Prescribing long-acting insulin is not the best option because it cannot be rapidly adjusted,
and the patient may be at risk for hypoglycemia or hyperglycemia during the surgery.
E) Prescribing guanyl guanidines is not recommended because they are not commonly used to
control blood sugar levels in DM-2 patients.
Medication:
-Maninilum is a sulfonylurea medication that stimulates the pancreas to produce more insulin,
which helps to lower blood sugar levels. It can cause hypoglycemia and weight gain as side
effects.
-Fast-acting insulin is a type of insulin that is injected subcutaneously and has a rapid onset of
action, typically within 15 minutes. Its duration of action is typically 3-5 hours, and it can be
used to treat hyperglycemia or prevent hypoglycemia. Side effects can include hypoglycemia,
weight gain, and injection site reactions.
endocrinology

140 Diabetes

A 33 y.o. woman has been suffering from DM (diabetes mellitus) for 5 years. For the last 3
years she has been taking more than 100 units of insulin per day. Body weight has
increased up to 10 kg. Fasting blood glucose is 13 mmol/L, glucoseuria - 3%. Generalized
microangiopathy. By increasing the dose of insulin the parameters of glycemia do not
change. The diagnosis is:
A DM 1st type, severe form, decompensation, insulin resistant
B DM 2nd type, severe form, decompensation
C DM st type, severe form, subcompensation, Somoji phenomenon
D DM 2nd type, moderate form, Zabrodi phenomenon
E DM 1st type, severe form, decompensation, allergic reaction to insulin

Correct Answer: A DM 1st type, severe form, decompensation, insulin resistant.


Key Points:
• A 33-year-old woman has been suffering from DM for 5 years.
• She has been taking more than 100 units of insulin per day for the last 3 years.
• Her body weight has increased up to 10 kg.
• Her fasting blood glucose is 13 mmol/L, glucoseuria is 3%.
• She has generalized microangiopathy.
• Increasing the dose of insulin does not change the parameters of glycemia.
Explanation:

Therapy
Based on the given information, the correct diagnosis is DM 1st type, severe form,
decompensation, insulin-resistant.
The patient has been suffering from DM for 5 years, and her blood glucose control has
deteriorated despite increasing the dose of insulin, which indicates decompensation. She is
taking a high dose of insulin, which suggests insulin resistance. Moreover, her increased body
weight and generalized microangiopathy may indicate insulin resistance.
Other Options:
• B: DM 2nd type, severe form, decompensation - This option is incorrect because the patient's
clinical features are more consistent with DM 1st type.
• C: DM st type, severe form, subcompensation, Somoji phenomenon - This option is incorrect
because there is no evidence of Somoji phenomenon, which is characterized by hypoglycemia
followed by rebound hyperglycemia.
• D: DM 2nd type, moderate form, Zabrodi phenomenon - This option is incorrect because there
is no evidence of Zabrodi phenomenon, which refers to the development of insulin resistance in
response to insulin therapy.
• E: DM 1st type, severe form, decompensation, allergic reaction to insulin - This option is
incorrect because there is no evidence of an allergic reaction to insulin.
Making Other Options Right:
• B: DM 1st type, severe form, decompensation, insulin-resistant - This option would be correct
if the patient's clinical features were consistent with DM 1st type, such as positive
autoantibodies and rapid onset of symptoms.
• C: DM 1st type, severe form, decompensation, insulin-resistant - This option would be correct
if the patient's blood glucose control improved with insulin therapy and if there was no evidence
of Somoji phenomenon.
• D: DM 2nd type, severe form, decompensation, insulin-resistant - This option would be correct
if the patient's clinical features were consistent with DM 2nd type, such as insulin resistance,
obesity, and a slow onset of symptoms.
• E: DM 1st type, severe form, decompensation, insulin-resistant - This option would be correct
if the patient's clinical features were consistent with DM 1st type and if there was evidence of an
allergic reaction to insulin.
Medication:
endocrinology

• Insulin: Insulin is a hormone that regulates blood glucose levels. It lowers blood glucose levels
by stimulating glucose uptake by cells and promoting glycogen synthesis. Side effects of insulin
therapy may include hypoglycemia, weight gain, and injection site reactions.
Words:
• DM: Diabetes Mellitus
• Glycemia: The concentration of glucose in the blood.
Findings:
• Fasting blood glucose: Normal range is 3.9-5.5 mmol/L. A fasting blood glucose level of 13
mmol/L indicates hyperglycemia.
• Glucoseuria: Normal range is negative. Glucoseuria of 3% indicates hyperglycemia with renal
involvement.
In general, hyperglycemia damages blood vessels' small capillaries and leads to
microangiopathy in many organs, including the eyes, kidneys, and nerves. The small blood
vessels' basement membranes become thickened, and the capillaries' lumen narrows, leading to
a reduction in blood supply to these organs, which can result in damage to them.
In this patient's case, the presence of microangiopathy and kidney damage is an indication of
severe complications resulting from long-term uncontrolled diabetes mellitus. This finding
further supports the diagnosis of severe decompensated diabetes mellitus.

Therapy
Resources Section
1. Link to Bases:
• Access the complete set of bases for the Krok 2 examination.
• Questions that share similarities across bases have been highlighted, providing a comprehensive
overview of recurring topics.
2. Normal Values for Krok 2 Labs:
• Download and print this PDF document containing a comprehensive list of normal laboratory
values relevant to the Krok 2 examination.
• Find detailed information on normal reference ranges for various blood tests, urine tests, and
other diagnostic investigations.
• Understand the significance of deviations from normal values and their diagnostic implications.
• Having a printed copy of the normal values can serve as a quick reference during your studies.
3. Question Papers from 2005-2023:
• Explore a valuable collection of question papers from previous Krok 2 examinations.
• Review and practice with questions from past years to enhance your understanding and
familiarity with the exam format.
• Learn from the patterns and trends observed in previous exams to better prepare for the current
examination.
- Note: The provided links will direct you to external resources. Please ensure a stable internet
connection to access these materials effectively. It is advisable to cross-reference the information
obtained from external sources with other reputable educational references.
Remember, these resources are meant to supplement your learning and should be used in conjunction with
comprehensive study materials and guidance from your faculty or educational institution.

Keep up your dedication and continue striving for excellence in your medical education!
Best regards, Mohammed Abdul Danish
Reference Lab Values

Complete blood count

Male: 4.3 − 5.9 · 1012/L


Erythrocyte count
Female: 3.5 − 5.5 · 1012/L

Male: 135-175 g/L


Hemoglobin, blood
Female: 120-160g/L

Color index 0.85 – 1.05

Reticulocyte count 0.5 – 1.5%


150 − 400 · 109/L
Platelet count

Leukocyte count 4.0 − 9.0 · 109/L

Basophils count 0 − 0.065 · 109/L (0-1%)

Eosinophils 0.02 − 0.30 · 109/L (0.5 – 5.0%)

Band neutrophils 0.04 − 0.30 · 109/L (1 – 6%)

Segmented neutrophils 2.0 − 5.50 · 109/L (47 – 72%)

Monocytes 0.09 − 0.60 · 109/L (3 – 11%)

Lymphocytes 1.2 − 3.0 · 109/L (19 – 37%)

Male: 0 – 15 mm/h
Erythrocyte sedimentation rate (ESR)
Female: 0 – 20 mm/h

Male: 41 – 53%
Hematocrit
Female: 36 – 46%
Biochemical blood analysis (Metabolic panel)

Total proteins 60-78 g/L

Albumin 35-50 g/L (52-65%)


Globulin 23-35 g/L (35-48%)
α1-Globulin 2-4 g/L (4.2-7.2%)
α2-Globulin 5-9 g/L (6.8-12%)
β-Globulin 6-11 g/L (9.3-15%)
γ-Globulin 11-15 g/L (15-19%)
IgD 0 – 0.15 g/L
IgG 6.5-15 g/L
IgM 0.4-3.45 g/L
IgA 0.76-3.90 g/L
IgE 0-380 kU/L
Total Bilirubin 2-17 mcmol/L
Indirect (unconjugated) Bilirubin 2-17 mcmol/L
Direct (conjugated) Bilirubin 0-5 mcmol/L
Triglycerides 0.59-1.77 mmol/L
Total cholesterol 3.9-6.2 mmol/L
High-density lipoproteins (β-Lipoproteins) <4.2 mmol/L
Low-density lipoproteins (α-Lipoproteins) 0.8-1.8 mmol/L
Glucose, blood 3.3-6.1 mmol/L (fasting)
Glycated hemoglobin 6%
Iron, blood 9-30 mcmol/L
Potassium, plasma 3.5-5.0 mmol/L
Sodium, plasma 136-145 mmol/L
Calcium, plasma 0.75-2.5 mmol/L
Magnesium, plasma 0.75 – 1 mmol/L
Phosphorus (inorganic), serum 0.646-1.292 mmol/L
Chloride, blood 95-105 mmol/L
Urea, serum 3.33 – 8.32 mmol/L
Creatinine 53 – 106 mcmol/L
Male: 0.12-0.38 mcmol/L
Uric acid
Female: 0.12-0.46 mcmol/L
Lactate dehydrogenase 45-90 U/L
α-Amylase (diastase), blood 25-125 U/L
Aspartate aminotransferase (AST) 15-40 U/L
Alanine aminotransferase (ALT) 10-40 U/L
Phosphatase (alkaline) 30-115 U/L
Coagulogram
Acid-base balance
80-100% (< 12
Prothrombin time
seconds)
pH arterial blood 7.35-7.45
Fibrinogen 2-4 g/L

Pco2 arterial blood 33-45 mm Hg


Partial thromboplastin time
< 28 seconds
(activated)

Po2 arterial blood 75 – 105 mm Hg


Lee-White coagulation time 5-10 minutes
Bicarbonate, blood
Bleeding time (Duke 22-28 mmol/L
< 4 minutes (HCO3)
method)

Other blood values Urine

Cortisol, serum 230 -750 nmol/L


Urine specific gravity 1.016-1.022
Osmolality, serum 275 – 295 mOsmol/kg Nechiporenko’s test:
< 2 · 106/л
leukocytes
Thyroid-stimulating Nechiporenko’s test:
0.5-5 mIU/L < 1 · 106/л
hormone, serum or plasma erythrocytes

Thyroxine (T4), serum 65-155 nmol/L <45.0 – 75.0 mg/24 h


Proteins, total
(<0.033 g/L)
Thyroid-stimulating
0.5-5 mIU/L
hormone, serum or plasma
Calcium 2.5 – 7.5 mmol/24h
Triiodothyronine (T3),
1.77 – 2.43 nmol/L Male: 97 – 137 mL/min
serum Creatinine clearance
Female: 88 – 128mL/min
Male:15-200 mcg/L
Ferritin, serum Oxalate 90 – 445 mcmol/L
Female: 12-150 mcg/L

Thymol turbidity test <5 U Male: 27.7 – 79.7


mcmol/24h
17-Ketosteroids (17-KS)
Female: 17.4 – 55.4
C-reactive protein (CRP) <6 mg/dL mcmol/24h

17-Hydroxycorticosteroids
Antistreptolysin 0 (ASL0) 250 U 5.5 – 27.6 mcmol/24 h
(17-OCS)
Antistreptohyaluronidase
250 U
(ASH) α-Amylase (diastase), urine 28-160 U/L
List of abbreviations
ACTH => Adrenocorticotrophic hormone IDL => Intermediate-density lipoproteins
ADP => Adenosine diphosphate IU => International unit
ALT, ALAT => Alanin aminotranspherase IUPAC => International Union of Pure and
Applied Chemistry
ALV => Artificial lung ventilation
LDH => Lactate dehydrogenase
AP => Arterial (blood) pressure
LDL => Low-density lipoproteins
AST, AspAT => Aspartat aminotranspherase
LOX => Lipoxygenase
ATP => Adenosine triphosphate
MAO => Monoamine oxidase
ВСG => Bacillus Calmette-Guérin
MRI => Magnetic resonance imaging
BP => Blood (arterial) pressure
NSAID => Nonsteroidal anti-inflammatory drug
CK => Creatine kinase
PE (PATE) => Pulmonary embolism (Pulmonary
CNS => Central nervous system artery thromboembolism)
СоА => Coenzyme A PSA => Prostate-specific antigen
CRP => C-reactive protein RBC => Red blood count
CT => Computed tomography RR => Respiratory rate
COX => Cyclooxygenase SES => Sanitary-and-epidemiologic station
DIC => Disseminated intravascular coagulation STD => Sexually transmitted disease
DTP (DPT) => Diphtheria-tetanus-pertussis STI => Sexually transmitted infection
vaccine
T/l => Trillion/liter
EDTA => Ethylenediamine tetra-acetic acid
TABT => Typhoid-paratyphoid A and B plus
ELISA => Enzyme-linked immunosorbent assay tetanus toxoid vaccine
ENT => Ear, nose, and throat (as a department TMJ => Temporomandibular joint
in a hospital)
TSH => Thyroid-stimulating hormone
EPR (ER) => Endoplasmic reticulum
TU => Tuberculin unit
ESR => Erythrocyte sedimentation rate
URTI => Upper respiratory tract infection
EV => Enterovirus
V/f => Vision field
FC => Functional class
VLDL => Very-low-density lipoproteins
GABA => Gamma aminobutyric acid
WBC => White blood count
Hct Ht => Hematocrit
WPW => Wolff-Parkinson-White syndrome
HDL => High-density lipoproteins
HR => Heart rate

You might also like